Vous êtes sur la page 1sur 210

www.crackjee.

xyz

www.crackjee.xyz
Head Office : B-32, Shivalik Main Road, Malviya Nagar, New Delhi-110017

• Sales Office : B-48, Shivalik Main Road, Malviya Nagar, New Delhi-110017
Tel. : 011-26691021 / 26691713

Page Layout : Prakash Chandra Sahoo

Typeset by Disha DTP Team

DISHA PUBLICATION
ALL RIGHTS RESERVED

© Copyright Author

No part of this publication may be reproduced in any form without prior permission of the publisher. The author and the
publisher do not take any legal responsibility for any errors or misrepresentations that might have crept in. We have
tried and made our best efforts to provide accurate up-to-date information in this book.

For further information about the books from DISHA,


Log on to www.dishapublication.com or email to info@dishapublication.com
www.crackjee.xyz
STUDY PACKAGE IN PHYSICS FOR JEE MAIN & ADVANCED
Booklet No. Title Chapter Nos. Page Nos.
Ch 0. Mathematics Used in Physics
Ch 1. Units and Measurements
Units, Measurements &
1 Ch 2. Vectors 1-202
Motion
Ch 3. Motion in a Straight Line
Ch 4. Motion in a Plane
Laws of Motion and Ch 5. Laws of Motion and Equilibrium
2 203-318
Circular Motion Ch 6. Circular Motion
Ch 7. Work, Energy and Power
Work Energy, Power &
3 Ch 8. Collisions and Centre of Mass 319-480
Gravitation
Ch 9. Gravitation
4 Rotational Motion Ch 1. Rotational Mechanics 1-120
Ch 2. Properties of Matter
Properties of Matter &
5 Ch 3. Fluid Mechanics 121-364
SHM
Ch 4. Simple Harmonic Motion
Ch 5. Thermometry, Expansion &
Calorimetry
6 Heat & Thermodynamics Ch 6. Kinetic Theory of Gases 365-570
Ch 7. Laws of Thermodynamics
Ch 8. Heat Transfer
Ch 9. Wave – I
7 Waves 571-698
Ch 10. Wave –II
Ch 0. Mathematics Used in Physics
8 Electrostatics Ch 1. Electrostatics 1-216
Ch 2. Capacitance & Capacitors
Ch 3. DC and DC circuits
9 Current Electricity Ch 4. Thermal and Chemical effects of 217-338
Current"
Ch 5. Magnetic Force on Moving
Charges & Conductor
Ch 6. Magnetic Effects of Current
10 Magnetism, EMI & AC Ch 7. Permanent Magnet & Magnetic 339-618
Properties of Substance
Ch 8. Electromagnetic Induction
Ch 9. AC and EM Waves
Ch 1. Reflection of Light
Ch 2. Refraction and Dispersion
11 Ray & Wave Optics Ch 3. Refraction at Spherical Surface, 1-244
Lenses and Photometry
Ch 4. Wave optics
Ch 5. Electron, Photon, Atoms,
Photoelectric Effect and X-rays
12 Modern Physics 245-384
Ch 6. Nuclear Physics
Ch 7. Electronics & Communication
www.crackjee.xyz
Contents
Contents
Study Package Booklet 6 - Heat & Thermodynamics
5. Thermometry, Expansion & 7.7 Reversible process 464
Calorimetry 365-414 7.8 Thermodynamical processes 464
5.1 Introduction 366 7.9 Change in internal energy 469
5.2 Temperature 366 7.10 Polytropic process 470
5.3 Thermal expansion of solids 370 7.11 Weakness of first law and need of 477
5.4 Expansion of liquids 373 second law
5.5 Expansion of gases 376 7.12 Second law of thermodynamics 478
5.6 Heat and calorimetry 382 7.13 Entropy 478
5.7 Special heat 382 7.14 Heat engine 479
5.8 Heat capacity or thermal capacity 385 7.15 Carnot reversible heat engine 479
5.9 Water equivalent 385 7.16 Refrigerator or heat - pump 482
5.10 Change in phase 386 Review of formulae & important points 485
5.11 Latent heat 387 Exercise 7.1 - Exercise 7.6 486-508
5.12 Law of mixture or law of calorimetry 389
Hints & solutions 509-526
Review of formulae & important points 392
Exercise 5.1 - Exercise 5.6 393-404 8. Heat Transfer 527-570
Hints & solutions 405-414
8.1 Modes of heat transfer 528
6. Kinetic Theory of Gases 415-456 8.2 Thermal conduction 528
8.3 Steady state and temperature gradient 528
6.1 Introduction 416
8.4 Rate of flow of heat : heat current 529
6.2 Gas laws 416
8.5 Thermal resistance 529
6.3 Ideal gas 418
6.4 Kinetic theory of gases 423 8.6 Determination of thermal conductivity 530
6.5 Translational kinetic energy 424 8.7 Combination of metallic rods 532
6.6 Graham’s law of diffusion 425 8.8 Redial flow of heat 533
6.7 Dalton’s law of partial pressure 426 8.9 Cylindrical flow of heat 534
6.8 The distribution of molecular speed 426 8.10 Formation of ice on ponds 534
6.9 Degrees of freedom 429 8.11 Application of conductivity in daily life 539
6.10 Law of equipartition of energy 431 8.12 Convection 539
6.11 Mean free path 432 8.13 Phenomenon based on convection 539
6.12 Phases and phase diagrams 433 8.14 Radiation 541
6.13 Vapour pressure 436 8.15 Spectral absorptive power 541
6.14 Dew point 437
8.16 Spectral emissive power 541
6.15 Humidity 437
8.17 Emissivity 542
Review of formulae & important points 438
Exercise 6.1 - Exercise 6.6 439-448 8.18 Black body 542
Hints & solutions 449-456 8.19 Kirchhoff’s law 542
8.20 Prevost theory of heat exchange (1792) 543
7. Laws of Thermodynamics 457-526
8.21 Stefan -boltzmann law 543
7.1 Thermodynamical terms 458 8.22 Newton’s law of cooling 543
7.2 Internal energy 458 8.23 Wien’s displacement law 547
7.3 Internal energy of an ideal gas 458 8.24 Solar constant 547
7.4 Work in volume change 459 Review of formulae & important points 548
7.5 Work done in cyclic process 460 Exercise 8.1 - Exercise 8.6 549-560
7.6 First law of thermodynamics 461 Hints & solutions 561-570
www.crackjee.xyz
www.crackjee.xyz
366 MECHANICS, HEAT, THERMODYNAMICS & WAVES
5.1 INTRODUCTION
Mechanics deals with the quantities like; acceleration, force, mechanical energy (external
energy) and Newton’s second law and conservation of energy. Thermodynamics deals
with temperature, heat, internal energy, entropy and the first and second law of
thermodynamics. The fundamental quantity in thermodynamics is temperature, so we
start our study by developing the concept of temperature from its foundation.

5.2 TEMPERATURE
Temperature is a macroscopic quantity of thermodynamics which gives the sensation of
hotness or coldness. Its SI unit is kelvin. The lowest temperature that has achieved in lab
is 2 × 10–8 K, in the process, called nuclear spin. Scientist are trying to see how close
they can come to absolute zero temperature. The highest laboratory temperature is
108 K.
Zeroth law of thermodynamics : Thermal equilibrium
A system is said to be in the state of thermodynamic equilibrium if the variable describing
the thermodynamic state of a system do not change with time. A system in thermal
equilibrium have no unbalanced force between the system and the surroundings and
each part of the system and the surrounding are at the same temperature. According to
zeroth law of thermodynamics, if two bodies A and B are in thermal equilibrium with third
body C separately, then bodies A and B are also be in thermal equilibrium. The zeroth law,
came in light in 1930 s, after the first and second laws of thermodynamics had been
discovered. Temperature on being central concept should have the lowest number, hence
the zero.
Measuring temperature
Any device which is used to measure the temperature is called thermometer. A thermometer
makes use of some property like; length of mercury column, pressure of gas etc. whose
variation with temperature is defined by a law. There is no property which varies by a law
for all range of temperature. Therefore we need different properties and hence different
thermometers.
Fixed points on a temperature scale
The old method was based on two fixed points of temperature which can be easily
reproduced in laboratory. The temperature of melting ice at 1 atm (called ice point or
freezing point) and the temperature of boiling water at 1 atm (called steam point or
boiling point). On Celsius scale the freezing point is assumed as 0 °C and the boiling
point as 100 °C.
Suppose a property ‘X’ which is defined by assuming a linear relation with t as
Xt = at b … (i)
If X0 and X100 denote the values of property at 0°C and 100°C respectively, then from
equation (i), we have
X0 = a 0 b … (ii)
and X 100 = a 100 b . … (iii)
After solving equations (ii) and (iii), we get
b = X0
X100 X 0
and a = .
Fig. 5.1 100
Substituting these values in equation (i), we get

Xt X0
t = 100 degree.
X100 X 0
Here property X may be length of mercury column, resistance of metal etc.
www.crackjee.xyz
THERMOMETRY, EXPANSION AND CALORIMETRY 367
1. Mercury thermometer
Its working is based on the fact that mercury expand uniformly on heating. If 0,
100 and t be the lengths of mercury column at 0°C, 100°C and at unknown
temperature t°C respectively, then the unknown temperature t can be calculated as

t 0
t = 100 .
100 0

The range of mercury thermometer is quite large because of low freezing point
(–39°C) and high boiling point (357°C).
2. Platinum resistance thermometer
The electric resistance of metal wire increases with temperature as, Rt R0 (1 t) ,
where is the temperature coefficient of resistance. If R0 and R100 denote the
resistances of metal wire at 0°C and 100°C respectively, then we can define the
temperature as

Rt R0
t = 100 degree .
R100 R0

Platinum resistance thermometer can be used in temperature range – 170°C to


200°C.
Temperature scales
1. The Celsius scale
On this scale, the lower fixed point is taken as 0°C the upper fixed point as 100°C.
The interval between these fixed points is divided into hundred equal divisions
and each division is called 1 °C.
2. The Fahrenheit scale
On this scale lower and upper fixed points are taken 32 °F and 212 °F respectively.
The interval between these fixed points is equally divided into 180 division. The
each division is equal to 1 °F.
3. The Kelvin scale
On this scale the lower and upper fixed points are taken as 273.15 K and 373.15 K
respectively. The interval between these fixed points is divided equally in 100
divisions. Thus each division represents 1 K.
1 division of C = 1.8 division of F = 1 division of K

Fig. 5.2
www.crackjee.xyz
368 MECHANICS, HEAT, THERMODYNAMICS & WAVES
Conversion of temperature from one scale to another
This can be done by using following relation
Temerature on any scale - lower fixed point
Constant for each scale
Upper fixed point - lower fixed point

X F .P. C 0 F 32 K 273.15
Thus we = = =
B.P. F .P. 100 0 212 32 373.15 273.15
Fig. 5.3 Here X is the temperature on any scale. F.P. and B.P. stand for freezing
point and boiling point of water respectively.
From the above equation, we have
C F 32 K 273.15
= =
5 9 5

9
or F = C 32
5
and K = C + 273.15
Absolute zero and absolute scale of temperature
Fig. 5.4
3RT
According to kinetic theory of gases vrms . The gas is supposed to have no
M
molecular motion (Also zero volume and zero pressure) at T = 0. This temperature is
called absolute zero temperature. Kelvin scale is called absolute temperature scale.
Triple point of water
The triple point of water is the state at which the three phases of water namely ice, water
and vapour are co-exist in equilibrium. It is unique because it occurs at a specific
temperature of 273.16 K and at a pressure of 4 mm of mercury. Thus for water.
Tr = 273.16 K or 0.01°C
Pr = 4 mm of mercury

Note:
In modern thermometry, the triple point of water is chosen to be one of the fixed points.
The melting point of ice and boiling point of water both change with pressure but triple
point is characterised by a unique temperature and pressure, so it is preferred over the
conventional fixed points.
In the absolute scale, the triple point of water is assigned the value 273.16 K. The
absolute zero is taken as the other fixed point on this scale.
Constant volume gas thermometer : Ideal gas temperature
If P and Pr are the pressures of constant volume of a gas at temperatures T and at the
triple point Tr respectively, then by Charle’s law, we have
T P
Ttr
= Ptr

P
Fig. 5.5 or T = Ttr
Ptr

P
or T = 273.16 K. ...(1)
Ptr
The temperature defined by the above equation depends slightly on the nature of the
gas and its pressure. But at high temperature and low pressure, in the limit Ptr 0 , all
www.crackjee.xyz
THERMOMETRY, EXPANSION AND CALORIMETRY 369
the different gas thermometers give the same value of temperature 373.15 K for steam
point. So we define a temperature scale by the equation

Lim P
T = Ptr 0
273.16 K. ...(2)
Ptr

The temperature scale defined by equation (2) is called ideal gas temperature.

Note: Fig. 5.6

One can also define a centigrade scale with gas thermometers. Suppose P0 and P100
are the pressure of a constant volume of gas at melting ice and in a steam bath
respectively, then temperature t corresponding to pressure Pt of the gas is defined by

Pt P0
t = 100 C
P100 P0

Radiation pyrometer
It is based on Stefan’s law, which states that the radiation emitted by a black body per
unit area per second is proportional to the fourth power of the absolute temperature i.e.,
E = T4, where is called Stefan’s constant. By noting the radiation E we can calculate
temperature T as 1
E 4
T = K.

The main advantage of the pyrometer is that it is capable of measuring the temperature,
however high it may be and whatever the distance of the object from the pyrometer. It
can accurately measure temperature greater than 1000 K.

Ex. 1 What is the value of the following in °F?


Ex. 3 The electrical resistance in ohm of a certain the
(i) 40 °C temperature,
(ii) 40 °C difference of temperature. rmometer varies with temperature according to the approximate
law:
Sol.
9 R = R0 [1 + 5 ×10-3 (T - T0 )]
(i) We know that F = C 32
5 The resistance is 101.6 at the triple point of water and 165.5 at
9 the normal melting point of lead (600.5 K ). What is the temperature
F = 40 32 when the resistance is 123.4 ?
5
= 104 °F Sol.
(ii) 1 °C difference is equal to
At T = 273 K, R = 101.6
= 1.8 °F
40 °C difference is equal to 101.6 = R0 [1+ 5 × 10–3 (273 – T0)] ...(i)
= 1.8×40 When T = 600.5 K, R = 165 .5
= 72 °F 165.5 = R0[1 + 5 × 10–3 (600.5 – T0)] ...(ii)
Ex. 2 A faulty thermometer reads freezing point and boiling After solving equations (i) and (ii), we get
point of water as 5 °C and 95 °C respectively. What is the correct T 0 = – 49.3 K
value of temperature as it reads 60 °C on faulty thermometer? Substituting the value of T0 in equation (i), we get
Sol. 101.6 = R0(1 + 5 × 10–3 (273 + 49.3)]
If X is the value of temperature on faulty thermometer, then
101.6
X F .P. C 0 or R0R = (1 5 10 3 322.3) 38.9
=
B.P. F .P. 100 0
For R = 123.4 ;
60 5 C
or = 123.4 = 38.9 [1+5×10–3 (T+ 49.3)]
95 5 100
After solving, we get
After solving , we get
T = 384.8 K Ans.
C = 61.11 °C Ans.
www.crackjee.xyz
370 MECHANICS, HEAT, THERMODYNAMICS & WAVES

Ex. 4 Two ideal gas thermometers A and B use oxygen and P


hydrogen respectively. The following observations are made: We have T= Ttr
Ptr

Pressure Pressure 1.797 105


Temperature 273 392.46 K
thermometer A thermometer B 1.250 105
Triple point of water 1.250×105 Pa 0.200×105 Pa For thermometer B;
Normal melting point 5 5
T tr = 273 K,
1.797×10 Pa 0.287×10 Pa
of sulpher Ptr = 0.200 × 105 Pa
T = ?, P = 0.287 × 105 Pa
(a) What is the absolute temperature of normal melting point of
sulphur as read by thermometer A and B? P
We have T = P Ttr
(b) What do you think is the reason for slightly different answers tr

from A and B?
0.287 105 273
391.75K
Sol. 0.200 105
(a) For thermometer A; (b) The slight difference in the temperatures as read by two
Ttr = 273 K , Ptr = 1.250 × 105Pa thermometers are due to the fact that oxygen and hydrogen do not
behave like an ideal gas.
and T = ?, P = 1.797 × 105 Pa

5.3 THERMAL EXPANSION OF SOLIDS


Substances expand on heating. The atoms of a solid are held together in a three dimensional
lattice by interatomic forces. The individual atoms vibrate about these lattice. When solid
is heated, the amplitude of vibration of atoms increases, and so the average interatomic
separation increases. This result in the thermal expansion of solid.
Coefficients of expansion of solid
1. Coefficient of linear expansion:

Increase in length
=
Original length × rise in temperature

L
or =
L0 T
If L0 and Lt are the lengths at 0°C and t°C respectively, then L Lt L0 and
T t 0 t C
Lt L0
=
L0 t
or Lt = L0 (1 t)

Note:
1 dL
1. In differential form can be written as ;
L0 dt
2. The above equation is applicable for small value of t (t < 100 °C) , till the expansion
of the material can be taken uniform. For higher range of temperature, Lt is given
by; 2
Lt L0 (1 1t 2t ....), where 1 2 ....
2. Coefficient of superficial expansion
Increase in area
=
Original area × rise in temperature

A
or =
A0 T
www.crackjee.xyz
THERMOMETRY, EXPANSION AND CALORIMETRY 371
If A0 and At are the areas at 0°C and t°C respectively, then A At A0 and
T t 0 t C
At A0
=
A0 t
or At = A0 (1 t)
3. Coefficient of cubical expansion

Increase in volume
=
Original volume ×rise in temperature

V
or =
V0 T

If V0 and Vt are the volumes at 0°C and t°C respectively, then V Vt V0 and
T t 0 t C

Vt V0
= V0 t

or Vt = V0 (1 t)

Note: Value of , and depends on material of the body..

Variation of with temperature


The value of is not constant at all range of temperature. The variation of cubical
expansion of copper with temperature is shown in Fig. 5.7.
At high temperature (above 500 K) its value becomes almost constant.
Relationship between , and
(i) Consider a sheet of length a and width b, the area of the sheet
Fig. 5.7
A = ab ...(i)
For small change in area ( A), we can write

A a b
= ...(ii)
A a b
Dividing both sides of the equation by T , we have
A a b
=
A T a T b T

or = 1 2
where 1and 2are the coefficients of linear expansions along length and width
respectively. For a material having expansion coefficient same in all directions, we
have 1 2

= 2
(ii) Take a cuboid of sides a, b and c respectively its volume ,
V = abc ...(i)

V a b c
We can write, = ...(ii)
V a b c
www.crackjee.xyz
372 MECHANICS, HEAT, THERMODYNAMICS & WAVES
Dividing both sides of the equation (ii) by T , we have
V a b c
=
V T a T b T c T

or = 1 2 3
For a material having expansion coefficient same in all directions, we have,
1 2 3

= 3

Note:
It is difficult to determine and directly. For solids, the value of is determined
and value of and are taken approximately equal to 2 and 3 respectively..

Variation of density with temperature


(i) Suppose 0 is the density of the substance at 0°C. At any temperature, let it
becomes t . As mass of the substance remains constant at any temperature, so we
have

0V0 = tVt
Here V0 and Vt are the volumes of the substance at 0°C and t°C respectively.
Also Vt = V0 (1 t)

0V0 = t V0 (1 t)

0
or t =
(1 t)
We can write,

= 1
t 0 (1 t)
For small value of , we can approximate, it as;

t 0 (1 t)

(ii) If 1 and 2 are the densities at t1 and t2 respectively, then we can write

1V1 = 2V2

or 1V0 (1 t1 ) = 2V0 (1 t2 )

(1 t2 )
or 1 = 2
(1 t1 )

= 2 (1 t2 )(1 t1 )

= 2 [1 (t2 t1 )]
{neglecting 2 on being small}

1 2
=
2 (t2 t1 )
www.crackjee.xyz
THERMOMETRY, EXPANSION AND CALORIMETRY 373

5.4 EXPANSION OF LIQUIDS


When a liquid is heated, the containing vessel also expand and hence the observed
increase in volume of the liquid is the apparent increase in volume. Thus:
Real expansion of liquid = Apparent expansion of liquid + expansion of vessel.

Coefficient of real expansion of liquid ( r )


If V0 and Vt are the volumes of liquid at 0°C and t°C respectively, then

V
r =
V0 T

Here, V = Vt V0 and T t 0 t

Vt V0
r =
V0 t

or Vt = V0 (1 rt)

If aand g are the apparent coefficients of expansion of liquid and volume coefficient
of expansion of container respectively, then

or r = a g

Determination of a
Method - I : By Dilatometer
Let V0 and Vt are the apparent volume noted (volume w.r.t container)
at 0°C and t°C respectively, then
Fig. 5.8
Vt V0
a = .
V0 t

Determination of the coefficient of real expansion of liquid


by Dulong and Petit’s method
The apparatus consists of a long glass tube. The tube contains the liquid whose
coefficient of real expansion is to be determined. Let the temperature of the cold and the
hot limbs are t1 °C and t2 °C respectively. Again let h1 and h2 be the height of liquid
columns and 1 and 2 be the densities of the liquids at temperatures t1 and t 2
respectively, then
Pressure at B = Pressure at C
Pa h1 1g = Pa h2 2 g … (1)
where Pa is the atmospheric pressure.
The equation (1) reduces to h1 1 = h2 2 … (2)
0 0
Here, 1 = and 2
1 1 r t2
r t1
0 0
h1 = h2 Fig. 5.10
1 r t1 1 r t2

or h1 (1 r t2 ) = h2 (1 r t1 )

h1 h1 r t2 = h2 h2 r t1

r ( h1t 2 h2t1 ) = h2 h1
www.crackjee.xyz
374 MECHANICS, HEAT, THERMODYNAMICS & WAVES

h2 h1
or r =
h1t2 h2 t1

h h0
If t1 = 0, h1 = h0 and t2 = t, h2 = h, then r =
h0 t

Determination of the coefficient of real expansion of liquid


: Second method
With the experimental set-up and data given in the figure, we have to find the value of r
In the Fig. 5.11 t1, t2, t3 and t4 denotes the temperatures.
If 0 is the density of mercury at 0°C, then

0 0 0 0
1
1
; 2
1
; 3
1
; 4
1
r t1 r t2 r t3 r t4

It is clear from the figure that, the pressures at D due to the columns of mercury in the hot
and cold limbs must be equal .
h1 2 g H1 1 g = h2 2g H 2 3 g h3 4 g

or h1 2 H1 1 = h2 2 H2 3 h3 4 … (1)

Substituting values of 1, 2 , 3 and 4 in equation (1), we have

0 0 0 0 0
h1 H1 = h2 H2 h3
1 r t2 1 r t1 1 r t2 1 r t3 1 r t4
Fig. 5.11
h1 H1 h2 H2 h3
or =
1 r t2 1 r t1 1 r t2 1 r t3 1 r t4

h1 h2 H1 H2 h3
or =
1 r t2 1 r t1 1 r t3 1 r t4

By putting the values of heights and temperatures, the value of r can be calculated.

Anomalous expansion of water


Most of the substances expand on heating. But water contracts instead of expanding,
when heated from 0°C to 4 °C, after which it expands in the usual manner. Thus water has
its least volume and maximum density at 4 °C. It can be seen from the Fig. 5.12.
Correction for barometric reading
The scale of barometer is usually calibrated at 0°C. If observation is taken at a different
temperature, then there need correction for brass-scale. Suppose the height of mercury
at 0°C is H0 and true scale reading is H. If is the coefficient of linear expansion of
brass, then true height of brass scale at temperature t,
Ht = H (1 t) … (1)
Fig. 5.12
As atmospheric pressure is constant at all temperature, so we have
Pressure at 0°C = Pressure at t°C
or H0 0 g = Ht t g … (2)
www.crackjee.xyz
THERMOMETRY, EXPANSION AND CALORIMETRY 375

Here 0 and t are the densities of mercury at 0°C and t°C respectively. Also we have

0
t , being volume coefficient of mercury. Thus from equations (1) and (2), we
1 t
have

0
H0 0 g = H (1 t) g
1 t

or = H (1 1
H0 t )(1 t) Fig. 5.13
H0 = H (1 t )(1 t)

H0 = H [1 t t t2 ] H [1 ( )t ]

Since and both are small (~ 10 5


/ C) , so their product becomes very small, and
therefore can be neglected.
H0
H =
[1 ( )t ]

Bimetallic strip
Bimetallic strip is made of two metal strips placed in contact (see Fig. 5.14). The strip
works as an electric contact breaker in an electrical circuit.

Fig. 5.14

When bimetallic strip is heated, it curves due to the difference in the coefficients of linear
expansion of the two metals, and the circuit breaks (figure b).
The metal of higher coefficient of expansion bends more than the other of lesser coefficient
of expansion. When temperature falls, the bimetallic strip contracts and the contact at P
is restored.

Ex. 5 The coefficient of cubical expansion of mercury is True reading at 30 °C,


0.00018 /°C and that of brass 0.00006/°C. If a barometer having a
brass-scale were to read 74.5 cm at 30 °C, find the true barometric 30 = observed
height at 0°C. The scale is supposed to be correct at 15 °C. = 74.5 0.02235
= 74.522 cm
Sol. Coefficient of linear expansion of brass,
As pressure is constant at all temperature, so
0.00006 Pressure at 0°C = Pressure at 30 °C
=
3 3
or h0 0 g h30 30 g
= 0.00002/°C
The brass scale is true at 15 °C, therefore at 30 °C its graduation will h30 30 h30 0
increase in length and so observed reading will be less than true reading
h0 = =
0 0 (1 Hg t )
by,
= t 74.5 0.00002 15 74.522
= 0.02235 cm === 74.122 cm Ans.
1 0.00018 30
www.crackjee.xyz
376 MECHANICS, HEAT, THERMODYNAMICS & WAVES
Sol.
Fact to know : Yes, let 1 and 2 are the lengths of brass and steel rods at 0°C and 1
Invar is an alloy of nickel and steel which has very small and 2 are their respective coefficients of expansion. If change in lengths
coefficient of expansion. That is why the pendulum clock of the rods are equal, then their difference remain constant at all
provided with invar pendulum, whose length and therefore temperature. i.e.,
time period remains almost constant.
1 = 2

Ex. 6 The difference between length of a certain brass rod or 1 1t = 2 2t


and that of a steel rod is claimed to be constant at all temperature or 1 1 = 2 2
is this possible?

5.5 EXPANSION OF GASES


When gases are heated, they may change in pressure and volume both. Hence there are
two coefficients of expansion of a gases.
(i) Volume coefficient of a gas, v :
It can be defined as ;
Increase in volume (keeping pressure constant)
v = Original volume × rise in temperature

V
=
V0 T
If V0 and Vt are the volumes of gas keeping pressure constant at 0°C and t°C
respectively, then
V = Vt V0 , T t 0 t
Vt V0
v = V0 t
or Vt = V0 (1 vt)
Value of v for an ideal gas :
For an ideal gas, PV = nRT …(1)
At constant pressure
P V = nR T … (2)
Dividing (2) by (1), we get
V T
=
V T
V 1
or =
V T T
V
As = v
V T
1 1
v = / C
T 273
At T = 273 + 0
V = V0
(ii) Pressure coefficient of gas, p :
It can be defined as ;
Increase in pressure (at constant volume)
p = Initial pressure × rise in temperature
P
=
P0 T
www.crackjee.xyz
THERMOMETRY, EXPANSION AND CALORIMETRY 377
If P0 and Pt are the pressures at 0°C and t°C respectively, then
P = Pt P0 , T t 0 t
Pt P0
p = P0 t

or Pt = P0 (1 pt)

Value of p for an ideal gas : Fig. 5.15


1
p / C
=
273
If P1 and P2 are the pressures, of a given mass of gas at temperatures t1 and t2
respectively, then
P1 = P0 (1 pt1 ) and P2 P0 (1 p t2 )

P2 1 p t2
= 1
P1 p t1
After solving, we get
P2 P1
p = P1t 2 P2 t1
.

Unoccupied length of tube


Suppose x0 be the length of mercury column at 0°C and xt at t°C.
Let a0 and at are areas of cross-section of tube at 0°C and t°C respectively, then
a0 (1 Fig. 5.16
at = gt) a0 (1 2 g t)

where g is the linear coefficient of expansion of glass.


Length of glass tube unoccupied by mercury
= AB t xt
(Vglass )t (VHg )t
=
at at

(Vglass )0 (1 3 gt) (VHg )0 (1 Hg t )


= a0 (1 2 g t)
a0 0 (1 3 g t) a0 x0 (1 Hg t )
= a0 (1 2 g t)

0 (1 3 g t) x0 (1 Hg t )
= (1 2 Fig. 5.19
g t)

1
= 0 (1 3 g t )(1 2 g t ) x0 (1 Hg t )(1 2 g t) 1

0 (1 3 g t )(1 2 g t) x0 (1 Hg t )(1 2 gt)

2
Neglecting product ( Hg g ) and g , we have
= 0 (1 g t) x0 [1 ( Hg 2 g )t ]
to be constant at all temperature, so
d( )
= 0
dt
d
[ 0 (1 gt) x0 {1 ( Hg 2 g )t ] = 0
dt
www.crackjee.xyz
378 MECHANICS, HEAT, THERMODYNAMICS & WAVES
d
or [ 0 (1 gt) x0 {1 ( Hg 2 g )t ] = 0
dt
0 g x0 ( Hg 2 g) = 0

0 g
x0 = 2 g
Hg

Change in time period of simple pendulum


If 0 be the length of the pendulum at 0°C, then its time period

0
T0 = 2 … (1)
g
At any temperature t, the time period of the pendulum is given by,

T = 2
g
Here, = 0 (1 t)

0 (1 t)
T = 2
g

= 2
0
(1 t )1/ 2
g

= T0 (1 t )1/ 2 …(2)

t
T0 1
2
T t
or 1 =
T0 2
T T0 t
=
T0 2
T t
or =
T0 2

t
T = T0
2

Note:
(i) If temperature increases, time period also increases and clock runs slow, so here
is loss in time.
(ii) If temperature decreases, time period also decreases and clock runs fast, so there
is gain in time.
Apparent weight of body at any temperature
Suppose a body of weight W in a submerged in liquid, its apparent weight at any
temperature t is given by
Wt = W Ft … (1)
Here Ft is the buoyant force exerted by liquid
www.crackjee.xyz
THERMOMETRY, EXPANSION AND CALORIMETRY 379

At 0°C, F0 = V0 0g … (2)
Here 0 is the density of liquid and V0 is the volume of the body at 0°C.
At t°C, Ft = Vt t g
Here Vt is the volume of body at t°C, which is;
Vt = V0 (1 st )

Here s is the coefficient of cubical expansion of body, t is the density of liquid at t°C,
which is given by;
0
t =
1 t
Here is the coefficient of cubical expansion of liquid.

0
Ft = V0 (1 st) g
1 t

= V0 1
0 g (1 s t )(1 t)
F0 (1 s t )(1 t)

= F0 (1 st t s t2 )

Neglecting s t 2 on being small.

Ft F0 [1 ( s )t ] … (3)
Substituting this value in equation (1), we get

Wt = W F0 (1 ( s )t … (4)

It is clear from equation (3) and (4) that with increase in temperature of the system
buoyant force decreases and apparent weight increases. If s , Wt W F0 , at all
temperatures.
Thermal expansion : An atomic view
Graph shows the potential energy U of two neighbouring atoms in a solid and their
interatomic separation r0.
At T0 = 0 K, the atoms remain at the equilibrium separation r0 and their energy E0 is
minimum (only P.E). At the higher temperature, under the influence of thermal energy,
atoms vibrate about their mean positions, the amplitude of vibrations increase with
increase in temperature. Referring to Fig. 5.20, at temperature T1, the amplitude of
vibration is a1b1 and at T2 (T2 > T1), the amplitude is a2 b2 . The corresponding mean
spacing between the atoms are giving by r1 and r2 respectively. As the repulsive force
is short range in nature as compared to the attractive force, the potential energy curve is
steeper on the left side of r 0 than on the right side. The equilibrium position shift to the
right of the curve i.e., r0 < r1 < r2 . In other words, the material exhibits thermal expansion. Fig. 5.20

Note:
It is due to lack of symmetry of the potential energy curve that accounts for the
thermal expansion. If a solid has a symmetrical potential energy curve would not
expand with temperature (see figure dotted curve). And the solid remains a solid and
does not melt or vaporise.
www.crackjee.xyz
380 MECHANICS, HEAT, THERMODYNAMICS & WAVES
Ex. 7 The Fig. 5.17 shows a rectangular plate of size (a × b)
from which two circular holes of radii R1 and R2 has been cut. The
separation between the holes is x. If a ', b ', R1' , R2' and x ' are the
respective values at higher temperature t, then find

a' b' R' R' x'


, , 1 , 2 and
a b R R2 x .

Sol.
If given data are at 0°C, then at any temperature t,

a ' = a(1 t) Fig. 5.18

1 = R1
b ' = b(1 t)
t
= R
R1' = R1 (1 t) 2
t
R 0 (1 T) … (i)
R2' = R2 (1 t) 2
B

and x ' = x(1 t) Similarly for copper strip, 2 0 (1 Ct) R2


' ' t
a ' b ' R1 R2 x ' or R = 0 (1 T) … (ii)
= = = = 1 t 2 C
a b R1 R2 x
Dividing equation (i) by (ii), we get

t
R 1 B T
2 =
t 1 C T
R
2
t t
R (1 C T) = R (1 B T)
2 2

t t t t
R R C T C T R R B T B T
2 2 2 2
If T is small, is still small, so we can neglect their product. And,
therefore, we have

R B T R C T = t
t
or R = Ans.
Fig. 5.17 ( B C) T

Ex. 8 A bimetallic strip is formed out of two identical strips,


one of copper and the other of brass. The coefficients of linear
Ex. 9 A long mercury glass tube with a uniform capillary bore
expansions of two metals are C and B . On heating, the has in it a thread of mercury which is 1 m long at 0°C. What will be
temperature of the strip goes up by T and the strip bends to form its length at 100°C if the real coefficient of expansion of mercury
an arc of radius R. Find R. is 0.000182 and coefficient of cubical expansion of glass equal to
0.000025/°C
Sol.
Sol.
Let 0 be the length and t the thickness of each strip. On heating, length
Suppose V0 and Vt are the volumes of mercury thread at 0°C and t°C
of brass rod respectively, then

1 = 0 (1 B T) Vt = V0 (1 rt)
By the geometry of the figure, we have Let a0 and at are the areas of cross-section of the tube at 0°C and t°C
respectively, then
www.crackjee.xyz
THERMOMETRY, EXPANSION AND CALORIMETRY 381
Ex. 11 A bar with a crack at its centre buckles as a result of
at = a0 (1 t)
temperature rise of 32 °C. If the fixed distance L0 is 3.77 m and the
coefficient of linear expansion of the bar is 25 × 10–6 /°C find the rise
= a0 (1 2 t ) { 2 } x of the centre.
The length of the thread at 0°C, Sol. Consider one half of the bar, its initial length
V0 L0
= 0 =
0 a0 2
The length of thread at t°C, Its length after increase in temperature t,
Vt = 0 (1 T)
t = By Pythogoras theorem
at

V0 (1 r t )
=
a0 (1 2 t )
Fig. 5.21
= 0 (1 r t )(1 2 t)
Substituting the given values, we get

0.000025
t = 1(1 0.000182 100)(1 2 100) Fig. 5.22
3 2 2 2
x = 0
= 1.0182 × 0.9983 2
= 0 (1 T )2 0
2
= 1.016 m Ans.
2
0 2 T
or x = 2 T
Ex. 10 A cube of coefficient of linear expansion is floating s
0
in a bath containing a liquid of coefficient of volume expansion . 3.77
= 2(25 10 6 32)
When the temperature is raised by T, the depth upto which the 2
cube is submerged in the liquid remain the same. Find the relation = 7.5 × 10–2 m Ans.
between s and showing all the steps.
Ex. 12 The temperature compensated pendulum is designed to
Sol. compensate for the change in length due to temperature rise. Fig.
Suppose initially be the length of the cube and y the depth of the cube 5.23 shows one such pendulum. It consists of an isosceles triangular
frame as shown in the figure. The pendulum is supported at mid-
submerged. Then by law of floatation.
point of side AB, and it remains horizontal. Determine the ratio
Weight of cube = Weight of liquid displaced
1 / 2 , so that the length of the pendulum remain same at all
Mg = ( 2 temperatures.
or y) g … (i)
Sol.
With the increase in temperature, the weight remain constant. Thus at
higher temperature

Mg = ( '2 y ) '
g … (ii)

From equations (i) and (ii)


( 2 y) g = ( '2 y ) '
g

2
or = '2 '

Fig. 5.23
'
But ' = (1 s T ), =
(1 T) The dotted line shows configuration after rise in temperature. As the
height of the pendulum remain same, so end C remain at its position.
2
= [ (1 s T )]2 The increase in length of the rod AB
(1 T)
1 = 1 1 T
or 1 T = (1 T )2
s As the length increases equally on both sides of AB, so
As T 1 , so by binomial theorem we can write
s
1
AA ' = 1 1 T … (i)
1 T = 1 2 s T 2
= 2 s Ans.
www.crackjee.xyz
382 MECHANICS, HEAT, THERMODYNAMICS & WAVES
Draw normal from A to A ' C , the increase in length of AC From equations (i), (ii) and (iii), we get

A'N = 2 2 T … (ii) 1 1
T = 1 1 T
2 2 2 2
From the AA ' N , 2

A'N = AA 'cos , as is small 1 2


= 2 Ans.
2 1
1
Here cos = 2 … (iii)
2

5.6 HEAT AND CALORIMETRY


Heat is the energy in transit
When two bodies at different temperature make in contact, something is transferred
between them . The word heat is meaningful only when energy is being transferred. The
expressions like, heat of a body or heat in a body are meaningless. So the heat can be
defined as the energy in transit that flows from one body to another due to difference of
temperature between them. Once heat is transferred to a body, it becomes the part of its
internal energy.
Unit of heat
CGS unit of heat : The CGS unit of heat is calorie.
Definition of calorie : It is the amount of heat energy required to raise the temperature of
one gram of water through 1 °C (from 14.5 °C to 15.5 °C).
SI unit of heat : The SI unit of heat is joule (J).
Joule’s mechanical equivalent of heat
James Precott Joule (1818 – 1889) performed a series of experiments and proved that heat
is a form of energy. He showed that if an amount of work W (or any other form of energy)
is converted into heat, the equal amount of heat is produced. Thus

W Q

or W = JQ

W
or J = Q

If Q = 1, then J = W
The proportionality constant J is called Joule’s mechanical equivalent of heat.
The value of J : J = 4.186 J/cal

Note: J is not a physical quantity. It just a conversion factor..


British thermal unit : It is the amount of heat required to raise the temperature of 1
pound of water through 1 °F.
1 BTU = 252 calorie
5.7 SPECIAL HEAT
It is defined as the amount of heat required to raise the temperature of unit mass of a
substance through 1 K (1 °C). Suppose Q amount of heat is supplied to m amount of
substance, the rise in temperature of substance is T , then specific heat is given by

Q
c =
m T
or we can write, Q = mc T
www.crackjee.xyz
THERMOMETRY, EXPANSION AND CALORIMETRY 383
Units of specific heat
(i) In CGS system, the unit of Q is calorie, m is gram and T in °C. Therefore unit of
c in this system becomes
c = cal/g –°C
(ii) In SI system, the unit of Q is Joule, m in kg and T in kelvin. Therefore in this
system unit of specific heat becomes J/kg-K.
1. Specific heat of water is 1cal/g-°C or 4200 J/kg-K.
2. Specific heat of ice is 0.5 cal/g-°C or 2100 J/kg-K.
3. The maximum value of specific heat is 3.5 cal/g°C for hydrogen.
4. The minimum value of specific heat is 0.022 cal/g°C for radon.
The specific heat of a substance is not constant at all temperature. Therefore specific
heat used in the above formula is the mean value of specific heats. When c varies
considerably with temperature, then for small change in temperature dT, we can write
dQ = mcdT
T2
mcdT
Q =
T1

Here T1 and T2 are the initial and final temperatures.


Dulong and Petit’s Law (1819)
“At near about room temperature the molar specific heat of most of the solids is equal to
3R or 6 cal/mol-K at constant volume”.
In case of solids, the significant motion of atoms are vibratory motion. During vibration,
the kinetic energy (Ek) of an atom changes periodically into potential energy (EP) and
vice-versa. So the average values of Ek and Ep are equal.
For each form of energy there are three degrees of freedom. Therefore a molecule has six
degrees of freedom (3 for kinetic + 3 for potential). According to law of equi-partition of
kT
energy, each degree of freedom possesses energy per atom. Therefore total energy
2
associated with one mole of a substance at a temperature T is given by;
kT 3
Ek = 3 kT
2 2
kT 3
EP = 3 kT
2 2
Average vibrational energy per atom
= Ek Ep 3kT
The internal energy (due to vibration) of one mole of an atom of the solid is given by
U = (3kT ) N
= 3(kN)T
= 3RT (kN = R)
dU
Also, we have CV =
dT
d (3RT )
=
dT
or CV = 3R
dU
Proof of formula CV , will be discussed in the next chapter..
dT
www.crackjee.xyz
384 MECHANICS, HEAT, THERMODYNAMICS & WAVES
Variation of specific heat of solid with temperature
The figure shows the variation of molar specific heat (CV) as a function of temperature.
It can be easily understand that at higher temperature the molar specific heat of all solids
is close a value 3R.

Specific heat of gas


Limit of specific heat of gas : Consider a gas of mass m and volume V at a pressure P.
(i) Suppose gas is compressed suddenly without supplying heat :
Let the temperature of the gas rises by T.
Q
c =
m T
But Q 0, c = 0.
(ii) Heat is supplied to the gas and it is allowed to expand in such a way that there is no
rise in temperature. i.e., T 0

Q
c =
m T
Fig. 5.24 Q
=
m 0
Thus the specific heat of a gas may varies from zero to infinity. It may have any
positive or negative value. The exact value depends on the conditions of pressure
and volume when heat is being supplied. Out of the many specific heats of a gas,
two are of prime significance.
1. Molar specific heat at constant volume CV
It is the amount of heat required to raise the temperature of 1 mole of a gas through
1 K (1 °C) at constant volume. If QV is the heat given to n moles of a gas at constant
volume and change in temperature be T , then

Qv
CV =
n T

or QV = nCv T
2. Molar specific heat at constant pressure CP
It is the amount of heat required to raise the temperature of 1 mole of a gas through
1K (1°C) at constant pressure. If QP is the heat given to n moles of a gas at
constant pressure and change in temperature be T , then

QP
CP =
n T

or QP = nCP T

Relation between CV and CP : Mayer’s formula


Heat supplied to a gas at constant volume entirely used to raise its temperature. When a
gas is heated at constant pressure, it expand to keep pressure constant and therefore
some mechanical work is to be done in addition to raise the temperature of the gas. Hence
more heat is required at constant pressure than that at constant volume. Thus for one
mole of a gas, we have
www.crackjee.xyz
THERMOMETRY, EXPANSION AND CALORIMETRY 385
CP – CV = Work done
= P V … (1)
At constant pressure, we have
PV1 = RT … (2)
and PV2 = R (T 1) … (3)
where V1 is the volume of gas at temperature T and V2 is the volume of gas at temperature
(T + 1). Subtracting equation (2) from (3), we get
P (V2 V1 ) = R
or P V = R
Substituting this value in equation (1), we get
CP CV = R Mayer’s formula
Fig. 5.25

Note:
1. Substance which expand on heating, P V is positive and therefore,
CP CV ve or CP CV . If any substance contracts on heating, P V will be
negative and therefore, CP CV ve or CP CV .

2. CP CV R holds good for all ideal gases.

3. For one gram of a gas, we have cV and cPand we can write CV McV and
CP McP . Also, cP cV = r
R
Here r which is different for different gases.
M
5.8 HEAT CAPACITY OR THERMAL CAPACITY
It is the amount of heat required to raise the temperature of whole amount of substance
through 1K (1°C). By definition, we have
Heat capacity = mass × specific heat
or S = mc
The CGS unit of heat capacity is cal/°C and SI unit is J/K.
5.9 WATER EQUIVALENT
The water equivalent of a body is defined as the mass of water which requires the same
amount of heat as is required by the given body for the equal rise of temperature. Let the
mass of substance be m, specific heat c and rise in temperature is T, then
Q = mc T … (i)
If water equivalent is w, then
Q = w 1 T (c of water is 1 cal/g-°C)
w 1 T = mc T
or w = mc

Note: Water equivalent numerically equal to the heat capacity but the unit of
water equivalent is gm or kg and that of heat capacity is cal/°C or J/°C.
www.crackjee.xyz
386 MECHANICS, HEAT, THERMODYNAMICS & WAVES
Ex. 13 70 cal of heat is required to raise the temperature of 2 20
mole of an ideal gas at constant pressure from 30 °C to 35 °C. What 3 t2
= t 2
is the amount of heat required to raise the temperature of same 10
gas through the same range (30 °C to 35 °C) at constant volume?
(R = 2 cal/mol-K) 202 102
203 103
Sol. Heat required at constant pressure = 2 2
QP = nCP T = 8200 – 1050
or 70 = 2 CP (35 – 30) = 7150 cal Ans.
C P = 7 cal/mol-K
Ex. 15 A metal sphere of radius R and specific heat C is rotated
As CP CV = R about an axis passing through its centre at a speed n rotation /
second. It is suddenly stopped and 50% of its energy is used in
C V = CP – R = 7 – 2
increasing its temperature, then find the rise in temperature of
= 5 cal/ mol - K the sphere.
Heat required at constant volume
Sol. The rotation K.E. of the sphere
QV = nCV T 1 2
K = I
= 2 5 (35 – 30) 2
= 50 cal Ans. 1 2 4 2
= MR 2 (2 n) 2 n 2 MR 2
Ex. 14 The specific heat of a substance varies as + t) × (3t2 10– 2 5 5
3 cal/g-°C. What is the amount of heat required to raise the
Kinetic energy used to raise the temperature
temperature of 1 kg of substance through 10°C to 20 °C?
Sol. For small change in temperature dt, heat required, 50 4 2n2
= 100 MR 2
dQ = mcdt 5
t2 2 2
2 n
mcdt = MR 2
Q = 5
t1
Let T be the rise in temperature, then
Given; m = 1000 g, c 2 2 2
(3t t) 2 n
MC T = MR 2
5
20
2 2 2
1000(3t 2 t ) 10 3 dt 2 n R
Q = T = Ans.
10 5C
5.10 CHANGE IN PHASE
A substance can exist in three possible phases viz., solid, liquid and gas. Transition from
one phase to another are accompanied by the absorption or liberation of heat and
usually by change in volume, even when the transition occurs at constant temperature.
As an example take small piece of ice in a container at –20 °C, and heat is supplied to the
container at a uniform rate. The temperature starts increasing steadily, as shown by the
segment a to b in Fig. 5.26, until the temperature rises to 0 °C. Thereafter ice starts
melting.

Fig. 5.26
www.crackjee.xyz
THERMOMETRY, EXPANSION AND CALORIMETRY 387
The melting process is a change in phase, from the solid phase to the liquid phase. But
the thermometer does not show any rise in temperature. When the whole ice has melted
(point c) the temperature of water now rises at a uniform rate (from c to d) although this
rate is slower than that from a to b. When temperature of water reaches 100 °C, it begins
to boil. The temperature remains constant until whole water has converted into water
vapour. Another change of phase has therefore taken place from liquid phase to the
gaseous phase. If heating is still continue (from e to f), the temperature of vapour starts
rising. The gaseous state would now be called superheated steam.

5.11 LATENT HEAT


The heat which is used to change the phase of substance at constant temperature is
called latent heat or hidden heat.

Fig. 5.27
Latent heat of fusion : The amount of heat required to convert unit mass of solid into
liquid at its melting point is called latent heat of fusion.
Latent heat of vapourisation : The amount of heat required to convert unit mass of liquid
into vapour at its boiling point is called latent heat of vapourisation.
The term heat of transformation is better used to both heats of fusion and heats of
vapourisation, and let both are denoted by L. If Q is the amount of heat absorbed or
liberated by m amount of substance, then L is defined as ;
Q
L =
m

or Q = mL
Unit of L
The CGS unit of L is cal/g.
The SI unit of L is J/kg.
Some values
Latent heat of fusion of ice is 80 cal/g or 336 kJ/kg.
Latent heat of vapourisation of water is 540 cal/g or 2259 kJ/kg at one atmospheric
pressure.

IMPORTANT POINTS
1. Melting and freezing occur at the same temperature and therefore melting point
and freezing point are equal.
2. As long as the change of phase (state) takes place, the temperature remains constant.
3. Some substances show increase in volume on melting. e.g., wax, ghee etc while
some other substances show decrease in volume on melting e.g., ice.
4. The melting point of those substances (water, antimony) which contracts on melting,
decreases with increase in pressure. The melting point of ice is 0°C at 1 atm and –
1°C at 133 atm, –2°C at 265 atm.
5. The melting point of those substances which expand on melting increases with
increase in pressure.
6. Impurities lower freezing point. When salt is mixed with ice, the temperature of the
mixture decreases.
www.crackjee.xyz
388 MECHANICS, HEAT, THERMODYNAMICS & WAVES
About boiling
1. Boiling and condensing occur at the same temperature.
2. As long as the change of phase takes place, the temperature remains constant.
3. All liquids show increase in volume on vapourisation.
4. The boiling point of liquid increases with increase in pressure. The boiling point of
water is 100°C at 1 atm and 130 °C at 2 atm.
5. In pressure cooker, the cooking occurs at 2 atm pressure.

The change in M.P. or B.P. with pressure can be calculate by using Clausius Clapeyron’s
P L
equation (which is out of syllabus). The equation is ; .
T T (V2 V1 )
Ex. 16 The melting point of ice is 0°C at 1 atm. At what pressure P 80 4.2 103
it will be –1°C? or ( 1) = 1 3
273 1 10
Sol. Here T (–1 – 0) = –1, T = 273 + 0 = 273 K 0.9

1 3 5
P = 132 10 N/m
2
and V2 V1 = 1 10 m3
0.9
132 atm
(given)
L = 80 cal/g or P2 P1 = 132 atm
P L P2 = 132 P1 133 atm Ans.
We have, = T (V V )
T 2 1

Regelation : Take an ice slab and place it on two supports. The weights are suspended
on the ice slab with the help of wire. The wire will pass through the slab without spliting
it. Just below the wire, ice melts at a lower temperature due to increase in pressure. When
wire has moved down, the water above the wire freezes again. The phenomenon of
melting of ice and its resolidification is called regelation.

FACTS TO KNOW
1. Supercooled water : Water below 0 °C is known as supercooled water. But this
can be possible at a pressure greater than atmospheric pressure.
2. Superheated steam : Steam at a temperature greater than 100°C is known as
Fig. 5.28 superheated steam.
3. Dry ice : Solid carbon dioxide is called dry ice. Carbon dioxide at –78 °C remains
in solid state. Solid carbon dioxide does not melt when exposed to air. It directly
evaporates and forms its vapours.
4. Vapour and gas : These are the gaseous states of the substance. These two
states have distinct boundary which is governed by a particular temperature
called critical temperature. The gaseous state of substance below the critical
temperature is called vapour and above critical temperature is called gas. A gas
cannot be liquefied by mere application of pressure, however high it may be . A
vapour can be liquefied by applying pressure. Thus to liquefy a gas, first it bring
below critical temperature.

Critical temperature
Substance Critical temperatrue °C
Water 374
Ammonia 132
Carbon dioxide 31
Oxygen –119
Hydrogen –240
Helium –268
www.crackjee.xyz
THERMOMETRY, EXPANSION AND CALORIMETRY 389

Ex. 17 A solid material is supplied heat at a constant rate. The (ii) CD = 2AB or Q4 2Q2 , it shows that latent heat of vaporisation
temperature of the material is changing with the heat input as is twice than that of latent heat of fusion.
shown in Fig. 5.28. Study the graph carefully and answer the
(iii) Let C1 and C2 are the specific heats of solid and liquid states
following questions :
respectively, then
(i) What do the horizontal regions AB and CD represent?
Q1 = mC1 T1
(ii) If CD = 2AB, what do you infer ?
(iii) The slope of OA > the slope of BC. What does this indicate?
1 1
(iv) What does the slope DE represent? or C1 =
T1 m slope of OA
m
Q1

1
Similarly C2 = m slope of BC

Since slope of OA > slope of BC, C1 C2 .


(iv) If C3 is the specific heat of vapour state (region DE), then

1
C3 = m slope of DE
Fig. 5.29
Sol. 1 1
or Slope of DE = mC heat capacity
(i) In the regions AB and CD the temperature of the material remain 3

constant. So AB represents fusion and CD represents vaporisation.

5.12 LAW OF MIXTURE OR LAW OF CALORIMETRY


When two or more non reacting substances are placed in contact, the heat lost by the
hot substance is equal to the heat gained by cold substances. That is ;
Heat lost = Heat gained
Two substances at temperatures T1 and T2 are taken together in a calorimeter. Let T is
the equilibrium temperature of the system.
(i) When substances of same phase are taken together
T1 T T2 , if T1 T2
(ii) When substances of different phases are taken together,
(a) T1 < T < T2 or
(b) T = T1 or T = T2.

Ex. 18 1g ice at 0°C is placed in a calorimeter having 1g water remain 0°C. Let m is the amount of ice melt due to 40 cal heat, then
at 40 °C. Find equilibrium temperature and final contents. m × 80 = 40
Assuming heat capacity of calorimeter is negligibly small. 1
or m = g
Sol. 2
1 1
The heat required to melt the ice completely Final contents : ice = 1 g
2 2
= mL = 1 × 80 = 80 cal 1 3
The heat available on water water = 1 g Ans.
2 2
= mc T 1 1 (40 – 0) Ex. 19 1g ice at – 40°C is placed in a container having 1g water
= 40 cal. at 10°C. Find equilibrium temperature. Assume heat capacity of
Entire heat of water is utilised to melt the ice and its temperature container is negligibly small.
falls to 0°C. Ice still at 0°C. So equilibrium temperature of contents
www.crackjee.xyz
390 MECHANICS, HEAT, THERMODYNAMICS & WAVES
Sol. 1 4
water = 1 g
The heat available on water to cool from 10°C to 0°C 3 3

= mc T 1 1 (10 – 0) 10 cal 1 2
steam = 1 g Ans.
Let temperature of ice becomes T after taking this heat 3 3

micec T = 10 Ex. 21 The temperature of equal masses of three different


liquids A, B and C are 12 °C, 19 °C and 28°C respectively. The
or 1 0.5 [T – ( 40)] = 10
temperature when A and B are mixed in 16 °C and when B and C
T + 40 = 20 are mixed it is 23 °C. What should be the temperature when A and
T = –20°C C are mixed?
Now the system have 1g ice at –20°C and 1g water at 0°C. Let m gram Sol.
water get freezed to bring the ice from –20°C to 0°C,
Given TA 12°C , TB 19°C and TC 28°C . Let CA, CB and CC are
heat gained by ice = heat lost by water
the specific heats of respective liquids.
micec [0 – (–20)] = m × 80
When liquid A and B are mixed, the temperature of mixture becomes
or 1 × 0.5 × 20 = m × 80
16 °C, then
1
m = g mC A (16 12) = mCB (19 16)
8

Thus equilibrium temperature becomes 0°C, as both ice and water change 4
or CB = CA … (i)
into 0°C. 3
Final contents : When liquid B and C are mixed, the temperature of mixture becomes
23 °C, then
1 9
ice = 1g g g
8 8 mCB (23 19) = mCC (28 23)

1 7 5
water = 1 g Ans. or CB =
4
CC … (ii)
8 8

Ex. 20 1g steam at 100°C is passed in a insulating vessel having From (i) and (ii), we get

1g ice at 0°C. Find the equilibrium temperature of the mixture. 15


Neglecting heat capacity of the vessel. CA = CC
16
Sol. Now when A and C are mixed, let equilibrium temperature of mixture is T,
Heat available on steam (changes into steam to water) then
= mL = 1 × 540 = 540 cal mC A (T 12) = mCC (28 T )
Heat gained by ice to change into water and then rise its temperature to
100°C 15
or CC (T 12) = CC (28 T )
16
= miceL mwat c T
31T = 628
= 1 80 1 1 (100 – 0)
or T = 20.26 °C Ans.
= 180 cal.
The above calculations show that some part of steam will condense to
change the ice into water of 100°C. Let m is the mass of steam condensed, Ex. 22 A 10 kW drilling machine is used to drill a bore in a
then small aluminium block of mass 8.0 kg. How much is the rise in
m × 540 = 180 temperature of the block in 2.5 minute? Assuming 50% of power is
used up in heating the machine itself or lost the surroundings
180 1 specific heat of aluminium = 0.91 J/g- °C.
or m = g
540 3

Final contents: ice = 0g


www.crackjee.xyz
THERMOMETRY, EXPANSION AND CALORIMETRY 391

Sol. Ex. 24 A child running a temperature of 101°F is given an


Total energy used by drilling machine antipyrine (i.e. a medicine that lowers fever) which causes an
increase in the rate of evaporation of sweat from his body. If the
= Pt
fever is brought down to 98°F in 20 min, what is the average rate of
= (10 × 103) × (2.5 × 60) extra evaporation caused by the drug? Assume the evaporation
= 1.5 × 106 J mechanism to be the only way by which heat is lost. The mass of
The energy absorbed by the aluminium block the child is 30 kg. The specific heat of human body is approximately
the same as that of water, and latent heat of evaporation of water at
50
= 1.5 106 that temperature is about 580 cal /g.
100
Sol.
= 0.75 × 106 J
Mass of the child, M = 30 kg
Let T be the rise in temperature of the aluminium block, then The fall in temperature of body of child,
6 T = 101 98 3 F
mC T = 0.75 10

(8 × 103) × 0.91 × T = 0.75 × 106 5 5


= 3 C
9 3
T = 103.02°C Ans.
Specific heat of human body
Ex. 23 In an experiment on the specific heat of a metal a 0.20
C = 1 cal/g °C
kg block of the metal at 150 °C is dropped in a copper calorimeter
The heat lost by the body of child
(of water equivalent 0.025 kg) containing 150 cm3 of water at 27 °C.
The final temperature is 40 °C. Compute the specific heat of the 5
metal. = MC T 30 103 1
3
Sol. = 5 × 104 cal
Let specific heat of the metal is C, the heat lost by metal block mC T If M gram of sweat evaporate from the body of the child, then heat
absorbed by sweat
= 0.20 C (150 – 40) = M 'L M ' 580 cal

Heat gained = Heat lost


= 22 C
or M × 580 = 5 × 104
Mass of water in the calorimeter
M ' = 86.2 g
= V 1000 (150 10 6 ) 0.15kg
Time taken by sweat to evaporate = 20 min
Heat gained by water and calorimeter
Rate of evaporation of sweat
= mwC T C T
Mass of sweat
= (0.15 × 4200 + 0.025 × 4200) × (40 – 27) = time of evaporation
= 735 × 13
= 9555 J 86.2
=
By principle of calorimetry, we have 20

22C = 9555 = 4.31 g/min Ans.

9555
C =
22

= 434.3 J/kg- °C Ans.


www.crackjee.xyz
392 MECHANICS, HEAT, THERMODYNAMICS & WAVES

Review of formulae & Important points


1. Thermometer : If X is the property which varies linearly, then 9. Expansion of liquids :
temperature If a and g are the apparent coefficient of expansion of liquid
and volume coefficient of expansion of container, then
Xt X 0
t = 100 degree r = a g
X100 X 0
Vt V0
Here property X may be length of mercury column, resistance (i) =
a V0t
of metal etc.
2. Temperature scales : M0 M
(ii) a =
C F 32 K 273.15 Mt
= 10. Expansion of gases :
5 9 5
1 div of C = 1.8 div of F = 1 div of K. 1
v = C
3. Triple point of water is 273.16 K at 4 mm of mercury. 273

P 1
4. T = Ttr p = C
Ptr 273
11. Fractional charge of M.I. of the rod due to small change in
5. Radiation pyrometer : It is based on Stefan's law. temperature T
1 I
E 4 = 2 T
T = kelvin I
12. Mechanical equivalent of heat
6. Expansion of solids : W
(i) Coefficient of linear expansion J =
Q
L 1 cal = 4.2 J
= L0 t 13. If c is the specific heat of substance, then
Q = mc T
(ii) Coefficient of superficial expansion
If c is the function of temperature, then
A T2
= A0 t
Q = mc dT
(iii) Coefficient of volume expansion T1

V 14. Specific heat of gas :


= V0 t (i) If Cv is the specific heat of gas at constant volume, then
Qv = nCv T
(iv) 2 ; 3 (ii) If C p is the specific heat at constant pressure
7. Density of material at any temperature Q = nCp T
15. Cp – Cv = R
0
= R R
t (1 t) Cv and C p
1 1
8. If 1 and 2 are the densities at t1 and t2 respectively, then
16. Latent heat : Q = mL
coefficient of volume expansion
Latent heat of fusion of ice = 80 cal/g or 336 kJ/kg.
Latent heat of vapourisation of water 540 cal/g or 2259 kJ/kg
1 2
= at one atmospheric pressure.
2 (t2 t1 )
17. Law of mixture :
Heat lost = heat gained
www.crackjee.xyz
THERMOMETRY, EXPANSION AND CALORIMETRY 393

MCQ Type 1 Exercise 5.1

Level -1
Only one option correct
1. A beaker is filled with water at 4°C. At one time the temperature Temp
is increased by few degrees above 4°C and at another time it is
decreased by a few degrees below 4°C. One shall observe that:
(a) The level remains constant in each case (c)
(b) In first case water flows while in second case its level comes
down
(c) In second case water over flows while in first case its comes –10°C Heat supplied
down
(d) Water overflows in both the cases
2. The coefficient of apparent expansion of a liquid is C when heated Temp
in a copper vessel and it is S when heated in a silver vessel. If A is
the coefficient of linear expansion of copper, then that of silver is:
(d)
C S – 3A C 3A – S
(a) (b)
3 3
S 3A – C C S 3A –10°C Heat supplied
(c) (d)
3 3
5. 1 calorie is the heat required to increase the temperature of 1g of
3. Liquid oxygen at 50 K is heated to 300 K at constant pressure of
water by 1°C from :
1 atm. The rate of heating is constant. Which of the following
(a) 13.5°C to 14.5°C at 76 mm of Hg
graphs represents the variation of temperature with time?
(b) 14.5°C to 15.5°C at 760 mm of Hg
Temp Temp (c) 13.5°C to 15.5°C at 76 mm of Hg
(d) 15.5°C to 16.5°C at 700 mm of Hg
(a) (b) 6. Assuming no heat losses, the heat released by the condensation of
Time Time x g of steam at 100°C can be used to convert y g of ice at 0°C into
water at 100°C, the ratio x : y is :
(a) 1 : 1 (b) 1 : 2
Temp Temp (c) 1 : 3 (d) 3 : 1
7. A solid substance is supplied heat at a constant rate and the variation
(c) (d) of temperature with heat input is shown in the figure. Choose the
Time Time correct statement :

4. A block of ice at –10°C is slowly heated and converted to steam at


Temperature

100°C. Which of the following curves represents the phenomenon E


qualitatively : C
D
Temp A
B
O Heat input
(a)
(a) AB and CD represent changes of phase from liquid to vapour
–10°C Heat supplied and from solid to liquid respectively.
Temp (b) The latent heat of vaporization is half the latent heat of
fusion.
(c) The specific heat in the solid state is less than that in the
(b) liquid state.
(d) The specific heat in the solid state is more than that in the
liquid state.
–10°C Heat supplied

Answer Key 1 (d) 3 (c) 5 (b) 7 (c)


Sol. from page 405 2 (b) 4 (a) 6 (c)
www.crackjee.xyz
394 MECHANICS, HEAT, THERMODYNAMICS & WAVES
8. A spherical shell of copper is completely filled with a liquid at a (a) Maintaining vacuum above mercury column in the stem of
temperaturet°C. The bulk modulus of the liquid is K and coefficient the thermometer
of volume expansion is . If the temperature of the liquid and the (b) Filling nitrogen gas at high pressure above the mercury
shell is increased by T, then the outward pressure p on the shell column
that results from the temperatu re increase is given by (c) Filling nitrogen gas at low pressure above the mercury level
( is the coefficient of linear expansion of the material of the (d) Filling oxygen gas at high pressure above the mercury column
shell):
14. A uniform metal rod is used as a bar pendulum. If the room
(a) K( – 3 ) T (b) K(3 – ) T temperature rises by 10°C, and the coefficient of linear expansion
(c) 3 (K – ) T (d) (3 – K) T of the metal of the rod is 2 × 10 –6 per °C, the period of the
9. Three different materials of indentical masses are placed, in turn, pendulum will have percentage increase of
in a special freezer that can extract energy from a material at a
(a) – 2 × 10–3 (b) – 1 × 10–3
certain constant rate. During the cooling process, each material
begins in the liquid state and ends in the solid state : Figure shows (c) 2 × 10–3 (d) 1 × 10–3
graphs of the temperature T versus time t for the three materials. 15. When a rod is heated but prevented from expanding, the stress
The material which has the greatest heat of fusion is : developed is independent of
(a) material of the rod (b) rise in temperature
(c) length of rod (d) none of above
16. At some temperature T, a bronze pin is a little large to fit into a
hole drilled in a steel block. The change in temperature required
for an exact fit is minimum when
(a) Only the block is heated
(b) Both block and pin are heated together
(c) Both block and pin are cooled together
(d) Only the pin is cooled
(a) 1
17. Density of substance at 0°C is 10 gm/cc and at 100°C, its density
(b) 2
is 9.7 gm/cc. The coefficient of linear expansion of the substance
(c) 3
will be
(d) Informations are not sufficient
(a) 102 (b) 10–2
10. A cylindrical metal rod is shaped into a ring with a small gap as
shown. On heating the system : (c) 10–3 (d) 10–4
18. Two uniform brass rods A and B of length l and 2l and radii 2r and
r respectively are heated to the same temperature. The ratio of
the increase in the volume of A to that of B is
(a) 1 : 1 (b) 1 : 2
(c) 2 : 1 (d) 1 : 4
19. The coefficient of apparent expansion of a liquid when determined
(a) x decreases, r and d increase using two different vessels A and B are 1 and 2 respectively. If
(b) x and r increase, d decreases the coefficient of linear expansion of the vessel A is , the
(c) x, r and d all increase coefficient of linear expansion of the vessel B is.
(d) x and r decreased, d remains constant 1 2 1 – 2
(a) (b)
11. At what temperature the centigrade (Celsius) and Fahrenheit, 1 2 2
readings are the same 1 – 2 1 – 2
(a) – 40° (b) + 40° (c) (d)
3 3
(c) 36.6° (d) – 37° 20. Melting point of ice
12. A constant volume gas thermometer shows pressure reading of (a) increases with increasing pressure
50cm and 90cm of mercury at 0°C and 100°C respectively. When (b) decreases with increasing pressure
the pressure reading is 60cm of mercury, the temperature is (c) is independent of pressure
(a) 25°C (b) 40°C (d) is proportional to pressure
(c) 15°C (d) 12.5°C
21. Work done in converting one gram of ice at –10°C into steam at
13. Mercury boils at 367°C. However, mercury thermometers are 100°C is
made such that they can measure temperature up to 500°C. This (a) 3045 J (b) 6056 J
is done by (c) 721 J (d) 616 J

Answer Key 8 (a) 10 (c) 12 (a) 14 (d) 16 (a) 18 (c) 20 (b)


Sol. from page 405 9 (b) 11 (a) 13 (b) 15 (c) 17 (d) 19 (d) 21 (a)
www.crackjee.xyz
THERMOMETRY, EXPANSION AND CALORIMETRY 395
22. In a water-fall the water falls from a height of 100 m. If the entire (a) Slope of line AB is 9/5 (b) Slope of line AB is 5/9
K.E. of water is converted into heat, the rise in temperature of
(c) Slope of line AB is 1/9 (d) Slope of line AB is 3/9
water will be
(a) 0.23°C (b) 0.46°C 26. 1 gm of ice is passed into a container having 2gm water at 35°C.
The equilibrium temperature of the mixture is :
(c) 2.3°C (d) 0.023°C
(a) 0 °C (b) 12 °C
23. Two metal strips that constitute a thermostat must necessarily
differ in their (c) 10 °C (d) 35 °C
(a) Mass (b) Length 27. A solid substance is at 30°C. To this substance heat energy is
(c) Resistivity supplied at a constant rate. Then temperature versus time graph
(d) Coefficient of linear expansion is as shown in the figure. The substance is in liquid state for the
24. Steam is passed into 22g of water at 20°C. The mass of water that portion (of the graph)
will be present when the water acquires a temperature of 90°C

Temperature (T°C)
(Latent heat of steam is 540 cal/g) is F
(a) 24.8 g (b) 24 g 240
(c) 36.6 g (d) 30 g 210 D
E
25. The graph AB shown in figure is a plot of temperature of a body in 60 B
C
degree celsius and degree Fahrenheit. Then 30 Time
A
100°C B
(a) BC (b) CD
Centigrade

(c) ED (d) EF

32°F 212°F Fahrenheit


A

Answer Key 22 (d) 24 (a) 26 (a)


Sol. from page 405 23 (d) 25 (b) 27 (b)

Level -2
1. A volume V pressure P graph was obtained from state 1 to state 2 2. A metallic solid sphere is rotating about its diameter as axis of
when a given mass of a gas is subjected to temperature changes. rotation. If the temperature is increased by 20 °C, the percentage
During the process the gas is : increase in its moment of inertia is (coefficient of linear expansion
of metal = 10-5 per °C) :
P (a) 0.1 (b) 0.2
(c) 0.03 (d) 0.04
1 3. When a block of iron floats in mercury at 0°C, a fraction k1 of its
volume is submerged , with at a temperature 60°C, a fraction k2 is
seen to be submerged. If the coefficient of volume expansion of
iron is Fe and that of mercury is Hg , then the ratio
k1 / k2 can be expressed as :
2
V 1 60 Fe 1 – 60 Fe
(a) 1 60 Hg (b) 1 60 Hg
(a) Heated continuously
(b) Cooled continuously 1 – 60 Fe 1 60 Hg
(c) Heated in the beginning and cooled towards end (c) (d)
1 – 60 Hg 1 60
(d) Cooled in the beginning and heated towards end Fe

Answer Key
1 (c) 2 (d) 3 (a)
Sol. from page 406
www.crackjee.xyz
396 MECHANICS, HEAT, THERMODYNAMICS & WAVES
4. Two rods one of aluminium and the other of steel, having initial
length l1 and l2 are connected together to form a single rod of length
(l1 + l2). The coefficients of linear expansion for aluminium and
steel are Al and S respectively. If the length of each rod increases
(c) (d)
by the same amount when their temperature are raised by t°C,
1
then the ratio is : P P
1 2
7. Two litre of water at 27°C is heated by 1 kW heater in an open
s Al
(a) (b) container. On an average heat is lost to surroundings at the rate
Al s
160 J/s. The time required for the temperature to reach 77°C is :
(a) 8 min 20 s (b) 10 min
s Al
(c) ( ) (d) ( ) (c) 14 min (d) 7 min
Al s Al s
8. A steel rod of diameter 1.0 cm is clamped firmly at each end when
5. An ideal gas is initially at temperature T and volume V. Its volume its temperature is 25°C so that it can not contract on cooling. The
is increased by V due to an increase in temperature T , pressure tension in the rod at 0°C is ( = 1 × 10–5/°C, Y = 2 × 1011 N/m2):
remaining constant. The quantity = V / V T varies with (a) 3925 N (b) 7000 N
temperature (in kelvin) as : (c) 7400 N (d) 4700 N
9. A glass bottle of capacity 50 cc at 0°C is filled with paraffin
at 15°C. Given that the density of paraffin at 0°C is 0.82 g/cc,
coefficient of expansion of paraffin for the range 0 to 15°C
is 0.0009/°C and coefficient of linear expansion of glass is
(a)
0.000009/°C, the mass of paraffin in the bottle is :
(a) 40.5 g (b) 54.0 g
Temp (K) (c) 50.4 g (d) 5.04 g
T T+ T 10. A block of ice of mass m = 10 kg is moved back and forth over the
flat horizontal surface of a large block of ice. Both blocks are at
0°C and the force that produces the back and forth motion acts
only horizontally. The coefficient of friction between the two
(b) surfaces is 0.060. If 15.2 g of water is produced, the total distance
travelled by the upper block relative to the lower is
Temp (K) (Lice = 3.34 × 105 J/kg)
T T+ T
(a) 432 m (b) 863 m
(c) 368 m (d) 216 m
11. A glass sinker has a mass M in air. When weighed in a liquid at
(c)
temperature t1, the apparent mass is M1 and when weighed in the
same liquid at temperature t2, the apparent mass is M 2. If the
Temp (K) coefficient of cubical expansion of the glass is g, then the real
T T+ T coefficient of expansion of the liquid is :

M 2 – M1 1 M 2 – M1 1
(a) –
(t2 – t1 ) (b)
g g
M – M2 M – M2 (t2 – t1 )
(d)
M – M2 1
(c) –
(t2 – t1 ) (d)
g
M 2 – M1
Temp (K)
T T+ T
6. Which of the following graphs correctly represents the variation M 2 – M1 1
of = (dV/dP) / V with P for an ideal gas of constant temperature: g
M 2 M1 (t2 – t1 )
12. A mercury thermometer is to be made with glass tubing of internal
bore 0.5 mm and the distance between the fixed points is to be
(a) (b) 20 cm. The coefficient of expansion of mercury is 0.000180/°C
and the coefficient of linear expansion of glass is 0.000009/°C.
The internal volume of the bulb and stem below the lower fixed
P P point is :
(a) 0.527 cc (b) 5.27 cc
(c) 2.57 cc (d) 3.27 cc

Answer Key 4 (c) 6 (a) 8 (a) 10 (b) 12 (c)


Sol. from page 406 5 (c) 7 (a) 9 (a) 11 (a)
www.crackjee.xyz
THERMOMETRY, EXPANSION AND CALORIMETRY 397
13. A 5.0 g bullet (specific heat of material of bullet = 128 J/kg°C) 19. A metal block of mass 2 kg ( = 5 × 103 kg/m3) is suspended from
moving with a velocity of 200 m/s enters a sand bag and stops. If an ideal spring of force constant 25 N/m. The spring block system
the entire kinetic energy of the bullet is changed into heat energy is immersed in 500 g of water contained in a vessel such that the
that is added to the bullet, then the rise in the temperature of the block is at a height h from the bottom of the vessel. The system is
bullet is: released so as to sink to the bottom, resulting in a rise of temperature
(a) 312.5°C (b) 156°C of 0.004°C of water. The height h through which the block went
(c) 500°C (d) 624°C down is (Given : specific heat of the block = 250 J/kg °K and that
14. A calorimeter (of water equivalent 50 g ) contains 250 g of water of water = 4200 J/kg °K, heat capacity of vessel and spring are
and 50 g of ice at 0°C. 30 g of water at 80°C is added to it. The final negligible. Take g = 10 m/s2)
condition of the system will be :
(a) 0.35 m (b) 0.16 m
(a) the temperature of the system will be 4.2°C.
(c) 1.60 m (d) 0.26 m
(b) the temperature of the system will still be 0°C and the entire
20. A tungsten wire with a diameter of 0.20 mm is stretched until the
ice will melt.
tension is 50 N. The wire is then clamped to a stout aluminium
(c) the temperature will be 0°C and half of the ice will melt.
(d) the temperature will be 0°C and 20 g of ice will left. rod when both are at a temperature of 20°C. The tension in the
wire when both are brought to a temperature of 150°C is (Y Tungsten
15. 100 g of steam at 100°C is passed into 200 g of water and 20 g of
= 34 × 1010 N/m2, T = 4.5 x 10–6/°C)
ice at 0°C in a calorimeter whose water equivalent is 50 g. (Lsteam
= 540 cal/g and Lice = 80 cal/g). The observed result is (a) 85.0 N (b) 57.5 N
(c) 100 N (d) 56.2 N
(a) the temperature of the system becomes 169°C.
21. The table gives the initial length L, change in temperature T and
(b) half of the ice is melted and the temperature of the system
change in length L of four rods. The rod, which has greatest
remains 0°C.
coefficient of expansion
(c) the temperature remains 100°C and 53 g of steam condenses.
(d) the temperature remains 100°C and the entire steam Rod L(m) T(ºC) L(m)
condenses. a 2 10 4 × 10–4
16. The coefficients of linear expansion of steel and brass are b 1 20 4 × 10–4
11 × 10–6/°C and 19 × 10–6/ °C respectively. If their difference in c 2 10 8 × 10–4
lengths at all temperatures has to be kept constant is 30 cm, their d 4 5 4 × 10–4
lengths at 0°C should be
(a) 71.25 cm and 41.25 cm (a) a (b) b
(b) 82 cm and 52 cm (c) c (d) d
(c) 92 cm and 62 cm 22. An ideal gas is expanding such that PT2 = constant. The coefficient
of volume expansion of the gas is
(d) 62.25 cm and 32.25 cm
1 2
17. Two vertical glass tubes filled with a liquid are connected by a (a) (b)
capillary tube as shown in the figure. The tube on the left is put in T T
an ice bath at 0°C while the one on the right is kept at 30°C in a 3 4
water bath. The difference in the levels of the liquid in the two (c) (d)
T T
tubes is 4.0 cm while the height of the liquid column at 0°C is
23. A piece of metal weighs 45g in air and 25g in a liquid of density
120 cm. The coefficient of volume expansion of the liquid is
1.5 × 103kg – m–3 kept at 30°C. When the temperature of the
liquid is raised to 40°C, the metal piece weighs 27g. The density
of liquid at 40°C, is 1.25 × 103 kg –m–3. The coefficient of linear
4cm expansion of metal is
(a) 1.3 × 10–3/°C (b) 5.2 × 10–3/°C
120 cm 30°C (c) 2.6 × 10 /°C–3 (d) 0.26 × 10–3/°C
24. Two rigid boxes containing different ideal gases are placed on a
0°C
table. Box A contains one mole of nitrogen at temperature T0,
while Box B contains one mole of helium at temperature (7/3) T0.
(a) 22 × 10–4/°C (b) 1.1 x 10–4/°C
–4 The boxes are then put into thermal contact with each other and
(c) 11 x 10 /°C (d) 2.2 x 10–4/°C
heat flows between them until the gases reach a common final
18. A brass scale of a barometer gives correct reading at 0°C.
temperature (Ignore the heat capacity of boxes). Then, the final
4
Hg 0.6 10 / C Brass = 0.00002/°C. The barometer reads temperature of the gases Tf in terms of T0 is
7 3
75 cm at 27°C. The atmospheric pressure at 0°C is (a) T f T0 (b) T f T0
3 2
(a) 74.20 cm (b) 74.62 cm
5 3
(c) 74.92 cm (d) 75.04 cm (c) T f T0 (d) T f T0
2 7

Answer Key 13 (b) 15 (c) 17 (c) 19 (d) 21 (c) 23 (c)


Sol. from page 406 14 (d) 16 (a) 18 (c) 20 (d) 22 (c) 24 (b)
www.crackjee.xyz
398 MECHANICS, HEAT, THERMODYNAMICS & WAVES
25. A substance of mass m kg requires a power input of P watts to 28. Three rods of equal length are joined to form an equilateral triangle
remain in the molten state at its melting point. When the power is PQR. O is the mid point of PQ. Distance OR remains same for
turned off, the sample completely solidifies in time t sec. What is small change in temperature. Coefficient of linear expansion for
the latent heat of fusion of the substance ? PR and RQ is same, i.e., 2 but that for PQ is 1. Then
Pm Pt (a) 2 3 1 R
(a) (b)
t m
(b) 2 4 1
m t
(c) (d) 3
Pt Pm (c) 1 2

26. 2 kg of ice at – 20°C is mixed with 5 kg of water at 20°C in an (d) 4


1 2
insulating vessel having a negligible heat capacity. Calculate the P Q
final mass of water remaining in the container. It is given that the O
specific heats of water and ice are 1 kcal/kg per °C and 0.5 kcal/kg/ 29. A horizontal tube open at both ends contains a column of liquid.
°C while the latent heat of fusion of ice is 80 kcal/kg The length of this liquid column does not change with temperature.
Let = coefficient of volume expansion of the liquid and
(a) 7 kg (b) 6 kg
= coefficient of linear expansion of the material of the tube
(c) 4 kg (d) 2 kg
(a) = (b) =2
27. In a vertical U-tube containing a liquid, the two arms are maintained (c) =3 (d) =0
at different temperatures t1 and t2. The liquid columns in the two 30. A cylindrical steel llay is inserted into a circular hole of diameter
arms have heights l1 and l2 respectively. The coefficient of volume 2.60 m is a brass plate. When the plug and the plante are at a
expansion of the liquid is equal to temperature 20 °C, the diameter of the plug is 0.010 mm smaller
than that of the hole. The temperature at which the plug will just

fit in is (given 6 6
steel 11 10 / C and brass 19 10 / C)
t1
(a) – 20° C (b) – 48 °C
t2
(c) 10 °C (d) none of these.
l1
l2 31. In two experiments with a continuous flow calorimeter to determine
the specific heat capacity of a liquid , an input power of
60 W produced a rise of 10 K in the liquid. When the power was
doubled, the same temperature rise was achieved by making the
l1 – l2 l1 – l2 rate of flow of liquid three times faster. The power lost to the
(a) l2t1 – l1t2 (b) l1t1 – l2t2 surrounding in each case was
(a) 20 W (b) 30 W
l1 l2 l1 l2
(c) (d) (c) 40 W (d) 120 W
l2t1 l1t2 l1t1 l2t2

Answer Key 25 (b) 27 (a) 29 (b) 31 (b)


Sol. from page 406 26 (b) 28 (d) 30 (b)
www.crackjee.xyz
THERMOMETRY, EXPANSION AND CALORIMETRY 399

MCQType 2 Exercise 5.2


Multiple correct options 4. A vessel is partly filled with liquid. When the vessel is cooled to a
1. A bimetallic strip is formed out of two identical strips, one of lower temperature, the space in the vessel unoccupied by the
copper and other of brass. The coefficients of linear expansion of liquid remains constant. Then the volume of the liquid (VL), volume
the two metals are C and B. On heating, the temperature of the of the vessel (Vv), the coefficients of cubical expansion of the
strip goes up by T and the strip bends to form an arc of radius of material of the vessel ( v ) and of the liquid ( L) are related as
curvature R. Then R is
(a) Proportional to T (a) L v (b) L v
(b) Inversely proportion a to T (c) v/ L Vv / VL (d) v/ L VL / Vv
(c) Proportional to B – C
5. If , and are coefficients of linear, superficial and volume
(d) Inversely proportional to B – C
expansion respectively, then
2. Heat is supplied to a certain homogeneous sample of matter, at a 1 2
uniform rate. Its temperature is plotted against time, as shown. (a) (b)
2 3
Which of the following conclusions can be drawn ?
3
(c) (d)
1
Temperature

6. A metal sphere of radius R and specific heat C is rotated about an


axis passing through its centre at a speed n rotation /second. It is
suddenly stopped and 50% of its energy is used in increasing its
Time temperature, then choose the correct statement(s) from the
(a) Its specific heat capacity is greater in the solid state than in following
the liquid state (a) Kinetic energy used to raise the temperature of the sphere is
(b) Its specific heat capacity is greater in the liquid state than in 2 2
2 n
the solid state MR 2
5
(c) Its latent heat of vaporization is greater than its latent heat
(b) Kinetic energy used to raise the temperature of the sphere is
of fusion
(d) Its latent heat of vaporization is smaller than its latent of 5 2n
MR
fusion 3
4 2n2 R 2
3. When the temperature of a copper coin is raised by 80°C, its (c) The rise in the temperature of the sphere is
7C
diameter increases by 0.2%, then
(a) percentage rise in the area of a face is 0.4% 2 2n2 R 2
(d) The rise in the temperature of the sphere is
(b) percentage rise in the thickness is 0.4% 5C
(c) percentage rise in the volume is 0.6%
(d) coefficient of linear expansion of copper is 0.25×10–4/°C

Answer Key 1 (b, d) 3 (a, c, d) 5 (b, c)


Sol. from page 408 2 (b, c) 4 (a, d) 6 (a, d)
www.crackjee.xyz
400 MECHANICS, HEAT, THERMODYNAMICS & WAVES

Statement Questions Exercise 5.3


Read the two statements carefully to mark the correct option out of the options given below:
(a) If both the statements are true and the statement - 2 is the correct explanation of statement - 1.
(b) If both the statements are true but statement - 2 is not the correct explanation of the statement - 1.
(c) If statement - 1 true but statement - 2 is false.
(d) If statement - 1 is false but statement - 2 is true.

1. Statement - 1 Statement - 2
A body is in equilibrium in an inertial frame but not be equilibrium
9
in an non-inertial frame. F= C + 32.
5
Statement - 2
The body can be in thermal equilibrium in both the frames. 8. Statement - 1
2. Statement - 1 Specific heat capacity is the cause of formation of land and sea
For the thermal equilibrium of two bodies, they must be in contact. breeze.
Statement - 2 Statement - 2
Two bodies in contact must be in thermal equilibrium. The specific heat of water is more than land.
3. Statement - 1 9. Statement - 1
A tightly closed metal lid of a glass bottle can be opened more Water kept in an open vessel will quickly evaporate on the surface
easily if it is put in hot water for some time. of the moon.
Statement - 2 Statement - 2
The coefficent of expansion of metal lid is greater than that of The temperature at the surface of the moon is much higher than
glass. boiling point of the water.
4. Statement - 1 10. Statement - 1
The coefficient of linear expansion has dimension K–1. Two bodies at different temperatures, if brought in contact do not
Statement - 2 necessary settle to the mean temperature.
The coefficient of volume expansion has dimension K–1. Statement - 2
5. Statement - 1 The two bodies may have different thermal capacites.
The melting point of ice decreases with increase of pressure. 11. Statement - 1
Statement - 2 Heat is a conserved quantity.
Ice contracts on melting. Statement - 2
6. Statement - 1 Energy of an isolated system remain conserved.
Fahrenheit is the smallest unit measuring temperature. 12. Statement - 1
Statement - 2 When solid melts or a liquid boils, the temperature does not increase
Fahrenheit was the first temperature scale used for measuring when heat is supplied.
temperature. Statement - 2
7. Statement - 1 The heat supplied is used to increase internal potential energy.
The temperature at which Centrigrade and Fahrenheit thermometers
read the same is – 40°.

Answer Key 1 (b) 3 (a) 5 (a) 7 (a) 9 (a) 11 (d)


Sol. from page 409 2 (d) 4 (b) 6 (c) 8 (a) 10 (a) 12 (a)
www.crackjee.xyz
THERMOMETRY, EXPANSION AND CALORIMETRY 401

Passage & Mat r ix Exercise 3.4

Passage for (Q. 1 - 3) : 6. A metal disc having circular hole at its centre is heated. If the metal
At very low temperatures, the molar heat capacity of rock salt varies expands on heating, the diameter of the hole will
with temperature according to Debye’s law; thus (a) increase
T 3 (b) decrease
C K 3 (c) remain unchanged
where K = 1940J/mol–K and = 281K (d) increases or decreases depending upon the metal
1. The heat required to raise the temperature of 2 moles of rock salt 7. On heating a liquid of coefficient of cubical expansion in a
from 10K to 50K is container having coefficient of linear expansion /3, the level of
(a) 240J (b) 273J liquid in the container will
(c) 348J (d) 472J
(a) rise
2. The mean molar heat capacity in this range is
(a) 3.42 J/mol–K (b) 4.12 J/mol–K (b) fall
(c) 4.82 J/mol–K (d) 5.08 J/mol–K (c) remain almost stationary
3. The true molar heat capacity at 50K is (d) none of these
(a) 8.3 J/mol–K (b) 9.6 J/mol–K
8. The coefficient of linear expansion of brass and steel are and
(c) 10.3 J/mol–K (d) 10.9 J/mol–K 1

2. If we take a brass rod of length 1 and a steel rod of length 2


Passage for (Q. 4 - 5) :
at 0°C, their difference in length will remain the same at any
A uniform solid brass cylinder of mass M = 0.50 kg and radius
temperature if
R = 0.030 m is placed in frictionless bearings and set to rotate about its
geometrical axis with an angular velocity of 60 rad/s. 2 2
(a) 1 2 2 1 (b) 1 2 2 1
4. The angular momentum of cylinder and required to reach this state
of rotation, starting from rest is (c) 2 2 (d)
1 2 2 1 1 1 2 2
(a) 0.75 × 10–2 J–s (b) 1.35 × 10–2 J–s
(c) 2.25 × 10 J–s–2 (d) 2.70 × 10–1 J–s Passage for (Q. 9 - 11) :
5. After the cylinder has reached the specified state of rotation, it is A uniform disc is spinning about geometric axis in free space. Its
heated without any mechanical contact from room temperature temperature is increased by T. The coefficient of linear expansion is .
20°C to 100°C, the fractional change in angular velocity of cylinder
is (Take = 2.0 × 10–5/ 0C) 9. The fractional change in its angular velocity is
(a) –3.2 × 10–3. (b) –6.4 × 10–3 (a) T (b) – T
(c) 1.6 × 10 –3 (d) 4.8 × 10–3
(c) 2 T (d) –2 T
Passage for (Q. 6 - 8) : 10. The fractional change in speed of rim is
When a solid is heated, its length changes according to the relation (a) T (b) 2 T
0 (1 T) where = final length, 0 = initial length, T = (c) – T (d) –2 T
change in temperature and = coefficient of linear expansion. 11. The fractional change in its rotational kinetic energy is
The change in area of solid upon heating is called superficial expansion.
(a) T (b) – T
The area changes according to the following relation A A0 (1 T)
(c) 2 T (d) –2 T
where A = final area, A0 = initial area and = coefficient of superficial
expansion.

Answer Key 1 (b) 3 (d) 5 (a) 7 (c) 9 (d) 11 (d)


Sol. from page 409 2 (a) 4 (b) 6 (a) 8 (d) 10 (c)
www.crackjee.xyz
402 MECHANICS, HEAT, THERMODYNAMICS & WAVES

12. Three liquids A, B and C having same specific heat and mass m, 2m and 3m have temperatures 20°C, 40°C and 60°C respectively. Temperature
of the mixture when
Column -I Column -II
(A) A and B are mixed p. 33.3°C
(B) A and C are mixed q. 52°C
(C) B and C are mixed r. 50°C
(D) A, B and C all three are mixed s. 46.67°C
13. A sphere is spinning about its diameter. Its temperature is gradually increased by T. The coefficient of linear expansion of sphere is . Match
the columns :
Column - I Column - II
(A) Fractional change in angular momentum p. T
(B) Fractional change in moment of inertia q. –2 T
(C) Fractional change in angular velocity r. 2 T
(D) Fractional change in kinetic energy s. zero
14. Column - I Column - II
(A) Bimetalic strip p. Radiation from a body
(B) Steam engine q. Energy conversion
(C) Incondescent lamp r. Melting
(D) Electric fuse s. Thermal expansion

Answer Key 12 A (p); B (r); C (q); D (s) 13 A (s); B (r); C (q); D (q)
Sol. from page 409 14 A (s, q); B (q); C (p, q); D (q, r)

Subjective Integer Type Exercise 5.5


Solution from page 410
1. A faulty thermometer reads 5°C in melting ice and 99°C in steam. 6. The volume of a glass vessel is 1000 cc at 20°C. What volume of
Find the correct temperature in °F when the faulty thermometer mercury should be poured into it at this temperature so that the
reads 52°C. Ans. 122°F. volume of the remaining space does not change with temperature?
Coefficients of cubical expansion of mercury and glass are
2. The pressure of air in the bulb of constant volume air thermometer
1.8 × 10–4/°C and 9.0 × 10–6/°C respectively. Ans. 50 cc.
is 75 cm of mercury at 0°C, 100 cm at 100°C and 80 cm at the
room temperature. Calculate the room temperature.Ans. 20°C. 7. How many grams of ice at –14°C are needed to cool 200 g of
water from 25°C to 10°C ? Take specific heat of ice = 0.5 cal/g/°C
3. A one litre flask contains some mercury. It is found that at different
and latent heat of ice = 80 cal/g. Ans. 31 g.
temperatures, the volume of air inside the flask remains the same.
What is the volume of mercury in this flask ? Given for glass 8. The internal energy of a monoatomic gas is 1.5 nRT. One mole of
9.0 × 10–6/ °C and for mercury = 1.8 × 10–4/ °C.Ans. 150 cm3. helium is kept in a cylinder of cross–section 8.5 cm2. The cylinder
is closed by a light frictionless piston. The gas is heated slowly in
4. Three equal length straight rods of aluminium, Invar and steel all
a process during which a total of 42 J heat is given to the gas. If
at 20°C, form an equilateral triangle with hinge pins at the vertices.
the temperature rises through 2°C, find the distance moved by the
At what temperature will the angle opposite the Invar rod be
piston. Atmospheric pressure = 100 kPa. Ans. 20 cm.
59.95° ? Ans. 66 ° C.
5. A railway track (made of iron) is laid in winter when the average
temperature is 18°C. The track consists of sections of 12.0 m
placed one after the other. How much gap should be left between
two such sections so that there is no compression during summer
when the maximum temperature goes to 48°C ? Coefficient of
linear expansion of iron = 11 × 10–6/ °C. Ans : 0.4 cm.
www.crackjee.xyz
THERMOMETRY, EXPANSION AND CALORIMETRY 403

Subjective Exercise 5.6


Solution from page 411
1. A constant volume gas thermometer using helium records a pressure 10. Suppose that one early morning when the temperature is 10°C, a
of 20.0 kPa at the triple–point of water and pressure of 14.3 Pa at driver of an automobile gets his gasoline tank which is made of
the temperature of ‘dry ice’ (solid CO2). What is the temperature steel, filled with 75 litre of gasoline, which is also at 10°C. During
of‘dry ice’ ? Ans. 195.30 K. the day, the temperature rises to 30°C. How much gasoline will
2. An ungraduated thermometer of uniform bore is attached to a overflow ? Given for steel = 1.2 × 10–5/ °C and for gasoline
= 9.5 × 10–4/ °C. Ans. 1.37 litre.
centimeter scale and is found to read 10.3 cm in melting ice, 26.8
11. If coefficient of linear expansion is treated as variable, dependent
cm in boiling water and 6.5 cm in freezing mixture. Calculate the
on temperature T, then show that the length L at temperature T,
temperature of the freezing mixture. Ans. –23.03°C.
T
3. A hole is drilled in a copper sheet. The diameter of the hole is 4.24 L L0 1 (T )dt
cm at 27.0°C. What is the change in the diameter of the hole when T0
the sheet is heated to 227°C ? Coefficient of linear expansion of where L0 is length at reference temperature T0.
copper = 1.70 x 10–5/°C. Ans. 1.44 × 10–2 cm. 12. A 1.28 m long vertical glass tube is half filled with a liquid at 20°C.
4. A steel tape 1 m long is correctly calibrated for a temperature of How much will the height of the liquid column change when the
27.0°C. The length of a steel rod measured by this tape is found to tube is heated to 30°C ? Take glass = 1.0 × 10–5/°C and liquid
be 63.0 cm on a hot day when the temperature is 45.0°C. What is = 4.0 × 10–5/°C. Ans. 1.3 × 10–4 m.
the actual length of the steel rod on that day ? What is the length 13. In figure shown, left arm of a U–tube is immersed in a hot water
of the same steel rod on a day when the temperature is 27.0°C ? bath at temperature T, and right arm is immersed in a bath of
melting ice; the height of manometric liquid in respective columns
Coefficient of linear–expansion of steel = 1.20 ×10–5/ °C ?
is ht and h0. Determine the coefficient of expansion of the liquid.
Ans. 63.0136 cm.
Water at
5. A large steel wheel is to be fitted on to a shaft of the same material. temperature
At 27°C, the outer diameter of the shaft is 8.70 cm and the diameter t°C Melting ice
of the central hole in the wheel is 8.69 cm. The shaft is cooled
using ‘dry ice’ (solid carbon dioxide). At what temperature of the
ht
shaft does the wheel slip on the shaft ? Assume coefficient of h0
linear expansion of the steel to be constant over the required
temperature range. steel = 1.20 × 10–5 / K. Ans. – 68.8°C.
6. A brass rod of length 50 cm and diameter 3.0 mm is joined to a
Ans. = [(ht – h0)/h0t].
steel rod of the same length and diameter. What is the change in
length of the combined rod at 250°C, if the original lengths are at 14. Three rods A, B and C for an equilateral triangle at 0°C. Rods AB
40.0°C ? Is there a ‘thermal stress’ developed at the junction ? and BC have same coefficient of expansion 1 and rod AC has 2.
The ends of the rod are free to expand. Coefficient of linear If temperature of the system is increased by T°C, what is the
change in angle formed by rods AB and BC ?
expansion of brass
= 2.0 × 10–5/°C and that of steel = 1.2 × 10–5/ °C.
Ans. 0.34 cm, No. A
7. A brass wire 1.8 m long at 27°C is held taut with little tension
between two rigid supports. If the wire is cooled to a temperature
1

of – 39°C, what is the tension developed in the wire, if its diameter


l 3,
l1 ,

is 2.0 mm ? Coefficient of linear expansion of brass =2.0 × 10–5/


2

°C, Young’s modulus of brass = 0.91 × 1011 Pa. 3d


Ans. T = .
Ans. 3.77 ×102 N. 2( 2 1)

8. Density , mass m and volume V are related as = m/V. Prove that B l2, 1
C

1d
– .
dT 15. A metre scale made of steel reads accurately at 20°C. In a sensitive
experiment, distances accurate upto 0.055 mm in 1 m are required.
9. What should be the lengths of steel and copper rods at 0°C that
Find the range of temperature in which the experiment can be
the length of steel rod is 5 cm longer than copper at all temperatures?
performed with this metre scale.Coefficient of linear expansion of
Given for copper = 1.7 × 10 –5/°C and for steel = 1.1 ×
steel = 11 × 10–6/ °C. Ans : 15°C to 25°C.
10 –5 /°C. Ans. 9.17 cm, 14.17 cm.
www.crackjee.xyz
404 MECHANICS, HEAT, THERMODYNAMICS & WAVES
16. Two steel rods and an aluminium rod of equal length l0 and equal 22. In an industrial process 10 kg of water per hour is to be heated
cross–section are joined rigidly at their ends as shown in the from 20°C is passed from a boiler into a copper coil immersed in
figure below. All the rods are in a state of zero tension at 0°C. Find water. The steam condenses in the coil and is returned to the boiler
the length of the system when the temperature is raised to . as water at 90°C. How many kg of steam are reqiured per hour ?
Coefficient of linear expansion of aluminium and steel are a and Ans : 1.27 kg.
23. An ice cube of mass 0.1 kg at 0°C is placed in an isolated container
s respectively. Young’s modulus of aluminium is Ya and of steel which is at 227°C. The specific heat ‘S’ of the container varies
is Ys.
with temperature T according to empirical relation S = A + BT,
where A = 100 cal/kg and B = 2 × 10–2 cal/kg–K2. If the final
Steel
temperature of the container is 27°C, determine the mass of the
Aluminium container. Ans : 0.495 kg.
Steel 24. The specific heat of substance varies with temperature according to
equation c = (2t2 + t) × 10–3 cal/g°C. Calculate the amount of heat
required to raise the temperature of 100 g of substance from 20°C to
aYa 2 sYs
Ans : l0 1 . 40°C. Ans : 37.9 kcal.
Ya 2Ys
25. A fat man is used to consuming about 3000 k–cal worth of food
17. A torsional pendulum consists of a solid disc connected to a thin everyday. His food contains 50 g of butter plus a plate of sweets
wire ( = 2.4 × 10–5/°C) at its centre. Find the percentage change everyday, besides items which provide him with other nutrients
(proteins, vitamins, minerals, etc.) in addition to fats and
in the time period between peak winter (5°C) and peak summer
carbohydrates. The caloric value of 10 g of butter is 60 kcal and
(45°C). Ans : 9.6 × 10–2.
that of a plate of sweets is of average 700 kcal. What dietary
18. A circular disc made of iron is rotated about its axis at a constant
strategy should he adopt to cut down his calories to about 2100
velocity . Calculate the percentage change in the linear speed of a kcal per day ? Assume the man cannot resist eating the full plate
particle of the rim as the disc is slowly heated from 20°C to 50°C of sweets once it is offered to him.
keeping the angular velocity constant.Coefficient of linear expansion Ans. The man intends to cut down 3000 – 2100 = 900 kcal. But
of iron = 1.2 × 10–5/ °C. Ans : 3.6 × 10–2. avoiding sweets completely, he will cut down 700 kcal. To cut
19. A glass window is to fit in an aluminium frame. The temperature down another 200 kcal, he should cut down butter by (10/60) ×
on the working day is 40°C and the glass window measures exactly 200 = 33 g per day. He should not cut down consumption of food,
20 cm × 30 cm. What should be the size of the aluminium frame so that provides him with vitamins and other vital nutrients.
that there is no stress on the glass in winter even if the temperature 26. A geyser heats water flowing at the rate of 30 l / min from 27°C to
drops to 0°C ? Coefficient of linear expansion for glass and 77°C. If the geyser operates on a gas burner, what is the rate of
aluminium are 9.0 x 10–6/°C and 24 × 10–6/°C respectively. consumption of the fuel if its heat of combustion is 4.0 × 104 J/g?
Ans : 157.5 g/min.
Ans. 20.012 cm × 30.018 cm.
27. Steam at 100°C is allowed to pass into a vessel containing 10 g of
20. The apparatus shown in the figure consists of four glass columns
ice, 100 g of water at 0°C, until all the ice is melted and the
connected by horizontal sections. The height of two central
temperature is raised to 5°C. Neglecting water equivalent of the
columns B and C are 49 cm each. The two outer columns A and D vessel and the loss due to radiation etc. Calculate how much steam
are open to atmosphere. A and C are maintained at a temperature is condensed. The latent heat of steam is 536 cal/g and latent heat
of 95°C while the columns B and D are maintained at 5°C. The of ice is 80 cal/g. Ans : 2.13 g.
height of the liquid in A and D measured from the base line are 52.8 28. A copper block of mass 2.5 kg is heated in a furnace to a temperature
cm and 51 cm respectively. Determine the coefficient of thermal of 500°C and then placed on a large ice block. What is the maximum
expansion of the liquid. amount of ice that can melt ? (specific heat of copper
= 0.39 J/g–°C and heat of fussion of water = 335 J/g).
Ans : 1.45 kg.
29. A 0.1 kg steel ball falls from a height of 10 m and bounces to a
height 7 m.
(a) Why does it not bounce back to its original height ?
(b) If all the dissipated energy were absorbed by the ball as heat,
how much will its temperature rise ? (specific heat of steel =
0.11 k cal/kg°C, 1 cal = 4.2 J) Ans : 0.064°C.
30. A lead bullet just melts when stopped by an obstacle. Assuming
that 25 percent of the heat is absorbed by the obstacle, find the
Ans. 2 ×10–4/°C. velocity of the bullet if its initial temperature is 27°C. (Melting
21. 0.05 kg steam at 373 K is mixed with 0.45 kg of ice at 253 K. Find point of lead = 327°C, specific heat of lead = 0.03 cal/g°C, latent
the temperature of the mixture. Latent heat of vaporization for heat of fusion of lead = 6 cal/g°C, J = 4.2 J/cal)
steam = 540 cal/g, latent heat of fusion of ice = 80 cal/g and Ans : v = 409.9 m/s.
specific heat of ice = 0.5 cal/g°C Ans : 0°C.
www.crackjee.xyz
THERMOMETRY, EXPANSION AND CALORIMETRY 405

Solutions Exercise 5.1Level -1


1. (d) Water expands on both sides of 4 °C.
T T
14. (d) 100 = 100
2. (b) C = 3A and S = silver T 2
After solving above equations, we get
6
2 10 10 100
silver = C 3A S =
2
3. (c) First the temperature of liquid oxygen increases thereafter it
= 10 –3.
comes in gaseous state.
4. (b) First ice comes from –10 °C to 0 °C. Then it melts and its 15. (c) The thermal stress, f Y T ; Clearly f does not depend
temperature rises to boiling point. Thereafter it will convert on length of the rod.
into steam. 16. (a) On heating the block, the hole size will increase.
5. (b) It is the definition of calorie.
0
6. (c) The heat lost in condensation = x × 540 cal. 17. (d) =
t 1 t
x 540 = y 80 y 1 (100 – 0)
10
x 1 9.7 = = 10–4/°C
or = . 1 100 ,
y 3
7. (c) As dQ = mCdT 18. (c) V = V T

1 dQ 1 1 V1 (2r ) 2 T 2
C = = m = 2: 1
m dT dT / dQ V2 r 2
2 T 1

The slope of BC OA , Csolid Cliquid . 19. (d) = 3 and –3 '


1 2

8. (a) V = V apparent T After solving, we get

V 1 2
or = ( –3 ) T ' = .
V 3
20. (b) Melting point of ice decreases with increase in pressure
Bulk modulus, K = P (experimentally).
V 21. (a) Heat needed,
V
Q = 1 × 0.5 × 10 + 1 × 80 + 1 × 1 × 100 + 1 × 540
= 3045 J.
V
P = K K( – 3 ) T 22. (d) mg × 100 = m × 4200 × T
V
T = 0.023 °C
9. (b) The change of state occurs at constant temperature, 23. (d) The rod should be expanded differently.
so material graph 2 will have greatest heat of fusion 24. (a) 22 × 1 × (90 – 20) = m × 540 + m × 1 × (100 – 90)
i.e., Q = H t ; t is largest for 2 m = 2.8 g
10. (b) Material expands outward and so x, r increases. Due to linear The total mass of water = 22 + 2.8 = 24.8 g.
expansion diameter of rod will increase.
C F 32 ; or 5 160
x 25. (b) C F .
x 32 5 9 9 9
11. (a) = , x 40
5 9
Thus the slope of line AB is 5/9.
Pt – P0 60 50 26. (a) The amount of heat available in water = 2 × 1 × (35 – 0) = 70
12. (a) t 100 100 25 C cal. The heat needed to melt the ice completely is 1 × 80 = 80
P100 – P0 90 – 50
cal. So ice will not melt completely and temperature of water
13. (b) By increasing pressure above mercury column, the boiling falls to 0°C.
point can be increased. 27. (b) At C, solid completely will convert into liquid.
www.crackjee.xyz
406 MECHANICS, HEAT, THERMODYNAMICS & WAVES

Solutions Exercise5.1 Level -2


1. (c) 10. (b) The heat consumed in melting 15.2 g of ice
P Q = mL = (15.2 × 10–3) × 3.34 × 105
= 5.07 × 103J
T2
T1 T1 > T2 Thus ( mg ) x = 5.07 × 103
or 0.060 × 10 × 9.8 x = 5.07 × 103
x = 863 m
11. (a) M1g = Mg – V 1 1
g

V or M1g = Mg – V1 1g … (i)

Figure shows two isotherms in which T1 > T2. Clearly gas is 1


and M 2 g = Mg V1[1 g (t2 t1)] g
heated in the beginning and cool at the end. [1 (t2 t1 )]
I … (ii)
2. (d) = 2 T After simplifying, we get
I
I M 2 M1 1
or 100 = [2 × 10–5× 20 ] × 100 = g
M M 2 (t2 t1 ) .
I
= 0.04
3. (a) mg = Fb 12. (c) 100 °C
Fe V0 g = ( k1V0 )( Hg ) 0 g … (i)
and ( Fe )60Vt g = ( k2Vt )( Hg )60 g

( Fe )0
( Hg )0
or 1 g = k2 g … (ii)
Fe 60 1 Hg 60
From above equations, we get
k1 1 60 Fe
= 0 °C
k2 1 60 Hg V
4. (c) 1 = 1 A t and 2 2 st
As 1 A t 2 st , 1 Al 2 s V = V T
or ( A) = V ( Hg – g ) T
1 1 s s
1 A or 20 × (0.25 × 10–1)2 = V(0.000180 – 0.000009)× (100)
1 2 A s Al s .
1 V = 2.57 cm3
s
1 2
5. (c) PV = nRT and P V nR T 13. (b) mv = mC T
2
V 1 1
; It is like y . v2 200 2
V T T x T =
2C 2 128
6. (a) PV = constant
or PdV VdP 0 = 156°C
14. (d) The heat available on water = 30 × 1 × (80 – 0) = 2400 cal.
dV 1 1 To melt the ice, it need = mL = 50 × 80 = 4000 cal
; it is like y .
V ( dP) P x So if m amount of ice is melt, then
m × 80 = 2400
7. (a) Pt 160t mC T or m = 30 g
or 1000t 160t 2 4200 (77 27) The ice left = 20 g
t 500 s or 8min 20s 15. (c) If m is the mass of steam condenses, then
m × 540 = (250) × 1 × (100 – 0) + 20 × 80
8. (a) Tension, F (Y T)A + 20 × 1 × (10000)
m = 53 g
= (2 10 ) (10 5 ) (25 – 0) 11
(0.5 10 2 ) 2 The entire contents will reach the temperature 100°C.
= 3925 N.
9. (a) The density of the paraffin 16. (a) s b = 30 … (i)

0.82 Also b s = 0
0
= 0.809g/cm3
1 t 1 0.0009 15 or b b T = s s T
The mass of the paraffin at this temperature 6 6
or b 19 10 = s 11 10 … (ii)
m V 0.809 50 = 40.5 g.
www.crackjee.xyz
THERMOMETRY, EXPANSION AND CALORIMETRY 407
11 dV dV 3T 2
or b = s …(iii) 3T 2 = C ;
19 dT dT C
From above equations, we get
1
s 71.25 cm and b 41.25 cm dV 3T 2 3
Thus = T3 .
17. (c) The pressure at the bottom of both the limbs must be same, VdT C T
so C

0 gh0 = t ght (45 – 25)g (45 – 27)g


23. (c) V1 and V2
d1 d2
0
or 0 h0 = ht
1 30 V1 20 d2
V2 = 18 d1
124
or 120 = 1 30 V1 20 d1
or =
4 V1 (1 rmetal 10) 18 (1 r 10) d1
= 11 10 / C
18. (c) Suppose the correct barometer height at 0°C is 0, then After simplifying, we get metal
2.6 10 3
/ C
3
0 = 75
5R 3R 7T0
or 0 75 ( Hg b) 27 75 24. (b) 1 (T f – T0 ) = 1 – Tf
2 2 3
4
or 0 75 (0.6 10 – 0.2 10 4 ) 27 75 3T0
Tf = .
0 = 74.92 cm 2
19. (d) If y be the extension of the spring, then Pt
25. (b) Pt = mL ; L=
.
m
26. (b) 2 × 0.5 × (20 – 0) + m × 80 = 5 × 1 × 20
m = 1 kg
k The total mass of water = 5kg + 1 kg = 6 kg.
27. (a) P1 = P2
m or 1g 1 = 2g 2

h 0 0
or 1 t1
1 = 2
1 t2
mg = ky + V wg
1 2
2 = .
10 3
g 2t1 1t2
or 2g = 25 × y + 3
5 10
28. (d) (OR ) 2 = (PR)2 – (PO)2 = 2
/2
2
y = 0.64 m
The energy of the spring block-spring system 2
2
1 2 = [ (1 2t )] (1 1t )
= ky mgh 2
2
After simplifying and neglecting 2 2 2 2
1 2 2 t and 1t , we get
Thus mwCw T mbCb T = ky mgh
2 1 = 4 2
1 29. (b) Let A 0 and A t be the areas of cross-section of the tube at
(0.5 × 4200 + 2 × 250) × 0.004 = 25 0.64 2 + 2× 10×h temperature 0 °C and t °C respectively,
2
h = 0.26 m l = length of the liquid column (constant)
20. (d) F = F0 +Y T × A V0 and Vt be the volumes of the liquid at temperature 0 °C
= 50 + (34 × 1010) × (4.5 × 10–6) × and t °C respectively,
(150 – 20) × (0.1 × 10–3)2 V0 = A0 Vt = At
= 56.2 N. Vt = V0(1 + t) At = A0 (1+2 t)
L Vt = A0(1+2 t) = V0(1+ t) = A0(1+ t) or = 2 .
21. (c) = ; It found greatest for rod (c). 30. (b) 2.60 + 2.60 × steel T = 2.60 brass T
L T
31. (b) 60 = mC T W ' … (i)
nRT
22. (c) Given PT 2 constant ; PV nRT or P
V and 120 = (3m)C T W ' … (ii)
After solving above equations, we get
nRT
T2 = constant W' = 30 W.
V
or T3 = CV
www.crackjee.xyz
408 MECHANICS, HEAT, THERMODYNAMICS & WAVES

Solutions Exercise 5.2


1. (b, d) Let 0 be the length and t the thickness of each strip. On dQ mCdT
heating, length of brass rod or Hdt mCdT
1 = 0 (1 B T) H 1
C
By the geometry of the figure, we have m (dT / dt )
dT
As of CD is smaller, so Cliquid Csolid
dt
Q1 H ( t1) and Q2 H ( t2 )
As t2 t1 , Q2 Q1 .
3. (a, c, d) Given,
D R
100 0.2% or 100 0.2%
D R
V R t
100 2 100 2 0.2 0.2
Fig. 5.18 V R t
1 = R1 = 0.6 %
t D 0.2
= R Also =
2 D 100
t D 80 0.2
R 0 (1 B T) … (i) or =
2 D 100
4
0.25 10 / C.
Similarly for copper strip, 2 0 (1 Ct) R2
4. (a, d) Obviously L v .
t
or R = 0 (1 C T) … (ii) VL = Vv
2
Dividing equation (i) by (ii), we get or VL L T = Vv v T
VL L = Vv v .
t
R 1 T 5. (b, c) 2 and 3
2 =
B
t 1 C T 2
R and 3
2 3
t t 6. (a, d) Kinetic energy of the sphere,
R (1 C T) = R (1 B T)
2 2 1 2
K I
2
t t
R R C T T 1 2
2 2
C MR 2 (2 n )2
2 5
t t 4 2 2
R R B T B T n MR 2 .
2 2 5
Kinetic energy used to raise, the temperature
If T is small, is still small, so we can neglect their
product. And, therefore, we have 4 2 2
= 0.5 n MR 2
5
R B T R C T = t
t 2 2 2
or R = Ans. = n mR 2
( B 5
C) T
T If T be the raise in temperature of the sphere, then
2 2 2
D E
MC T = 5 n MR 2
2. (b, c) B C
2 2 2
2 n R
A t T = .
t1 t2 5C
www.crackjee.xyz
THERMOMETRY, EXPANSION AND CALORIMETRY 409

Solutions Exercise-5.3
1. (b) In non-inertial frame, there is a pseudo force, which disturb 8. (a) The specific heat of water is greater than specific heat of
the equilibrium. Thermal equilibrium has no effect of force. land. Because of this water looses heat slowly in comparision
2. (d) Without contact the heat can be transfer due to radiation. to land, which causes temperature difference between two
3. (a) correctly explained and so land and sea breeze.
4. (b) Coefficient of linear expansion and coefficient of volume
9. (a) Correctly explained
expansion both have same unit, i.e., 1° C or / K.
5. (a) Correctly explained 10. (a) m1C1(T1 – T ) = m2C2 (T – T2 )
6. (c) In Fahrenheit, the fixed points are equally divided into 180
division instead of 100 division. m1C1T1 m2C2T2
T = m1C1 m2C2
9
7. (a) F = C 32
5
11. (d) Heat is not a conserved quantity because it is the moving
9 energy.
or x = x 32
5 12. (a) At the change of state, the internal potential energy increases.
x = – 40°

Solutions Exercise-5.4
Passage (Q.1 - 3)
2 R
= – –2 T
50 50 R
3
1. (b) Q = T
nCdT 2 k
3
dT = – 2 × 2 × 10–5 × 80
10 10 = – 3.2 × 10–3.
Passage (Q.6 - 8)
50
2 1940 6. (a) Due to expansion of material, the diameter of hole will
= T 3dT
(281)3 increase.
10

7. (c) apparent liquid cont 3 0


2 1940 | T 4 |10
50 3
=
(281)3 4
So V V T 0.
apparent

= 273 J. 8. (d) For lengths of the rods to be constant

50 1 = 2
CdT 50 or T = T
2. (a) Cmean = 10 1 T3 1 1 2 2
K 3
dT
(50 – 10) 40 =
10 1 1 2 2

Passage (Q.9 - 11)


= 3.42 J/mol - K.
9. (d) R = R T .
3 3
T 50 In the process, the angular momentum of the disc remains
3. (d) C = K 3
1940 10.9J/mol-K
2813 constant, so
Passage (Q.4 - 5)
MR 2
L = I
MR 2 0.50(0.030) 2 2
4. (b) L = I = 60
2 2 2 R
L
or =
= 1.35 × 10–2 J-s L R

5. (a) In the process L , remains constant.


or 0 = 2 T
R = R T

2 R
Also =0 = 2 T
R
www.crackjee.xyz
410 MECHANICS, HEAT, THERMODYNAMICS & WAVES
10. (c) The angular momentum can also be written as 13. A s; B r; C q; D q
L = mvR
2
L v R I = mR 2
or = 5
L v R
I M 2 R
v
T ;
v
T
=
or 0 = I M R
v v
= 0+ 2 T = 2 T
L2 In the process, angular momentum is constant and so,
11. (d) As, K = ,
2I
L
k 2 L I 0.
= L
k L I
As L = I
= 0– 2 T .
L I
= – 2 T , =
L I
12. A p; B r; C q; D s
A and B: (m) (s) ( – 20) = (2m)(s)(40 – )
or 0 = 2 T
100
or 33.3 C
3
or = –2 T.
A and C : (m)(s)( – 20) = (3m)(s)(60 – )
= 50°C
k
B and C : (2m) (s) ( – 40) = (3m) (s) (60 – ) Also = 2 T.
k
= 52 °C
A, B and C : (m) (s) ( – 20) + (2m)(s)(60 – ) 14. A s, q; B q;C p, q; D q, r
= (3m) (s) (60 – ) Theoritical.
= 46.67°C

Solutions Exercise-5.5
1. If x is the temperature on faulty thermometer, then 6. Suppose V be the volume of mercury. For the volume of empty
space to be constant,
X FP C 0
= Vmercury = Vglass
BP FP 100 0
or V mercury t = 1000 glass t
52 5 C
or =
99 5 100 1000 9 10 6

C = 50° C V = 4
1.8 10
= 122° F Ans.
= 50 cm3 Ans.
Pt P0 7. If m be the required mass of the ice, then
2. Temperature t = 100 m × 0.5 ×[0 – ( – 14)] + m × 80 + m × 1 × (10 – 0)
P100 P0
= 200 × 1 × (25 – 10)
m = 31 g. Ans.
80 75 8. Given, U = 1.5 nRT
= 100
100 75 dU = 1.5 nR(dT)
= 20° C Ans. = 1.5 × 1 × 8.3 × 2
= 24.9 J
3. Suppose V be the volume of the mercury in the flask. For volume
We know that, Q = dU + W
of air in the flask to be constant, increase in volume of mercury
W = Q – dU
= increase in volume of flask
= 42 – 24.9
or V Hg t = (103) g t = 17.1 J
As W = P V
103 (3 9 10 6 )
or V = = P (Ax)
4
1.8 10 = PA( x)
= 150 cm3 Ans. W
4. See the solution of problem 14 from exercise 5.6. x =
PA
5. The expansion of each rail
17.1
= t =
= 12 × 11 × 10–6 × (48 – 18) 100 10 8.5 10 4
3

= 0.4 × 10–2 m Ans. = 20 × 10–2 m Ans.


www.crackjee.xyz
THERMOMETRY, EXPANSION AND CALORIMETRY 411

Solutions Exercise-5.6
7. The thermal stress in the wire
P T
1. We know that = fth = Y t
Ptr Ttr
= 0.91 × 1011 × 2.0 ×
10–5 × | – 39 – 27 |
Ttr
T = P = 1.20 × 108 N/m2
Ptr
The tension in the wire
F = Fth A
274.16
= 14.3 = f× r2
20
= 1.20 × 108 × (10–3)2
= 195.30 k Ans.
= 3.77 ×102 N. Ans.

t 0 m
2. The temperature t = 100 8. Given =
100 0 V
= mV –1

6.5 10.3 Differentiating both sides w.r.t. temperature we have


= 100
26.8 10.3
d 2 dV
= mV [m = constant]
= – 23.03° C. Ans. dt dt
3. The change in diameter is given by
m dV
D = D t =
V Vdt
= 4.24 × (1.70 × 10–5) × (227 – 27)
= 1.44 × 10–2 cm Ans. dV
As =
4. The length of the rod at hot day Vdt

2 = 1[ 1+ (t2 – t1) ]
d
= 63 [ 1 + 1.2 0 × 10–5 (45 – 27) ] dT
= 63.013 cm Ans.
1d
5. For the slip of the wheel on the shaft or = Proved
dt
D = D t
or – 0.01 = 8.70 × 1.20 × 10–5 × t 9. Suppose 01 , and 02 be the lengths of steel and copper rods at
t = – 95.78° C 0° C respectively.
Final temperature t = 27 + (– 95.78) Their lengths at temperature t are given by
= – 68.78° C = (1+
1 01 1t)
= – 68.8° C Ans.
6. The change in length in brass rod and 2 = 02
(1+ 2t)

b = t Given 1– 2 = 5 cm
= 50 × 2.0 × 10–5 × (250 – 40) or (1+ – (1+ =5 .......(i)
01 1t) 02 2t)
= 0.21 cm
Differentiating equation (i) w.r.t.t, we have
For steel rod s = t
01 – = 0
= 50 × 1.2 × 10–5 × (250 – 40) 1 02 2

= 0.13 cm
2
The total change = 01 = 02
b+ s 1
= 0.21 + 0.13
5
= 0.34 cm 1.7 10
= 5 02
As the rods are free to expand, so no stress will developed 1.1 10
anywhere.
www.crackjee.xyz
412 MECHANICS, HEAT, THERMODYNAMICS & WAVES
On differentiating equation (i), we get
or 01 = 1.545 02
......(i)
(2 1 cos ) d 2 + (2 2 cos ) d 1 – (2 1 2) sin d
= 2 1 (d 1) +2 2 (d 2) – 2 3 (d 3) .....(ii)
At t = 0
When temperature of the system is increased by T,
01 – 02 = 5 .......(ii) d = T; d = T ; and
1 1 1 2 2 1
After solving equations (i) and (ii), we get d 3= 3 2 T;
If T is small, then 1 = 2
01 = 14.17 cm,
= 3

= 9.17 cm Ans. and = 60°.


02

10. The volume of overflow gasoline Putting these values in above equations and simplifying, we get

V = V app t
3d
= V[ gas – st] t T = Ans.
2( 2 1)
= 75 [ 9.5 × 10–4 – 0.12 × ]10–4
× (30 – 10) 15. The change in length, = t
= 1.37 litre. Ans. 0.055 × 10–3 = 1 × 11× 10–6 × t
11. If is the function of temperature, then with small change in t = 5° C
temperature (dT), the length
Thusthe range of temperature = 20 C 5 C
L0 = L0 dT
= 15° C to 25° C Ans.
T 16. Suppose A be the cross – section of the each rod.
Thus L = L0 L0 dT Ans.
0 F
2A Steel
12. The change in height of liquid column
A Al F
1.28
h = liquid t 1.28 glass t
2 Change in length of steel rod
= +
1.28 st t F
= (4 10 5 ) 10 1.28 1 10 5
10
2 F 0
= + AY ......(i)
= 1.3 × 10–5 m. Ans. 0 s
a
13. By Pascal's law the pressure intensity at the bottom of left arm is
Change in length of aluminium rod
equal to the pressure intensity at the bottom of right arm. Thus
= –
ht t g = h0 0 g A t F

F 0
0 = – AY .....(ii)
or h0 ( 1 + t) × = h0 0 a
(1 t) 0 a

After simplifying, we get At equilibrium, st = A

F 0 F 0
ht h0 or + = – AY
= h0t Ans. 0 s 2 AYs 0 a
a

14. Suppose the length of the rods becomes 1, 2 and 3 when they 1 1
heat. If is the angle between A and B, then or F = A( a – s)
2Ys Ya
2 2 2
1 2 3
cos = A(
2 1 2 a s)
F
1 1
or 2 2 2 2 2Ys Ya
1 2 cos 1 2 3 ...(i)
www.crackjee.xyz
THERMOMETRY, EXPANSION AND CALORIMETRY 413
19. Suppose size of the aluminium from on the worky day be ×b
A( s) 0
From equation (i), =
a At 0° C, the length of aluminium frame
st 0 s
1 1 = (1 – × 40 )
2A Ys
2Ys Ya
= (1 – 24 × 10–6 × 40 )
The length of glass window at the same temperature
aYa 2 sYs = 30 (1 – × 40)
0
Ya 2Ys = 30 ( 1 – 9 × 10–6 × 40)
For no stress, their sizes must be equal and so
The length of the system = 0+ st
(1 – 24 × 10–6 × 40) = 30 (1 – 9 × 10–6 × 40)
2 sYs = 30.018 cm.
= aYa
0
Ya 2Ys
0 Similarly b = 20.012 cm Ans.
20. Using Pascal's law, we have
Pa + ghA – ghB + c ghc – DghD = Pa
aYa 2 sYs A B
= 0 1 Ans. Here =
Ya 2Ys A c
= 95
17. Time period of torsional pendulum and B = D

I = 5
T = 2 ( hA + hc) = ( hB + hD )
C 95 5

T 1 I 0
(52.8 49) = 0
(49 51)
= or
T 2 I 1 95 1 5

As I = kL 2 After simplifying, we get


I 2 L = 2 × 10–4 /°C
= Ans.
I L
21. Heat required to melt the ice completely
Due to small change in temperature T, = 450 × 0.5 × (20) + 450 × 80
L = L T = 40500 cal
I The heat available on stream
= 2 T = 50 × 540 + 50 × 1 × (100 – 0)
I
= 32000 cal.
T 1 It shows that steam will loose its entire heat and comes down to
and = 2 T
T 2 0°C. The ice will not melt completely and so equilibrium
= T temperature is 0°C. If m amount of ice, is melted, then
450 × 0.5 × 20 + m × 80 = 32000
= 2.4 × 10–5 × (45 – 5)
m = 343.75 g Ans.
= 9.6 0 × 10–4
22. If m amount steam is needed per hour, then
= 9.6 × 10–2 % Ans.
10 × 1× (90 – 20) = m × 540 + m × 1 × (100 – 90)
18. We know that v = r ( constant)
m = 1.27 kg Ans.
v r 23. The heat gained by iceH = 0.1 × 80 + 0.1 × 1 × (27 – 0)
=
v r = 10.7 kcal
With small change in temperature = 10.7 × 103 cal
T,
If m be the mass of the contains, then heat lost by it
r = r t
500 500
v r t msdT = m (A BT ) dT
= =
v r 300 300

= t
500
= 1.2 × 10–5 × ( 50 – 20 ) BT 2
= m AT 2
= 3.6 × 10–4 300
= 3.6 × 10–2 % Ans.
www.crackjee.xyz
414 MECHANICS, HEAT, THERMODYNAMICS & WAVES
If m amount of steam condenses, then
2 2
2 10 500 2 10 300 2 m × 536 + m × 1 × (100 – 5) = 1350
= m 100 500 100 300
2 2
m = 2.13 g. Ans.
= 21600 m 28. The heat available
Thus 21600 m = 10.7 × 103 H = mC T
m = 0.495 kg. Ans.
= 2.5 × 0.39 × (500 – 0)
24. Given, C = (2t 2 t ) 10 3 Cal/g°C. = 487.5 k cal
If m amount of ice is melted due to this heat, then
t2

The heat required H = mCdt 487.5


m = 1.45 kg. Ans.
t1 335

29. (a) The ball does not bounce back to the same height because
40
some of its kinetic energy is lost in collision, which will
= 100 (2t 2 t ) 10 3 dt convert into heat.
20
(b) The loss in kinetic energy
40 = mg(h1 – h2)
3
0.1 2t t2
= = 0.1 × 9.8 × (10 – 7)
3 2 20
= 2.94 J.
= 3790 Cal = 3.79 K cal. Ans. If T be the rise in temperature of the ball, then
25. Answer is the solution
0.1 × (0.11 × 103 × 4.2) T = 2.94
26. The heat required to raise the temperature of water.
H = 30 × 1 × (77 – 27) T = 0.064°C Ans.
= 1500 k cal / min 30. If v be the speed of the bullet, then
= 1500 × 103 × 4.2 = 63 × 105 J/min.
Thus fuel needed per minutes 1 2
0.75 × mv = mC T mL
2
5
63 10
= 157.5 g/ min Ans.
4 104 v2
0.75 = (0.03 4.2 103 ) (327 – 27) 6 4.2 103
27. The heat required 2
H = [10 × 80 + 10 ×1 ×(5 – 0)] v = 409.9 m/s Ans.
+ [100 × 1 × (5 – 0)]
= 1350 cal.
www.crackjee.xyz
KINETIC THEORY OF GASES 415
www.crackjee.xyz
416 MECHANICS, HEAT, THERMODYNAMICS & WAVES
6.1 INTRODUCTION
It can be simply understand that any gas occupies the volume of its container due to the
freedom of motion of its molecules. Pressure exerted by a gas is due to collisions of gas
molecules on the walls of its container. And temperature of a gas can be related to the
kinetic energy of these molecules. In the present chapter we shall relate volume, pressure
and temperature to the microscopic properties like speed and kinetic energy of gas
molecules.

6.2 GAS LAWS

Boyle’s law (1660)


It states that the volume of a given mass of a gas is inversely proportional to its
pressure, provided the temperature remain constant. That is
1
V
P
k
or V = PV = k
P

If Pi and Vi are the initial values of pressure and volume and Pf and Vf are the final values,
then
PiVi = Pf Vf
Boyle’s law is true for an ideal gas, it is obeyed approximately by real gases and is not a
fundamental law like Newton’s laws.
1
The following graphs shows the variation of P with V and P with for a given mass of
V
a gas at constant temperature.
Charle’s law (1787)
It states that at constant pressure, the volume of a given mass of a gas increases or
1
decreases by of volume at 0°C for each 1°C rise or fall of temperature.
273.15
If V0 is the volume of gas at 0 °C, then volume of the gas at t °C,
t 273.15 t
Vt = V0 1 = V0
273.15 273.15
As 273.15 + 0 = T0 and 273.15 + t = T
T
Vt = V0
T0
Vt V0
Fig. 6.1 or =
T T0

V
or = Constant or V T
T
Thus Charle’s law can be started in another way that at constant pressure, the volume of
given mass of a gas is directly proportional to its absolute temperature. The following
graph shows the variation of volume of given mass of a gas with temperature.
Gay-Lussac’s law or Pressure law
It states that if the volume remains constant, the pressure of a given mass of a gas
1
Fig. 6.2 increases or decreases by of its pressure at 0°C for each 1°C rise or fall in the
273.15
temperature.
If P0 is the pressure of a given mass of gas at 0°C, then its pressure at t°C
www.crackjee.xyz
KINETIC THEORY OF GASES 417

t (273.15 t )
Pt = P0 1 = P0
273.15 273.15

T
= P0
T0

Pt P0
or = T0
T
P
or = Constant or P T
T

Thus Gay-Lussac’s law can be stated in another form that at constant volume, the
pressure of a given mass of a gas is directly proportional to the absolute temperature.
The following figure shows the variation of pressure of a given mass of a gas with
temperature.

Ideal / perfect gas equation


If we relate P, V and T all together by gas laws, we will get

PV = nRT
Fig. 6.3
where n is the number of moles and R is called universal gas constant. This equation is
called ideal gas equation.
More about ideal gas equation
1. In deriving ideal gas equation the following assumptions are used :
(i) The size of molecules are negligibly small in comparison to the size of the
container.
(ii) Intermolecular forces are negligibly small.
m
2. If m is the mass and M is the molecular mass of the gas, then, n =
M
m
PV = RT
M
3. For 1 mole of a gas, n = 1
PV = RT
4. If V is the volume of gas of mass m, then
m
P V = RT
M
( m/ V)
or P = RT
M
RT m
=
M V
µ
5. If µ is the number of molecules and N is the Avogadro number, then n =
N
µ
PV = RT
N
R
= µ T
N
R
Here = k, is called Boltzmann’s constant. Thus we can write
N
PV = µkT
6. For one molecule , µ = 1, PV = kT
www.crackjee.xyz
418 MECHANICS, HEAT, THERMODYNAMICS & WAVES
Universal gas constant R
From ideal gas equation, we can write
PV
R =
nT
Work
=
number of moles × temperature
SI unit of R = J/mol-K
Numerical value of R
For one mole of a gas at STP,
P0V0
R =
T0
Here P0 = 1.013 × 105 N/m2, V0 = 22.4 litre = 22.4 × 10–3 m3 and T0 = 273 + 0 = 273 K

(1.013 105 ) (22.4 10 3 )


R =
273
= 8.31 J/mol-K
4.31
=
4.2
= 1.98 cal/mol-K
Numerical value of k
R
We know that, k =
N
8.31
=
6.02 1023
= 1.38 × 10–23 J/K
6.3 IDEAL GAS
Fig. 6.4 We have already used the phrase ‘ideal or perfect gas’ . The question arises that; what is
an ideal gas ? The gas law PV = nRT that governs macroscopic properties gives the
answer. Although there is no such thing in nature which is truely ideal gas, all gases
approach the ideal state at low pressure and high temperature.

Note:
For practical purposes, real gases below a pressure of two atmosphere can be
considered as ideal gases. A saturated vapour in equilibrium with own liquid can be
considered to have the properties of an ideal gas.

Fig. 6.5 Deviation from ideal behaviour


PV
(i) The ratio is plotted as a function of P and T. For an ideal gas this quantity is
nT
constant and equal to R, but for real gases it varies with temperature. At high
temperature and low pressure this quantity approaches the value R.

(ii) Figure shows the comparison between the theoretical curves predicated by Boyle’s
law and experimental curve at two different temperatures.

(iii) Figure shows the comparison between the theoretical curve predicted by Charle’s
Fig. 6.6 law and the experimental curve at two different temperature.
www.crackjee.xyz
KINETIC THEORY OF GASES 419
Vander Waal's equation
The ideal gas equation PV = nRT, can be used for real gases at high temperature and low
pressure at which intermolecular forces are negligibly small. Vander waal modified this
equation so that it can be used for real gases at a wide range of temperature and pressure.
He made following corrections :
(i) Volume correction : Due to finite size of molecule, some part of volume of gas is
occupied by the molecules. So the space available for the motion of molecules of
gas will be slightly less (b) than the volume of gas. Hence effective volume becomes
(V – b).
(ii) Pressure correction : Due to intermolecular forces, molecules do not exert that
force on the walls of the container as they would have exerted in absence of
intermolecular forces. The effective pressure as calculated is ; P a .
Vander Waal’s equation for 1 mole of a gas, V2

a
P (V b) = RT
V2
for n moles of a gas,
an2
P (V nb) = nRT
V2
Critical constants
The critical temperature and the corresponding values of pressure and volume at the
critical point are called the critical constants. At the critical point, the rate of change of
pressure with volume (dP/dV) is zero. This point is called the point of inflexion.
According to Vander Waal’s equation
a
P (V b) = RT
V2

RT a
P = … (1)
V b V2
Differentiating equation (1) w.r.t. V, we have
dP RT 2a
= 2
… (2)
dV (V b) V3

dP
At the critical point 0 , T = Tc and V= Vc
dV
RTc 2a
2 = 0
(Vc b) Vc3

2a RTc
or 3 = … (3)
Vc (Vc b)2
Differentiating equation (2), we have
d 2P 2 RT 6a
= 3
dV 2 (V b) V4

d 2P
At the critical point is also zero. Putting
dV 2
T = Tc and V = Vc
2 RTc 6a
3 = 0 … (4)
(Vc b) Vc 4
www.crackjee.xyz
420 MECHANICS, HEAT, THERMODYNAMICS & WAVES
Solving equations (3) and (4), we get
Vc = 3b
Substituting the value of Vc in equation (3), we have
2a RTc
3 =
27b 4b2
8a
or Tc =
27bR
Substituting values of Vc and Tc in equation (1), we have
R 8a a
Pc = 27 Rb(2b) 9b2

a
or Pc =
27b2

Variation of atmospheric pressure with height


Suppose P and (P + dP) are the pressures at h and h + dh respectively, then
–dP = gdh …(i)
Let V be the volume of m mass of the gas, then density of gas
m
=
V
m
Also P V = RT
M
( m / V ) RT
or P =
M
RT
P =
M
PM
or
RT
Substituting value of P in equation (i), we get
PM
dP = gdh
RT
dP Mg
or = dh … (ii)
P RT
Integrating both sides of equation (ii), we get
P h
Fig. 6.7 dP Mg
= dh
P RT
P0 0

P Mg
or ln P P = | h |0h
0 RT
P Mgh
or ln =
P0 RT
Mgh
P =
P0 e RT

Here P0 is the pressure at the ground level, h = 0. Also we have assume that the
temperature of the atmosphere does not change with height.
www.crackjee.xyz
KINETIC THEORY OF GASES 421
Ex. 1 A closed container of volume 0.02 m 3 contains Now by Boyle’s law, we have
a mixture of neon and argon gases at 27 °C temperature and P1V1 = P2V2
1.0 × 105 N/m2 pressure. If the gram-molecular weight of neon and P × 40 A = (P – 22) × 58 A
argon are 20 and 40 respectively, find the masses of the individual or 18 P = 22 × 58
gases in the container. Assuming them to be ideal ( R = 8.314 J/mol-
P = 70.9 cm Ans.
K). Total mass of the mixture is 28 g.
Ex. 3 An ideal monoatomic gas is confined in a cylinder by a
Sol. spring-loaded piston of cross-section 8.0 × 10 –3 m2. Initially the
Let m be the mass of the neon gas, then mass of argon will be 28 – m. gas is at 300 K and occupies a volume of 2.4 × 10–3 m3 and the
spring is in the relaxed state (fig. 6.9). The gas is heated by a small
Number of moles of neon,
heater untill the piston moves out slowly by 0.1 m. Calculate the
m final temperature of the gas. The force constant of the spring is
n1 = 8000 N/m, and the atmospheric pressure is 1.0 ×10 5N/m2. The
20
cylinder and the piston are thermally insulated. The piston and
Number of moles of argon,
the spring are massless and there is no friction between the piston
and the cylinder. Neglect any heat-loss through the lead wires of
28 m
n2 = the heater. The heat capacity of the heater coil is negligible.
40
Now by ideal gas equation, we have
PV = nRT

m 28 m
Here n = n1 + n2 = , P = 1.0 × 105 N/m2
20 40
T = 273 + 27 = 300 K, V = 0.02 m3

m (28 m)
(1.0 105 ) 0.02 = 8.314 300 Fig. 6.9
20 40
After solving, we get, m = 4.07 g
Sol.
Mass of neon gas = 4.07 g Given; initial volume of the gas,
V1 = 2.4 × 10–3 m3
Mass of argon gas = 28 – m
The spring is in relaxed state, so,
= 28 – 4.07 = 23.93 g Ans.
P1 = atmospheric pressure
Ex. 2 A narrow uniform glass tube 80 cm long and open at both
= 1.0 × 105 N/m2
ends is half immersed in mercury. Then, the top of the tube is
closed and it is taken out of mercury. A column of mercury 22 cm T 1 = 300 K
long then remains in the tube. What is the atmospheric pressure? When piston moves by 0.1 m, the final volume of gas becomes,
V2 = V1 + Ax
Sol.
= 2.4 × 10–3 + 8.0 × 10–3 × 0.1
Suppose A is the area of cross-section of the tube and P is the atmospheric
pressure. When half tube is inside mercury then volume of air in the tube. = 3.2 × 10–3 m3

V1 = A × 40 = 40 A cm3 kx
Final pressure, P2 = P1
A
P1 = P
5 8000 0.1
= 1.0 10 3
8.0 10
= 2.0 × 105 N/m2
Let T2 be the final temperature of the gas, then by gas law, we have

PV
1 1 P2V2
=
T1 T2
Fig. 6.8
P2V2
When the tube is taken out of mercury, the volume of air, or T2 = T1
PV
1 1
V2 = A × 58 = 58A
If P2 is the pressure of air in the tube, then (2.0 105 ) (3.2 10 3 )
P2 + 22 =P = 300
(1.0 105 ) (2.4 10 3 )
or P2 =P – 22
T 2 = 800 K. Ans.
www.crackjee.xyz
422 MECHANICS, HEAT, THERMODYNAMICS & WAVES
Ex. 4 A mercury manometer consists of two unequal arms of Sol.
equal cross-section and length 100 cm and 50 cm. The two open Mass of molecular nitrogen
ends are sealed with air in the tube at a pressure of 80 cm of 70
mercury. Some amount of mercury is now introduced in the = 1.4 = 0.98 g
100
manometer through the stopcock connected to it. If mercury rises
in the shorter tube to a length 10 cm in steady state, find the length 30
Mass of atomic nitrogen = 1.4 = 0.42 g
of the mercury column risen in the longer tube. 100
Sol. Number of moles of molecular nitrogen,
For shorter arm;
Initial volume of air = A × 50 cm3
0.98
n1 = 0.035
28
Number of moles of atomic nitrogen
0.42
n2 = 0.03
14
The pressure of the gas = pressure exerted by molecular nitrogen +
pressure exerted by atomic nitrogen
i.e., P = P1 + P2

n1 RT n2 RT ( n1 n2 ) RT
= =
V V V
Fig. 6.10 (0.035 0.03) 8.31 1800
When mercury is introduced in the tube, the volume of air remains = 3
5 10
= A × 40 cm3
= 1.94 × 105 N/m2 Ans.
Let P1 be the pressure of the air in shorter arm, then by PV = constant,
we have Ex. 6 A horizontal tube of length closed at both ends contains
80 × (A × 50) = P1 × (A × 40) an ideal gas of molecular weight M. The tube is rotated at a constant
P1 = 100 cm angular velocity about a vertical axis passing through an end.
For longer arm; Assuming the temperature to be uniform and constant, show that
Initial volume of air = A × 100 cm3 M 2 2
P2 =
Let 0 be the length of mercury, then volume of air remains P1e 2 RT
where P2 and P1 denote the pressures at the free end and the fixed
= A × (100 – 0 ) end respectively.
Let P2 be the pressure of the air in the longer air, then by Pascal’s law Sol.
PA = PB Consider an element of the gas of thickness dx at a distance x from the
axis of rotation. Let P and P + dP are the pressures on left and right side
0 P2 = 10 P1 of the element, then net force acting on the element of gas of mass dm
P2 = (10 P1 ) 0 = (10 100) 0

= (110 0 ) cm
Now by PV = constant, we have
(110 0) A(100 0)
Fig. 6.11
= 80 ( A 100) = ( F dF ) F
= ( P dP) A PA
or (110 0 )(100 0) =(dP)A
= 8000 As the element is moving in a circle so by Newton’s second law
(dP ) A = (dm) 2 … (i)
2
210 0 3000 = 0 x
0
Using PV = nRT for the element, we have
0 = 15.5 0 15.5 cm Ans.
dm
P ( Adx) = RT
Ex. 5 A vessel of volume V = 5.0 litre contains 1.4 g of nitrogen M
at temperature, T = 1800 K. Find the pressure of the gas if 30% of MPA
its molecules are dissociated into atoms at this temperature. or dm = dx
RT
www.crackjee.xyz
KINETIC THEORY OF GASES 423
Substituting the value of dm in equation (i), we get
P2
MPA 2 dP M 2
(dP ) A = xdx xdx
RT P = RT
P1 0
MP 2 M 2 x2
or dP = xdx ln P P2
P
RT = RT 2
1
0
2 P 2 2
dP M ln 2 M
or = xdx … (ii) or P1 =
P RT 2 RT
Integrating both sides of equation (ii), we have M 2 2
P2 = Pe 2 RT
1

6.4 KINETIC THEORY OF GASES


Kinetic theory of gases was developed by Maxwell. He made the following assumptions:
1. The gas is composed of small indivisible large number of particles called molecules.
The properties of the individual molecules are same as that of the gas as a whole.
2. The size of molecule is negligibly small in comparison to the average distance
between the molecules.
3. Intermolecular forces between the molecules are negligibly small. Thus all internal
energy of the gas is kinetic.
4. Effect of gravity on them is negligibly small and Newton laws are valid for their
motion.
5. The molecules are in a state of continuous random motion, moving in all directions
with all possible velocities.
6. The collision between the molecules and with the walls of the container are perfectly
elastic. Between two collision a molecule moves in a straight path with a uniform
velocity. The average distance moved by a molecule between two successive
collisions is called mean free path.
7. The time of collision is negligibly small in comparison to the time of the free path
between the molecules.

Note:
It is not possible to define macro property like pressure by taking small number of
molecules (say two) in the container.
Pressure and temperature : A molecular view
Pressure exerted by a gas
The molecules of a gas are in a state of continuous random motion. They collide with one
another and also with the walls of the container. Due to collisions with the walls, the
momentum of molecules will change. As collisions of molecules with the walls of the
container are elastic, so an equal amount of momentum is transferred to the walls of the
container. According to Newton’s second law of motion, the rate of transfer of momentum
to the wall is equal to the force exerted on the wall. The force exerted per unit area of the
wall is the pressure of the gas.
Expression of pressure
Consider an ideal gas enclosed in a cubical container of volume V. Suppose the sides of Fig. 6.12
the container are parallel to the co-ordinate axes and held at temperature T.
Now consider a typical molecule of mass m, moving with velocity v (vx , v y , vz ) which is
about to collide with the wall (see figure 6.12). As the collision is elastic, the molecule will
rebound with the same speed. The velocity of molecule after collision become
( vx , v y , vz ) . This means that the only change in the momentum is then along the x-axis,
and that change is
mvx mv x = 2mvx
www.crackjee.xyz
424 MECHANICS, HEAT, THERMODYNAMICS & WAVES
By the conservation of momentum, the momentum delivered P to the wall by the
molecules is 2mvx.
If t be the time, the molecule takes to travel between the parallel walls at speed vx, then
2L
t = .
vx
The rate at which momentum is delivered to the wall by the single molecule is
P 2 mvx mvx 2
= =
t 2L L
vx
If n be the total number of molecules is the container then total force exerted on the wall
mv 2x mvx2 mv x2
F = 1 2 n
.......
L L L
2
nm v x1 v x2 .... v x2
2 n
=
L n

vx2 vx2 .... vx2


Here 1 2 n is the average value of the square of the x-components, which
n
can be written as v x2 .
nm 2
F = vx … (1)
L
The pressure of the gas
F (nmvx2 ) / L
P = =
A L2
nmv x2 nmv x2
= = … (2)
L3 V

For any molecule, v 2 v x2 v 2y v z2 . As there are many molecules and they are moving
in random directions, so the average values of the squares of their velocity components
v2
are equal. Thus we have v 2 3v 2x or vx2 = . Equation (2) becomes
3
nmv 2
P = … (3)
3V

The square root of v 2 is called the root-mean square speed of the molecules and can be
denoted by vrms. Thus we can write
2
nmvrms
P = … (4)
3V
nm
As nm = mass of the gas, so = density of gas. Thus equation (4) becomes
V
1 2
P = vrms … (5)
3
6.5 TRANSLATIONAL KINETIC ENERGY
Kinetic interpretation of temperature
Consider one mole of a gas, and let P, V, T and M be the pressure, volume, temperature
and molecular mass of the gas respectively, then
www.crackjee.xyz
KINETIC THEORY OF GASES 425

Nm 2
P = vrms (n = N)
3V
2
Mvrms
= (Nm = M) … (1)
3V

2 1 2
or PV = Mvrms
3 2

1 2
But Mvrms is the average translational kinetic energy E of one mole of the gas
2
2E
PV =
3
For one mole of an ideal gas, we havePV = RT
2E
RT =
3
3
or E =RT per mole … (2)
2
If N is the Avogadro’s number, then average kinetic energy per molecule is given by
3 RT
e =
2 N
R
As = k, Boltzmann’s constant
N
3
e =kT per molecules … (3)
2
Thus the mean kinetic energy is proportional to the absolute temperature of the gas. It
does not depend on the pressure, volume and nature of the gas.
Again from equation (1)
3PV
v rms =
M

3RT
or v rms = … (4)
M
i.e., v rms T
From above treatment, it can be concluded that the temperature of a gas is the measure
of the average kinetic energy of its molecules. This is what we mean by the kinetic
interpretation of temperature.
Also,at T = 0, vrms = 0, so we can define the absolute zero is the temperature at which
motion of all the molecules stop.

6.6 GRAHAM’S LAW OF DIFFUSION


Let us consider two gases of densities 1 and 2 and their respective rms speeds are v1
and v2.
1 2
Pressure exerted by gas 1, P1 = 1v1
3

1 2
and pressure exerted by gas 2, P2 = 2 v2
3
www.crackjee.xyz
426 MECHANICS, HEAT, THERMODYNAMICS & WAVES
When gases diffuse one another and steady condition is reached, we have
P1 = P2
1 2 1 2
or 1v1 = 2 v2
3 3

v1 2
=
v2 1
As we know that rate of diffusion is directly proportional to its rms speed. Thus if r1 and
r2 be the respective rates of diffusion, then
r1 v1 2
= =
r2 v2 1
Thus the rate of diffusion of a gas is inversely proportional to the square root of its
density.
6.7 DALTON’S LAW OF PARTIAL PRESSURE
Consider number of gases in a container of volume V. The pressure exerted by gases
when taken in the container separately are ;

1 n1m1v12 1 n2 m2 v22
P1 = , P2 =, ,......
3 V 3 V
When gases are taken together in the container, the total pressure
= P1 + P2 + .....

1 n1m1v12 1 n2 m2 v2 2
= .........
3 V 3 V
As the temperature of all the gases in the mixture is the same, therefore
1 1 1
m1v12 = m2v22 ...........= mv 2 (say)
2 2 2
or m1v12 = m2 v2 2 .............. = mv2

1
P1 P2 .... = (n1 n2 ...)mv 2
3V
where n1 n2 = n, , is the total number of molecules in the container..

nmv 2
P1 P2 ....... =
3V
or P1 + P2 + ......... = P
Thus the total pressure exerted by number of nonreacting gases when taken together
in a container is equal to the sum of their partial pressure.
6.8 THE DISTRIBUTION OF MOLECULAR SPEED
The root-mean-square speed vrms gives a general idea of the molecular speed for a gas
at a given temperature. This does not mean that speed of each molecule is vrms. Many of
the molecules have speed less than vrms and many have speed greater than vrms. Maxwell
derived an equation giving the distribution of molecules in different speed. If dN represents
the number of molecules with speeds between v and v + dv, then

3/ 2 mv 2
m 2
dN = 4 N v e 2 kT dv
2 kT

dN
Figure shows the plot of against v at any temperature T.
dv
www.crackjee.xyz
KINETIC THEORY OF GASES 427
The important features of the speed distribution curve are as follows :
(i) At any temperature, the speed of molecules varies from zero to infinity.
(ii) The number of molecules with speeds v to v + dv is equal to the area of the strip
shown.
dN
(iii) The speed at which is maximum, is called most probable speed vmp. Its value
dv
is given by
2kT
vmp =
m
Different types of speeds of gas molecules
1. Root mean square speed . It is defined as the square root of mean of squares of the
speed of different molecules, i.e.,

v12 v22 ...... vn2


v rms =
n
According to kinetic theory of gases,

3RT 3kT
vrms = =
M m
when M is the mass of one mole while m is the mass of one molecule.
(i) At room temperature the rms speed of hydrogen gas will be

3RT 3 8.31 300 Fig. 6.13


vH = = 1920 m / s
3
M 2 10

RT
(ii) As speed of sound in a gas v =
M
v
vrms
=
3

v = vrms
3
2. Average speed. It is defined as the arithmetic mean of the speeds of the molecules
of a gas at a given temperature. If v1, v2, ......, vn are the speeds of the n gas
molecules, then the average speed v is given by
v1 v2 ..... vn
vav =
n
By Maxwell speed distribution law, it can be shown that

8RT 8kT
vav = = 0.92 vrms
M m
3. Most probable speed It is the speed with which the largest number of molecules in
a gas are moving. At any temperature it is given by
2RT
vmp =
M
= 0.816 vrms
It can be seen from above that
vrms vav vmp
www.crackjee.xyz
428 MECHANICS, HEAT, THERMODYNAMICS & WAVES
Ex. 7 At what temperature will the average velocity of oxygen
molecule be sufficient so as to escape from the earth? Escape velocity
from earth is 11.0 km/s and mass of one molecule of oxygen is
5.34 × 10–26 kg. Boltzmann constant k = 1.38 × 1023 J/molecule - K
Sol.
The average velocity of molecule at any temperature is given by

2 kT
vav =
m
For the molecule to just escape from the earth
Fig. 6.14
vav = v e
Let x-axis indicates the direction of normal then change in momentum
2kT Px = – mv cos 45 – mv cos 45
or = ve
m
= –2mv cos 45
mve2 2mv
T = 2mv
2k =
2
26
5.34 10 (11.0 103 )2 and Py = mv sin 45 ( mv sin 45 ) = 0
= 23
2 1.38 10
The momentum delivered to the wall = 2mv
= 2.34 × 105 K Ans.
Thus pressure exerted,
Ex. 8 N molecules each of mass m of gas A and 2N molecules F Px / t Px n 2mnv
each of mass 2m of gas B are contained in the same vessel which is P = = = =
A A A A
maintained at a temperature T. The mean square of the velocity of
the molecules of B type is denoted by v2 and the mean square of the where, n = 1023 per second, A = 2 10 4 cm 2 , v = 103 m/s
x-component of the velocity of A type is denoted by 2, then what is
2 27
2 (3.32 10 ) (10 23 ) 103
the ratio of =? P =
v2 2 10 4

Sol.
3kT = 2.347 103 N / m2 Ans.
The mean square velocity of gas molecule is given by v 2 = .
m
3kT Ex. 10 16g of oxygen and 14 g of nitrogen are mixed in an
For gas A, v 2A = m …(i) enclosure of volume 10 litre and temperature 27°C. Calculate the
pressure exerted by the mixture.
For a gas molecule, v 2 = v 2x v 2y v 2z
Sol. We know that
= 3vx2 (v 2x v 2y v z2 ) PV = nRT
v2 m
or PV = RT
or vx2 = M
3
From equation (i), we get mRT
P =
MV
v 2A 3kT / m kT
vx2 = =
3
= …(ii) Pressure exerted by oxygen;
3 m
3kT 3kT mRT 16 8.31 300
For gas B, vB2 = v2 = m = 2 m …(iii) Poxygen = =
B MV 32 10 10 3
Dividing equation (ii) by (iii), we get = 1.246 × 104 N/m2
2
kT / m 2 Pressure exerted by nitrogen;
2 = = Ans.
v 3kT / 2m 3 mRT 14 8.31 300
Pnitrogen = 3
Ex. 9 The mass of hydrogen molecules is 3.32 × kg. If 10 –27 MV 28 10 10
1023 hydrogen molecules strike a fixed wall of area 2 cm2 per second = 1.246 × 105 N/m2
at an angle of 45° to the normal and rebound elastically with a Now by Dalton’s law of partial pressure, the total pressure
speed of 103m/s, calculate the pressure on the wall. P = Poxygen + Pnitrogen
Sol.
= 1.246 × 105 + 1.246 × 105
As the collision of molecule with the wall is elastic, so the magnitude of
momentum of molecule before collision will equal to after collision. = 2.493 × 105 N/m2 Ans.
www.crackjee.xyz
KINETIC THEORY OF GASES 429

Ex. 11 Two perfect gases at absolute temperatures T1 and T 2 (iii) The root-mean square speed is given by
are mixed. There is no loss of energy. Find the temperature of 1/ 2
mixture if masses of molecules are m1 and m2 and the number of n1v12 n2v22 n3v32 n4v42 n5v52
vrms =
molecules in the gases are n1 and n 2 respectively. n1 n2 n3 n4 n5
Sol.
3 1/ 2
The average kinetic energy of a gas molecule = kT . 2 12 4 22 9 3 2 5 4 2 3 52
2 =
The total kinetic energy before mixing the two gases 2 4 9 5 3

3 3
= n1 kT1 n2 kT2 1/ 2
2 2 254
= = 3.32 m/s
If T is the temperature of the mixture, then the mean kinetic energy after 23
mixing Ex. 13 During an experiment an ideal gas is found to obey an
3
= (n1 + n2) × kT additional law VP2 = constant. The gas is initially at temperature
2 T and volume V. When it expands to a volume 2V, calculate the
As there is no loss of energy, so
resulting temperature.
3 3 3
( n1 n2 )
2
kT = n1
2
kT1 n2
2
kT2 Sol.
The ideal gas equation
n1T1 n2T2
T = PV = nRT … (i)
n1 n2
The gas obeys an additional law
Ex. 12 You are given the following group of particles, n i
VP2 = C (constant) … (ii)
representing the number of molecules with speed vi
ni vi (m/s) From equation (i),
2 1.0 nRT
4 2.0 P =
9 3.0 V
5 4.0 Substituting this value in equation (ii), we get
3 5.0
Calculate (i) most probable speed (ii) average speed (iii) root-mean- 2
nRT
square speed. V = C
V
Sol.
(i) The most probable speed is 3.0 m/s, by which the largest molecules
are moving at any time. T2 C
or = = k (new constant)
(ii) The average speed is given by V n2R 2
n1v1 n2v2 n3v3 n4v4 n5v5 If T' is the temperature corresponding to volume 2V, then
vav = n1 n2 n3 n4 n5
T2 T '2
2 1 4 2 9 3 5 4 3 5 =
= V 2V
2 4 9 5 3
After simplifying, we get
72
= = 3.13 m/s T' = 2T Ans.
23

6.9 DEGREES OF FREEDOM


The degrees of freedom are the minimum number of informations required to know about
the system.
For thermodynamic system (moving particles), these are the total number of independent
terms of energy. These are :
The independent possible motions are translational, rotational and vibrational, so there
are three types of degrees of freedom. These are :
(i) Translational degrees of freedom : The maximum number of translational degrees of
1 2 1 2 1 2
freedom can be three. These are mvx , mv y , mvz .
2 2 2
(ii) Rotational degrees of freedom : The maximum number of rotational degrees of
1 2 1 2 1 2
freedom can be three . These are Ix x, Iy y, Iz z.
2 2 2
www.crackjee.xyz
430 MECHANICS, HEAT, THERMODYNAMICS & WAVES
(iii) Vibrational degrees of freedom : Their numbers depend on atoms in the molecule
and their arrangement. These degrees of freedom are consider at a very high
temperature.

Note:
At room temperature only translational and rotational degrees of freedom are taken
into account.

General expression of degrees of freedom f


f = 3N – k
where, N = number of particles in the system (number of atoms in a molecule)
k = number of independent relations between the particles.
1. Monoatomic gas : It has assumed that the molecules of a gas are negligible in size,
so moment of inertia and hence rotational kinetic energy of monoatomic gas
molecules about the axis passes through itself will be zero.
The degrees of freedom of monoatomic gas molecules are due to three independent
1 2
Fig. 6.15 translational motions along x, y, and z-axis. The degree of freedoms are mvx ,
2
1 2 1 2
mv y , mvz .
2 2
2. Diatomic gas : In diatomic gases the molecules are assumed to be in the shape of
dumbbells; two atoms of negligible size at some separation. In addition to
translational motion, the molecule, can rotate about an axis, so the degrees of
freedom of diatomic gas molecules are due to translation and due to rotation both.
If the line joining the two atoms (particles) is taken as the z-axis, then moment of
inertia and hence rotational kinetic energy about z-axis becomes zero. The molecule
has three degrees of freedom of translation and two degrees of freedom of rotation.
1 2 1 2 1 2 1 2 1 2
These are : mv x , mv y , mv z , I x x, Iy y
2 2 2 2 2

Note:
If vibrational degrees of freedom are taken into account, then total number of degrees
of freedom of diatomic molecule becomes seven. These are :

1 2 1 2 1 2 1 1 1 1
mv x , mv y , mvz , I x 2x , I y 2y , µv 2 , kr 2 .
2 2 2 2 2 2 2
1 2
Here µv corresponds to kinetic energy of vibration (µ is the reduced mass) and
2
1 2
kr corresponds to potential energy of vibration (k is the force constant, r is the
2
separation between the atoms).

3. Triatomic or polyatomic gas : A non-linear molecule has non zero moment of


inertia about any axis, so there are three translational and three rotational degrees
of freedom. Total number of degrees of freedom are six. These are :

Fig. 6.16 1 2 1 2 1 2 1 2 1 2 1 2
mv x , mv y , mv z , I x x, Iy y, Iz z.
2 2 2 2 2 2
www.crackjee.xyz
KINETIC THEORY OF GASES 431
Degrees of freedom of different gases

Atomicity of gas Example Figure Degrees of freedom f


Monoatomic He, Ne, Ar 3 translational
Diatomic H2, N2, O2, Co 3 translational + 2 rotational = 5
Triatomic
H2O, SO2 3 translational + 3 rotational = 6
(non-linear)
Ex. 14 What are the degrees of freedom in the following cases. Sol.
(i) A particle moves along a straight line. (i) One, translational.
(ii) A particle is projected in space. (ii) Two, translational
(iii) Motion of fan. (iii) One, rotational.
(iv) Motion of ant. (iv) Two, translational.
(v) Motion of mosquito or fly (no rotation). (v) Three, translational.

6.10 LAW OF EQUIPARTITION OF ENERGY


According to this law, for a system in thermal equilibrium, the total energy of a dynamic
system is equally distributed among its various degrees of freedom. The energy
kT RT
associated with each degree of freedom is per molecule or per mole.
2 2
CV, CP and for different gases
1. Monoatomic gas
Degrees of freedom f = 3
RT 3RT
Kinetic energy per mole E = U=3 =
2 2
Molar specific heat at constant volume
U 3R
CV = =
T 2
Molar specific heat at constant pressure
5R
C P = CV R
2
CP 5R / 2
Ratio of specific heats = =
CV 3R / 2
5
= = 1.67
3
2. Diatomic Gas
Degree of freedom f = 5
RT 5RT
Kinetic energy per mole U = 5 =
2 2
Molar specific heat at constant volume
U 5R
CV = =
T 2
Molar specific heat at constant pressure
7R
C P = CV R =
2
CP 7R / 2
Ratio of specific heats = =
CV 5R / 2
7
= = 1.4
5
www.crackjee.xyz
432 MECHANICS, HEAT, THERMODYNAMICS & WAVES
3. Triatomic or polyatomic
Degrees of freedom f = 6
RT
Kinetic energy per mole U = 6 = 3RT
2
Molar specific heat at constant volume
U
CV = = 3R
T
Molar specific heat at constant pressure
C P = CV + R = 4R
CP 4R
Ratio of specific heats = =
CV 3R
4
= = 1.33
3
For a molecule with f degrees of freedom
RT fRT
Kinetic energy per mole U = f× =
2 2
Molar specific heat at constant volume
U fR
CV = =
T 2
Molar specific heat at constant pressure
fR
C P = CV + R = R
2
f
= R 1
2
CP R f /2 1
Ratio of specific heats = CV
=
fR
2
2
or = 1
f
6.11 MEAN FREE PATH
The mean free path of a gas molecule may be defined as the average distance travelled
by the molecule between two successive collisions.
Fig. 6.17 Let 1 , 2 , ....., n be the distances travelled by a gas molecule during n collisions, then
the mean free path of gas molecule is given by
1 ...... n
2
=
n
During the collision, a molecule of a gas moves in a straight line with constant velocity.
The statistical study of heat gives the mean free path as following :
1
=
2 nd 2
where d is the diameter of molecule, n is the number molecules per unit volume
If we take one mole of a gas, then
N N P P
n = = = =
V RT R kT
T
P N

kT
Fig. 6.18 =
2 d 2P
www.crackjee.xyz
KINETIC THEORY OF GASES 433
Ex. 15 The temperature of the walls of a vessel containing a
1.37 10 23
290
gas at a temperature T is T wall. In which case is the pressure = 10 2
exerted by the gas on the vessel walls higher : when the vessel 2 (2 10 ) (2 1.013 105 )
walls are colder than the gas (Twall < T) or when they are warmer = 1.1 × 10–7 m
than the gas (Twall > T)? Root mean square speed of nitrogen molecules
Sol.
3RT
If the temperature of walls of the vessel Twall is equal to the temperature vrms =
T of the gas, then a molecule striking the wall changes the normal M
component Px to its momentum by – Px. So the change in the momentum
will be 2Px. When T wall > T, the gas is heated. This means that gas 3 8.31 290
= 3 = 5.1 10 2 m / s
molecules bounce off the wall at a higher velocity than that at which they 28 10
impinge the wall, and hence a higher momentum. As a result, the change The time interval between two successive collisions
in momentum will be larger than 2Px. If, however, Twall < T, the gas is
cooled, i.e., gas molecules bounce off the wall with a smaller momentum 1.1 10 7
and hence change in momentum will be smaller than 2Px. Thus the pressure t = v = = 2.2 10 10
s
exerted by the gas on the walls is higher when the walls are warmer.
rms 5.1 102
The frequency of collision
Ex. 16 Estimate the mean free path and collision frequency of 1
nitrogen molecule in a cylinder containing nitrogen at 2.0 atm and 1 vrms
n = = ( /v ) =
temperature 17°C. Take the radius of a nitrogen molecule to be t rms
roughly 1.0° Å. Compare the collision time with the time the 5.1 10 2
molecule moves freely between two successive collisions. = 7 = 4.6 × 109 Hz
(Molecular mass of N2 = 28.0g) 1.1 10
Time taken for the collision
Sol.
Here T = 273 + 17 = 290 K diameter of molecule
=
d = 2 × 1.0 × 10–10 m v rms
P = 2.0 atm = 2 × 1.013 × 105 N/m2
10
k = 1.37 × 10–23 J/K 2.0 10
= = 4 × 10–13 s
5.1 102
kT
Mean free path, = The time between two successive collisions is nearly 500 times the time
2 d 2P taken for a collision.

6.12 PHASES AND PHASE DIAGRAMS


Isotherm of water : A graph plotted between the pressure and volume of a system at
constant temperature is called an isotherm. Figure 6.19 shows isotherms for water in the
temperature range 350°C to 380°C.
From the figure it can be concluded that :
(i) At 350°C, part AB of isotherm represents vapour phase of water. In this region
Boyle’s law is obeyed (PV = constant). With increase in pressure some part of
vapour condenses into water, so part BC represents the transition phase. When
entire vapour changes into water, there is no appreciable change in volume with
pressure (part CD). Fig. 6.19
(ii) With the increase in temperature, liquification becomes difficult, and so liquid-
vapour region becomes shorter. At 374.1°C, it reduces to point H, called critical
point and the corresponding temperature is called critical temperature.
(iii) The state above critical temperature is called gaseous phase.
Pressure-temperature phase diagram for water
(i) In the space above the steam-line and right of ice-line, water exists in liquid phase.
(ii) In the space below the steam line and right of hoar frost line, water exists in gaseous
phase (steam).
(iii) In the space above the hoar-frost line and on left of ice-line, water exists in solid
Fig. 6.20
phase as ice.
Fig. 6.20 shows the P-T phase diagram
Triple point : It is the point on P-T diagram at which all the three phases of a substance
for water. From the graph it can be
can co-exist in equilibrium with each other. For water it is 273.16 K at 4.6 mm of Hg. concluded that ;
www.crackjee.xyz
434 MECHANICS, HEAT, THERMODYNAMICS & WAVES
Ex. 17 The volume vs. temperature T graphs for a certain
amount of a perfect gas are at two pressures P1 and P2 are shown in
figure 6.21. Which one is greater P1 or P2?
Sol. For a perfect gas
PV = nRT
V nR constant
= =
T P P
Thus slope of V–T is inversely proportional to pressure.
As slope of 1 is smaller, so P1 P2 .
Fig. 6.23
PB PA
From the figure, it is clear that TB TA

VA VB .
Ex. 20 An electric bulb of volume 250 cm3 was sealed off during
manufacture at a pressure of 10–3mm of mercury at 27°C. Compute
the number of air molecules contained in the bulb. Given that
R 8.31 J / mol K and N 6.02 10 23 / mol .

Fig. 6.21 Sol.


6
Ex. 18 From the graph for a perfect gas state whether m1 or m2 Pressure inside bulb P = h g 10 (13.6 103 ) 9.8
is greater. = 0.13 N/m2
Assuming air as an ideal gas, we can use
Sol. For a perfect gas PV = nRT
PV = nRT
PV 6
0.13 250 10
m or n = =
= RT RT 8.31 300
M
= 1.3 × 10–8 mole.
T MV constant Number of molecules = (1.3 × 10–8) × (6.02 × 1023)
= =
P mR m = 7.83 × 1015 Ans.
Ex. 21 A vertical hollow cylinder of height 1.52 m is fitted with
a movable piston of negligible mass and thickness. The lower half
portion of the cylinder contains an ideal gas and the upper half is
filled with mercury. The cylinder is initially at 300 K. When the
temperature is raised half of the mercury comes out of the cylinder.
Find the temperature assuming the thermal expansion of mercury
to be negligible.
Sol.
Let A be the area of cross-section of the piston.
Fig. 6.22 Initially, V1 = 76A, P1 = pa + 76 cm of Hg
Thus slope of T –P is inversely proportional to mass of the gas. As slope = 2pa (pa = 76 cm)
T 1 = 300 K
of 1 is smaller, so m1 m2 .
Finally, V2 = 114 A,
P2 = pa + 38 cm of Hg
Ex. 19 The P–T graph for the given mass of a perfect gas is
shown in figure. What inference can be drawn regarding the change pa 3 pa
= pa =
in volume. 2 2

Sol.
For a perfect gas
PV = nRT
P nR constant
= =
T V V
1
or V
P
T Fig. 6.24
www.crackjee.xyz
KINETIC THEORY OF GASES 435
If T2 is the final temperature then by ideal gas equation (b) When the vessel is stopped, its K.E. will be converted into internal
energy, so we have
PV
1 1 P2V2
= 1 2
T1 T2 mv0 = nCV T
2
P2V2T1
T2 = 5
PV
1 1 = n R T
2
3 pa
(114 A) 300 1
2 or (32 10 3 ) v02
= 2
(2 pa ) (76 A)
= 337.5 K Ans. 5
= 1 8.31 1
2
Ex. 22 One gram mole of oxygen at 27°C and 1 atmospheric
After solving , we get v0 = 36 m/s Ans.
pressure is enclosed in a vessel. (a) Assuming the molecules to the
moving with vrms, find the number of collisions per second which the Ex. 23 Two closed vessels of equal volume contain air at 105 k
molecules make with one square meter area of the vessel wall. (b) Pa, 300 K and are connected throw a narrow tube. If one of the
the vessel is next thermally insulated and moved with a constant vessels is now maintained at 300 K and the other at 400 K, what
speed v0. It is then suddenly stopped. The process results in a rise of will be the pressure in the vessels?
temperature of the gas by 1°C. Calculate speed v0. Sol.
(k = 1.38 × 10–23 J/K and N = 6.02 × 1023 /mol.). Initially, pressure, volume and temperature of each vessel be P0, V0 and
Sol. P0V0
Mass of one molecule of oxygen T0. Let n be the number of moles in each vessel, then n RT0 . When
mass of one mole other vessel is kept at temperature T = 400 K, the pressure of the
= system changes. Let new pressure becomes P. If n1 and n 2 are the
no. of molecules in one mole
number of moles of air in first and second vessel respectively, then
3
32 10 PV0
= 23 = 5.316 × 10–26 kg PV0
6.02 10 n1 = RT , n2 = .
0 RT
(a) The speed of the oxygen molecule For an isolated system
3kT n + n = n1+n 2
vrms = or 2n = n1+n 2
m
Momentum of the oxygen molecule P0V0 PV0 PV0
2 =
p = mv RT0 RT0 RT

3kT 2P0 1 1
= m = 3mkT P
m T0 = T0 T
According to kinetic theory of gases, the molecule makes elastic
collision with the wall, so momentum after collision becomes –p.
The change in momentum
p = 2p = 2 3mkT
Fig. 6.25
= 2 3 5.316 10 26 1.38 10 23 300
2 P0T
= 5.139 10 23 N s P = T T
0
If there are n collisions per second, then force exerted
2 105 400
p 1 = = 120 kPa Ans
F = = n p n 400 300
t t
CV and CP :
F n p CP
Pressure, P = = = n p We have, CP CV = R and C =
A 1
V
or 1.01 105 = n 5.139 10 23
R R
CV = and CP =
n = 1.965 10 27 Ans. 1 1
www.crackjee.xyz
436 MECHANICS, HEAT, THERMODYNAMICS & WAVES
of the mixture : Substituting these values in above equation, we get
(i) If n1moles of CV , n2 moles of CV2 , _____ are mixed, then by 1 n2 1 n2
1
conservation of energy, we have 5 7 =
1 1 19
U1+ U2 +.............. = U 1
3 5 13
or n1CV1 T ........ = ( n1 n2 .....)(CV )mix T
n2CV2 T
3 5n2 13(1 n2 )
Here(CV)mix is the specific heat of the mixture at constant volume or =
2 2 6
After solving above equation, we get
n1CV1 n2CV2 ..... or 3(3 5n2 ) = 13(1 n2 )
(CV )mix =
n1 n2 ... 9 15n2 = 13 13n2
and (CP )mix = (CV ) R n2 = 2 gm mole Ans.

(CP )mix Ex. 25 A gas is enclosed in a vessel of volume V at a pressure P.


mix = (C ) It is being pumped out of the vessel by means of a piston pump with
V mix
a stroke volume v. What is the final pressure in the vessel after n
(ii) If n1 moles of 1and n2 moles of 2,........... are mixed, then by strokes of the pump? Assume temperature remains constant.
conservation of energy , we have
U1 + U2+ ........ = U Sol.
or n1CV1 T n2CV2 T ........ = ( n1 n2 .....)(CV )mix T When v volume of a gas is pumped out from the vessel, the remaining gas
will occupy the volume of the container. The total volume of gas thus
R R R becomes (V + v), let P1 be the pressure of the gas of volume (V + v), then
or n1 n2 ..... = ( n1 n2 ....) by Boyle’s law
1 1 2 1 1
PV = P1(V v)
n1 n2 ( n1 n2 .......)
or ..... = V
1 1 2 1 mix 1 P1 = P … (i)
V v

Note:
It is not correct to get of the mixture by using a formula
n1 1 n2 2
mix = n1 n2 for mixture of two gases.

Fig. 6.26
Ex. 24 A gaseous mixture enclosed in a vessel consists of one Similar treatment after second stroke gives
gm mole of a gas A with ( = 5 / 3) and another B with ( = 7 / 5) at a
P1V = P2 (V v)
temperature T. The gases A and B do not react with each other and
assumed to be ideal. Find the number of gm moles of the gas if V
P2 = P1 … (ii)
of the gaseous mixture is 19/13. V v
Substituting the value of P1 from (i) in (ii), we get
Sol. For two gases, we can write
2
n1 n2 n1 n2 V
P2 = P
1 1 2 1 = mean 1 V v
After n strokes, let Pn be the pressure of the gas in the vessel, then
Here, n1 = 1 and n2 = ? n
V
1 = 5 / 3 and 2 7/5 Pn = P Ans.
V v
6.13 VAPOUR PRESSURE
Consider a closed vessel having some water. Due to evaporation some water transforms
into vapour and adjusted between the air molecules in the space above water. This
vapour exerted pressure on the water, which is called vapour pressure. After some time
the volume of water becomes constant and the vapour above it becomes saturated.
When space above water contains the maximum possible amount of vapour, the vapour
is called saturated, and the corresponding vapour pressure is called saturation vapour
pressure (SVP). The saturation vapour pressure of a substance is constant at a given
temperature. It increases with increase in temperature. Figure (6.27) shows the saturation
Fig. 6.27 vapour pressure of water, as a function of temperature.
www.crackjee.xyz
KINETIC THEORY OF GASES 437
Saturation vapour pressure of water

T (°C) Vapour pressure (mm of Hg) T (°C) Vapour pressure (mm of Hg)
0 4.58 100 760
5 6.51 120 1490
10 8.94 140 2710
15 12.67 160 4630
20 17.5 180 7510
40 55.1 200 11650
60 149 220 17390
80 355
6.14 DEW POINT
The temperature at which the saturation vapour pressure becomes equal to the existing
vapour pressure is called due point. If the temperature is decreased below the dew point,
some of the vapour condenses.
6.15 HUMIDITY
The amount of water vapour present in unit volume of air is called absolute humidity. Its
SI unit is kg/m3

Amount of water vapour present in a given


volume of air at a given temperature
Relative humidity: RH =
Amount of water vapour required to saturated
the same volume of air at the same temperature
As the pressure exerted by vapour is directly proportional to the amount of vapour
m
present in a given volume PV RT , so relative humidity can also be defined as
M
Vapour pressure of air
RH =
SVP at the same temperature

SVP at dew point


or RH =
SVP at the existing temperature

Ex. 26 The relative humidity in a closed room at 15°C is 60%. Ex. 27 A jar contains a gas and a few drops of water at TK. The
If the temperature rises to 20°C what will be the relative humidity? pressure in the jar is 830 mm of Hg. The temperature of the jar is
SVP at 15°C and 20°C are 12.67 and 17.36 mm of Hg respectively. reduced by 1%. The saturated vapour pressures of water at the two
Sol. The SVP at 15°C = 12.67 mm of Hg temperatures are 30 and 25 mm of Hg. Calculate the new pressure
in the jar.
Vapour pressure at 15°C ( P15 )
RH = Sol. The pressure in the jar due to gas and due to water vapours = 830
SVP at 15°C mm of Hg.
P15 Saturated vapour pressure at temperature T = 30 mm of Hg.
or 0.60 = P15 = 7.6 mm of Hg So pressure of the gas at temperature T, P1 = 830 –30 = 800 mm of Hg.
16.67
The final temperature of the jar T2 = T – 0.01T = 0.99T
From P15 , we have to find the vapour pressure at 20°C (P20) . By gas
Let P2 be the final pressure of the gas, then by gas law
equation PV = nRT, we have
PV
1 1 P2V2
P15 273 15 288 =
P20 = = T1 T2
273 20 293
800V P2V
293 293 or =
or P20 = P15 = 7.6 T 0.99T
288 288
or P2 = 792 mm of Hg
= 7.73 mm of Hg
The saturated vapour pressure at
P20 7.73 T 2 = 25 mm of Hg
RH at 20°C = =
SVP at 20°C 17.36 The final total pressure in the jar
= 0.445 = 44.5% Ans = 792 + 25 = 817 mm of Hg Ans.
www.crackjee.xyz
438 MECHANICS, HEAT, THERMODYNAMICS & WAVES

Review of formulae & Important Points


1. Boyle's law (ii) Diatomic gas
PV = constant Degree of freedom = 5 (3 translational + 2 rotational)
2. Charle's law 5RT
Kinetic energy per mole =
V 2
= constant
T
5R
3. Lay-Lussac's law Cv =
2
P
= constant 7R
T Cp =
4. Perfect gas equation 2
PV = nRT 7
5. Vander Waal's equation for real gases =
5
an2 (iii) Triatomic or polyatomic gas
P (V nb) = Degrees of freedom = 6
V 2 nRT
(3 translational + 3 rotational)
2
nmvrms Kinetic energy per mole = 3RT
6. Pressure, P = Cv = 3 RT
3V
7. Kinetic energy of gas Cp = 4R

3 4
(i) Translational K.E. RT per mole for each gas =
2 3
(ii) For diatomic gas
2
11. In general, = 1
5 f
Translational + rotational K.E. = RT per mole
2
kT
3RT 12. Mean free path, =
8. Root mean square velocity vrms 2 d 2P
M
13. of the mixture :
9. Dalton's law of partial pressure
(i) When Cv and Cp are given : For two gases with n1
P = P1 + P2 + .....
and n2 moles
10. Degrees of freedom :
(i) Manoatomic gas n1Cv1 n2Cv2
Degrees of freedom f = 3 translational [Cv ]mix = n1 n2
3
Kinetic energy per mole = RT n1C p1 n2C p2
2
[C p ]mix = n1 n2
U 3
Cv = R
T 2
[C p ]mix
5 mix = [Cv ]mix
Cp = Cv R R
2
(ii) For two gases of 1 and 2 with n1 and n2 moles
Cp 5
= ( n1 n2 ) n1 n2
Cv 3 =
mix 1 1 1 2 1
www.crackjee.xyz
KINETIC THEORY OF GASES 439

MCQ Type 1 Exercise 6.1

Level -1
Only one correct option 7. The average transitional energy and the rms speed of molecules in
1. Two identical cylinders contain helium at 2.5 atmosphere and a sample of oxygen gas at 300 K are 6.21 × 10–21 J and 484 m/s
argon at 1 atmosphere respectively. If both the gases are filled in respectively. The corresponding values at 600 K are nearly
one of the cylinders, the pressure would be (assuming ideal gas behaviour)
(a) 3.5 atmosphere (b) 1.75 atmosphere (a) 12.42 × 10–21 J, 968 m/s (b) 8.78 × 10–21 J, 684 m/s
(c) 1.5 atmosphere (d) 1 atmosphere
(c) 6.21 × 10–21 J, 968 m/s (d) 12.42 × 10–21 J, 684 m/s
2. The air density at Mount Everest is less than that at the sea level.
8. The average transitional kinetic energy of O 2 (molar mass 32)
It is found by mountaineers that for one trip lasting a few hours,
the extra oxygen needed by them corresponds to 30,000 cc at sea molecules at a particular temperature is 0.048 eV. The transitional
level (pressure 1 atmosphere, temperature 27°C). Assuming that kinetic energy of N2 (molar mass 28) molecules in eV at the same
the temperature around Mount Everest is – 73°C and that the temperature is
oxygen cylinder has capacity of 5.2 liters, the pressure at which (a) 0.0015 (b) 0.003
oxygen be filled (at site) in the cylinder is (c) 0.048 (d) 0.768
(a) 3.86 atmospheres (b) 5.00 atmospheres 9. A vessel contains 1 mole of O 2 gas (molar mass 32) at a temperature
(c) 5.77 atmospheres (d) 1 atmospheres T. The pressure of the gas is P. An identical vessel containing one
3. We have a jar A filled with gas characterised by parameters P, V and mole of He gas (Molar mass 4) at a temperature 2T, has a pressure
T and another jar B filled with a gas with parameters 2P, V/4 and of
2T, where the symbols have the usual meanings. The ratio of the
number of molecules of jar A to those of jar B is P
(a) (b) P
(a) 1 : 1 (b) 1 : 2 8
(c) 2 : 1 (d) 4 : 1
(c) 2P (d) 8P
4. At room temperature the r.m.s. speed of the molecules of a certain
10. Which of the following molecular properties is the same for all
diatomic gas is found to be 1920 m/s. The gas is:
ideal gases at a given temperature are
(a) H 2 (b) F 2
(a) rms momentum (b) rms velocity
(c) O 2 (d) Cl2
5. Three closed vessels A, B and C are at the same temperature T and (c) mean kinetic energy (d) mean free path
contain gases which obey the Maxwellian distribution of velocities. 11. The temperature of an ideal gas is increased from 120 K is 480 K.
Vessel A contains only O2, B only N2 and C a mixture of equal If at 120 K, the root mean square velocity of the gas molecules is
quantities of O2 and N2. If the average speed of the O2 molecules v, then at 480 K, it will be
in vessel A is v1, that of the N2 molecules in vessel B is v2, the (a) 4v (b) 2v
average speed of the O2 molecules in vessel C is
v1 v2 v v
(a) (c) (d)
2 2 4
(b) v1 12. The plot of isotherms will not be a straight line when it is a plot
(c) (v 1 v 2) 1/2 between
3kT (a) p and T (b) V and p
(d) where m is the mass of an oxygen molecule (c) S (entropy) and T (d) pV and V
m
6. A vessel contains a mixture of 1 mole of oxygen and two moles of 13. The internal energy of an ideal gas decreases by the same amount
nitrogen at 300 K. The ratio of the average rotational kinetic energy as the work done by the system
per O2 molecule to that per N2 molecule is (a) the process must isothermal
(a) 1 : 1 (b) the process must be adiabatic
(b) 1 : 2
(c) the process must isobaric
(c) 2 : 1
(d) depends on the moment of inertia of the two molecules (d) the temperature of the system must increase

Answer Key 1 (a) 3 (d) 5 (b) 7 (d) 9 (c) 11 (b) 13 (b)


Sol. from page 449 2 (a) 4 (a) 6 (a) 8 (c) 10 (c) 12 (b)
www.crackjee.xyz
440 MECHANICS, HEAT, THERMODYNAMICS & WAVES
14. Figure shows two flasks connected to each other. The volume of R
the flask 1 is twice that of flask 2. The system is filled with an 23. For a gas 0.67 . This gas is made up of molecules which
CV
ideal gas at temperature 100 K and 200 K respectively. If the mass are
of the gas in 1 be m, then what is the mass of the gas in flask 2 (a) diatomic
(b) mixture of diatomic and polyatomic molecules
100 K 200 K (c) monoatomic
(d) polyatomic
1 2 24. If cp and cv denote the specific heats of nitrogen per unit mass at
(a) m (b) m/2
constant pressure and constant volume respectively, then
(c) m/4 (d) m/8
15. If pressure of a gas contained in a closed vessel is increased by (a) cp – cv = R / 28 (b) cp – cv = R / 14
0.4% when heated by 1°C, the initial temperature must be (c) cp – cv = R (d) cp – cv = 28 R
(a) 250 K (b) 250°C
(c) 2500 K (d) 25°C 25. The specific heats at constant pressure is greater than that of the
16. To double the volume of a given mass of an ideal gas at 27°C same gas at constant volume because
keeping the pressure constant, one must raise the temperature in (a) At constant pressure work is done in expanding the gas
degree centigrade to
(b) At constant volume work is done in expanding the gas
(a) 54° (b) 270°
(c) 327° (d) 600° (c) The molecular attraction increases more at constant pressure
17. Air is pumped into an automobile tube upto a pressure of 200 (d) The molecular vibration increases more at constant pressure
kPa in the morning when the air temperature is 22°C. During the 26. The specific heat of a gas
day, temperature rises to 42°C and the tube expands by 2%. The
(a) has only two values cp and cv
pressure of the air in the tube at this temperature, will be
approximately (b) has a unique value at a given temperature
(a) 212 kPa (b) 209 kPa (c) can have any value between 0 and
(c) 206 kPa (d) 200 kPa (d) depends upon the mass of the gas
18. The gas in vessel is subjected to a pressure of 20 atmosphere at a
27. Gas at a pressure P0 is contained in a vessel. If the masses of all the
temperature 27°C. The pressure of the gas in a vessel after one
half of the gas is released from the vessel and the temperature of molecules are halved and their speeds are doubled, the resulting
the remainder is raised by 50°C is pressure P will be equal to
(a) 8.5 atm (b) 10.8 atm (a) 4P0 (b) 2P0
(c) 11.7 atm (d) 17 atm P0
19. In Vander Waal’s equation a and b represent (c) P0 (d)
2
a
P V – b RT 28. One kg of a diatomic gas is at a pressure of 8 × 104 N/m2. The
V2
(a) Both a and b represent correction in volume density of the gas is 4 kg/m3. What is the energy of the gas due to
(b) Both a and b represent adhesive force between molecules its thermal motion
(c) a represents adhesive force between molecules and b (a) 3 × 104 J (b) 5 × 104 J
correction in volume
(c) 6× 104 J (d) 7 × 104 J
(d) a represents correction in volume and b represents adhesive
force between molecules 29. At 0 K which of the following properties of a gas will be zero
20. Which of the following statements is true? (a) Kinetic energy (b) Potential energy
(a) Absolute zero temperature is zero energy temperature. (c) Vibrational energy (d) Density
(b) Two different gases at the same temperature pressure have
30. A closed compartment containing gas is moving with some
equal root mean square velocities.
acceleration in horizontal direction. Neglect effect of gravity. Then
(c) The root mean square speed of the molecules of different
ideal gases, maintained at the same temperature are the same. the pressure in the compartment is
(d) Given sample of 1cc of hydrogen and 1 cc of oxygen both at (a) same everywhere (b) lower in the front side
NTP; oxygen sample has a large number of molecules. (c) lower in the rear side (d) lower in the upper side
21. According to the kinetic theory of gases, at absolute temperature
31. A gas is filled in the cylinder shown in the figure. The two pistons
(a) water freezes (b) liquid helium freezes
(c) molecular motion stops (d) liquid hydrogen freezes are joined by a string. If the gas is heated, the pistons will
22. The speeds of 5 molecules of a gas (in arbitrary units) are as (a) move towards left
follows : 2, 3, 4, 5, 6. The root mean square speed for these (b) move towards right
molecules is (c) remain stationary Gas
(a) 2.91 (b) 3.52
(c) 4.00 (d) 4.24 (d) none of these

Answer Key 14 (c) 16 (c) 18 (b) 20 (a) 22 (d) 24 (a) 26 (c) 28 (a) 30 (b)
Sol. from page 449 15 (a) 17 (b) 19 (c) 21 (c) 23 (c) 25 (a) 27 (b) 29 (a) 31 (b)
www.crackjee.xyz
KINETIC THEORY OF GASES 441
32. An experiment is carried on a fixed amount of gas at different 34. From the following P-T graph what inference can be drawn
temperatures and at high pressure such that it deviates from the
T V2
PV
ideal gas behaviour. The variation of with P is shown in the
RT
diagram. The correct variation will correspond to V1
2
(a) Curve A PV/RT 1
P
2.0 A
(b) Curve B (a) V2 > V1 (b) V2 < V1
B
1.0 (c) V2 = V1 (d) None of the above
(c) Curve C C
D 35. Which one the following graphs represents the behaviour of an
(d) Curve D ideal gas?
0, 0 20 40 60 80 100 P (atm)
33. The change in volume V with respect to an increase in pressure P PV PV
has been shown in the figure for a non-ideal gas at four different
temperatures T1, T2, T3 and T4. The critical temperature of the gas
(a) (b)
is
V V
(a) T1 P
(b) T2 T1
PV PV
T2
(c) T3 T3
T4 (c) (d)
(d) T4 (0, 0) V
V V

Answer Key (b) (b) (a) (b)


32 33 34 35
Sol. from page 449

Level -2
5
1. If one mole of monoatomic gas is mixed with one mole of 3 1 2
3 (a) (b)
7 2 1 1
diatomic gas , the value of for the mixture is
5
(a) 1.40 (b) 1.50 9 25
(c) ( 1) (d) ( 1)
(c) 1.53 (d) 3.07 2 2
2. The temperature at which the average transitional kinetic energy 5. Two ideal gas at absolute temperature T1 and T2 are mixed. There
of a molecule is equal to the energy gained by an electron in is no loss of energy. The masses of the molecules are m1 and m2
accelerating from rest through a potential difference of 1 volt is and the number of molecules in the gases are n1 and n2 respectively.
(a) 4.6 × 103 K (b) 11.6 × 103 K The temperature of the mixture is
(c) 23.2 × 103 K (d) 7.7 × 103 K T1 T2
T1 T2
3. A diatomic gas is heated at constant pressure. The fraction of the (a) (b) n1n2
2
heat energy is used to increase the internal energy is

3 3 n1T1 n2T2
(a) (b) (c) T1 T2 (d) n1 n2
5 7
6. Half mole of helium gas is contained at STP. The heat energy
5 5
(c) (d) needed to double the pressure of the gas keeping the volume
7 9
constant (specific heat of the gas = 3J/g –K) is
4. For a gas if ratio of specific heat at constant pressure and volume (a) 3276 J (b) 1638 J
is then the value of degree of freedom is (c) 819 J (d) 409.5 J

Answer Key (b) (d) (c) (b) (d) (b)


1 2 3 4 5 6
Sol. from page 450
www.crackjee.xyz
442 MECHANICS, HEAT, THERMODYNAMICS & WAVES
7. Two thermally insulated vessels 1 and 2 are filled with air at 12. A gas mixture consists of 2 moles of oxygen and 4 moles of argon
temperatures (T 1, T 2), volume (V 1, V 2) and pressure (P1, P2) at temperature T. Neglecting all vibrational modes, the total internal
respectively. If the valve joining the two vessels is opened, the energy of the system is
temperature inside the vessel at equilibrium will be (a) 4 RT (b) 15 RT
(a) T1 + T2 (b) (T1 + T2) / 2 (c) 9 RT (d) 11 RT
13. Two different masses m and 3m of an ideal gas are heated separately
T1T2 PV
1 1 PV
2 2 T1T2 PV
1 1 PV
2 2 in a vessel of constant volume, the pressure P and absolute
(c) PV (d) temperature T, graphs for these two cases are shown in the figure
1 1T2 PV
2 2T1 1 1T1
PV PV
2 2T2
as A and B. The ratio of slopes of curves B to A is
8. The adjoining figure shows graph of pressure and volume of a gas P B
at two different temperatures T1 and T2. Which of the following
inferences is correct? P 3m A
(a) T1 > T2 m
T
(b) T1 = T2 (a) 3 : 1 (b) 1 : 3
(c) T1 < T2 T2 (c) 9 : 1 (d) 1 : 9
T1 14. One gram mole of nitrogen at 27°C and 1 atm pressure is contained
(d) none of these V
in a vessel and the molecules are moving with their rms speed. The
9. The root mean square of the molecules of a diatomic gas is v. number of collisions per second which the molecules make with an
When the temperature is double, the molecules dissociate into area of 1m2 on the vessel's wall is
two atoms. The new root mean square speed of the atom is (a) 2 × 1027 (b) 2 × 1020
(c) 2 × 10 10 (d) 2 × 1024
(a) v (b) 2v
15. Consider the following statements :
(c) 2v (d) 4v A gas can be liquefied by increasing the pressure
10. The eq uation of state of a gas is given 1. above the critical pressure only
2. Only when the temperature of the enclosed gas is below the
aT 2 c critical temperature
by P V RT b , where a, b, c and R are constants.
V 3. Only when the volume of the enclosed gas is below the
critical volume
The isotherms can be represented by P = AVm – BVn, where A and
Which of the statements given above is / are correct :
B depend only on temperature and
(a) 1 and 2 (b) 2 only
(a) m = – c and n = – 1 (b) m = c and n = – 1
(c) 3 only (d) 2 and 3
(c) m = – c and n = 1 (d) m = c and n = – 1
16. One mole of an ideal gas is taken from an initial state (p, v, T) to a
11. A horizontal uniform glass tube of 100 cm, length sealed at both final state (2p, 2V, 4T) by two different paths as shown in the
ends contain 10 cm mercury column in the middle. The temperature figures 1 and 2 given. If the changes in internal energy between the
and pressure of air on either side of mercury column are final and the initial states of the gas along the paths I and II are
respectively 81°C and 76 cm of mercury. If the air column at one denoted by UI and UII respectively, then :
end is kept at 0°C and the other end at 273°C, the pressure of air
which is at 0°C is (in cm of Hg) p p
C (2p, 2v, 4T) (p,v/2,T)
Hg B C
(2p, 2v, 4T)
A (p,v,T)

A (p,v,T)
B (p/2,2v,T)
v v
10 cm
100 cm (a) UI = UII (b) UI > UII
(a) 76 (b) 68.2 (c) UI < UII (d) UI = 0.66 UII
(c) 102.4 (d) 122

Answer Key 7 (c) 9 (b) 11 (c) 13 (a) 15 (d)


Sol. from page 450 8 (c) 10 (a) 12 (d) 14 (a) 16 (a)
www.crackjee.xyz
KINETIC THEORY OF GASES 443
17. Five gas molecules chosen at random are found to have speeds of (a) 8.2 km (b) 12.4 km
500, 600, 700, 800 and 900 m/s: (c) 10.6 km (d) 15.2 km
(a) The root mean square speed and the average speed are the 19. A vessel containing one gram mole of oxygen at 27°C and 1 atm
same. pressure is thermally insulated and moles with a constant speed
(b) The root mean square speed is 14 m/s higher than the average v0. It is then suddenly stopped and this results in a rise of
speed. temperature of the gas by 1°C the speed v0 is
(c) The root mean square speed is 14 m/s lower than the average (a) 63.03 m/s (b) 30.63 m/s
speed. (c) 36.03 m/s (d) 33.06 m/s
(d) The root mean square speed is 14 m/s higher than the average 20. A tube closed at one end is lowered, with the open end down, into
speed. a fresh water lake. If one third of the tube is filled with water, the
18. A nitrogen molecule has some rms speed at 0°C on the surface of distance between the surface of the lake and level of water in the
the earth. With this speed, it goes straight up. If therei is no tube is
collisions with other molecules, the molecule will rise up to a (a) 6.41 m (b) 5.82 m
height of (c) 5.10 m (d) 4.20 m

Answer Key
17 (b) 18 (b) 19 (c) 20 (c)
Sol. from page 450

MCQ Type 2 Exercise 6.2


Multiple correct options 4. A gas in container A is in thermal equilibrium with another gas in
1. Cv and Cp denote the molar specific heat capacities of a gas at container B. Both contain equal masses of the two gases in the
constant volume and constant pressure, respectively. Then respective containers. Which of the following can be true?
(a) Cp – Cv is larger for a diatomic ideal gas than for a monoatomic
ideal gas (a) PAVA PBVB (b) PA PB , VA VB
(b) Cp + Cv is larger for a diatomic ideal gas than for a monoatomic PA PB
ideal gas (c) PA PB , VA VB (d) VA VB
(c) Cp / Cv is larger for a diatomic ideal gas than for a monoatomic
ideal gas 5. Graph shows a hypothetical speed distribution for a sample of N
(d) Cp . Cv is larger for a diatomic ideal gas than for a monoatomic dN
ideal gas gas particle (for v > v0 ; 0)
dv
2. A box contains a mixture of H2 and He gases. Which of the following
statements are corrects?
(a) The average translational kinetic energies of H2 molecules dN
and He atoms are same dv
(b) The average energies of H2 molecules and He atoms are same a
(c) H2 molecules have greater average energy than that of He
atoms
(d) The average speed of H2 molecules and He atoms are same
3. From the following statements, concerning ideal gas at any given
temperature T, select the correct one(s) v0 speed v
(a) The coefficient of volume expansion at constant pressure is (a) The value of v0 is 2N
same for all ideal gases
(b) The ratio vavg /v0 is equal to 2/3
(b) The average translational kinetic energy per molecule of
oxygen gas is 3KT (K being Boltzmann constant) (c) The ratio vrms /v0 is equal to 1/ 2
(c) In a gaseous mixture, the average translational kinetic energy (d) Three fourth of the total particle has a speed between 0.5v0
of the molecules of each component is same and v0
(d) The mean free path of molecules increases with the decrease
in pressure

Answer Key 1 (b, d) 3 (a, c, d) 5 (a, b, c, d)


Sol. from page 451 2 (a, c) 4 (b, c)
www.crackjee.xyz
444 MECHANICS, HEAT, THERMODYNAMICS & WAVES

6. Let v , vrms and vp respectively denote the mean speed, the root- 12. The total kinetic energy of translatory motion of all
mean-square speed and the most probable speed of the molecules the molecules of 5 litres of nitrogen exerting a pressure P is 3000 J.
in an ideal monoatomic gas at absolute temperature T. The mass of (a) the total K.E. of 10 litres of N2 at a pressure of 2P is 3000J
a molecule is m. Then (b) the total K.E. of 10 litres of He at a pressure of 2P is 3000 J
(a) No molecule can have speed greater than vrms (c) the total K.E. of 10 litres of O2 at a pressure of 2P is 12000 J
vp (d) the total K.E. of 10 litres of Ne at a pressure of 2P is 12000 J
(b) No molecule can have speed less than
2 13. A mixture of ideal gases 7 kg of nitrogen and 11 kg of CO2. Then
(a) equivalent molecular weight of the mixture is 36
(c) vp v vrms
(b) equivalent molecular weight of the mixture is 18
3 2
(d) mv p
The average kinetic energy of a molecule is (c) for the mixture is 5/2
4
7. 3
Hydrogen gas and oxygen gas have volume 1 cm each at NTP (d) for the mixture is 47/35
(a) Number of molecules is same in both the gases (Take for nitrogen and CO2 as 1.4 and 1.3 respectively)
(b) The rms velocity of molecules of both the gases is the same 14. Hydrogen gas and oxygen gas have volume 1cm3 each at N.T.P.,
(c) The internal energy of each gas is the same Then
(d) The average velocity of molecules of each gas is the same (a) number of molecules is same in both the gases.
8. Velocities of four gas molecules are 2, 3, – 4, –1 m/sec respectively. (b) the rms velocity of molecules of both the gases is the same.
Then
(c) the internal energy of each gases is the same.
(a) the mean velocity of molecules is zero
(b) the mean speed of molecules is 2.5 m/s (d) the average velocity of molecules of each gas is the same.
(c) the mean square velocity of molecules is 7.5 m2/s2 15. In case of hydrogen and oxygen at N.T.P., which of the following
(d) the root mean square velocity of molecules is 2.74 m/s quantities is/are the same?
9. Choose the correct statement(s)? (a) average momentum per molecule
(a) In an adiabatic expansion the product of pressure and volume
(b) average kinetic energy per molecule
decreases.
(b) The rms translational speed for all ideal gas molecules at the (c) kinetic energy per unit volume
same temperature is not the same but it depends on the (d) kinetic energy per unit mass
mass.
16. For two different gases X and Y, having degrees of freedom f1 and
(c) Temperature of an ideal gas is doubled from 100°C to 200°C.
The average kinetic energy of each particle is also doubled. f2 and molar heat capacities at constant volume Cv1 and Cv2
(d) The rms translational speed for all ideal gas molecules at the respectively, the lnP versus lnV graph is plotted for adiabatic
same temperature is not the same and does not depend on process, as shown
the mass. ln P
(a) f1 > f2
10. A gas expands such that its initial and final temperature are equal. y
(b) f2 > f1
Also, the process followed by the gas traces a straight line on the x
P-V diagram : (c) Cv2 > Cv1
(a) The temperature of the gas remains constant throughout (d) Cv1 > Cv2 ln V
(b) The temperature of the gas first increases and then decreases
17. The volume of a gas and the number of gas molecules within that
(c) The temperature of the gas first decreases and then increases volume for four situations are (1) 2V0 and N0, (2) 3V0 and 3N0, (3)
(d) The straight line has a negative slope 8V0 and 4N0 (4) 3V0 and 9N0. Which of them has mean free path
11. Two vessels of the same volume contain the same gas at same greatest :
temperature. If the pressure in the vessels be in the ratio of 1 : 2,
(a) 1 (b) 2
then
(a) the ratio of the average kinetic energy is 1 : 2 (c) 3 (d) 4
(b) the ratio of the root mean square velocity is 1 : 1
(c) the ratio of the average velocity is 1 : 2
(d) the ratio of number of molecules is 1 : 2

Answer Key 6 (c, d) 8 (a, b, c, d) 10 (b, d) 12 (c, d) 14 (a, c, d) 16 (b, c)


Sol. from page 451 7 (a, c, d) 9 (a, b) 11 (b, d) 13 (a, d) 15 (a, b, c) 17 (a, c)
www.crackjee.xyz
KINETIC THEORY OF GASES 445

Statement Questions Exercise 6.3

Read the two statements carefully to mark the correct option out of the options given below:
(a) If both the statements are true and the statement - 2 is the correct explanation of statement - 1.
(b) If both the statements are true but statement - 2 is not the correct explanation of the statement - 1.
(c) If statement - 1 true but statement - 2 is false.
(d) If statement - 1 is false but statement - 2 is true.

1. Statement - 1 7. Statement - 1
Consider a gas of neutrons, it to behave much better as an ideal gas A gas has unique value of specific heat.
as compared to hydrogen gas at the same pressure and temperature.
Statement - 2
Statement - 2
Specific heat is defined as the amount of heat required to raise the
Internal potential energy of gas of neutrons is zero.
temperature of unit mass of the substance through one degree
2. Statement - 1 centrigrade.
When we place a gas cylinder on a moving train, its internal kinetic
8. Statement - 1
energy increases.
Absolute zero is the temperature corresponding to zero energy.
Statement - 2
Its temperature remains constant. Statement - 2
3. Statement - 1 The temperature at which no molecular motion cease is called
absolute zero temperature.
When you come out of a river after a dip, you feel cold.
Statement - 2 9. Statement - 1
The temperature of atmosphere is less than that of river water. At low density, variable of gases P, V and T follows the equation
4. Statement - 1 PV = RT.
Cooking is faster in pressure cooker. Statement - 2
Statement - 2 At low density real gases are more closely to ideal gases.
Aluminium is good conductor of heat. 10. Statement - 1
5. Statement - 1 For an ideal gas, at constant temperature, the product of pressure
In pressure temperature ( P-T) phase diagram of water, the slope and volume is constant.
of the melting curve is found to be negative. Statement - 2
Statement - 2 The mean square velocity of the molecules is inversely proportional
Ice contracts on melting. to mass.
6. Statement - 1 11. Statement - 1
For an atom the number of degrees of freedom is 3. For an ideal gas (in mole)
Statement - 2
Cp – Cv = R.
The ratio of specific heats at constant pressure and volume is a
Statement - 2
constant. i.e.,
At constant pressure, some heat is spent in expansion of the gas.
Cp
.
Cv

Answer Key 1 (a) 3 (c) 5 (a) 7 (d) 9 (a) 11 (a)


Sol. from page 452 2 (d) 4 (b) 6 (b) 8 (d) 10 (b)
www.crackjee.xyz
446 MECHANICS, HEAT, THERMODYNAMICS & WAVES

Passage & Matrix Exercise 6.4

Passage for (Q. 1 - 3) : Passage for (Q. 7 - 9) :


A balloon whose volume is 500m 3 is to be filled with hydrogen at A cubical box of side 1 m contains helium gas (atomic weight 4) at a
atmospheric pressure. pressure of 100 N/m2. During an observation time of 1 s , an atom
travelling with the root mean square speed parallel to one of its edges
1. If hydrogen is stored in cylinders of volume 0.05m 3 at an of the cube, was found to make 500 collisions with a particular wall,
atmospheric pressure of 15 × 105 Pa. Then number of cylinders without any collision with other atoms. Taking R = (25/3) J/mol–K and
required are k = 1.38× 10–23 J/K. Evaluate

(a) 425 (b) 570


(c) 675 (d) 690 7. The temperature of the gas is

2. The total weight can be supported by the balloon in air at standard (a) 160 K (b) 210 K
condition is (c) 280 K (d) 320 K
(a) 3775 N (b) 4500 N 8. The average kinetic energy per atom is
(c) 5934 N (d) 6231 N (a) 1.612 × 10–21J (b) 3.312 × 10–21 J
3. The weight supported by the balloon, if filled with helium instead (c) 3.521 × 10–21 J (d) 4.20 × 10–21 J
of hydrogen is 9. The total mass of helium gas in the box is
(a) 5498 N (b) 3212 N (a) 0.1 g (b) 0.2 g
(c) 2000 N (d) 1825 N (c) 0.3 g (d) 0.4 g

Passage for (Q. 4 - 6) : Passage for (Q. 10 - 12) :


A gaseous mixture enclosed in a volume V consists of one gram mole of A cylinder contains nitrogen gas at 2.0 atm pressure and temperature
a gas A with (Cp / Cv) = 5 / 3 and another has B with = 7/5 at a certain 17°C. Take the radius of a nitrogen molecule to be roughly 1.0° Å.
temperature T. The gram molecular weights of the gases A and B are 4 (Molecular mass of N2 = 28.0g)
and 32 respectively. The gases A and B do not react with each other and
are assumed to be ideal. The gaseous mixture follows the equation
10. The mean free path of nitrogen molecules is
PV19/13 = constant, in adiabatic processes.
(a) 1.1 × 10–7 m (b) 1.1 × 10 –6 m
(c) 2.3 × 10–5 m (d) 4.5 × 10–8 m
4. The number of gram moles of the gas B in the gaseous mixture is
11. The collision frequency of nitrogen molecules is
(a) 2 g mole (b) 1 gm mole
(a) 4.6 × 107 Hz (b) 4.6 × 109 Hz
(c) 3g mole (d) 4 gm mole
(c) 5.6 × 108 Hz (d) 5.5 × 109 Hz
5. The speed of the sound in the gaseous mixture at
T = 300 K is 12. The time taken for a collision is
(a) 300.0 m/s (b) 400.7 m/s (a) 5 × 10–9 s (b) 6 × 10–12 s
(c) 480 m/s (d) 512 m/s (c) 4 × 10–13 s (d) 4 × 10–14 s
6. If T is raised by 1 K from 300 K, the percentage change in the
speed of sound in the gaseous mixture is
(a) 0.10 % (b) 0.20 %
(c) 0.17 % (d) none of these

Answer Key 1 (c) 3 (a) 5 (b) 7 (a) 9 (c) 11 (b)


Sol. from page 452 2 (c) 4 (a) 6 (c) 8 (b) 10 (a) 12 (c)
www.crackjee.xyz
KINETIC THEORY OF GASES 447

13. X and Y are two equal size containers. X contains 5 mole of H2 and Y has 10 mole of O2 at the same temperature. Assuming that the gases are
ideal, match columns
Column I Column II
A. In the container X (p) pressure of the gas is more
B. In the container Y (q) rms speed of gas molecules is more
C. Since the number of molecules is more (r) average thermal energy of a molecule is the same as that of a
molecule in the other container
D. Since the gas has a smaller molecular mass (s) internal energy of the system is more than that of the system in
the other container
14. Column I Column II
A. An ideal gas obeys gas equation (p) with decrease in pressure
B. A real gas behaves as an ideal gas at low pressure (q) at all temperature
C. Mean free path of molecules increases (r) same for all gases
(s) at high temperature
15. Match Column I (Physical Variables) with Column II (Expressions) and select the correct answer using the codes given below (n = number
of gas molecules present per unit volume, k = Boltzmann constant, T = absolute temperature, m = mass of the particle) :
Column I Column II
A. Most probable velocity (p) nkT
B. Energy per degree of freedom (q) (3kT / m)
C. Pressure (r) (2kT / m)
D. R.M.S. velocity (s) kT / 2
16. Column I Column II

A. v av 3RT
(p)
M

B. v rms 8RT
(q)
M

C. v mp Rt
(r)
M
2RT
D. vsound (s)
M

Answer Key 13 A (q, r); B (p, r, s); C (p, s); D (q) 15 A (r); B (s); C (p); D (q)
Sol. from page 452 14 A (r, s); B (r, s); C (p) 16 A (q); B (s); C (p); D (q)

Subjective Integer Type Exercise 5.5


Solution from page 454
1. An ideal gas is kept in a long cylindrical vessel fitted with a 2. A gas consisting of rigid diatomic molecules is expanded
frictionless piston of cross–sectional area 10 cm2 and weight 1 kg. adiabatically. How many time has the gas to be expanded to reduce
The vessel itself is kept in a big chamber containing air at the root mean square velocity of the molecules = 1.50 times.
atmospheric pressure 100 kPa. Ans : 7.6 times.
3. 0.014 kg of nitrogen is enclosed in a vessel at a temperature of
27°C. How much heat has to be transferred to the gas to double
the r.m.s. velocity of its molecules ? Ans : 9315 J.
4. A jar contains a gas and a few drops of water at TK. The pressure
in the jar is 830 mm of mercury. The temperature in the jar is
reduced by 1%. The saturated vapour pressure at the two
The length of the gas column is 20 cm. If the chamber is now temperatures are 30 mm and 25 mm of mercury. Calculate the
completely evacuated by an exhaust pump, what will be the length new pressure in the jar. Ans. 817 mm of mercury.
of the gas column? Assume the temperature to remain constant
throughout the process. Ans : 2.2 m.
www.crackjee.xyz
448 MECHANICS, HEAT, THERMODYNAMICS & WAVES

Subjective Exercise 6.6


Solution from page 454
1. A metre long narrow bore held horizontally (and closed at one temperature of 300 K. The tube is slowly heated and it is found
end) contains a 76 cm long mercury thread, which traps a 15 cm that the cork pops out when the temperature reaches 600 K. Let
column of air. What happens if the tube is held vertically with the dN denotes the magnitude of the normal contact force exerted by
open end at the bottom ? a small length dl of the cork along the periphery (see the figure).
Assuming that the temperature of the gas is uniform at any instant,
Ans. The mercury thread will decrease in length by 23.8 cm.
2. An air bubble of volume 1.0 cm3 rises from the bottom of a lake dN
calculate . Ans : 1.25 × 104 N/m.
40 m deep at a temperature of 12°C. To what volume does it d
grow, when it reaches the surface which is at a temperature of 6. In a certain region of space there are 5 molecules/cm3 on an average.
35°C ? The temperature there is 3 K. What is the average pressure of this
Ans. 5.275 ×10–6 m3 . very dilute gas ? Ans : 2.07 ×10–16 N/m2.
3. An oxygen cylinder of volume 30 litre has an initial gauge pressure 7. How will the rate of collision of rigid diatomic molecules against
of 15 atm and a temperature of 27°C. After some oxygen is the vessel’s wall will change, if the gas expanded adiabatically
withdrawn from the cylinder, the gauge pressure drops to 11 atm times ? Ans : rate decreases by (1+i)/i .
8. Two glass bulbs of equal volume are connected by a narrow tube
and its temperature drop to 17°C. Estimate the mass of oxygen
and are filled with a gas at 0°C and a pressure of 76 cm of mercury.
taken out of the cylinder, R = 8.3 J mol–1 K–1, molecule weight of
One of the bulbs is then placed in melting ice and the other is
oxygen = 32. Ans : 0.141 kg. placed in water bath maintained at 62°C. What is the new value of
4. Figure shows plot of PV / T versus P for 1.00 × 10–3 kg of oxygen the pressure inside the bulbs ? The volume of the connecting tube
gas at two different temperatures. is negligible. Ans : 83.75 cm of mercury.
9. Calculate the ratio of the speed of sound in neon to that in water
vapour at any temperature (molar weight of neon = 2.02 × 10–
2 kg / mole and molar weight of water vapour = 1.8 × 10–2 kg/
mole).
Ans : 1.06.
10. A monoatomic ideal gas expanded adiabatically to times of its
initial volume. Calculate ratio of final rate of collisions of molecules
with unit area of container’s walls to the initial rate.
Ans : 1 / 4/3.
(a) What does the dotted plot signify ? 11. The piston cylinder arrangement shown contains a diatomic gas at
(b) Which is true : T1 > T2 or T1 < T2 ? temperature 300 K. The cross–sectional area of the cylinder is
(c) What is the value of PV/T where the curves meet on the 1 m 2. Initially the height of the piston above the base of the
y – axis ? cylinder is 1 m. The temperature is now raised to 400 K at constant
(d) If we obtained similar plots for 1.00 × 10–3 kg of hydrogen, pressure.
would we get the same value of PV / T at the point where
the curves meet on the y – axis ? If not what mass of hydrogen
yields the same value of PV / T (for low pressure high
temperature region of the plot) ?
(Molecular mass of H2 = 2.02 u, of
O2 = 32.0 u, R = 8.31 J /mol–K)
Ans : (a) Ideal behaviour of the gas
(b) T1 > T2 (c) 0.26 J/K (d) 6.3 × 10–5 kg.
5. Figure shows a cylindrical tube of radius 5 cm and length 20 cm.
It is closed by a tight –fitting cork. The friction coefficient between
Find the new height of the piston above the base of the cylinder. If
the cork and the tube is 0.20.
the piston is now brought back to its original height without any
heat loss, find the new equilibrium temperature of the gas. You can
leave the answer in fraction.
4
Ans. m, 488.8 K.
3
12. The pressure of an ideal gas varies according to the law P = P0 –
AV 2, where P0 and A are positive constants. Find the highest
temperature that can be attained by the gas.
2 P0 P0
Ans. Tmax
3nR 3 A
The tube contains an ideal gas at a pressure of 1 atm and a
www.crackjee.xyz
KINETIC THEORY OF GASES 449

Solutions Exercise6.1 Level -1


1. (a) By Dalton’s law 15. (a) P = kT
P P1 P2 2.5 1 3.5 atmosphere P T
=
2. (a) P1 1 atm , V1 30 litre , T1 273 27 300 K P T
0.4 1
P2 = ? , V2 5.2 litre , T2 273 73 200 K or =
100 T
PV
1 1 P2V2 PV
1 1 T2 1 30 200 T = 250 K
Using , P2
T1 T2 T1 V2 300 5.2 V 2V
= 3.85 atmosphere. 16. (c) =
T T'
PV n1 PV / T T ' 2T 2(273 27) 600 K
3. (d) , 4 :1
RT n2 V = 600 – 273 = 327 °C
2P
4 17. (b) V1 A , T1 273 22 295 K , P1 = 200 kPa
2T
V2 A(1.02 ) T2 273 42 315 K ,
3RT 3 8.31 300
4. (a) vrms 1920 PV
1 1 T2 200 A 315
M M P2
T1 V2 295 1.02 A
M 2 10 3 kg 2g = 209 kPa
It is hydrogen. m
18. (b) 20 × V = R 300
8 RT M
5. (b) The average speed; v ; T .
M v m/2
6. (a) Each degree of rotation of diatomic molecule has energy and PV = R (273 50)
M
RT P = 10.8 atm
per mole. 19. (c) a represents correction for intermolecular forces and b for
2
finite size of molecule.
3 3
7. (d) E 300 ; E ' R(600) = 2E = 2 × 6.21 × 10–21 3
2 2 20. (a) E RT ; T 0, E 0.
= 12.42 × 10–21 J. 2

3R 300 3R 600 3RT


vrms ; v 'rms 2 vrms 21. (c) vrms ;T 0 , vrms 0.
M M M
= 682.44 m/s
8. (c) The average translational energy depends only on 22 32 42 52 62
22. (d) vrms 4.24
temperature. 5
9. (c) PV 1 RT and P 'V 1 R 2T R R 3R
23. (c) 0.67 Cv .
P ' 2P Cv 0.67 2
3 Thus gas is monoatomic.
10. (c) E RT , so it is same for all ideal gases at same
2 24. (a) cP– cv = R
temperature. or Mcp – Mcv = R
v1 T1 120 1 R R
cp cv =
11. (b) v2 T2 480 4 M 28
25. (a) Work done is to be done in expanding the gas at constant
or v2 2v1 pressure.
12. (b) PV = constant ; so graph between P and V will be a rectangular Q
hyperbola. 26. (c) C ; If T 0, C and if Q 0 , then C = 0
m T
13. (b) Q U W
1 mnc 2 and P ' 1 m
or Q U– U 0 ; It is an adiabatic process. 27. (b) P0 n (2c) 2 = 2 P0.
3 V 3 2
14. (c)
m m'
P(2V ) R 100 and PV R 200 1 2 2 3P
M M 28. (a) P c or c
3
M
m' . 1 2 1 8 104
4 E mc 1 3 = 3 × 104 J
2 2 4
www.crackjee.xyz
450 MECHANICS, HEAT, THERMODYNAMICS & WAVES
33. (b) Critical temperature is the temperature above which real gas
3 RT 3R 0 can not be liquify by applying pressure.
29. (a) vrms 0
M M 34. (a) PV = nRT
K.E is zero at zero Kelvin.
T
or V = nR nR tan
P
30. (b) Due to pseudo force the pressure at backside becomes greater
and at front side becomes smaller. As 2 1; V2 V1
31. (b) The force on the larger piston F = PA will be greater because
A is layer. So piston will move towards right. 35. (b) PV nRT ; PV = constant at constant temperature.
32. (b) With the increase in pressure, the gas deviates from its ideal
behaviour.

Solutions Exercise6.1 Level -2


n1 n2 n1 n2
1. (b) = aT 2 V c (RT b) P ( RT b)V c
– (aT 2 )V 1
–1 1 1 2–1 10. (a) P
V
2 1 1
or =5 7 On comparing with equation, P AV m – BV n,
–1 –1 –1
3 5 m = – c and n = –1.

3 11. (c) 1 2 45 cm
r = . The pressure must be same on both sides. Hence
2
3 1 2 1 2 2
2. (d) 1 eV = kT 1
2 T1 T2 273 273 273 2
3 23
or 1.6 10 19 = 1.38 10 T Also 1 2 90 ; 1 30cm and 2 60cm
2
T = 7.7 × 103 K. P1( 1 A) P( A)
3. (c) Friction is given by, Now =
T1 T
1 1 5
F = = = . P1 30 76 45
7 /5 7 or P1 102.4 cm .
273 273 81
2 2
4. (b) = 1 ; f
f 1 5 3
12. (d) E = 2 RT 4 RT = 11 RT..
5. (d) From conservation of energy, 2 2
E = E1 + E2 m P
13. (a) PV = RT ; Cm
3 3 3 M T
( n1 n2 ) kT = n1 kT1 n2 kT2
2 2 2 slope of B mB 3m
n1T1 n2T2 or = 3
T =
slope of A mA m
n1 n2
14. (a) Number of collisions can be calculated as
6. (b) Cv Mcv 4 3 12 J/mol-K n(2mvrms ) = PA
To doubling the pressure, the temperature will be doubled, so
T = T2 T1 273 K PA 3kT
= v
1
n 2mvrms ; rms m
Thus Q = nCv T 12 273 1638 J
2 15. (d) The real gas cannot be liquified above critical temperature
PV P2V2 by applying pressure.
P(V1 V2 ) 1 1
7. (c) = … (i) 16. (a) The change in internal energy depends only on initial and
T T1 T2
final state, so UI U II .
Also P(V1 V2 ) = PV 1 1 P2V2 … (ii)
After solving above equations, we get 500 600 700 800 900
17. (b) vav = 700 m/s
( PV 5
1 1 P2V2 )T1T2
T = PV and
1 1T2 P2V2T1
8. (c) With increase in temperature, the product (PV) increases.
5002 600 2 700 2 8002 900 2
3RT 3R(2T )
vrms = 714 m/s
9. (b) vrms ; and vrms 2vrms . 5
M M /2
Thus vrms is greater than average speed by 14 m/s.
www.crackjee.xyz
KINETIC THEORY OF GASES 451

3RT 3 8.31 273 2v


18. (b) vrms = 493 m/s 20. (c) (y g Pa ) = PaV
M 28 10 3 3

1 2 mgh
Thus mvrms . After substituting the values and
2 h
1
R y
simplifying, we get h = 12.4 km
5 1 2
19. (c) U = R T mv0 ;
2 2
m = 32 × 10–3 kg After substituting values and simplifying,
After substituting values and simplifying, we get we get y = 5.10 m
v0 36.03 m/s

Solutions Exercise 6.2


5R
3R 3RT 8 RT RT
1. (b, d) CP Cv = 4 R for monoatomic gas 6. (c,d) vrms ; v 2.55
2
2 M M M
7 R 5R
and CP Cv = 6 R for diatomic gas 2 RT
2 2 vp
Similarly CPCv is larger for diatomic gas. M
2. (a, c) Average translational energy of any ideal gas is 7. (a,c,d) Same volume means same molecules, same temperature
3
Etranslational = RT f
2 means same internal energy U nRT .
2
While total energy of diatomic molecule will be greater
than that of monoatomic. Average velocity is zero.
5RT 3 2 3 4 1
Ediatomic = , Emonoatomic RT 8. (a,b,c,d) Mean velocity = 0 ;
2 2 4
1 2 3 4 1
3. (a, c, d) v p /K Mean speed = 2.5 m/sec
273 4
3
E RT is same for all ideal gases at same tempera- 22 32 ( 4) 2 ( 1) 2
2 Mean square velocity v 2
4
kT
ture. The mean free path, ; so it increases 4 9 16 1
2 d 2P = 7.5 m²/sec²
with decrease in pressure. 4
m m
4. (b, c) PAV A nRT RT and PBVB RT Root mean square velocity vrms v2 7.5
MA MB
= 2.7386 m/sec.
So if PA PB , VAM A VB M B . 9. (a, b)
5. (a,b,c,d) Area under the curve is equal to number of molecules of (a) In an adiabatic expansion, internal energy decreases and
the gas sample. hence temperature decreases.
1 From equation of state of ideal gas PV = nRT,
Hence N av0 av0 2N
2 The product of P and V decreases.
v0 3kT
1 1 a 2 (b) vrms
vavg v N (v )dv C v dv v0 m
N N v0 3
0 0 (c) The temperature in kelvin scale is not doubled.
vavg 2 10. (b, d) T1 T2 ; So on expansion the temperature first increases,
v0 3 then decreases.
v0 P
2 1 1 a v02
vrms v 2 N (v ) dv v 2
v dv T1
N N v0 2
0 0
T2
vrms 1
v0 2
V
3
Area under the curve from 0.5 v0 to v0 is of total area.
4
www.crackjee.xyz
452 MECHANICS, HEAT, THERMODYNAMICS & WAVES
11. (b, d) As the temperature is same so vrms must be same. As
3 3
1 15. (b, c) K1 kT per molecule and K 2 PV
P mnC 2 ; so number of molecules are in the ratio of 2 2
3
Clearly both K1 and K2 are equal for hydrogen and oxygen.
their pressure.
3 PV 16. (b, c) For adiabatic process, PV constant (k)
12. (c, d) vrms =
M Also n P nV = nk
1 2 3
K Mvrms PV nP = – nV nk
2 2
As slope of x is greater than y, so
3
K' (2 P 2V ) 4K 4 3000 12000 J .
2 x > y

M1 M 2 28 44
13. (a, d) M = 36 R
1 1 2 As Cv = ; so Cv1 Cv2
1
n1 n2 n1 n2
= 2
mi x 1 1 1 2 1
1
Also =
f
1 1
4 4 1/ 4 1/ 4
= 2
1 1.4 1 1.3 1 or =
mi x f 1 ; f1 f2 .
2 1 1
or 1 = 1
mi x 0.4 0.3 17. (a, c) Mean free path, =
2 d 2n
mi x 1.343
PV N
n n ; which greatest for least value of n. n is least in
14. (a, c, d) PV = nRT ; ; As P,V and T are same, so n V
RT
must be same. case (a) and (c).
Both the gases are diatomic, so their internal energy is
equal.

Solutions Exercise-6.3
1. (a) Neutron is chargeless particle, so intermolecular force 6. (b) An atom can have only translatory motion, so its degrees of
between them is neglegible small. 1 2 1 2 1 2
2. (d) Internal energy can be increased when molecules of gas will freedom can be mvx , mv y , mvz .
2 2 2
get greater velocity w.r.t. container.
7. (d) The specific heat of a gas can be from 0 to .
3. (c) The heat will be extracted from body in evaporation of
8. (d) At absolute zero temperature, K.E. of the gas becomes zero.
water.
But internal energy may be due to its internal potential
4. (b) In pressure cooker, boiling point is increased due to increased
energy.
pressure. Aluminium is a good conductor of heat.
9. (a) At high temperature and low pressure (low density), real gas
5. (a) The P-T diagram is shown in figure.
behaves like an ideal gas.
P 3kT
10. (b) PV constant and vrms
m

11. (a) CP Cv = Work done


= P V R T R 1 R.
T

Solutions Exercise-6.4
Passage (Q.1– 3)
1. (c) If n be the number of cylinders then PVM 1.013 105 500 2 10 3
m = 44.5 kg
PV = n P V RT 8.31 273
The total weight supported
or 1.013 105 500 = n 15 105 0.05
n = 675. = Fb – mg V air g mg
m = 500 1.3 9.8 44.5 9.8
2. (c) PV nRT PV RT = 5934 N
M
www.crackjee.xyz
KINETIC THEORY OF GASES 453
m PVM 8. (b) Mean kinetic energy per atom
3. (a) PV RT m
M RT 3
= kT
5 3 2
1.013 10 500 4 10
= 89 kg 3
8.31 273 = × 1.38 × 10–23× 160
The total weight supported by helium balloon 2
= 500 ×1.3 ×9.8 – 89 × 9.8 = 3.312 × 10–21 J.
= 5498 N. m
9. (c) We know that PV = RT
Passage (Q. 4 - 6) : M
4. (a) of the mixture is given by PVM
m =
n1 n2 n1 n2 RT
= 1 1
1 1 2 100 1 4
=
25
1 n2 1 n2 160
or = 3
1 5 7
1 1 = 0.3 g Ans.
3 5
n2 = 2 g mole Ans. Passage (Q. 10 - 12) :

rRT
5. (b) V =
M 10. (a) Here T = 273 + 17 = 290 K
n1M1 n2 M 2 d = 2 × 1.0 × 10–10 m
Here, M = n1 n2 P = 2.0 atm = 2 × 1.013 × 105 N/m2
k = 1.37 × 10–23 J/K
1 4 2 32 68
= = kT
1 2 3 Mean free path,
2 d 2P
(19 /13) 8.31 300
v = 3 23
(68/ 3) 10 1.37 10 290
10 2
= 400.7 m/s Ans. 2 (2 10 ) (2 1.013 105 )
RT = 1.1 × 10–7 m
6. (c) As v =
M 11. (b) Root mean square speed of nitrogen molecules
v 1 T 3RT 3 8.31 290
= vrms 5.1 102 m / s
v 2 T M 28 10 3

1 1
= 1 1 vrms
2 300 The frequency of collision n
= 0.17% t ( / vrms )
Passage (Q. 7 - 9) :
7. (a) The time between successive collisions, 5.1 102
7
4.6 109 Hz
1 1.1 10
t = s
500 diameter of molecule
The root mean square speed 12. (c) Time taken for the collision
vrms
2
vrms = 10
t 2.0 10 13
4 10 s
2 1 5.1 102
=
1/ 500 13. (A) q, r; (B) p, r, s; (C) p, s; (D) q
= 1000 m/s. 14. (A) r, q ; (B) r,s; (C) p
By the definition of rms speed, (A) All ideal gases obey gas equation at all temperature.
3RT (B) All real gases obey gas equation at high temperature, at which
vrms = intermolecular forces are zero.
M
3RT kT
or 100 = (C) Mean free path, ; so it decreases with increase
M 2 d 2P
in pressure.
3 25 / 3 T
or 100 = 3 15. (A) r; (B) s; (C) p ; (D) q
4 10
16. (A) q; (B) p; (C) s ; (D) r
T = 160 K
www.crackjee.xyz
454 MECHANICS, HEAT, THERMODYNAMICS & WAVES

Solutions Exercise-6.5
1. The pressure exerted by the weight of the piston For diatomic gas, = 5
mg and = 1.5
= After simplifying, we get
A
V2 = 7.6V1 Ans.
1 10
=
10 10 4 v1 T1
= 1× 104 N/m2 3. We have v2 = T2
Thus initial pressure of the gas
P 1 = 100 × 10–3 + 104 T1
= 110 × 103 N/m2 1
= T2
V1 = A 1 2
= A × 0.20 or T2 = 4 T1
P 2 = 104 N/m2
Using P1V 1 = P2V 2 T = T2 – T1
or 3T 1 = 3 × (273 + 27)
or P1(A 1) = P2 (A 2)
= 900 K
P1 1
2 = The heat required Q = Cv T
P2
14 5
110 103 = 8.31 900
= 0.20 28 2
1 10 4 = 9315 J Ans.
= 2.2 m Ans. 4. The initial pressure of gas
2. v1 and v2 are the rms velocities at temperatures T 1 and T 2
P1 = 830 mm – 30 mm
respectively, then
= 800 mm
v1 T1 and T1 = T
v2 = T2 If P is now pressure in the jar, then final pressure of the gas
P 2 = (P – 25)
T1
or = T
T2 and T2 = T
100
T1
T2 = 2
P1 P2
We know that T1 = T2
For the adiabatic process,
1
T1 800 P 25
V2 or =
= T2 T T
V1 T
100
= 2
P = 817 mm of Hg Ans.

Solutions Exercise-6.6
1. Suppose the air column in the tube becomes (15 + x), then 2. At the bottom of the lake, V1 = 1 cm3,
Pa P1 = Pa + 40 m
Air P1 = 10.3 + 40
= 50.3 m of water.
15 cm 76 cm T1 = 273 + 12
V1 = A × 15, = 285K.
P1 = Pa P2 100 – x At the surface, P2 = Pa
= 10.3 m of water.
T2 = 273 + 35
= 308 K.
V2 = A (15 + x),
x PV
1 1 P2V2
Using =
P2 + x = P a T1 T2
P 2 = (Pa – x) PV
1 1T2
By using Boyle's law, we have P1V1 = P2V2 Pa or V2 =
P2T1
or Pa× (15A) = (Pa – x) (100 – x) A
or 76 × 15 = (76 – x) (100 – x) 50.3 1 10 6 308
or x2 –176x + 6460 = 0 =
10.3 285
After solving, x = 52. 17 cm
= 5.275 × 10–6 m3 Ans.
Thus mercury thread will decrease in length by 23.8 cm.
www.crackjee.xyz
KINETIC THEORY OF GASES 455
3. Given, P1 = 15 + 1 fr= (P2 – Pa) A
= 16 atm (absolute) = 1 atm × A
V1 = 30 × 10–3 m3, = (1.013 × 105) × r2
T1 = 273 + 27 If normal reaction N is distributed uniform over the perimeter,
= 300 K. then
P2 = 12 atm,
T2 = 273 + 17 dN
= 290 K. fr = 2 r
dl
m
We have PV = RT dN fr
M =
dl 2 r
PVM
1
m = RT1 1.013 105 r2
=
0.2 2 r
(16 1.013 105 ) (30 10 3 ) (32 10 3 )
= 8.31 300 1.013 105 0.05
=
= 0.624 kg. 0.2 2
P2VM = 1.25 × 104 N/m Ans.
Finally m' = 6. We know that, for n molecules
RT2
PV = nkT
(12 1.013 105 ) (30 10 3 ) (32 10 3 ) n
= P = kT
8.31 290 V
= 0.484 kg = ( 5 × 106) × ( 1.38 × 10–23)×3
The mass of the oxygen taken out = m – m' = 2.07 × 10–16 N/m2 Ans.
= 0.624 – 0.484 7. In adiabatic process,
= 0.141kg. Ans.
V2 1T2 = V1 1
T1
PV
4. (c) The value, = R 1
T T2 V1
T1 =
1 V2
= 8.31
32 1
1
= 0.26 J/K Ans. = …(i)
PV
(d) For hydrogen, = R The number of molecules in unit volume
T
N
1 n =
= 8.31 V
2
= 4.15 J/K Ans. V
= RT
For the same value as in case (c), we have
P
m
0.26 = M H × 8.31 P
=
R
m = 6.3 × 10–2 g T
= 6.3 × 10–5 kg Ans. N

P1 P2 P
5. By gas law, = =
T1 T2 kT
The volume swept by the molecules in 1 second
T2 = A × distance travelled in 1 sec.
P2 = T1 P1 = A × vrms
These molecules move along six possible directions : x, y and
600 z.
= 1
300 1
Thus in any direction, the numbers are = nAvrms
= 2 atm 6
For the equilibrium of the cork, we have These are the number of collisions per second
P2 A = fr + Pa A
3RT
As vrms =
M
fr
P2 1 P 3RT
Pa A =
6 kT M
www.crackjee.xyz
456 MECHANICS, HEAT, THERMODYNAMICS & WAVES

1 N 3RT 10. See the solution of quesion 7.


Also A = V1 V2
6 V M
11. At constant pressure T = T2
1
T
A A1 A 2
V2 or =
T1 T2
A1 T1 V22 T2
Thus = …(ii)
A2 T2 V12 2 = T1
1

From equations (i) and (ii), we get 400


A1 = 1
1 2 300
A2 =
4
= m Ans.
1 3
2 Again by TV –1 = constant, we have
= Ans.
In the answer given in exercise, i used for number of degrees of 1
= V2
freedom. T3 T2
8. By conservation of matter, we have V3
= 1 2 = 1.4 1
400 4 / 3
PV P 'V P 'V = 488.8 K. Ans.
or 2 = 12. The ideal gas equation
RT RT1 RT2
PV = nRT
2P 1 1 Given P = P0 – AV2,
or = P' (P0 – AV2)V = nRT
T T1 T2 or P0V – AV3 = nRT .....(i)
Differentiating above equation w.r.t. V, we have
2P T1T2
P' = dT
T T1 T2 P0 – 3AV2 = nR .
dV
2 76 273 (273 62) For highest value of T,dT/dV = 0
= or P0 – 3AV2 = 0
273 273 (273 62)
= 83.75 cm of mercury. Ans. P0
V =
9. The speed of sound in any gas is given by 3A
Now from equation (i), we have
RT
v = . P0 P0
M
P0 A
Thus for neon and water vapour, we have 3A 3A = nRTmax

vneon neon
M water vap 2 P0 P0
= Tmax = Ans.
vwater vap water vap M neon 3nR 3 A

1.67 1.8
=
1.33 2.02
= 1.06 Ans.
www.crackjee.xyz
www.crackjee.xyz
458 MECHANICS & THERMODYNAMICS

Thermodynamics versus mechanics


Mechanics deals with motion of the body (system) as a whole and its external K.E. and
P.E. Thermodynamics deals with the motion of particles of the body and its internal K.E.
and P.E.

7.1 THERMODYNAMICAL TERMS


(i) Thermodynamical system: A thermodynamical system is an assembly of large
number of particles which can be described by thermodynamic variables like
pressure (P), volume (V), temperature (T).
(ii) Surroundings: Everything outside the system which can have a direct effect on
the system is called surroundings. The gas cylinder in the kitchen is the
Fig. 7.1 thermodynamic system and the relevant part of the kitchen is the surroundings.
(iii) An adiabatic wall: The wall which prevent the passage of matter and energy.
(iv) Diathermic wall: It prevent the passage of matter but allow the passage of energy.
An aluminium can is an example of a container whose walls are diathermic.
(v) An isolated system: In this type of system neither the mass nor the energy can be
exchanged with the surroundings.
(vi) Equation of state: The relationship between the pressure, volume and temperature
of the thermodynamical system is called equation of state.
For example: PV = nRT.

7.2 INTERNAL ENERGY


There are intermolecular forces in real gases, so they possess internal potential energy
(Up ). If the volume of the gas increases, work is to be done by the gas against
k
intermolecular attraction and so its potential energy increases Up . The
r
molecules of a gas are always in motion. The motion may be translational, rotational or
vibrational. Hence molecules of gas possess internal kinetic energy (Uk). With increase
in temperature the average kinetic energy of the gas molecules also increases.
3
[ Uk RT (monoatomic gas)].
2
Thus internal energy of a system is the sum of its internal kinetic and internal potential
energy. The internal energy (U) can be written as:
U = Uk + UP..

7.3 INTERNAL ENERGY OF AN IDEAL GAS


In ideal gases there are no intermolecular forces, so internal potential energy of an ideal
gas is zero. The internal energy of an ideal gas is only due to its internal kinetic energy,
so we can write U = Uk. As internal kinetic energy is the function of temperature, so
internal energy of an ideal gas depends only on temperature.
More about internal energy
1. Internal energy of an ideal gas;
3
U = RT for monoatomic gas
2

5
U = RT for diatomic gas
2
www.crackjee.xyz
LAWS OF THERMODYNAMICS 459

2. If the temperature of the gas changes by T, then change in internal energy of the
gas
U = nCV T .
For one mole U = CV T

U dU
CV = or CV =
T dT

3. In isothermal process, U = 0 and hence U = 0.


4. In cyclic process, U = 0 and hence U = 0.
5. For an isolated system, Q = 0, W = 0and hence U = 0.
6. The change in internal energy depends only on the initial and final states of the
system.

7.4 WORK IN VOLUME CHANGE


Let us consider a gas or liquid contained in a cylinder equipped with a movable piston,
as shown in Fig. 7.2. Suppose that the cylinder has a cross-sectional area A and the Fig. 7.2
pressure exerted by the gas at the piston is P.
The force exerted by gas on the piston
F = PA
If the piston moves out a small distance dx, the work done

dW = Fdx = PAdx
= PdV
where dV = Adx, is the change in volume of the gas.

The total work done by the gas when its volume changes from Vi to Vf
Vf

W = PdV
Vi

If the pressure remain constant while volume changes, then the work done

W = P(V f Vi ) P V

Indicator diagram
The state of a thermodynamical system can be understand completely if only two Fig. 7.3
thermodynamical variables are known because the third variable gets automatically
fixed by PV = nRT. The graphical representation of the state of system with the help of
two variables is called an indicator diagram.
Cyclic process and non - cyclic process
If a system having gone through a change, returns to its initial state then process is
called a cyclic process. If system does not return to its initial state, the process is called
non-cyclic process.
Work done from P - V diagram
Fig. 7.5 shows a P - V diagram for a system under going expansion from the state A
(Pi,Vi) to B (Pf ,Vf). Suppose that the volume increases by small amount dV in which
pressure assumed to be constant (P). Fig. 7.4
The work done in small change in volume
dW = PdV
= area of the shaded strip
www.crackjee.xyz
460 MECHANICS & THERMODYNAMICS

The total work done can be obtained by adding the areas of all such strips between C
and D.
Thus work done
W = area ABCD
Sign conventions
Work done by the system (expansion) taken as position. Work done on the system
(compression) taken as negative.

7.5 WORK DONE IN CYCLIC PROCESS


Suppose gas expands from initial state A to final state B via the path AXB.
The work done in this expansion
Fig. 7.5 WX = + area A XBCDA
Now gas returns to its initial state B via path BYA.
Work done during this compression
WY = – area BYADCB
The net work done
W = WX + WY
= area AXBCDA – area BYADCB
= + area AXBYA
Thus for a cyclic process
(i) Work done in complete cycle is equal to the area of the loop representing the cycle.
(ii) If the closed loop is traced in the clockwise direction, the expansion curve lies
above the compression curve. (WX >WY), the area of loop is positive.
(iii) If the closed loop is traced in the anticlockwise direction, the expansion curve lies
Fig. 7.6 below the compression curve (WX <WY), the area of the loop is negative.
More cyclic processes

Fig. 7.7 Fig. 7.8

Fig. 7.9 Fig. 7.10


www.crackjee.xyz
LAWS OF THERMODYNAMICS 461

Work done in non-cyclic process

Fig. 7.11 Fig. 7.12 Fig. 7.13

Ex. 1 A gas expands in a piston - cylinder device from volume V1 RT


2
R 2T 2
a T = T0 = T0
to V2, the process being described by P b , a and b are P P2
V
2
constants. Find work done in the process. or TP2 = T0 P R 2T 2
Sol. or P = RT (T T0 ) –1/ 2 ...(ii)
Work done After differentiating , we get
V2
PdV dP 1/ 2 1 3/ 2
W = = R (T T0 ) T (T T0 )
V1 dT 2
For minimum pressure,
V2
a dP
= b dV = 0
V1
V dT
1/ 2 1 3/ 2
0 = R (T T0 ) T (T T0 )
V2 2
= a nV bV V1 After solving , T = 2T 0
= ( a nV2 bV2 ) (a nV1 bV1) From equation (ii)
Pmin = 1/ 2
R.2T0 (2T0 T0 )
V2
= a n V b(V2 V1 ) Ans.
1 = 2R T0

Ex. 2 Find the minimum attainable pressure of ideal gas in the

process T T0 V 2 , where T0 and are positive constants and V


is the volume of one mole of gas. Draw the approximate P vs V plot
of this process.

Sol.
Given , T = T0 V2 ...(i)
For one mole of a gasPV = RT
RT
or V =
P
Substituting this value in equation (i), we get Fig. 7.14

7.6 FIRST LAW OF THERMODYNAMICS


The first law of thermodynamics is based on conservation of energy. According to this
law heat Q supplied to a system is equal to the sum of the change in internal energy ( U)
and work done by the system (W). Thus we can write
Q = U+ W
More about first law of thermodynamics :
1. Heat supplied to the system taken as positive and heat given by the system taken
as negative.
2. It makes no different between heat and work. It does not indicate that why the
whole of heat energy cannot be converted into work.
www.crackjee.xyz
462 MECHANICS & THERMODYNAMICS
3. Heat and work depend on the initial and final states but on the path also. The
change in internal energy depends only on initial and final states of the system.
4. The work done by the system against constant pressure P is W = P V. So the first
law of thermodynamics can be written as Q U P V .
5. Differential form of the first law;
dQ = dU + dW
or dQ = dU + PdV.

Ex. 3 1.0 m3 of water is converted into 1671m3 of steam at Ex. 5 A system is taken from an initial state i to a final state
atmospheric pressure and 100°C temperature. The latent heat of f as shown in the Fig. 7.15. When it goes from i to f via iaf it is found
vapourisation of water is 2.3×10 6 J/ kg. If 2.0 kg of water be that Q = 70 cal and W = 40 cal, and along the path ibf, Q = 40 cal.
converted into steam at atmosph eric pressure and 100°C
temperature, then how much will be the increase in its internal
energy? Density of water is 1.0×103 kg/m3, atmospheric pressure
= 1.01×105 N/m2.
Sol.
Heat given to water to change into steam
Q = ML 2.0 2.3 106
= 4.6 × 106J
Mass 2.0 Fig. 7.15
Volume of water V = density
103 (a) What is W along the path ibf ?
= 2.0×10–3 m3 (b) If W = –15 cal for the curved path fi, what is Q for the path?
Volume of steam formed will be (c) If internal energy at i (Ui) =20 cal, what is Uf.
= 2.0×10–3×1671 (d) If Ub = 25 cal, what is Q for the process ib?
= 3342 ×10–3m3
Sol.
The change in volume in the process
From first law of thermodynamics
V = V 3342 10 3 2.0 10 3 U = Q–W
= 3340 ×10–3m3 or Uf – Ui = 70 – 40 = 30 cal
The work done against the atmospheric pressure
As U is independent of path, so it remain same for ibf or if.
W = P V
= (1.01×105)×(3340×10–3) (a) Along path ibf, Q = + 40 cal, U +30 cal
= 0.337 ×106J As Q = U+W
By first law of thermodynamics W = Q – U = + 40 – 30 = 10 cal
Q = U+W (b) For the return path fi,
U = Q–W
U = Ui – Uf = – (Uf –Ui) = – 30 cal
= 4.6×106 – 0.337×106
= 4.263×106 J` Ans. and W = –15 cal
Here positive value of U indicates that internal energy in the process Q = W+ U
increases. = – 15 – 30 = – 45 cal
Ex. 4 At 0°C and normal atmospheric pressure, the volume of (c) U i = 20 cal
1g of water increases from 1cm3 to 1.091 cm3 on freezing. We have, Uf – Ui = + 30
What will be the change in its internal energy? Normal atmospheric
U f = 30 + Ui = 30 + 20 = 50 cal
pressure is 1.013×10 5 N/m2 and latent heat of fusion of ice
= 80 cal/ g. (d) Ui = 20 cal and Ub = 25 cal
Sol. U = Ub– Ui = 25 – 20 = 5 cal
Heat released by water on freezing Wib = Wibf = 10 cal
Q = – mL Q = U +W
= – 1× 80 = – 80 cal = – 336 J = 5 + 10 = 15 cal
During freezing water expands against the atmospheric pressure. Hence
work done in the process Ex. 6 Air is contained in a piston - cylinder arrangement as
5
W = P V = (1.013 10 ) (1.091 1) 10 6 shown in Fig. 7.16 with a cross - sectional area of 4 cm2 and an
initial volume of 20 cc. The air is initially at a pressure of 1 atm
= 0.092 J and temperature of 20°C. The piston is connected to a spring whose
By first law of thermodynamics spring constant is k = 10 4 N/m, and the spring is initially
Q = U+W undeformed. How much heat must be added to the air to increase
or –336 = U + 0.092 the pressure to 3 atm. (For air, CV = 718 J/kg°C, molecular mass of
U = –336.092 J Ans. air 28.97)
www.crackjee.xyz
LAWS OF THERMODYNAMICS 463

Fig. 7.16
Fig. 7.17
Sol.
When pressure changes from 1 atm to 3 atm , the change in pressure From C to A, V = 0 WCA= 0
P = 2 atm For the whole cycle
= 2 × 1 × 105 N/m2
The force exerted on the piston WAB + WBC + WCA = W
F = PA = 2×105 × 4 ×10–4 = –1200
= 80 N As WCA = 0, WAB + WBC = –1200 J ...(i)
The compression of the spring Work done from A to B :
In the process V T , so pressure remains constant.
F 80
x = = 4 = 0.008m We know that PV = nRT
k 10
or P V = nR T
The change in volume of the air due to displacement of piston by x
WAB = P V = nR T = 2 ×8.31× (500 – 300)
V = Ax = 4 10 4 0.008
= 3324 J
= 3.2 ×10–6m3 Substituting this value in equation (i), we get
Final volume V2 = V1+ V
3324 + WBC = –1200
= 20 × 10–6 + 3.2 × 10–6
= 23.2 × 10–6 m3 W BC = – 4524 J Ans.
By equation of state
PV
1 1 P2V2 Ex. 8 Fig.7.18 shows the variation in the internal energy U
T1 = T2 with the volume V of 2.0 mole of an ideal gas in a cyclic process
abcda. The temperature of the gas at b and c are 500 K and 300 K
P2V2T1 respectively. Calculate the heat absorbed by the gas during the
T2 = PV process.
1 1

3 (23.2 10 6 )
= (273 20)
1 (20 10 –6 )
= 1020 K
The change in internal energy of air
U = mCV T
= (2.38 × 10–5) × 718 × (1020 – 293)
= 12.42 J
Work done in compressing the spring by x
1 2 10 4
W = kx = (0.008)2 = 0.32 J
2 2 Fig. 7.18
From first law of thermodynamics Sol.
Q = U + W =12.42 +0.32 = 12.74 J Ans. In the process a to b and c to d
Ex. 7 Consider the cyclic process ABCA, shown in Fig. 7.17 As U = 0, T = 0 or T constant
performed on a sample of 2.0 mole of an ideal gas. A total of 1200 J Vf
of heat is withdrawn from the sample in the process. Find the work
W = PdV
done by the gas during the part BC.
Vi
Sol.
In the cyclic process nRT
We have PV = nRT P =
U = 0 V
From first law of thermodynamics for the cyclic process
Q = U+W Vf
dV
W = Q – U = –1200 – 0 W = ( nRT )
= –1200 J Vi
V
www.crackjee.xyz
464 MECHANICS & THERMODYNAMICS

Wbc = Wda = 0
V Vf
= nRT | nV |V f = nRT n The work done in complete cycle
i Vi W = Wab + Wbc + Wcd + Wda
= 1000 R n2 + 0 – 600 R n2 + 0
2V0
Wab = nRTb n V = 2R × 500 n 2 = 1000 R n2 = 400 R n2
0
From first law of thermodynamics
V0 1 Q = U+W
and Wcd = nRTc n 2V = 2 R 300 n = 600 R n2 = 0 + 400 R n2
0 2
= 400 R n2 Ans.
There is no volume changes from b to c and from d to a, so

Quasi-static process
Quasi-static process is the nearly static process. At every state, the temperature and
pressure of the system differ only infinitesimally from those of the surroundings. A
quasistatic process is an ideal concept that is applicable to all thermodynamic systems
including electric and magnetic systems.
Consider the expansion of a gas in a closed cylinder fitted with a piston. Initially weights
are put on the piston and the pressure of the gas inside the cylinder is higher than the
atmospheric pressure. If the weight are small and are taken off slowly one by one, the
Fig. 7.19 process can be considered quasistatic. If, however, all the weights are removed at once,
the expansion takes place suddenly and it will be a non-equilibrium process (non quasi-
static).

7.7 REVERSIBLE PROCESS


Any process which return to its initial state of thermodynamical variables at each stage
of variation as it proceed in direct process is called reversible process. A complete
reversible process is an ideal concept as it can never be realised in practice because
dissipative forces cannot be completely eliminated.
(i) The process must be quasi - static.
(ii) The dissipative forces such as viscosity, friction etc. must be absent.
(iii) The work done in complete cycle will be zero.
Examples:
(i) The gradual extension and compression of an elastic spring is approximately
Fig. 7.20 Reversible process reversible.
(ii) The electrolysis process is reversible if internal resistance is negligibly small.
(iii) Slow compression and expansion of an ideal gas at constant temperature.

Irreversible process
A process is said to be irreversible if it cannot be retraced back exactly in the opposite
direction. During an irreversible process, work is always done to overcome friction.
Energy is also dissipated in the form of conduction and radiation. Most of the process
occurring in nature are irreversible.
Examples :
(i) Dissolution of salt in water.
(ii) Diffusion of gases.
(iii) Sudden expansion or compression of gas.
(iv) Passage of an electric current through a resistance.
(v) Rain fall.
(vi) Rusting of iron.

7.8 THERMODYNAMICAL PROCESSES


Any process may have own equation of state, but each thermodynamical process must
obey PV = nRT.
www.crackjee.xyz
LAWS OF THERMODYNAMICS 465

1. Isobaric process
If a thermodynamic system undergoes physical change at constant pressure, then
the process is called isobaric.
(i) Isobaric process obeys Charle’s law, V T
dP
(ii) Slope of P ~ V curve, = 0.
dV
(ii) Specific heat at constant pressure
5R 7R
CP = for monoatomic and CP = for diatomic
2 2
Fig. 7.21
(iv) Bulk modulus of elasticity: As P is constant, P = 0

P
and B = =0
V
V
(v) Work done: W = P V nR T
(vi) First law of thermodynamics in isobaric process
Q = U W = U P V = U nR T
= nCV T nR T = n(CV R) T
= nCP T Fig. 7.22
(vii) Examples: Boiling of water and freezing of water at constant pressure etc.

2. Isochoric or isometric process


A thermodynamical process in which volume of the system remain constant, is
called isochoric process.
(i) An isochoric process obeys Gay - Lussac’s Law, P T
dP
(ii) Slope of P – V curve, =
dV
(ii) Specific heat at constant volume
3R 5R
CV = for monoatomic and CV = for diatomic
2 2
(iv) Bulk modulus of elasticity : As V is constant, V = 0 Fig. 7.23

P
B = = `
V
V
(v) Work done : W = P V=0
(vi) First law of thermodynamics in ischoric process
Q = U + W = U+ 0
or Q = U
= nCV T
3. Isothermal process
A thermodynamical process in which pressure and volume of the system change
at constant temperature, is called isothermal process.
(i) An isothermal process obeys Boyle’s law PV = Constant.
(ii) The wall of the container must be perfectly conducting so that free exchange
of heat between the system and surroundings can take place.
(iii) The process must be very slow, so as to provide sufficient time for the
exchange of heat.
www.crackjee.xyz
466 MECHANICS & THERMODYNAMICS
(iv) Slope of P – V curve:
For isothermal process
PV = Constant
After differentiating w.r.t. volume, we get
dP
P V = 0
dV
dP P P
or = or tan =
dV V V
(v) Specific heat at constant temperature:
As T = 0,
Fig. 7.24 Q
C = =
n T
(vi) Isothermal elasticity: Bulk modulus at constant temperature is called isothermal
elasticity. It can be
defined as
P dP
Eiso = B = =
V dV
dP V V
From above = P
dV
V

Eiso = P
Vf

(vi) Work done : W = PdV


Vi

nRT
By PV = nRT P =
V
Vf
dV
W = nRT
V
Vi

V
nRT nV V f
i

Vf
or W = nRT n
Vi

Vf
Here Vi is called expansion ratio.

Vf Pi
Also PiVi = PfVf, Pf
Vi

Vf Pi
W = nRT n V = nRT n P
i f

(viii) First law of thermodynamics in isothermal process.


As T = 0, U=0
Q = U +W= 0 +W
or Q = W
www.crackjee.xyz
LAWS OF THERMODYNAMICS 467

4. Adiabatic process
An adiabatic process is one in which pressure, volume and temperature of the
system change but heat will not exchange between system and surroundings.
(i) Adiabatic process must be sudden, so that heat does not get time to exchange
between system and surroundings.
(ii) The walls of the container must be perfectly insulated.
(iii) Adiabatic relation between P and V
According to first law of thermodynamics
dQ = dU + dW
For adiabatic process, dQ = 0, dU + dW = 0 ...(1)
For one mole of gas
dU = CVdT and dW = PdV
Substituting these values in equation (1), we have
CVdT + PdV = 0 ...(2)
For one mole of an idea gas,
PV = RT ...(3)
After differentiating equation (3), we get
PdV + VdP = RdT
PdV +VdP
or dT =
R
From equation (2)
PdV VdP
CV PdV = 0
R
or CV PdV + CVVdP + RPdV = 0
or (CV + R) PdV + CVVdP = 0
or CPPdV + CVVdP = 0
After rearranging, we get
CP dV dP
= 0
CV V P

CP
Substituting = , we have
CV

dV dP
or = 0 ...(4)
V P
Integrating equation (4), we get
dV dP
= C
V P
or nV nP = C
or nV nP = C

or n( PV ) = C
or PV = eC
or PV = k
Adiabatic relation between V and T & P and T
For one mole of gas
RT
PV = RT, or P =
V
Substituting in PV = k, we get
www.crackjee.xyz
468 MECHANICS & THERMODYNAMICS

RT
V = k
V
k
or V 1
T = = new constant
R
RT
Also V =
P

RT
P = k
P

k
or P1 T = = another constant
R

(iv) Slope of P – V curve : We have PV k . On differentiating,


1 dP
we have P V V = 0
dV
dP P
or =
dV V
Fig. 7.25
P
or tan =
V

As slope of isothermal curve =


P
V

Slope of adiabatic curve = × slope of isothermal curve.


Since > 1, so slope of adiabatic always be greater than slope of isothermal
curve.
Q 0
(v) Specific heat : C = = =0
n T n T
Fig. 7.26 (vi) Adiabatic elasticity: Bulk modulus of gas at constant heat is called adiabatic
elasticity. If can be defined as
P dP
Ead = B= =
V dV
V V

dp
As = P
dV
V

Ead = P

As Eiso = P, Ead = Eiso


Vf

(vii) Work done : W = PdV


Vi

For adiabatic process PV = i i = Pf V f = k


PV
or P = kV –
www.crackjee.xyz
LAWS OF THERMODYNAMICS 469

Vf

W = kV dV
Vi
Vf
V1
= k (1 )
Vi
1
= (1 [kV f 1 kVi1 ]
)
1
= [( Pf V f )V f 1 ( PV
i i )Vi
1
]
(1 )
1
= Pf V f PV
i i
(1 )

( PV
i i Pf V f )
or W =
( 1)

Also PiVi = nRTi and PfVf = nRTf

nR
W = [Ti T f ]
1

(viii) First law of thermodynamics in adiabatic process

Q = U+ W
As Q = 0, U=– W
or Uf – Ui = – W
Uf = Ui – W
Summary of four gas processes :
Fig. 7.28 shows four different processes : Isobaric, isothermal, adiabatic and isochoric.
Constant Process Some results
Path
quantity type U = Q – W and U = nCV T for all paths
1 P Isobaric Q = nCP ΔT ; W = PΔV
2 T Isothermal Q W nRT n(V f / Vi ); U 0
1
3 PV , TV Adiabatic Q 0; W U Fig. 7.28 P – V diagram representing
4 V Isochoric Q U nCV T ; W 0 four different processes for an ideal gas

7.9 CHANGE IN INTERNAL ENERGY


1. In boiling : Suppose m mass of liquid transforms into vapour at boiling point. Its
volume changes from Vi to Vf. Thus work done by the liquid
W = P V = P(Vf – Vi)
If L be the latent heat of vapourisation of liquid, then amount of heat required,
Q = mL.
Now from first law of thermodynamics
Q = U+ W
or mL = U+ P (Vf – Vi)
U = mL – P (Vf – Vi)
www.crackjee.xyz
470 MECHANICS & THERMODYNAMICS

2. In melting : Suppose m mass of solid transforms into liquid at melting point. In this
process change in volume is negligibly small, V 0 , so W = 0.
If L be the latent heat of fusion of solid, then amount of heat required, Q = mL.
Now from first law of thermodynamics
Q = U+ W
Fig. 7.27 or mL = U 0
U = mL
3. In free expansion : Consider two insulated vessels, one contains a gas and other is
evacuated (see Fig. 7.27). When stop-cock is opened, the gas rushes into the
evacuated vessel and expand freely. As process is sudden (adiabatic)
so Q = 0, also gas expands freely, so
W = P V = 0 × V = 0.
From first law of thermodynamics
Q = U+ W
or 0 = U+0
U = 0
Thus in free expansion internal energy of the gas will not change.

7.10 POLYTROPIC PROCESS


A process PVr = constant is called polytropic process, in which r 1 or .
(i) Work done in polytropic process: As we have calculated in adiabatic process, here
also work done
nR
W = Ti T f
r 1
nR
= Tf Ti
r 1
nR T
or W =
r 1
For one mole of a gas n = 1
R T
W =
r 1
(ii) Specific heat : If C is the molar specific heat, then heat required to increase the
temperature of one mole of a gas by T
Q = C T
Form first law of thermodynamics
Q = U+ W
R T
or C T = CV T
r 1
R R R
C CV –
r 1 1 r 1

Ex. 9 An ideal gas expands according to law PV2 = constant. Ex. 10 An ideal gas (Cp/Cv = ) is taken through a process in
What is the value of molar heat capacity? which the pressure and the volume vary as P = aVb. Find the value
of b for which the specific heat capacity in the process is zero.
Sol.
Compare the given process PV2 = constant with PVr = constant, we have Sol.
r = 2. Given, P = aVb
R or PV–b = a
We know that C CV Compare with PVr = Constant, we have
r 1 r = –b
R R
= CV = CV – R Ans. We know that, C = CV
2 1 r 1
www.crackjee.xyz
LAWS OF THERMODYNAMICS 471

R Sol.
Here, C = 0, CV = Here, P1= 2 atm, T1= 273 +27 = 300 K
1
When tyre burst, P2 = 1 atm, T2 = ?
R R
0 = For air = 7/5.
1 b 1
As the process is sudden, so we have
or b = – Ans.
1 1
Ex. 11 A certain amount of gas occupies volume V0 at pressure P1 P2
=
P0 and temperature T0. It is allowed to expand (i) isobarically, (ii) T1 T2
adiabatically and (iii) isothermally . In which case the work done 1
is maximum and in which case it is minimum? P2 T2
or =
P1 T1
Sol.
Work done by all the processes given are shown in Fig. 7.29. (7 /5) 1 7 /5
1 T2
=
2 300
After solving, we get T 2 = 246.1 K Ans.

Ex. 14 Two samples of a gas initially at same temperature and


V
pressure are compressed from a volume V to . One sample is
2
compressed isothermally and the other adiabatically. In which
sample is the pressure greater?

Sol.
V Vi
Here Vi = V, Vf = , Vf = 2
2
Fig. 7.29 In isothermal process
Pi Vi = Pf Vf
It is clear from the figure that area AB1 CD is the largest and area AB3CD PV
i i
is the least one. If Wisob,Wiso and Wad represent work done in isobaric, Pf = V = Pi 2 = 2P1
isothermal and adiabatic processes respectively, then Wisob>Wiso>Wad. f
In adiabatic process
Ex. 12 The volume of an ideal gas is V at pressure P. On PV
i i = Pf V f
increasing the pressure by P, the change in volume of the gas is
V 1 under isothermal conditions and V 2 under adiabatic Vi
conditions. Under which of the two conditions, will the change in Pf = Pi = Pi (2)
Vf
volume be more?
Sol. As > 1, so 2 > 2 and hence the pressure in adiabatic pressure will be
greater.
Under isothermal conditions, the isothermal elasticity
Ex. 15 One mole of a gas is carried through the cycle shown in
P
Eiso = =P the Fig. 7.30. The gas expands at constant temperature T from
V1 volume V to 2V. It is then compressed to the initial volume at constant
V pressure and is finally brought back to its original state by heating
at constant volume. Calculate the work done by the gas in complete
PV cycle.
V1 =
P
Sol.
Under adiabatic conditions, the adiabatic elasticity Work done in isothermal process A to B
P
Ead = = P
V2
V

PV V1
V2 = =
P
As > 1, V 2< V 1 Ans.

Ex. 13 A motor car tyre has a pressure of 2 atmosphere at room


temperature of 27°C. If the tyre suddenly bursts, find the resulting Fig. 7.30
temperature.
www.crackjee.xyz
472 MECHANICS & THERMODYNAMICS

From first law of thermodynamics


Vf 2V Q = U+W
WAB = nRT n = 1RT n
Vi V 9 RT0
= RT0
= RT n 2 2
If pressure at B is PB, then 11 RT0
=
PAVA = PBVB 2
or PV = PB × 2V PB= P/2 11 RT0
Thus heat absorbed by the gas from A . Ans.C B is
2
Work done in isobaric process from B to C at constant pressure P/2
Ex. 17 A vessel of volume V0 contains an ideal gas at pressure
P0 and temperature T. Gas is continuously pumped out of this
WBC = PB (V f Vi )
dV
vessel at a constant volume- rate = r, keeping the temperaturee
P PV dT
= (V 2V ) =
2 2 constant. The pressure of the gas being taken out equals the
pressure inside the vessel. Find (a) the pressure of the gas as a
RT function of time, (b) the time taken before half the original gas is
=
2 pumped out.
Work done in isochoric process C to A, WCA = 0 Sol.
Work done in whole cycle (a) As the temperature in the process is constant, so we have
W = WAB+ WBC+WCA PV = k
Differentiating w.r.t. time, we get
RT
= RT n 2 0 dV dP
2 P V = 0
dt dt
1
= RT n2 Ans. dV
2 Upon substituting = r and V = V0 (which is constant) and
dt
Ex. 16 1 mole of a monoatomic gas is taken from a point A to rearranging, we get
another point B along the path ACB. The initial temperature at A dP r
is T 0. Calculate the heat absorbed by the gas in the process = V dt
P 0
A C B. Integrating above equation
Sol. P t
If TB be the temperature at B, then by gas law dP r
dt
P0
P = V0 0

rt
| nP |PP0 = V
0

rt
nP nP0 =
V0

Fig. 7.31 P rt
or n =
P0 V0
PAVA PBVB
TA = TB P rt
or =
PBVB (2 P0 )(2V0 )
P0 e V0
TB = P V TA = P0V0
T0
rt
A A
The change in internal energy from A to B or P = P0e V0 Ans.

3R P0
U = nCV T = 1 (4T0 T0 ) (b) If t is the time in which pressure reduces to , then
2 2
9 RT0 P0 rt
= = P e V0
2 2 0
Work done in the process A to C
WAC = P V = P0 (2V0 – V0)
or e rt / V0 = 2
= P0V0 = RT0 rt
and W CB = 0 or V0 = n2
Total work done from A C B
WAC + WCB = RT0 + 0 = RT0 V0 n 2
t = Ans.
r
www.crackjee.xyz
LAWS OF THERMODYNAMICS 473

Ex. 18 Two moles of monoatomic gas occupy two chambers of a The pressure of the left chamber is also PL =32 P0.
cylinder - piston system in which the piston is free to move and the If TL is the final temperature of the left chamber, then
walls of the cylinder and the piston are made of insulating material. 15V0
P0V0 (32 P0 )
The initial volumes, pressures and temperatures of the two 8
T0 = T = 60T0 Ans.
chambers are the same- P0, V0 and T0. The chamber in the left is TL
heated internally by some device resulting in expansion of the gas
(c) The change in internal energy of the gas in left chamber,
pushing the piston to the right. The gas in the right chamber is
compressed until the pressure becomes 32 times the initial pressure. 3R
U = nCV T = 1 (60T0 T0 )
Calculate: 2
= 88.5T 0 Ans.
(d) The heat absorbed by the left chamber = heat absorbed by right
chamber
Q = U+W
9
= 88.5 RT0 RT0
2
= 93.0 RT0 Ans.
Fig. 7.32
(a) the work done in compression,
(b) the final temperatures of the two chambers,
Ex. 19 Fig. 7.33 shows an adiabatic cylindrical tube of volume
(c) the change in internal energy of the left chamber, and V0, divided in two parts by a frictionless adiabatic separator. Initially,
(d) the heat absorbed by the left chamber. the separator is kept in the middle, an ideal gas at pressure P1 and
temperature T1 is injected into the left part and another ideal gas
Sol. at pressure P2 and temperature T2 is injected into the right part.
(a) As the system is made of insulating material so process between Cp
the chambers will be adiabatic. = is the same for both the gases. The separator is slid
Cv
When pressure in the right chamber becomes 32 times initial, let slowly and is released at a position where it can stay in equilibrium.
its volume becomes VR. Then Find
P0V0 = (32P0 )VR
1
1
or VR = V0
32
For monoatomic gas Fig. 7.33
5
=
3 3
1 5 V0
VR = V0 =
32 8
Fig. 7.34
If TR is the corresponding temperature of the right chamber, then
(a) the volumes of the two parts,
P0V0 (32 P0 )(V0 / 8) (b) the heat given to the gas in the left part and
T0 = TR (c) the final common pressure of the gases.
or T R = 4 T0 Sol.
The separator will stop in the position when pressure of both the parts
The work done on the gas in right chamber
of the tube become equal. Let it is P.
( PV
i i Pf V f ) If V1 and V2 are the volumes of two parts, then
W = V1 + V2 = V0 ...(i)
( 1)
(a) The process in each part is adiabatic, so
V0
P0V0 32 P0 V0
8 P1 = PV1 ...(ii)
= 5 2
1
3
9 P0V0 9 RT0 V0
and P2 = PV2 ...(iii)
= = 2
2 2
(b) Total volume of the two chambers = 2 V0. Ans. Dividing equation (ii) by (iii), we have
V0
Finally when volume of right chamber becomes , the volume P1 V1
8
of left chamber will be P2 =
V2
V0 15V0 1/
= 2V0 = P1
8 8 or V1 = V2 ...(iv)
P2
www.crackjee.xyz
474 MECHANICS & THERMODYNAMICS

Substituting this value in equation (i), we get P0S2 + F = PS2 + m2g


or F = (P – P0)S2 + m2g ...(ii)
V0 P21/ V0 P11/
V2 = and V1 = Equating equations (i) and (ii)
P11/ P21/ P11/ P21/
(P – P0) S1 – m1g =(P – P0) S2 + m2g
Ans. or (P – P0) (S1 – S2) = (m1 + m2)g
(b) As cylindrical tube and separator both are adiabatic, so no heat is Given, S1 S2 S and m1 m2 m
given in the process. Ans.
(c) Substituting value of V1 in equation (ii), we get
( P P0 ) S = mg
mg
P11/ P21/ or P = P0 ...(iii)
P = Ans. S
2 Now from equation of state, PV = nRT, we can write
P V = nR T
Ex. 20 A smooth vertical tube having two different sections is
open from both ends and equipped with two pistons of different mg V S
or P0 ( S) = 1 R T
areas (see Fig. 7.35). Each piston slides within a respective tube S n 1
section. One mole of ideal gas is enclosed between the pistons tied After simplifying, we get
with a non-stretchable thread.
( P0 S mg )
T = Ans.
R
Ex. 21 An ideal gas has a molar heat capacity CV at constant
volume. Find the molar heat capacity of this gas as a function of its
volume V, if the gas under goes the following process:
(a) T V (b) P V
T0e P0e
Sol.
If C be the molar heat capacity of the gas, then first law of
thermodynamics, Q = U + W can be written as
C T = CV T P V
V
or C = CV P
T
Fig. 7.35 T
For the ideal gas PV = RT or P = R
The cross - sectional area of upper piston is S greater than that of V
the lower one. The combined mass of two pistons is equal to m. The RT dV
outside air pressure is P0. By how much temperature must the gas C = CV ...(i)
V dT
between the pistons be heated to shift the piston through a distance ?
Sol. (a) For the given process T = T0e V
One mole of the gas is confined between upper and lower pistons. Let Differentiating w.r.t. volume, we get
m1 and m2 be their masses and S1 and S2 respective areas. If P is the dT d
= T0e V
pressure of the gas inside and F is the tension in the string, then for upper dV dV
piston
V
= T0e = T
dV 1
or = ...(ii)
dT T
Now from equation (i) and (ii), we get
RT 1
C = CV
V T
R
= CV Ans.
V
(b) For the process P V , we have
P0e
RT
= P0e V
V
Fig. 7.36 Differentiating w.r.t. volume, we get
RT R dT
P0S1 + F + m1g = P S1 2 = P0 e V
V V dV
or F = (P – P0) S1 – m1 g ...(i)
For lower piston R dT P0 V V
or = e P0 e
V dV V
www.crackjee.xyz
LAWS OF THERMODYNAMICS 475

R dT V 1 V PBVBTA VBTA
or = P0 e or TB = = (PA = PB)
V dV V PAVA VA
dT P0e V 0.084
or = (1 V) = TA = 3TA = 3 × 533 = 1599K
dV R 0.028
dV R At C : As process C A is isothermal , so
or = V
dT P0e (1 V) T C = 533 K
V C = VA
R R
= P (1 V ) = RT In the process B to C,
(1 V)
V PV1.5 = k

V nRT
By equation of state, V =
= T (1 V) ...(iii) P
From equations (i) and (iii), we get 1.5
nRT
RT V P = k
P
C = CV V T (1 V)

R T 1.5
= CV or = k
1 V
Ans. P0.5
Thus we have
Ex. 22 A mass of air is initially at 260°C and 700 k Pa and
occupies 0.028 m3. The air is expanded at constant pressure to 1.5 0.5
TB PB
0.084m3. A polytropic process with r = 1.5 is then carried out followed =
by a constant temperature process which completes a cycle. All the TC PC
processes are reversible. Sketch the cycle in the PV diagram. Find
(a) the heat required, (b) the heat rejected in the cycle, and (c) the 1.5/ 0.5 3
TC 1 PB
efficiency of the cycle. PC PB PB
CV = 0.718 kJ/kgK, Rair = 0.287 kJ/ kgK TB 3 27
Sol. For the process A B:
The complete cycle is shown in Fig. 7.37. For m kg of the air we can
CP = CV + Rair = 0.718 + 0.287 = 1.005 kJ/ kg K.
write
QAB = mCP (TB TA ) = 0.128 1.005 (1599 533)
= 137.13 kJ
For the process B C:

TB TC
WBC = mRair
r 1

and U = UC – UB= mCV (TC –TB)


QBC = U WBC

Fig. 7.37 TB TC
= mCV (TC TB ) mRair
PV 700 0.028 r 1
1 1
m= R T = = 0.128 kg
air 1 0.287 533
Rair
At A : = m(TC TB ) CV
PA = 700 k Pa r 1
VA = 0.028 m3
0.287
TA = 273 + 260 = 533 K = 0.128[533 1599] 0.718
1.5 1
At B : PB = 700 k Pa (As process is isobaric from A to B)
VB = 0.084 m 3 = 0.128 × (– 1066) × 0.144
TB = ? = – 19.65 k J
By equation of state For process C A : As
PAVA PBVB T C = TA = 0, U = UA – UC = 0
TA = T
B
www.crackjee.xyz
476 MECHANICS & THERMODYNAMICS

VA PC QCA = nCP (TA TC )


WCA = mRairTA n V = mRairTA n P
C A 5R P0V0 2P0V0
= 1
1 2 R R
= 0.128 0.287 533 n
27
5 P0V0
= – 64.53 k J =
2
(a) Heat received in the cycle,
As path AB is isochoric, so heat absorbed
Q AB = 137.13 kJ
(b) Heat rejected in the cycle, QAB = nCV (TB TA )
Q = 19.59 + 64.53 3R 3 P0V0 P0V0
= 84.12 kJ = 1
2 R R
(c) The efficiency of the cycle, = 3 P0V0
Q (c) For the complete cycle ABCA, U= 0.
= 1 Q From first law of thermodynamics, we have
AB
Q = U+W
84.12 = 0+W
= 1
137.13 or Q = W
= 0.39 or 39% or QAB + QBC + QCA = W
Ex. 23 One mole of an ideal monoatomic gas is taken round QAB = W – (QAB + QCA)
the cyclic process ABCA as shown in Fig. 7.38. Calculate
5
= P0V0 3P0V0 P0V0
2
P0V0
=
2
(d) We know that PV = RT. For T to be maximum, PV must be
maximum.
P – V relation for the process BC:
P = kV k …(i)
For point B, 3P0 = kV0 k ...(ii)

For point C, P0 = k (2V0 ) k ...(iii)


Fig. 7.38 After solving equations (i) and (ii), we get
(a) the work done by the gas
2P0
(b) the heat rejected by the gas in the path CA and the heat k=
absorbed by the gas in the path AB V0 and k = 5P0
(c) the net heat absorbed by the gas in the path BC Substituting these values in equation (i), we have
(d) the maximum temperature attained by the gas during the
2 P0V
cycle. P = 5 P0 ...(iv)
V0
Sol.
(a) The work done by the gas in entire cycle Substituting this value of P in PV = RT, we have
W = area of ABCA
2 P0V
5 P0 V = RT
1 V0
= (2V0 V0 ) (3P0 P0 )
2 P0 2V 2
1 or T = R 5V V0 ...(v)
= V0 2 P0 P0V0
2
dT
PV For maximum value, = 0,
(b) For one mole of a gas, we have T = dV
R
P0V0 (3P0 )V0 3P0V0 dT P0 4V
TA = , TB = = So = R 5 V0 =0
R R R dV
P0 (2V0 ) 2 P0V0 5V0
and TC = = which gives V =
R R 4
As path CA is isobaric, so heat rejected Putting this value of V in equation (v), we get
www.crackjee.xyz
LAWS OF THERMODYNAMICS 477

From equations (i) and (ii), we have


2
P0 5V0 5V0 1
Tmax = R 5 4 2
4 V0 2RT
dT T dP = 0
V
P0 25V0 25V0 or 2RT dT+ VT dP = 0
= VdP = – 2 RdT ...(iv)
R 4 8
Now from equations (iii) and (iv), we have
25 P0V0
= – 2RdT + VdP = 2RdT
8R
or PdV = 4 R dT ...(v)
Ex. 24 Two moles of an ideal monoatomic gas is taken through (a) The work done in the process AB
a cycle ABCA as shown in the P – T diagram. During the process
300
AB, pressure and temperature of the gas vary such that
PT = Constant . If T1=300 K, calculate WAB = PdV 4 R dT
600

= 4R | T |300
600 = 4 R (300 600)
= – 1200 R Ans.
(b) (i) As process B C is isobaric, so

5R
QBC = nCP T = 2 (600 300)
2
= 1500 R Ans.
(ii) Process C A is isothermal, so U = 0
QCA = U WCA = WCA

P1
Fig. 7.39 WCA = nRT n P
(a) the work done of the gas in the process AB and 2

(b) the heat absorbed or released by the gas in each of the 2 P1


processes. Give answers in terms of the gas constant R. = 2 R 600 n P
1
Sol. = 1200 R n 2
For the process A – B, it is given that QCA = 1200 R n2
PT = constant Again for the process A B
Differentiating above equation partially, we have QAB = U WAB
PdT + TdP = 0 ...(i)
= nCV T WAB
Equation of state for two moles of a gas
3R
2 RT = 2 (300 600) 1200 R
PV = 2RT or P = ...(ii) 2
V
= – 900 R – 1200 R
After differentiating equation (ii) partially, we get = – 2100 R
PdV + VdP = 2R dT ...(iii)

7.11 WEAKNESS OF FIRST LAW AND NEED OF SECOND LAW


(i) It does not tell about direction of flow of heat:
Heat always flows from hot body to a cold body. But first law has no explanation
why heat cannot flow from cold body to hot body.
(ii) In which conditions heat can be converted into work:
A revolving wheel stops due to friction.
According to first law, kinetic energy of wheel is converted into heat. But it fails to
explain why heat energy cannot be converted into kinetic energy of the wheel.
(iii) Amount of heat can be converted into work:
No practical device converts entire heat into work. First law has no explanation of
this fact.
Second law of thermodynamics provides answers of these questions.
www.crackjee.xyz
478 MECHANICS & THERMODYNAMICS

7.12 SECOND LAW OF THERMODYNAMICS


(i) Kelvin - Plank statement
It is impossible to construct an engine that can convert heat completely into work
without producing any other effect. According to the statement the efficiency of
any heat engine always be less than 100%.
(ii) Clausius statement
For a self acting machine, it is impossible to transfer heat from a colder body to a
hotter body without the aid of external agency.
7.13 ENTROPY
Entropy is the another thermodynamical variable which many times very useful to
understand the system. Entropy is related to the disorder or randomness in the system.
To understand this, let us consider two systems as shown in Fig. 7.40:
If S1 and S2 are the entropies of the system 1 and 2 respectively at any temperature, then
S1 < S2.
(i) Entropy is not a conserved quantity.
(ii) Entropy can be created but cannot be destroyed.
(iii) Entropy of the universe always increases.
If a system at temperature T is supplied a small amount of heat Q, then change in
entropy of the system can be defined as

Q
S = for constant T
T
Fig. 7.40
For a system with variable T, we have

Sf
dQ
S = Sf Si = T
Si

The second law of thermodynamics may be stated in terms of entropy as:


It is impossible to have a process in which the entropy of an isolated system is decreased.

T2
Ex. 25 One kilogram of ice at 0°C is melted and converted to dQ
water at 0°C. Compute the change in entropy. S = T
T1
Sol. For small change in temperature dT of the water of specific heat C, we
As the temperature in the process remain constant, so change in entropy, have
Q dQ = mCdT
S =
T T2
dT
mC T2
Here Q = mL = 1 334 103 = 334 103 J S =
T1
T = mC | nT |T1
and T = 273 + 0 = 273 K
T2
334 103 = mC n T
S = = 1223 J/K Ans. 1
273 273 100
= 1000 4.2 n
Ex. 26 One kilogram of water at 0°C is heated to 100°C. 273 0
Compute its change in entropy. 373
= 1000 4.2 n
Sol. 273
As temperature in the process is not constant, so = 1308 J/K Ans.
www.crackjee.xyz
LAWS OF THERMODYNAMICS 479

7.14 HEAT ENGINE


It is a device which is used to convert heat energy into mechanical energy in a cyclic
process.
There are two types of heat engine.
(i) External combustion engine: In this type of engine, heat needed for the engine is
produced by the burning of the fuel outside the cylinder.
Example: steam engine.
(ii) Internal combustion engine: In this type of heat engine, heat needed for the
engine is produced by the burning of the fuels inside the cylinder.
Example:diesel engines, petrol engines etc.
The efficiency of the steam engine is less than 20%.
The efficiency of petrol engine is about 25%.
The efficiency of diesel engine is about 40%.
Parts of a heat engine Fig. 7.41
(i) Source: It is a heat reservoir at higher temperature T1. Its thermal capacity should
be very large so that any amount of heat can be drawn from it without appreciable
change in temperature of source.
(ii) Sink: It is a heat reservoir at a lower temperature T2. Its thermal capacity is also
very large so that any amount of heat can be added without appreciable change in
temperature.
(iii) Working substance: Working substance is the material which performs mechanical
work. It may be solid, liquid or gas.
In heat engine, the engine draws heat from the source Q1 and after doing some
mechanical work W, it rejects the remaining heat to the sink. Thus conservation of
energy gives
Q1 = W + Q2
or W = Q1 – Q2
Efficiency of heat engine is given by
Output
=
Input

Work done by engine W


= Heat absorbed by engine from source = Q
1

Q1 Q2 Q2
= =1
Q1 Q1

Note:
The practical value of is always be less than 1 or 100%.
If engines of efficiencies 1, 2, ...... are put in series, then output efficiency of the
system of engines will be 1 2 .......

7.15 CARNOT REVERSIBLE HEAT ENGINE


Sadi Carnot in 1824 conceived a theoretical engine which is free from all the defects of
practical engines. It has the maximum efficiency and it is an ideal heat engine. It is based
on four operations, which constitutes a Carnot cycle. These are: Isothermal expansion ,
adiabatic expansion, isothermal compression and adiabatic compression.
www.crackjee.xyz
480 MECHANICS & THERMODYNAMICS

Carnot heat engine has three main parts:


(i) Source, T1 (ii) Sink, T2 (iii) Working substance, W
A perfectly insulating stand is also provided so that the working substance can undergo
adiabatic operation.

Fig. 7.42
Carnot cycle
Carnot cycle has four operations. Thermodynamic coordinates after each operation are
shown in Fig. 7.44. Initially at A coordinates are P1,V1,T1.
1. Isothermal expansion: If Q1 is the heat absorbed from the source and W1 is the
work done, then,
Vf
Q1 = W1 = nRT1 n (As U 0)
Vi

V2
= nRT1 n V
1

2. Adiabatic expansion: If W2 is the work done during the adiabatic expansion, then

Fig. 7.43 nR(Ti T f ) nR (T1 T2 )


W2 = =
1 1
3. Isothermal compression: If Q2 is the heat reject to the sink and W3 is the work
done during the process, then
Vf V4
Q2 = W3= nRT2 n V = nRT2 n V
i 3

As U
V
= nRT2 n 3
V4
4. Adiabatic compression: If W4 is the work done during the adiabatic compression,
then
nR(Ti T f ) nR (T2 T1 )
W4 = =
1 1

(T1 T2 )
= nR
1
Net work done in the whole cycle
W = W1 + W2 + W3 + W4
www.crackjee.xyz
LAWS OF THERMODYNAMICS 481

V2 nR (T1 T2 ) V (T1 T2 )
= nRT1 n nRT2 n 3 nR
V1 1 V4 1

V2 V3
= nR T1 n T2 n ...(i)
V1 V4
In the adiabatic expansion B C
1 1
T1V2 = T2V3

1
V2 T2
or = T1
...(ii)
V3
Similarly in the adiabatic compression D A
1 1
T2V4 = T1V1

1
V1 T2
or = ...(iii)
V4 T1

From equations (ii) and (iii) , we have


V2 V1
=
V3 V4

or V1V3 = V2 V 4 …(iv)
V2 V3
Also =
V1 V4
Efficiency of carnot engine
Work done byengine (W )
=
Heat absorbed by engine from source (Q1)

V2 V3
nR T1 n T2 n
V1 V4
=
V2
nRT1 n
V1

V2 V3
As =
V1 V4

T2 T2
= =1 .
T1 T1

Note:
1. The isothermal process will take place only when the piston moves very slowly to
give enough time for the heat transfer between source and working substance.
The adiabatic process will take place when the piston moves extremely fast to
avoid heat transfer. Any practical engine can not satisfy these conditions.
2. All practical engines have an efficiency less than the carnot engine (Carnot
theorem).
www.crackjee.xyz
482 MECHANICS & THERMODYNAMICS

More about
T2
1. For to be unity or 100%, 0 . It can be possible if either T = 0 K or
T1 2
T1= . Practically these are not possible (till now), so Carnot engine cannot have
efficiency 100%.
T1 T2
2. = = ,
T1 T1
with decrease in temperature difference (T1 – T2) between source and sink will
decrease the efficiency of the engine.
If T1 –T2 = 0 or T1 = T2, =0
Q2
3. As Q1
and =
T1
Q2 T2 Q1 Q2 Q
= or = = Constant
Q1 T1 T1 T2 T
4. When two engines are put in series , the net work done,
W = W1 + W2
By conservation of energy, we have
Q1 = Q3 + (W1+ W2)
Fig. 7.44 = Q3 + W
or W = Q1 – Q3
W Q1 Q3
= Q1 Q1

Q3 T3
1 1
Q1 T1

7.16 REFRIGERATOR OR HEAT - PUMP


Carnot cycle is reversible cycle. It can work as a heat engine and also as a refrigerator. In
a refrigerator, the working substance absorbs an amount of heat Q2 from the sink (cold
reservoir) at temperature T2. W is the work done by external agency and it rejects a larger
quantity of heat Q1 to the source (hot reservoir). By conservation of energy, we have,
Q1 = Q2 + W
or W = Q1 – Q2
Coefficient of performance of a refrigerator is defined as:
Heat absorbed from sink
=
Work done

Q2
=
Fig. 7.45 W

Q2 1
= =
Q1 Q2 Q1
1
Q2

1
or = T1 =
1
T2
www.crackjee.xyz
LAWS OF THERMODYNAMICS 483

Note:
In domestic refrigerator, the interior of it acts as the sink and the room (surroundings)
acts as the source. Work is done by the electrical energy and frown CCl2F2 is used as
a working substance.

More about
1. Coefficient of performance always be greater than 1. Practical refrigerators have
a coefficient of performance close to 10.
T2
2. As , so lesser the temperature difference T1 – T2, higher is the value
T1 T2
of .
3. When refrigerator works, the temperature of sink T2 decreases due to formation of
ice and T1 – T2 increases. This decrease the value of . So defrosting is necessary
to increase the performance of the refrigerator.
4. If the door of the refrigerator is opened, it rejects Q1 amount of heat which is
greater than heat absorbed from the room Q2, so the temperature of the room will
increase.
Ex. 27 One of the most efficient en gines ever developed 1 T2 65
operated between 2100 K and 700 K. Its actual efficiency is 40%. = 1 T1 ...(ii)
3
What percentage of its maximum possible efficiency is this?
Solving equations (i) and (ii) , we get
Sol. T 1 = 390 K and T2 = 325 K Ans.
Here T 1 = 2100 K and T2 = 700 K
T
1 2 = 1 700
Ex. 29 A Carnot engine whose heat sink is at 27°C has an
max = T1 2100 efficiency of 40%. By how many degrees should the temperature of
the source be changed to increase the efficiency by 10% of the
1 2 original efficiency?
= 1 = = 66.6%
3 3 Sol.
Here, T 2 = 273 + 27 = 300 K
Given, actual = 40%
The % of maximum efficiency T2
We know that = 1 T
1
actual 100
=
max 300
or 0.40 = 1 T1
40
= 100 60% Ans. or T 1 = 500 K
66.6
Increase in efficiency of engine = 10% of 40 = 4%
1 Thus new efficiency of the engine becomes = 40 + 4 = 44%
Ex. 28 The efficiency of a Carnot cycle is 6
. If on reducing
1 Let T1 is the new temperature of the source, then
the temperature of the sink by 65°C, the efficiency becomes ,
3 T2
find the initial and final temperatures between which the cycle is 0.44 = 1
working. T1
Sol. 300
or 0.44 = 1
If T1 and T2 are the temperatures of source and sink respectively, then T1
T
1 2 or T1 = 535.7 K
1 = T1
Increase in temperature of the source
1 T2
or = 1 T ...(i) = T1 ' T1 535.7 500
6 1 = 35.7 K Ans.
1
When temperature of sink reduces by 65°C, then 2 = 3
www.crackjee.xyz
484 MECHANICS & THERMODYNAMICS

Ex. 30 Five moles of an ideal gas taken in a Carnot engine


working between 100°C and 30°C. The useful work done in one T2 WA
or 1 = Q
cycle is 420 joule. Calculate the ratio of the volume of the gas at T1 1
the end and beginning of the isothermal expansion.
R = 8. 4 J/ mol. K T2 T
WA = Q1 1 T = Q1 1 ...(ii)
Sol. 1 800

Here, T 1 = 273 + 100 = 373 K and For engine B: T1 = TK and T2 = 300 K


T 2 = 273 + 30 = 303 K Efficiency of the engine
Q1 T1 373 WB
We know that Q2 = T2 = 303 =
B Q1

373 T2
or Q1 = Q2 ...(i) WB
303 or 1 = Q '
T1 1
Work done in the cycle W = Q1 – Q2 = 420 J ...(ii)
From equations (i) and (ii), we have T2 300
WB = Q1 1 = Q1 1 ...(iii)
373 T1 T
Q2 Q2 = 420
303
Since the engine B absorbs the heat rejected by the engine A.
or Q 2 = 1818 J
So = Q2
Q1
and Q 1 = Q2 + 420
Now equation (ii) becomes
= 1818 + 420 = 2238 J
When the gas is carried through Carnot cycle, the heat absorbed Q 1 300
W B = Q2 1 ...(iv)
during isothermal expansion will equal to the work done by the gas. If V1 T
and V2 are the volumes of the gas at the beginning and at the end of the Case (i) When outputs of the two engines are equal
isothermal expansion, then WA = W B

V2 T 300
Q1 = W = nRT n Q1 1 = Q2 1
V1 800 T

T Q2 300
V or 1 = Q 1 T
or 2238 = 5 × 8.4 × 373 n 2 800 1
V1
Q2 T
From (i) Q1 =
V2 800
or n = 0.1428
V1 T T 300
1 = 1
800 800 T
V1
or = 1.153 Ans. After solving, we get , T = 550 K Ans.
V2 Case (ii) When the efficiencies are equal
Ex. 31 Two Carnot engines A and B are operated in series. The A = B
first one A receives heat at 800 K and rejects to a reservoir at
temperature T K. The second engine B receives the heat rejected T 300
or 1 = 1
by the first engine and in turn rejects to a heat reservoir at 300 K. 800 T
Calculate the temperature TK for the following cases: After solving, we get
(i) When the outputs of the two engines are equal. T = 489.9 K Ans.
(ii) When the efficiencies of the two engines are equal. Ex. 32 How much energy in watt hour may be required to
Sol. convert 2 kg of water into ice at 0°C, assuming that the refrigerator
For engine A: T1 = 800 K and T2 = TK is ideal? Given temperature of freezer is –15°C, room temperature
Q2 T2 is 25°C and initial temperature of water is 25°C.
T
We know that, Q1 = T =
1 800
...(i) Sol.
Efficiency of the engine Here T 1 = 273 + 25 = 298 K and
T 2 = 273 –15 = 258 K
WA
A = Q Specific heat of water,
1
C = 4.2 × 103 J/ kg K
www.crackjee.xyz
LAWS OF THERMODYNAMICS 485
Latent heat of fusion of ice,
L = 3.36×105 J / kg T1
Q1 = Q2
The amount of heat required to transform water of 25°C into ice of 0°C T2

Q2 = mC T mL 298
5
= 8.82 10 10.19 105 J
= 2 × 4.2 × 103 × (25 – 0) + 2 × 3.36 × 105 258
= 2.1 × 105 + 6.72 × 105 Energy supplied to convert water into ice,
= 8.82 × 105J
W = Q1 Q2
Heat rejected to the surroundings;
= 10.15 ×105 – 8.82 ×105
Q1 T1
We have = T = 1.33×105 J
Q2 2
1.33 105
= = 36.96 Wh Ans.
3600

Review of formulae & Important Points


1. Internal energy of an ideal gas is due to its kinetic energy, 5. Polytropic process :
which is the function of temperature.
PVr = 1, r 1 or .
3
U = RT for monoatomic gas nR
2 Work done, W = [Ti Tf ]
r 1
5
= RT for diatomic gas R
2 Specific heat, C = CV
r 1
Also U = nCv T
6. Second law of thermodynamics
If T = 0, U 0. (i) Kelvin-Plank statement : It is impossible to construct
In cyclic process, an engine that can convert heat completely into work.
U = 0. (ii) Clausius statement : For a self acting machine, it is
impossible to transfer heat from a colder body to a hotter
Vf
body without the aid of external agency.
2. Work done, W = P dV 7. The change in entropy,
Vi
Q
3. First law of thermodynamics S = .
Q = U + W T
In diferential form, it can be written as For a system with variable T
dQ = dU + W Sf
4. Thermodynamic processes : dQ
PV = const for isothermal; PV r = constant for adiabatic. S = T
Si
(i) Work done in isobaric process,
W = P V = nR T 8. Efficiency of heat engine
(ii) Work done in isochoic process
W = 0 W Q1 Q2 Q2
= 1
Also Q = U + W = U + 0 = nC V T Q1 Q1 Q1
(iii) Work done in isothermal process 9. Carnot heat engine :
Vf Pi V1V 3 = V2V 4
W = nRT n nRT n
Vi Pf T2
= 1
E iso = P T1
(iv) Work done in adiabatic process 10. Regrigerator or heat pump: Coefficient of performance
( PV
i i Pf V f ) Q2 Q2 1
W = =
( 1) W Q1 Q2 Q1
1
Q2
nR
or W = (Ti Tf )
1
1
Ead = P =
T1 1
Ead = E iso 1
T2
www.crackjee.xyz
486 MECHANICS & THERMODYNAMICS

MCQ Type 1 Exercise 7.1


Level -1
Only one option correct 6. When an ideal diatomic gas is heated at constant pressure. The
1. An ideal monoatomic gas is taken round the cycle ABCDA as fraction of heat energy supplied which increases the internal energy
shown in figure. The work done during the cycle is of the gas is
2 3
(a) (b)
5 5
3 5
(c) (d)
7 7
7. In a given process on an ideal gas, dW = 0 and dQ < 0, then for the
gas
(a) the temperature will decrease.
(a) PV (b) 2 PV (b) the volume will increase.
(c) PV / 2 (d) Zero (c) the pressure will remain constant.
2. Two identical containers A and B with frictionless pistons contain
(d) the temperature will increase.
the same ideal gas at the same temperature and the same volume V.
8. Choose the correct statement from among the following?
The mass of the gas in A is mA and that in B is mB. The gas in each
(a) A monoatomic gas has three degrees of freedom because it
cylinder is now allowed to expand isothermally to the same final
undergoes translational as well as rotational motion.
volume 2V. The changes in pressure in A and B are found to be P
and 1.5 P respectively. Then (b) A diatomic molecule undergoes both translational and
(a) 4 mA = 9 mB (b) 2mA = 3mB rotational motion and has six degrees of freedom.
(c) 3mA = 2mB (d) 9mA = 4mB (c) A diatomic molecule is capable of rotating energetically about
3. The specific heat of a gas at constant pressure is greater than the each of three mutually perpendicular axes.
specific heat of the same gas at constant volume because : (d) The molecule of a polyatomic gas is capable of rotating
(a) work is done in the expansion of the gas at constant pressure. energetically about each of three mutually perpendicular axes
(b) work is done in the expansion of the gas at constant volume. and undergoes both translational and rotational motion.
(c) the attraction between the molecules increases at constant 9. In figure, three isothermal processes are shown for the same gas
pressure. and for same change in volume (Vi – Vf) but at different temperature.
(d) the molecular attraction increases at constant volume. If Q1, Q2, and Q3 are the heat transferred in the respective
4. One mole of a monoatomic gas is heated at constant pressure of process, then :
1 atmosphere from 0 K to 100 K. If the gas constant R is taken as
8.32 J/mol-K, the change in the internal energy of the gas is P
approximately
(a) 2.3 J (b) 46 J
(c) 1.25 × 103 J (d) 8.67 × 104 J 1
5. The P-V diagram of process on a system is shown in figure. During 2
the process, the work done by the system 3
V
Vi Vf

(a) Q1 = Q2 = Q3
(b) Q1 > Q2 > Q3
(c) Q1 < Q2 < Q3
(d) Q1 = Q2 = Q3 = 0
10. A gas, confined to an isolated cylinder, is compressed adiabatically
(a) increases continuously to half its volume. The entropy of the gas :
(b) decreases continuously (a) increases (b) decreases
(c) first increases, becomes maximum and then decreases (c) remains same (d) none of the above
(d) first decreases, becomes minimum and then increases

Answer Key 1 (a) 3 (a) 5 (a) 7 (a) 9 (b)


Sol. from page 509 2 (c) 4 (c) 6 (d) 8 (d) 10 (c)
www.crackjee.xyz
LAWS OF THERMODYNAMICS 487
11. An ideal gas is made to go through a cyclic thermodynamical 19. In a reversible isochoric change
process in four steps. The amount of heat involved are
(a) W 0 (b) Q 0
Q1 = 600J, Q2 = –400J, Q3 = – 300J and Q4 = 200J respectively.
The corresponding work involved are W 1 = 300J, (c) U 0 (d) all
W2 = – 200J, W3 = –150J and W4. The value of W4 is
20. If the door of a running refrigerator is kept open, then which of the
(a) –50J (b) 100J following is true
(c) 150J (d) 50J
(a) Room is cooled
12. A container of volume 1m3 is divided into two equal compartments
(b) Room is heated
by a partition. One of these compartments contains an ideal gas at
300 K. The other compartment is vacuum. The whole system is (c) Room is either cooled or heated
thermally isolated from its surroundings. The partition is removed (d) Room is neither cooled nor heated
and the gas expands to occupy the whole volume of the container. 21. If we consider solar system consisting of the earth and sun only as
Its temperature now would be one of the ideal thermodynamic system. The sun works as source
(a) 300 K (b) 239 K of energy having temperature 6000 K and the earth as sink having
(c) 200 K (d) 100 K temperature 300 K, the efficiency of solar system would be on the
13. For free expansion of the gas which of the following is true basis of exchange of radiations
(a) Q = W = 0 and Eint 0 (a) 30% (b) 65%
(b) Q = 0, W > 0 and Eint –W (c) 75% (d) 95%
(c) W = 0, Q > 0 and Eint Q
1
(d) W > 0, Q < 0 and Eint 0 22. A carnot engine, having an efficiency of as heat engine, is
10
14. Which of the following parameters does not characterize the
thermodynamic state of matter used as a refrigerator. If the work done on the system is 10 J, the
amount of energy absorbed from the reservoir at lower temperature
(a) Volume (b) Temperature
is
(c) Pressure (d) Work
(a) 99 J (b) 90 J
15. Which of the following statements is correct for any
thermodynamic system (c) 1J (d) 100 J
(a) The internal energy changes in all processes 23. In the following indicator diagram, the net amount of work done
(b) Internal energy and entropy are state functions will be
(c) The change in entropy can never be zero
P
(d) The work done in an adiabatic process is always zero
PV 1 2
16. The gas law = constant is true for
T
(a) Isothermal changes only V
(b) Adiabatic changes only
(a) Positive (b) Negative
(c) Both isothermal and adiabatic changes
(c) Zero (d) Infinity
(d) Neither isothermal nor adiabatic changes
24. The temperature-entropy diagram of a reversible engine cycle is
17. One mole of an ideal gas expands at a constant temperature of 300
given in the figure. Its efficiency is
K from an initial volume of 10 litres to a final volume of 20 litres.
The work done in expanding the gas is
(R = 8.31 J/mole-K) T
(a) 750 J (b) 1728 J
2T0
(c) 1500 J (d) 3456 J
18. During an adiabatic process, the pressure of a gas is found to be
T0
proportional to the cube of its absolute temperature. The ratio Cp/
S
Cv for the gas is S0 2S0
3 4
(a) (b)
2 3 (a) 1/3 (b) 2/3
5
(c) 2 (d) (c) 1/2 (d) 1/4
3

Answer Key 11 (c) 13 (a) 15 (b) 17 (b) 19 (d) 21 (d) 23 (b)


Sol. from page 509 12 (a) 14 (d) 16 (c) 18 (a) 20 (b) 22 (b) 24 (a)
www.crackjee.xyz
488 MECHANICS & THERMODYNAMICS
25. A thermodynamic system undergoes cyclic process ABCDA as
(b) VT 1/( –1) e / RT = constant
shown in figure. The work done by the system is
1
P (c) V –1Te / RT = constant
3P0 C B
–1
(d) V = constant
2P0 O 30. 5 g of a gas is carried through a cycle ABCDA in a piston cylinder
P0 assembly as shown. If in the portion from A to B, 5.5 kcal of heat
A D flows into the gas and the temperature of the gas at A is 427°C, Cv
V for the gas is
V0 2V0
P(atm)
(a) P0 V 0 (b) 2 P0V0 A
15
PV
0 0
(c) (d) Zero
2 10 B
26. The P-V diagram of a system undergoing thermodynamic
transformation is shown in figure. The work done by the system C
5 D
in going from A B C is 30J and 40J heat is given to the
system. The change in internal energy between A and C is V(l)
P 5 10 15
(a) 0.49 kJ/kg°C (b) 0.298 kJ/kg°C
C (c) 3.03 kJ/kg°C (d) 0.98 kJ/kg°C
31. 2 k mol of hydrogen at NTP expands isobarically to twice its
initial volume. The change in its internal energy is (Cv = 10 kJ/
A B kg.K and atm pressure = 1 × 105 N/m2)
V (a) 10.9 MJ (b) 9.10 MJ
(c) 109 MJ (d) 1.09 MJ
(a) 10 J (b) 70 J 32. A thermodynamic process is shown in the figure. In the process
(c) 84 J (d) 134 J AB 500 J of heat are added and in process BD, 150 J of heat are
27. In the following figure, four curves A, B, C and D are shown. The added. The change in the internal energy in the process ABD is :
curves are
P
P P
4
6×10 B D
Pa
A
B C
4
D 2×10 A C
V V Pa V(m3)
–3 –3
2×10 6×10
(a) Isothermal for A and D while adiabatic for B and C
(b) Adiabatic for A and C while isothermal for B and D (a) 890 J (b) 410 J
(c) Isothermal for A and B while adiabatic for C and D (c) 650 J (d) 240 J
(d) Isothermal for A and C while adiabatic for B and D 33. One mole of a monoatomic ideal gas is taken through a reversible
28. The equation of state of a gas is given as P(V – b) = nRT where b cycle ABC as shown in the figure. The process BC is adiabatic.
is a constant, n is the number of moles and R is the universal gas The work done per cycle is (given that temperatures of A, B and
constant. When 2 moles of this gas undergo reversible isothermal C are 300, 600 and 450 K) :
expansion from volume V to volume 2V, what is the work done by
P
the gas B
V –b 2V – b
(a) 2RT ln (b) 2RT ln
2V – b V –b
V –b 2V
(c) 2RT ln (d) 2RT ln
2V V –b
29. For an ideal gas the equation of a process for which the heat A C
V
capacity of the gas varies with temperature as C = /T ( is a
constant is given by) (a) 75 R (b) 150 R
(a) VlnT = constant (c) 112.5 R (d) 45 R

Answer Key 25 (d) 27 (d) 29 (d) 31 (a) 33 (a)


Sol. from page 509 26 (a) 28 (b) 30 (c) 32 (b)
www.crackjee.xyz
LAWS OF THERMODYNAMICS 489

Level -2
Only one option correct (a) – 5 J (b) – 10 J
1 (c) – 15 J (d) – 20 J
1. mole of helium gas is contained in a container at STP. The heat
2 7. An ideal gas heat engine operates in a Carnot cycle between 227°C
energy needed to double the pressure of the gas, keeping the
and 127°C. It absorbes 6.0 × 104 cal at the higher temperature.
volume constant (heat capacity of the gas = 3 J/g K) is
The amount of heat converted into work is equal to
(a) 3276 J (b) 1638 J
(a) 4.8 × 104 cal (b) 3.5 × 105 cal
(c) 819 J (d) 409.5 J 4
(c) 1.6 × 10 cal (d) 1.2 × 104 cal
2. A monoatomic ideal gas initially at temperature T1 is enclosed in
a cylinder fitted with a frictionless piston. The gas is allowed to 8. A carnot engine takes in 3000 kcal of heat from a reservoir at
expand adiabatically to a temperature T2 by releasing the piston 627°C and gives it to a sink at 27°C. The work done by the engine
suddenly. If L1 and L2 are the lengths of gas column before and is
(a) 4.2 × 106 J (b) 8.4 × 106 J
T1 (c) 16.8 × 10 J 6 (d) zero
after expansion respectively, then T is
2 5R
2/3
L1 9. The specific heat at constant pressure of an ideal gas, C p .
L1 2
(a) (b) L2
3
The gas is kept in a closed vessel of volume 0.0083 m at 300 K
L2
and a pressure of 1.6 × 106 N/m2. 2.49 × 104 J of heat energy is
2/3
L2 L2 supplied to the gas. The final temperature and the pressure
(c) L1 (d) respectively are
L1
(a) 567.2 K and 6.3 × 106 N/m2
3. A gas mixture consists of 2 moles of oxygen and 4 moles of argon (b) 675.2 K and 3.6 ×106 N/m2
at temperature T. Neglecting all vibrational modes, the total internal
(c) 275.2 K and 2.3 ×106 N/m2
energy of the system is
(d) 465.6 K and 4.2 × 106 N/m2
(a) 4 RT (b) 15 RT
10. A bubble has a volume of 3 mm3 at a depth of 20 m in a lake of
(c) 9 RT (d) 11 RT
pure water. If the bubble slowly rises upto the surface of the lake,
4. P-V plots of two gases during adiabatic processes are shown in
figure. Plots 1 and 2 should correspond respectively to its volume will be (atmospheric pressure is 100 kPa)
(a) 7.24 mm3 (b) 4.44 mm3
(c) 8.88 mm 3 (d) 6.66 mm3
11. A 0.5 hp motor is stirring 4 kg of water. Assuming there is no heat
loss except in heating the water, the time it will take to raise the
temperature of the water by 5°C is
(a) 4.73 min (b) 3.75 min
(c) 5 min (d) none of these
12. A 500 ml sealed cylinder contains nitrogen at a pressure of 1 atm.
A tiny glass tube lies at the bottom of the cylinder. Its volume is
(a) He and O2 (b) O2 and He 0.50 ml and it contains hydrogen at a pressure of 4.5 atm. The
(c) He and Ar (d) O2 and N2 glass tube is broken so that hydrogen also fills the cylinder. The
5. An ideal gas expands isothermally from a volume V 1 to V2 and new pressure in the cylinder is (1 atm = 1 × 105 N/m2)
then compressed adiabatically to original volume V, initial pressure (a) 76.34 cm Hg (b) 82.40 cm Hg
is P1 and final pressure is P2. If the total work done is W, then (c) 94.24 cm Hg (d) 104.34 cm Hg
(a) P2 > P1, W > 0 (b) P2 < P1, W < 0 13. 4 kg of oxygen gas is heated so as to raise its temperature from
(c) P2 > P1, W < 0 (d) P2 = P1, W = 0 20 to 120°C. If the heating is done at constant pressure, the
6. An ideal gas is taken through the cycle A B C A as shown external work done by the gas is (C p = 0.219 cal/g°C and
in figure. If the net heat supplied to the gas in the cycle is 5 J, the Cv = 0.157 cal/g°C)
work done the gas in the process C A is (a) 628 kJ (b) 104 kJ
(c) 366 kJ (d) 206 kJ
14. On P-V coordinates, the slope of an isothermal curve of a gas at a
pressure P = 1MPa and volume V = 0.0025 m3 is equal to –400
MPa/m3. If Cp / Cv = 1.4, the slope of the adiabatic curve passing
through this point is :
(a) –56 MPa/m3 (b) –400 MPa/m3
(c) –560 MPa/m 3 (d) none of these

Answer Key 1 (b) 3 (d) 5 (c) 7 (d) 9 (b) 11 (c) 13 (b)


Sol. from page 510 2 (d) 4 (b) 6 (a) 8 (b) 10 (c) 12 (a) 14 (c)
www.crackjee.xyz
490 MECHANICS & THERMODYNAMICS
15. The P-V diagram here shows six curved paths (connected by (a) Zero (b) P(V2 – V1)
vertical paths) that can be followed by a gas. Which two of them (c) L – P(V2 – V1) (d) L
should be part of a closed cycle if the network done by the gas is
20. Two kg of water is converted into steam by boiling at atmospheric
to be at its maximum positive value?
pressure. The volume changes from 2 × 10–3m3 to 3.34 m3. The
P work done by the system is about
a (a) – 340 kJ (b) –170 kJ
b
c (c) 170 kJ (d) 340 kJ
d 21. A closed system undergoes a process 1 2 for which the values
e
f W1-2 and Q1-2 are 50 kJ and –20 kJ respectively. If the system is
V returned to state 1 and Q2 1 is + 10 kJ the work done W2 is
1
(a) a, f (b) b, c
(a) 40 kJ (b) 50 kJ
(c) b, e (d) c, e
(c) –60 kJ (d) –50 kJ
16. In the P-V diagram of figure shown, the gas does 5J of work along
22. A cylindrical tube of uniform cross-sectional area A is fitted with
isotherm ab and 4J along adiabat bc. What is the change in the
two air tight frictionless pistons. The pistons are connected to
internal energy of the gas if the gas traverse the straight path from
each other by a metallic wire. Initially the pressure of the gas is P0
a to c?
and temperature is T0, atmospheric pressure is also P0. Now the
temperature of the gas is increased to 2T0 , the tension in the wire
P
will be
a
b
wire
c
V

(a) 1J (b) – 4J (a) 2 P0A (b) P0 A


(c) 5J (d) 9J P0 A
17. Water is heat on a stove. The temperature of the water rises (i) (c) (d) 4 P0A
2
from 20ºC to 30ºC (ii) from 30ºC to 40ºC (iii) from 80ºC to 90ºC.
23. An ideal gas is subjected to cyclic process involving four
In which case the change in entropy is greatest
thermodynamic states, the amounts of heat (Q) and work (W)
(a) (i) (b) (ii) involved in each of these states
(c) (iii) (d) (ii) and (iii) Q1 = 6000 J; Q2 = –5500 J; Q3 = –3000 J; Q4 = + 3500 J
18. An ideal gas has temperature T1 at the initial state i shown in the W1 = 2500 J; W2 = –1000 J; W3 = –1200 J; W4 = xJ
P-V diagram. The gas has a higher temperature T 2 at the final
The ratio of the net work done by the gas to the total heat absorbed
states a and b, which it can reach the paths shown. The change in
by the gas is . The values of x and respectively are
entropy:
(a) 500; 7.5% (b) 700; 10.5%
P (c) 1000; 21% (d) 1500; 15%
T2 24. A thermally insulated rigid container contains an ideal gas heated
a by a filament of resistance 100 through a current of 1A for 5 min
then change in internal energy is
T1
T2 (a) 0 kJ (b) 10 kJ
i b
V (c) 20 kJ (d) 30 kJ
25. Two cylinders A and B fitted with pistons contain equal amounts
(a) greatest in a (b) greatest in b of an ideal diatomic gas at 300 K. The piston of A is free to move
(c) same in a and b (d) nothing can be said while that of B is held fixed. The same amount of heat is given to
19. Unit mass of a liquid with volume V1 is completely changed into the gas in each cylinder. If the rise in temperature of the gas in A is
a gas of volume V2 at a constant external pressure P and temperature 30 K, then the rise in temperature of the gas in B is
T. If the latent heat of evaporation for the given mass is L, then the (a) 30 K (b) 18 K
increase in the internal energy of the system is (c) 50 K (d) 42 K

Answer Key 15 (d) 17 (a) 19 (c) 21 (c) 23 (b) 25 (d)


Sol. from page 510 16 (b) 18 (b) 20 (d) 22 (b) 24 (d)
www.crackjee.xyz
LAWS OF THERMODYNAMICS 491
26. 1 k mol of a monoatomic gas is taken from a point A to another
point B along the path ACB. The initial temperature at A is T0. P P
A B A
The heat absorbed by the gas in the process A C B is
(c) (d)
P C B
C
V V
2P0 B 29. Carbon monoxide is carried around a closed cycle abc in which bc
is an isothermal process as shown in the figure. The gas absorbs
7000 J of heat as its temperature increases from 300 K to 1000 K
in going from a to b. The quantity of heat rejected by the gas
P0 C
A during the process ca is
V
V0 2V0 P
P2 b
11 9 RT0
(a) RT0 (b)
2 2
a
15 RT0 5RT0 P1 c
(c) (d) V
2 2 V1 V2
27. A cyclic process ABCD is shown in the figure P-V diagram. Which
of the following curves represent the same process (a) 4200 J (b) 5000 J
(c) 9000 J (d) 9800 J
P A B
30. Six moles of an ideal gas performs a cycle shown in figure.
C If the temperature are TA = 600 K, TB = 800 K, TC = 2200 K and
TD = 1200 K the work done per cycle is
D
V P B C

P P A
A B A B D
(a) (b) T
C C
D D (a) 20 kJ (b) 30 kJ
T T (c) 40 kJ (d) 60 kJ
31. In the following P-V diagram two adiabatics cut two isothermals
P P
B Va
A A B at temperatures T1 and T2 (fig). The value of V will be
(c) (d) d

D C C D P
T T
a
28. A cyclic process ABCA is shown in the V-T diagram. Process on
b
the P-V diagram is T1

V d c
T2
C B
V
A Va Vd Vb Vc
T

P P Vb Vc
C (a) (b)
B Vc Vb

(a) A (b) Vd
B (c) (d) VbV c
A C Va
V V

Answer Key 26 (a) 28 (c) 30 (c)


Sol. from page 510 27 (a) 29 (d) 31 (a)
www.crackjee.xyz
492 MECHANICS & THERMODYNAMICS
32. One mole of an ideal gas having initial volume V, pressure 2P and
35. The efficiency of an ideal gas with adiabatic exponent ' ' for the
temperature T undergoes a cyclic process ABCDA as shown below
shown cyclic process would be

2P
A B v
2V0 C
Pressure P

P C
D
V0 A
B
T 2T
The net work done in the complete cycle is
T0 2T0 T
(a) Zero (b) RT ln2 ( 2 ln 2 1) (1 2 ln 2)
(a) (b) /( 1)
3 / 1
(c) RT ln2 (d) T ln2
RT
2 ( 2 ln 2 1) ( 2 ln 2 1)
(c) (d)
33. In P-V diagram shown in figure ABC is a semicircle. The work /( 1) /( 1)
done in the process ABC is
36. The specific heat capacity of a monoatomic gas for the process
2 TV2 = constant is (where R is gas constant)
P(N/m )
(a) R (b) 2R
3 A
R R
B (c) (d)
3 4
1 37. An ideal gas can be expanded from an initial state to a certain
C
volume through two different processes,
3
0 V(m )
1 2 (i) PV 2 = K and (ii) P = KV 2 , where K is a positive constant.
Then, choose the correct option from the following
(a) Final temperature in (i) will be greater than in (ii)
(a) 4J (b) J
2 (b) Final temperature in (ii) will be greater than in (i)
(c) Work done by the gas in both the processes would be equal
(c) J (d) zero (d) Total heat given to the gas in (i) is greater than in (ii)
2
34. One mole of an ideal gas is taken from state A to state B by three 38. The molar heat capacity C for an ideal gas going through a given
different processes, process is given by C = a/T , where a is a constant. If
= CP/CV , the work done by one mole of gas during heating from
P T0 to T0 through the given process will be

B 1 1
(a) ln (b) a ln – T0
1 RT
a
C D E (c) a ln – ( – 1) RT0 (d) none of these
39. In an H2 gas process , PV 2 = constant. The ratio of work done by
A gas to change in internal energy is
V
(a) 2/3 (b) 0.4
(i) ACB (ii) ADB (iii) AEB as shown in the P-V diagram. The heat
(c) – 0.4 (d) – 2/3
absorbed by the gas is
40. In a process the pressure of a gas is inversely proportional to the
(a) greater in process (ii) than in (i)
square of the volume. If temperature of the gas is increased, then
(b) the least in process (ii) work done by the gas
(c) the same in (i) and (iii)
(a) is positive (b) is negative
(d) less in (iii) than in (ii)
(c) is zero (d) may be positive

Answer Key 32 (c) 34 (d) 36 (a) 38 (b) 40 (c)


Sol. from page 510 33 (c) 35 (a) 37 (b) 39 (c)
www.crackjee.xyz
LAWS OF THERMODYNAMICS 493
41. A system consists of a cylinder, piston and a spring as shown.
R
The initial volume of the cylinder is 100 l and its pressure is (a) (b) RT0
100 kPa so that it just balances the atmosphere pressure plus the T0 / 2
piston weight. In this position, the spring connected to the piston
3
exerts no force on it. Heat is now transferred to the system so as (c) RT0 (d) 2RT0
to expand air to double its volume, at which the pressure in the 2
cylinder is 300 kPa. The work done by the system is (in case of a 46. An exhausted chamber with nonconducting walls is connected
spring F = – kx) : through a valve to the atmosphere where the pressure is p0 and
the temperature is T0 . The valve is opened slightly and air flows
into the chamber until the pressure within the chamber is p 0.
Assuming the air to behave like an ideal gas with constant heat
capacities, the final temperature of the air in the chamber is
Air (a) T0 (b) (T0 / )
(c) (1 / – 1)T0 (d) ( / 1 – ) T0
Heat
(a) 200 J (b) 2 × 106 J 47. A cube of side 5 cm made of iron and having a mass of 1500 g
is heated from 25° C to 400°C. The specific heat for iron is
(c) 20 kJ (d) 2 kJ
0.12 cal/g°C and the coefficient of volume expansion is
42. A one mol sample of an ideal gas is carried around the
3.5 × 10–5/°C, the change in the internal energy of the cube is (atm
thermodynamics cycle shown in the figure. The cycle consists of pressure = 1 × 105 N/m2)
three steps : (i) an isothermal expansion (a b), (ii) an isobaric
(a) 320 kJ (b) 282 kJ
compression (b c), and (iii) a constant volume increases in
(c) 141 kJ (d) 423 kJ
pressure (c a). If Ta = 400 K, Pa = 4 atm and Pb = Pc = 1 atm,
48. During the charging of a storage battery, the current is 22 A and the
the work done by the gas per cycle is
voltage is 12 V. The rate of heat transfer from the battery is 12 W.
P The rate of change of internal energy is
a (a) 504 J/s (b) 252 J/s
Pa
(c) 25.2 J/s (d) 126 J/s
T = 400 K 49. A vertical cylinder closed from both ends is equipped with an
easily moving piston dividing the volumes into two parts, each
b
P=
b
Pc c containing one mole of air. In equilibrium at T0 300 K, the ratio
V of the volume of the upper and lower parts is 4. The temperature
Va= Vc Vb
at which this ratio will be 3 is
(a) 211.6 J (b) 1158 J
(a) 320 K (b) 420 K
(c) 705.3 J (d) 2116 J (c) 480 K (d) 500 K
43. A closed system undergoes a change of state by process 1 2 for 50. A vessel of volume V is evacuated by means of a piston air pump.
which Q12 = 10 J and W12 = – 5 J. The system is now returned to One piston stroke captures the volume v0. The pressure in the
its initial state by a different path 2 1 for which Q21 is – 3J. The
total energy change for the cycle is 1
vessel is to be reduced to of its original pressure P0. If the
(a) – 8J (b) zero n
(c) – 2J (d) + 5J process is assumed to be isothermal and air is considered an ideal
44. Relation between U, P, V for an ideal gas is U = 3 + 2PV then gas gas, the number of strokes needed in the process is
is
nn nn
(a) monoatomic (b) diatomic
v v
(c) triatomic (a) n 1 0 (b) n 1 0
V V
(d) mixture of monoatomic and diatomic
45. 1 mole of a monoatomic gas at temperature T 0 expand slowly
v0
according to the law P2 = T. Its final temperature is 2T0, then heat n 1
(c) V (d) none of these
supplies to the gas is
n

Answer Key 41 (c) 43 (a) 45 (d) 47 (b) 49 (b)


Sol. from page 510 42 (d) 44 (d) 46 (a) 48 (b) 50 (b)
www.crackjee.xyz
494 MECHANICS & THERMODYNAMICS

MCQ Type 2 Exercise 7.2


Multiple options correct 5. For any monoatomic gas the quantity/quantities independent of
1. The figure shows four path on P-V diagram along which a gas can the nature of the gas at the same temperature is/are
be taken from state i to f. The path according to the greatest change (a) the number of molecules in one mole
in internal energy is : (b) the number of molecules in equal volume and pressure
P (c) the translational kinetic energy of one mole
1 (d) the kinetic energy of unit mass
6. Temperature versus pressure graph of an ideal gas is shown in
i figure. During the process AB
2
T
3
4 f
V
A B
(a) 1 (b) 2 (c) 3 (d) 4
2. In figure here shows four paths traversed by a gas on a P-V diagram.
If U1, U2, U3 and U4 are the change in internal energies in P
their respective paths, then :
(a) internal energy of the gas remains constant
P (b) volume of the gas is increased
(c) work done by the atmosphere on the gas is positive
1
(d) pressure is inversely proportional to volume
2 3 7. One mole of a gas is subjected to two processes AB and BC, one
T3
after the other as shown in the figure. BC is represented by
4 T1
T2 PVn = constant. We can conclude that (where T = temperature,
W = work done by gas, V = volume and U = internal energy).

(a) U1 = U2 = U3 = U4 (b) U1 + U2 = U3 + U4 P
(c) U1 > U2 > U3 > U4 (d) None of these A B
3. During the melting of a slab of ice at 273 K at atmospheric pres-
sure
(a) Positive work is done by ice-water system on the atmosphere C
(b) Positive work is done on the ice-water system by the
V
atmosphere V0 2V0 3V0
(c) The internal energy of the ice-water system increases
(d) the internal energy of the ice-water system decreases (a) TA = TB = TC (b) VA < VB, PB < PC
4. The figure shows the P-V plot of an ideal gas taken through a cycle (c) WAB > WBC (d) TA < TB
ABCDA. The part ABC is a semi-circle and CDA is half of an 8. One mole of an ideal gas is taken through the cyclic process shown
ellipse. Then, P in the V-T diagram, where V = volume and T = absolute temperature
of gas. Which of the following statements is/are correct?
A V
3 A B
2V0

2 D
B
V0 C
D
1
C

0 T0 2T0 T
1 2 3 V
(a) The process during the path A B is isothermal (a) Heat is given out by the gas in the whole process
(b) Heat flows out of the gas during the path B C D (b) Heat is absorbed by the gas in the complete cycle
(c) Work done during the path A B C is zero (c) The magnitude of the work by the gas is RT0 In 2
(d) Positive work is done by the gas in the cycle ABCDA (d) The magnitude of the work done by the gas is V0T0

Answer Key 1 (a, b, c, d) 3 (b, c) 5 (a, b, c) 7 (c, d)


Sol. from page 514 2 (a, b) 4 (b, d) 6 (a, d) 8 (a, c)
www.crackjee.xyz
LAWS OF THERMODYNAMICS 495
9. Heat is supplied to a certain homogeneous sample of matter at a 13. An idea gas is taken from state 1 to state 2 through optional path
uniform rate. A, B, C & D as shown in P-V diagram. Let Q, W and U represent
the heat supplied, work done & internal energy of the gas
T respectively. Then

P
2
A B
C
1
D
(0, 0) V
t
(a) QB – WB > QC – WC (b) QA – QD = WA – WD
Its temperature is plotted against time as shown in the figure. (c) WA < WB < WC < WD (d) QA > QB > QC > QD
Which of the following conclusions can be drawn ? 14. Two gases have the same initial pressure, volume and temperature.
(a) its specific heat capacity is greater in the solid state than in They expand to the same final volume, one adiabatically and the
the liquid state other isothermally
(b) its specific heat capacity is greater in the liquid state than in (a) The final temperature is greater for the isothermal process
the solid state (b) The final pressure is greater for the isothermal process
(c) its latent heat of vaporization is greater than its latent heat (c) The work done by the gas is greater for the isothermal process
of fusion (d) All the above options are incorrect
(d) its latent heat of vaporization is smaller than its latent heat 15. A cyclic process ABCD is shown in the P-V diagram. Which of the
of fusion following curves represents the same process if
10. An ideal gas is taken from the state A (pressure P, volume V) to the BC & DA are isothermal processes
state B (pressure P/2, volume 2V) along a straight line path in the
P-V diagram. Select the correct statement (s) from the following P
(a) The work done by the gas in the process A to B exceeds the A B
work that would be done by it if the system were taken from
A to B along the isotherm. C
(b) In the T-V diagram, the path AB becomes a part of a parabola D
V
(c) In the P-T diagram, the path AB becomes a part of a hyperbola
(d) In going from A to B, the temperature T of the gas first
P V
increases to a maximum value and then decreases
11. During an experiment, an ideal gas is found to obey a condition A B D C
VP2 = constant. The gas is initially at a temperature T, pressure P
(a) (b)
and volume V. The gas expands to volume 4V. C B
P D A
(a) The pressure of gas changes to T T
2
(b) The temperature of gas changes to 4T
P V
(c) The graph of above process on the P-T diagram is parabola
A B
(d) The graph of above process of the P-T diagram is hyperbola B
12. During an experiment, an ideal gas is found to obey a condition A
(c) (d)
P2
= constant [ = density of the gas]. The gas is initially at D C D C
T T
temperature T, pressure P and density . The gas expands such
that density changes to /2. 16. Four Carnot engines operate between reservoir temperature of (i)
(a) The pressure of gas changes to 2P 300 K and 400 K (ii) 400 K and 500 K (iii) 500 K and 600 K (iv)
(b) The temperature of gas changes to 2T 600 K and 800 K. Which engines has greatest thermal efficiency?
(c) The graph of above process on the P-T diagram is parabola (a) (i) (b) (ii)
(c) (iii) (d) (iv)
(d) The graph of above process of the P-T diagram is hyperbola

Answer Key 9 (a, c) 11 (a, d) 13 (b, d) 15 (a, b)


Sol. from page 514 10 (a, b, d) 12 (b, d) 14 (a, b, c) 16 (a, d)
www.crackjee.xyz
496 MECHANICS & THERMODYNAMICS

St at ement Quest ions Exercise 7.3


Read the two statements carefully to mark the correct option out of the options given below:
(a) If both the statements are true and the statement - 2 is the correct explanation of statement - 1.
(b) If both the statements are true but statement - 2 is not the correct explanation of the statement - 1.
(c) If statement - 1 true but statement - 2 is false.
(d) If statement - 1 is false but statement - 2 is true.

1. Statement - 1 Zeroth law of thermodynamics explain the concept Statement - 2


of energy. Adiabatic expansion of the gas causes lowering of temperature
Statement - 2 and condensation of water vapours.
Energy depends on temperature. 7. Statement - 1
2. Statement - 1 Air quickly leaking out of a balloon becomes cooler.
Work and heat are two equivalent forms of energy. Statement - 2
Statement - 2 The leaking air undergoes adiabatic expansion.
Work is the transfer of mechanical energy irrespective of 8. Statement - 1
temperature difference, whereas heat is the transfer of thermal In an electric fan be switched on in a closed room, the air of the
energy because of temperature difference only. room will be heated.
3. Statement - 1 Statement - 2
Reversible systems are difficult to find in real world. Electrical energy is converted into heat.
Statement - 2 9. Statement - 1
Most processes are dissipative in nature. In an adiabatic process, change in internal energy of a gas is equal
4. Statement - 1 to work done on or by the gas in the process.
Thermodynamic processes in nature are irreversible Statement - 2
Statement - 2 Temperature of gas remains constant in an adiabatic process.
Dissipative effects can not be eliminated. 10. Statement - 1
5. Statement - 1 An adiabatic process is an isoentropic process.
In isothermal process whole of the heat energy supplied to the Statement - 2
body is converted into internal energy. Change in entropy is zero in case of adiabatic process.
Statement - 2 11. Statement - 1
According to the first law of thermodynamics Efficiency of a Carnot engine increase on reducing the temperature
Q= U + W. of sink.
6. Statement - 1 Statement - 2
When a bottle of cold carbonated drink is opened a slight fog Efficiency of a Carnot engine is defined as the ratio of net
forms around the opening. mechanical work done per cycle by the gas to the amount of heat
energy absorbed per cycle from the source.

Answer Key 1 (d) 3 (a) 5 (d) 7 (a) 9 (c) 11 (a)


Sol. from page 515 2 (a) 4 (a) 6 (a) 8 (a) 10 (b)
www.crackjee.xyz
LAWS OF THERMODYNAMICS 497

Passage & Matrix Exercise 7.4

4. As the bubble moves upwards, besides the buoyancy force the


Passage for (Q. 1 - 3) : following forces are acting on it
Two moles of helium gas are taken over the cycle ABCDA, as (a) Only the force of gravity
shown in the P – T diagram. (b) The force due to gravity and the force due to the pressure of
the liquid
P (c) The force due to gravity, the force due to the pressure of the
A B liquid and force due to viscosity of liquid
5
2×10
(d) The force due to the gravity and the force due to viscosity of
P(Pa) the liquid
1×10
5 5. When the gas bubble is at height y from the bottom, its temperature
D C is

300 K 500 K T 2/5


P0 gh
(a) T0
1. Assuming the gas to be ideal the work done on the gas in P0 gy
taking it from A to B is
(a) 200 R (b) 300 R 2/5
P0 g( H y )
(b) T0
(c) 400 R (d) 500 R P0 gH
2. The work done on the gas in taking it from D to A is
(a) – 414 R (b) + 414 R 3/5
P0 gH
(c) – 690 R (d) + 690 R (c) T0
P0 gH
3. The net work done on the gas in the cycle ABCDA is
(a) zero (b) 276 R P0 g( H – y )
3/ 5
(c) 1076 R (d) 1904 R (d) T0
P0 gH

Passage for (Q. 4 -6) : 6. The buoyancy force acting on the gas bubble is (Assume R is the
universal gas constant)
5
A small spherical monoatomic ideal gas bubble is trapped
3 ( P0 gH )2 / 5
(a) nRgT0
inside a liquid of density 1 (see figure). Assume that the bubble does ( P0 gy )7 / 5
not exchange any heat with the liquid. The bubble contains n moles of
gas. The temperature of the gas when the bubble is at the bottom is T0,
the height of the liquid is H and the atmospheric pressure is P0 (Neglect nRgT0
(b)
surface tension) ( P0 gH )2 / 5[ P0 g(H y )]3/ 5

P0
( P0 gH )3/ 5
Liquid (c) nRgT0
( P0 gy )8 / 5

H
nRgT0
(d)
y ( P0 gH )3/ 5[ P0 g (H y )]2 / 5

Answer Key 1 (c) 3 (b) 5 (b)


Sol. from page 515 2 (a) 4 (d) 6 (b)
www.crackjee.xyz
498 MECHANICS & THERMODYNAMICS
15. How many calories would be produced if the mechanical work
Passage for (Q. 7 - 9) : required to operate the refrigerator in previous question were
When a system is taken from state a to state b in figure along the path converted directly to heat?
acb, 80 J of heat flow into the system and 30 J of work are done. (a) 400 cal (b) 800 cal
(c) 1200 cal (d) 1600 cal

Passage for (Q. 16 -18) :


c b A diathermic piston divides adiabatic cylinder of volume V0 into two
Pressure

equal parts as shown in the figure. Both parts contain ideal monoatomic
gases. The initial pressure and temperature of gas in left compartment
are P0 and T0 while that in right compartment are 2P0 and 2T0. Initially
a d the piston is kept fixed and the system is allowed to acquire a state of
thermal equilibrium.
Volume

7. The heat flows into the system along path adb if the work is 10J.
P0 , T0 2 P0 , 2T0
(a) 40 J (b) 50 J
(c) 60 J (d) 70 J
8. When the system is returned from b to a along the curved path, the
work done by the system is –20J. The heat liberate is 16. The pressure in left compartment after thermal equilibrium is
(a) 40 J (b) 50 J achieved is
(c) 60J (d) 70 J 3
(a) P 0 (b) P0
9. If Ua = 0 and Ud = 40J, the heat absorbed in the process ad is 2
(a) 20 J (b) 30 J 4
(c) 40 J (d) 50 J (c) P0 (d) none of these
3
17. The heat that flown from right compartment to left compartment
Passage for (Q. 10 - 12) : before thermal equilibrium is achieved is
Consider the isothermal compression of 0.10 mole of an ideal gas at 3
T = 0°C. The initial pressure is 1 atm and the final volume is 1/5 the (a) P0V 0 (b) P0V0
4
initial volume.
3 2
(c) P0V0 (d) P0V0
10. The work done in the process is 8 3
(a) 242 J (b) – 366 J 18. If the pin which was keeping the piston fixed in removed and the
(c) –432 J (d) 512 J pisiton is allowed to slide slowly such that a state of mechanical
11. The heat exchange in the process is equilibrium is achieved. The volume of left compartment when
(a) –312 J (b) –366 J pistion is in equilibrium is
(c) +332 J (d) +432 J 3 V0
12. The change in internal energy in the process is (a) V0 (b)
4 4
(a) 72 J (b) 212 J
(c) 302 J (d) Zero V0 2
(c) (d) V0
2 3
Passage for (Q. 13 - 15) :
A carnot engine is operated between two reservoirs at temperatures of Passage for (Q. 19 - 21) :
400 K and 300 K. Figure shows the variation of the internal energy U with the density
of one mole of ideal monoatomic gas for a thermodynamic cycle ABCA.
13. If the engine receives 1200 cal from the reservoir at 400 K in each Here process AB is a part of rectangular hyperbola.
cycle, how many calories per cycle does it reject to the reservoir at
300 K? U
A C
(a) 400 cal (b) 650 cal 5U0
(c) 750 cal (d) 900 cal
14. If the engine is operated in reverse, as a refrigerator and receives
1200 cal from the reservoir at 300 K, how many calories does it
2U0 B
deliver to the reservoir at 400 K?
(a) 400 cal (b) 800 cal
(c) 1200 cal (d) 1600 cal
2 0 5 0

Answer Key 7 (c) 9 (d) 11 (b) 13 (d) 15 (a) 17 (c)


Sol. from page 515 8 (d) 10 (b) 12 (d) 14 (d) 16 (b) 18 (c)
www.crackjee.xyz
LAWS OF THERMODYNAMICS 499
19. The P-V diagram of above process is
Passage for (Q. 25 -27) :
P C P B In the given figure an ideal gas changes its state from state A to state C by
C
two paths ABC and AC
(a) (b)
B A
A P
V 2
V P(N/m )
B
15 C
P C 10
5 A
E D 3
0 V (m )
(c) (d) None of these 2 4 6
A B

V
25. The paths along which work done is least
20. The total amount of heat absorbed by the system for cyclic process (a) AB (b) AC
is
(c) BC (d) all are equal
10 10 26. The internal energy of gas at A is 10 J and amount of heat supplied
(a) ln 2.5 – 2 U 0 (b) ln 0.4 – 2 U 0
3 5 to change its state to C through the path AC is 200 J
(c) 50 U0 (d) None of these Calculate the internal energy at C
21. The work done in process AB is (a) 100 J (b) 120 J
(a) –U 0 (b) –2U 0 (c) 170 J (d) 220 J
(c) –5U 0 (d) None of these
27. The internal energy of gas at state B is 20 J. Find the amount of
heat supplied to the gas to go from A to B.
Passage for (Q. 22 -24) :
(a) 5J (b) 8J
Two moles of helium gas undergo a cyclic process as shown in figure.
(c) 12 J (d) 10 J
Assuming the gas to be ideal, calculate the following quantities in this
process.

P Passage for (Q. 28 -30) :


20,000 J of heat energy is supplied to a metallic object of mass 1 kg at
2 atm. A B
atmospheric pressure at 20°C.
Given specific heat of metal = 400 J/kg/°C. Density of metal
= 9000 kg/m3, coefficient of expansion = 9 × 10–5/°C. Atmospheric
D
1 atm. C pressure = 105 N/m2.
T
300 K 400 K
22. The net change in the heat energy is 28. The final temperature of metal
(a) 1050 J (b) 1150.7 J (a) 50°C (b) 70°C
(c) 1225 J (d) 1260.3 J (c) 90°C (d) 95°C
23. The net work done is 29. The work done by the metal
(a) 1150.7 J (b) 1260.3 J (a) 0.01 J (b) 0.04
(c) 625 J (d) none of these
(c) 0.05 J (d) zero
24. The net change in internal energy is
30. The change in internal energy of the metal
(a) 1150. 7 J (b) 625 J
(a) 10000 J (b) 14500 J
(c) 550 J (d) zero
(c) 17550 J (d) 19999.95 J.

Answer Key 19 (a) 21 (b) 23 (a) 25 (b) 27 (d) 29 (c)


Sol. from page 515 20 (a) 22 (b) 24 (d) 26 (c) 28 (b) 30 (d)
www.crackjee.xyz
500 MECHANICS & THERMODYNAMICS

(a) g ( L0 – H ) 2 P0 ( L0 – H ) L0 P0 0
Passage for (Q. 31 -33) :
A fixed thermally conducting cylinder has a radius R and height L0. The (b) g ( L0 – H ) 2 – P0 ( L0 – H ) – L0 P0 0
cylinder is open at its bottom and has small hole at its top . A piston of
mass M is held at a distance L from the top surface. as shown in the
figure. The atmospheric pressure is P0.
(c) g ( L0 – H ) 2 P0 ( L0 – H ) – L0 P0 0

2R
(d) g ( L0 – H ) 2 – P0 ( L0 – H ) L0 P0 0
L

Passage for (Q. 34 -36) :


L0 Two cylinder A and B having pistons (massless) of cross sectional area
b
100 cm2 and 200 cm2 respectively. The pistons are connected by massless
rod. The pistons can move freely without friction. The cylinder A contains
100 gm of an ideal gas ( = 1.5) at pressure 105 N/m2 and temperature
Piston
T0. The cylinder B contains identical gas at same temperature T0 but has
31. The piston is now pulled out slowly and held at a distance 2L different mass. The pistons are held at the state such that volume of gas
from the top. The pressure in the cylinder between its top and in cylinder A and cylinder B are same and is equal to 10–2 m3. The walls
the pistom will then be and piston of cylinder A are thermally insulated where as gas in cylinder
B is maintained at temperature T0. The whole system is in vacuum.
P0 Now the temperature T0. The whole system is in vacuum. Now the
(a) P0 (b)
2 piston is slowly released and its moves towards left and mechanical
equilibrium is reached at the state when the volume of gas in cylinder A
P0 Mg P0 Mg beocmes 25 ×10–4 m3.
(c) 2 (d) –
2 R 2 R2
32. While the piston is at a distance 2L from the top, the hole at the
top is sealed. The piston is then released, to a position where it A B
can stay in equilibrium. In this condition, the distance of the
piston from the top is
34. The mass of gas in cylinder B is
2 P0 R 2 P0 R 2 – Mg
(a) (2 L ) (b) (2 L ) (a) 200 gm (b) 600 gm
R2 P0 Mg R 2 P0
(c) 500 gm (d) 1 kg
35. The change in internal energy of gas in cylinder A is
P0 R2 Mg P0 R 2 (a) 2000 J (b) 1000 J
(c) (2 L ) (d) (2 L )
R2 P0 R2 P0 – Mg
(c) 500 J (d) 3000 J
33. The piston is taken completely out of the cylinder. The hole at 36. The compressive force in the connecting rod at equilibrium is
the top is sealed. A water tanks is brought below the cylinder and (a) 2000 N (b) 4000 N
put in a position so that thewater surface in the tank is at the (c) 8000 N (d) 10000 N
same level as the top of the cylinder as shown in the figure. The
density of the water is r. In equilibrium, the height H of the water
column in the cylinder satisfies

L0

Answer Key 31 (a) 33 (c) 35 (a)


Sol. from page 515 32 (d) 34 (d) 36 (c)
www.crackjee.xyz
LAWS OF THERMODYNAMICS 501

37. Column I contains a list of processes involving expansion of an ideal gas. Match this with Column II describing the thermodynamic
change during this process.
Column - I Column - II
A. An insulated container has two chambers separated (p) The temperature of the gas decreases
by a valve. Chamber I contains an ideal gas and the
Chamber II has vacuum. the valve is opened.

I II
ideal gas vacuum

B. An ideal monoatomic gas expands to twice its original (q) The temperature of the gas increases or remains
1
volume such that its pressure P where V is constant
V2
the volume of the gas
C. An ideal monoatomic gas expands to twice its (r) The gas loses heat
1
original volume such that its pressure P
V 4 /3
where V, is its volume
D. An ideal monoatomic gas expands such that its (s) The gas gains heat
pressure P and volume V follows the behaviour

shown in the graph

V
V1 2V1

38. One end of a copper rod is immersed in boiling water at 100°C, the other end in ice water mixture at 0°C. The sides of the rod are insulated.
During a certain time interval, 0.5 kg of ice melts. Match the following columns:
Column - I Column - II
A. The entropy change of the boiling water (p) 610 J/K
B. The entropy change of the ice-water mixture (q) zero
C. The entropy change of the copper rod (r) – 446 J/K
D. The total entropy change of the entire system (s) 164 J/K
39. Column-I Column-II
A. The coefficient of volume expansion at constant pressure (p) decrease in pressure
B. At constant temperature, an increase in volume results in (q) at all temperature
C. An ideal gas obeys Boyle’s and Charle’s law (r) same for all gases
D. A real gas behaves as an ideal gas at low pressure (s) at high temperature

Answer Key 37 (A) → q; (B) → p, r; (C) → p, s; (D) → q, s 39 (A) → r; (B) → p; (C) → q; (D) → s
Sol. from page 515 38 (A) → r; (B) → p; (C) → q; (D) → s
www.crackjee.xyz
502 MECHANICS & THERMODYNAMICS
40. The P-V diagram of 0.2 mol of a diatomic ideal gas is shown in figure. Process BC is adiabatic, = 1.4.
P B

1 atm
A 455K C
V
Column I Column II
A. Q AB (J) (p) 602
B. WBC (J) (q) – 644
C. UCA (J) (r) 1246
D. UBC (J) (s) – 602
41. A gas undergoes a process according to the graph. P is pressure, V is volume, W is work done by the gas, U is change in
internal energy of the gas and Q is heat given to the system.
P

C B

D A
1/V
Column-I Column -II
A. For process AB (p) U > 0, Q > 0
B. For process BC (q) U < 0, Q < 0
C. For process CD (r) Q× U×W=0
D. For process DA (s) Q× U<0
42. Match the column
Column I Column II
(Processes for ideal gases) (Symbols have usual meaning)
A. Isothermal (p) U=0
B. Isobaric (q) Q = 0
C. Isochoric (r) W = 0
D. Adiabatic free expansions (s) U = nCV T
43. There is an ideal gas sample. The ratio of CP and CV for gas sample is . In initial state its pressure is P1 and volume is V1. Now it is expanded
isothermally from volume V1 to V2 . Then it is compressed adiabatically from volume V2 to V1 again.
Regarding the above situation, match the following
Column I Column II
A. Heat given to system (i.e. ideal gas sample) during (p) Positive
isothermal expansion. 1
P1 V1 V2
B. Work done by gas during adiabatic compression (q) 1
– 1 V1
C. Change in internal energy of gas sample during 1
P1 V1 V2
adiabatic process. (r) 1
1– V1

D. Change in internal energy of gas sample from most (s) Negative.


initial state to the final state.

Answer Key 40 (A) → r; (B) → p; (C) → q; (D) → s. 42 (A) → p, s; (B) → s; (C) → r, s; (D) → p, q, r, s
Sol. from page 515 41 (A) → p, r; (B) → p; (C) → q, r; (D) → q 43 (A) → p; (B) → r, s; (C) → p, q; (D) → p, q
www.crackjee.xyz
LAWS OF THERMODYNAMICS 503
44. Match Column I with Column II and select the correct answer :
A. Isothermal process (p) No heat exchange
B. Isentropic process (q) Constant temperature
C. Isochoric process (r) Constant pressure
D. Isobaric process (s) Constant volume
(t) Constant enthalpy
45. Match Column I with Column II and select the correct answer using the codes given below the lists :
A. Absolute zero (p) Gaseous state
B. Triple point (q) Entropy of the universe
C. Critical temperature (r) Third law of thermodynamics
D. Adiabatic work (s) Sublimation
(t) First law of thermodynamics
46. Match Diagram I with Diagram II and select the correct answer using the codes given below the Diagrams.

Diagram I Diagram II
(P - V) (T - S)
A. 1 P (p) T
T1 p
1
B. 2 (q)
4 2 s q
C. 3 (r)
T2 3 r
D. 4 V (s) S

47. V – T graph of a thermodynamic process is shown in figure. If heat given to process is taken to be positive. ( Q is heat given to the process,
U is change in internal energy in the process, W is the work doen in the process)

V(m3 )

4V0 M

2V0 J

V0 L
K
T (K)
T0 2T0

Column I Column II
A. JK (p) W 0
B. KL (q) Q<0
C. LM (r) W<0
D. MJ (s) Q>0
(t) U =0

Answer Key 44 (A) → q; (B) → p; (C) → s; (D) → r 46 (A) → p; (B) → s; (C) → r; (D) → q
Sol. from page 515 45 (A) → r; (B) → s; (C) → p; (D) → t 47 (A) → (q, r, t) ; (B) → s; (C) → (p, s, t) ; (D) → (q, r)
www.crackjee.xyz
504 MECHANICS & THERMODYNAMICS
48. A sample of gas goes from state A to state B in four different manners, as shown by the graphs. Let W be the work done by the gas and U
be change in internal energy along the path AB. Correctly match the graphs with the statements provided.
Column I Column II

V
A. A B (p) Both W and U are positive
P

P B
B. A (q) Both W and U are negative
T

T B
C. A (r) W is positive whereas U is negative
V

V A

D. (s) W is negative whereas U is positive


B
P

Answer Key
Sol. from page 515 48 (A) → s; (B) → q; (C) → r; (D) → q

Subjective Integer Type Exercise 7.5


Solution from page 519
1. An ideal gas having initial pressure P, volume V and temperature T
is allowed to expand adiabatically until its volume becomes 5.66 V
T
while its temperature fall to .
2
(i) How many degrees of freedom do the gas molecules have ?
(ii) Obtain the work done by the gas during the expansion as a
function of the initial pressure P and volume V.
Ans : (i) 5; (ii) 1.25 PV.
2. Two Carnot engines A and B are operated in series. The first one A
receives heat at 900 K and rejects to a reservoir at temperature T
K. The second engine B receives the heat rejected by the first
engine and in turn rejects to a heat reservoir at 400 K. Calculate the
temperature T for the situation when (i) the efficiencies of the two Ans : 2000 N/m, 1295 J.
engines are equal (ii) the work outputs of the two engines are 4. A weightless piston divides a thermally insulated cylinder into
equal. two parts of volumes V and 3V. 2 moles of an ideal gas at pressure
Ans. (i) 600 K, (ii) 650 K. P = 2 atmosphere are confined to the part with volume V = 1 litre.
3. Two moles of an ideal monoatomic gas are confined within a The remainder of the cylinder is evacuated. Initially the gas is at
cylinder by a massless and frictionless spring loaded piston of room temperature. The piston is now released and the gas expands
cross–sectional area 4 × 10–3 m2. The spring is initially, in its to fill the entirespace of the cylinder. The piston is then pressed
relaxed state. Now the gas is heated by an electric heater, placed back to the initial position. Find the increase of internal energy in
inside the cylinder, for some time. During this time, the gas expands the process . The ratio of the specific heat of the gas = 1.5.
and does 50 J of work in moving the piston through a distance Ans : 400 J.
0.10 m. The temperature of the gas increases by 50 K. Calculate 5. Calculate the work done when one mole of a perfect gas is
the spring constant and the heat supplied by the heater. compressed adiabatically. The initial pressure and volume of the
gas are 105 N/m2 and 6 litre respectively. The final volume of the
gas is 2 litre. Molar specific heat of the gas at constant volume is
3R
. Ans : 957 J.
2
www.crackjee.xyz
LAWS OF THERMODYNAMICS 505
6. Find the amount of workdone to increase the temperature of one 8. Determine the work done by an ideal gas during a closed cycle
mole of an ideal gas by 30°C if it is expanding under the condition 1 4 3 2 1 shown in figure if P1 = 105 Pa, P0 = 3 × 105
V T2/3. (R = 1.99 cal / mol–K) Pa, P2 = 4 × 105 Pa, V2 – V1= 10 litre, and segments 4–3 and 2–1
Ans : 167 J of the cycle are parallel to the V–axis.
7. Three moles of an ideal monoatomic gas perform a cycle shown in
figure. The gas temperatures in different states are T1 = 400 K,
T2 = 800 K, T3 = 2400 K, and T4 = 1200 K. Determine the work
done by the gas during the cycle.

Ans. 750 J.

Ans. W = 3R (T1 + T3 – T2 – T4) = 20 kJ.

Subjective Exercise 7.6


Solution from page 520
1. Answer the following questions : 4. A sample of gas ( = 1.5) is compressed adiabatically from a
(a) A vessel with a movable piston maintained at a constant volume of 1600 cm3 to 400 cm3. If the initial pressure is 150 kPa,
temperature by a thermostat contains a certain amount of what is the final pressure and how much work is done on the gas
in the process ?
liquid in equilibrium with its vapour. Does this vapour obey
Ans : 1200 kPa, – 480 J.
Boyle’s law ? In other words, what happens when the volume
5. Two gases have the same initial pressure P0, volume V 0 and
of vapour is decreased ? Does the vapour pressure
temperature T0. They expand to the same volume, one adiabatically
increase ?
and the other isothermally.
(b) What is meant by ‘superheated water’ and ‘supercooled (i) In which case is the final pressure greater ?
vapour’ ? Do these states of water lie on its P–V–T surface? (ii) In which case is the work done greater ?
Give some practical applications of these states of water. (iii) In which case is the final temperature greater ?
Ans. Ans : (i) Final pressure is greater for the isothermal expansion.
(ii) More work is done in the isothermal expansion.
(a) No, the vapour is equilibrium with its liquid does not obey
(iii) Final temperature is greater for the isothermal expansion.
Boyle’s law. When the volume of the vapour is decreased by 6. One mole of an ideal gas whose pressure changes with volume as
applying pressure, some of the vapours condense into liquid, P = V, where is a constant, is expanded so that its volume
maintaining the same pressure of the vapour at the given increases times. Find the change in internal energy and heat
temperature i.e., vapour pressure does not increase when capacity of the gas.
the volume of vapours is decreased.
(b) Superheated water : Water in liquid phase having V2 2 R 1
Ans : [ 1], .
temperature above the boiling point of water at the given 1 2 1
pressure is called superheated water. It is highly unstable 7. Two rectangular boxes shown in figure has a partition which can
stage. slide without friction along the length of the box has one mole of
Suppercooled vapour : Water in vapour phase having 5
temperature below its boiling point at the given pressure is a monoatomic ideal gas at a pressure P0 ,volume V0 and
3
called supercooled vapour. It is also highly unstable stage. temperature T0. The chamber on the left is slowly heated by an
2. An ideal gas has a specific heat at a constant pressure, Cp = (5/2)R. electric heater.
The gas is kept in a closed vessel of volume 0.0083 m 3 at a
temperature of 300 K and a pressure of 1.6 × 10 6 N/m2. An
amount of 2.49 × 104 J of heat energy is supplied to the gas.
Calculate the final temperature and pressure of the gas.
Ans : 675 K, 3.6 × 106 N/m2.
3. One mole of a monoatomic gas is mixed with three moles of a
diatomic gas. What is the molecular specific heat of the mixture at The walls of the box and the partitions are thermally insulated.
constant volume? Take R = 8.31 J mol–1 K–1 ? Heat loss through the lead wires of the heater is negligible.
Ans : 18.7 J mole–1 K–1. The gas in the left chamber expands, pushing the partition until
www.crackjee.xyz
506 MECHANICS & THERMODYNAMICS
(a) Sketch P-V and P-T diagrams for the complete process.
243P0
the final pressure in both chambers becomes . Determine (b) Calculate the new work done by the gas and net heat supplied
32 to the gas during the complete process.
(i) the final temperature of the gas in each chamber and
(ii) the work done by the gas in the right chamber.
9 15
Ans : (i) T (ii) PV
4 0 8 0 0 Ans : (a) ,

5
8. Two moles of helium gas ( = ) are initially at temperature 27 °C
3
and occupy a volume of 20 litre. The gas is first expanded at
constant pressure until the volume is doubled. Then it undergoes
(b) W = 4.81 TA, Q = W
an adiabatic change until the temperature returns to its initial value.
(i) Sketch the process on a P–V diagram. 13. Calculate the difference in efficiencies of a Carnot engine
(ii) What are the final volume and pressure of the gas ? working between
(i) 400 K and 350 K and (ii) between 350 K and 300 K.
(iii) What is the work done by the gas ?
Ans. 1.8 %.
14. An ideal engine operates by taking in steam from a boiler at a
temperature of 327°C and rejecting heat to the sink at a temperature
of 27°C. The engine runs at 500 rpm and the heat taken is 600
k-cal in each revolution. Calculate
(i) The Carnot efficiency of the engine (ii) the work done in
each cycle
Ans. (i) (iii) the heat rejected in each revolution and
(iv) the power output of this engine.
Ans : (i) 50 % (ii) 1.26 × 106 J (iii) 300 kcal
(iv) 1.05 x 107 W.
15. A Carnot cycle is performed by air initially at 327°C. Each stage
(ii) volume = 113.14 x 10–3 m3, pressure = 0.88 x 105 N/ m2 represents a compression or expansion in the ratio 1 : 6. Calculate
(iii) 5000 J, 12450 J. (i) the lowest temperature and (ii) efficiency of the cycle.
9. One mole of an ideal monoatomic gas undergoes the process (Given = 1.4).
P = T1/2 where is a constant. (i) Find the work done by the gas Ans : (i) 293 K or 20°C (ii) 51.2 %.
if its temperature increases by 50 K. (ii) Find the molar specific 16. A Carnot engine having a perfect gas as the working substance is
heat of the gas. driven backward and is used for freezing water already at 0°C. If
the engine is driven by a 500 W electric motor having a efficiency
R 1 of 60 %, how long will it take to freeze 15 kg of water. Take 15°C
Ans : (i) 207.7 J (ii) .
2 1 and 0°C as the working temperatures of the engine and assume
10. One mole of an ideal gas with heat capacity at constant pressure there are no heat losses in the refrigerating system. Latent heat of
Cp undergoes the process T = T0 + V, where T 0 and are ice = 333 × 103 J/kg.
constants. Find Ans : 914.8 s.
17 If you are asked to increase the efficiency of a Carnot engine by
(a) heat capacity of the gas as a function of its volume,
increasing the temperature of the source or by decreasing the
(b) the amount of heat transferred to the gas, if its volume
temperature of the sink by 10 K, which method would you prefer
increased from V1 to V2.
and why ?
RT0 V2 Ans : The efficiency of a Carnot engine can be increased by a
Ans : (a) Cp + (b) (V2 – V1) Cp + RT0 loge V . greater amount by decreasing the temperature of the sink.
V 1
18. A perfect Carnot engine utilises an ideal gas as the working
11. A piston divides a closed gas cylinder into two parts. Initially the
substance. The source temperature is 227°C and the sink
piston is kept pressed such that one part has a pressure P and
temperature is 127°C. Find the efficiency of this engine, and find
volume 5 V and the other part has pressure 8 P and volume V. The
the heat received from the source and the heat released to the sink
piston is now left free. Find the new pressure and volume for the
when 10,000 J of external work is done.
adiabatic and isothermal processes. For this gas = 1.5.
Ans. 20%, 5 × 104 J, 4 × 104 J.
13P 30 48V 10V 8V 19. An ideal gas is taken through a cyclic thermodynamic process
Ans : , V , ; 1.84 P, , .
6 13 13 3 3 through four steps. The amount of heat involved in these steps are
Q1 = 5960 J, Q2 = – 5585 J, Q3 = – 2980 J and Q4 = 3645 J
7 respectively. The corresponding works involved are W1 = 2200 J,
12. Three moles of an ideal gas (C p = R) at pressure PA and
2 W2 = – 825 J, W3 = –1100 J and W4 respectively.
temperature TA is isothermally expanded to twice its initial volume. (i) Find the value of W4.
It is then compressed at constant pressure to its original volume. (ii) What is the efficiency of the cycle ?
Finally the gas is compressed at constant volume to its original Ans : (a) 765 J (b) 10.82 %
pressure PA.
www.crackjee.xyz
LAWS OF THERMODYNAMICS 507
20. One mole of a monoatomic ideal gas is taken through the cycle 24. Two moles of an ideal monoatomic gas, initially at pressure P1 and
shown in figure. volume V1 undergo an adiabatic compression until its volume is
V2. Then the gas is given heat Q at constant volume V2.
(a) Sketch the complete process on a P – V diagram.
(b) Find the total work done by the gas, the total change in its
internal energy and the final temperature of the gas. (Given
your answers in terms of P1, V1, V2, Q and R).

A B adiabatic expansion
B C cooling at constant volume Ans : (a)
C D adiabatic compression
D A heating at constant volume
The pressure and temperature at A, B etc. are denoted by
PA, PB, .....; TA, TB, ........, etc. respectively.
2 1
Given that TA = 1000 K, PB = P and PC = P .
3 A 3 A 2/3
3 V1
Calculate the following quantities : (b) W = PV 1 ,
(i) The work done by the gas in process A B 2 1 1 V2
(ii) The heat lost by the gas in process B C
(iii) The temperature TD . 2/3 5/3
3 V1 Q PV
1 1
2 /5 U= Q+ PV 1 , T = 3R
2 2 1 1 V2 2RV2 2/ 3
Given that = 0.85
3
25. Two vessels of volumes V1 and V2 contain the same ideal gas. The
Ans : (i) 1869.83 J (ii) 5297.63 J (iii) TD = 500 K
21. There are two vessels. Each of then contains one mole of a pressures in the vessels are P1 and P2 and the temperatures are
monoatomic ideal gas. Initial volume of the gas in each vessel is T1 and T2 respectively. The two vessels are now connected to
8.3 x 10–3 m3 at 27°C. Equal amount of heat is supplied to each each other through a narrow tube. Assuming that no heat is
vessel. In one of the vessels, the volume of the gas is doubled exchanged between the surrounding and the vessels, find the
without change in its internal energy, where as the volume of the common pressure and temperature attained after the connection.
gas is held constant in the second vessel. The vessels are now
connected to allow free mixing of the gas. Find the final temperature
and pressure of the combined gas system.
Ans : 369.3 K, 2.46 × 105 N/ m2.
22. The pressure in monoatomic gas increases linearly from 4 × 105 T1T2 ( PV
1 1 P2V2
PV 1 1 P2V2 )
N/m2 to 8 ×105 N/m2, when its volume increases from 0.2 m3 to Ans. ,
0.5 m3. Calculate the following : V1 V2 PV
1 1T2 P2V2T1
(i) Work done by the gas, 26. Two moles of an ideal monoatomic gas is taken through a cycle
(ii) Increase in the internal energy,
ABCA as shown in the P–T diagram. During the process AB,
(iii) Amount of heat supplied,
pressure and temperature of the gas vary such that PT = constant.
(iv) Molar heat capacity of the gas.
Ans : (i) 1.8 × 105 J (ii) 4.8 × 105 J If T1 = 300 K. Calculate
(iii) 6.6 × 105 J (iv) 17 J. (a) the work done on the gas in the process AB and
23. One mole of a diatomic ideal gas ( = 1.4) is taken through a cyclic (b) the heat absorbed or released by the gas in each of the
process starting from point A. The process A B is an adiabatic
processes. Give answers in terms of the gas constant R.
compression, B C is isobaric, C D an adiabatic expansion
and D A is isochoric. The volume ratios are VA / VB = 16 and
VC / VB = 2 and the temperature at A is TA = 300°K. Calculate the
temperature of the gas at the points B and D and find the efficiency
of the cycle.

Ans. (a) – 1200 R (b) –2100 R, 1500 R, 831.7 R.

Ans : 909 K, 791.3 K, 61.4 %.


www.crackjee.xyz
508 MECHANICS & THERMODYNAMICS
27. A cyclic process (cycle) 1–2–3–4–1 consisting of two isobars 2–
3 and 4–1, isochor 1–2, and a certain process 3–4 represented
by a straight line on the P–V diagram involves n moles of an ideal
gas. The gas temperatures in states 1, 2 and 3 are T1, T2 and T3
respectively, and points 2 and 4 lie on the same isotherm. Determine
the work done by the gas during the cycle.
5 2
Ans. Q = Q – 3 nRT – kl .
2
31. A thermally insulated vessel is divided into two parts by a
heat–insulating piston which can move in the vessel without
friction. The left part of the vessel contains one mole of an ideal
monoatomic gas, and the right part is empty. The piston is
connected to the right wall of the vessel through a spring whose
length in free state is equal to the length of the vessel.
Determine the heat capacity C of the system, neglecting the heat
T2 T3 capacities of the vessel, piston and spring.
Ans. W = nR (T2 – T1) 2
T1 T2
28. A vessel of volume V = 30 l is separated into three equal parts by
stationary semipermeable thin particles. The left middle and right
parts are filled with mH 2 = 30 g of hydrogen , mO2 = 160 g of
oxygen and m N 2 = 70 g of nitrogen respectively. The left partition Ans. C = 2R.
32. A thin U–tube sealed at one end consists of three bends of length
lets through only hydrogen, while the right partition lets through
l = 250 mm each, forming right angles. The vertical parts of the
hydrogen and nitrogen. What will be the pressure in each part of
tube are filled with mercury to half the height.
the vessel after the equilibrium has been set in if the vessel is kept
at a constant temperature T = 300 K ?

H2 O2 N2

Ans. P1 = 1.3 GPa , P2 = 4.5 G Pa, P3 = 2.0 G Pa.


29. A sample of 2 kg of monoatomic helium (assumed ideal) is taken
through the process ABC and another sample of 2 kg of the same gas
is taken through the process ADC . Given molecular mass of helium
=4
(i) What is the temperature of helium in each of the states A, B, All of mercury can be displaced from the tube by heating slowly
C and D ?
the gas in the sealed end of the tube, which is separated from the
(ii) Is there any way of telling afterwards which sample of helium
atmospheric air by mercury. Determine the work done by the gas
went through the process ABC and which went through the
there by if the atmospheric pressure is P0 = 105 Pa, the density of
process ADC ? Write yes or no.
mercury is mer = 13.6 × 103 kg/m3, and the cross–sectional area
(iii) How much is the heat involved in each of the processes
of the tube is S = 1 cm2.
ABC and ADC ?
5 7
Ans. W = P0Sl + gSl2 7.7 J.
2 4 Hg
33. A monoatomic ideal gas of two moles is taken through a cyclic
process starting from A as shown in figure. The volume ratios
are VB / VA = 2 and VD / VA = 4. If the temperature TA at A is 27°C,
calculate

Ans : (i) 120.33 K, 240.66 K, 481.32 K, 240.66 K (ii) No


(iii) 3.25 × 106 J , 2.75 × 106 J
30. A horizontal cylindrical vessel of length 2 l is separated by a thin
heat – insulating piston into two equal parts each of which contains
n moles of an ideal monoatomic gas at a temperature T. The piston
is connected to the end faces of the vessel by undeformed spings
of rigidity k each. When an amount of heat Q is supplied to the gas (a) the temperature of the gas at point B,
in the right part, the piston is displaced to the left by a distance (b) heat absorbed or released by the gas in each process,
x = l/2. (c) the total work done by the gas during the complete cycle.
Determine the amount of heat Q’ given away at the temperatureT Express your answer in terms of gas constant R.
to a thermostat with which the gas in the left part is in thermal Ans : (a) 327°C
contact all the time. (b) 1500 R, 831.6 R, – 900 R, –831.6 R (c) 600 R
www.crackjee.xyz
LAWS OF THERMODYNAMICS 509

Solutions Exercise 7.1Level -1


1. (a) W =area enclosed = P × V= PV or 600 – 400 – 300 + 200 = 300 – 200 – 150 + W4
W 4 = 150 J
n A RT n A RT n A RT 12. (a) For the container, Q = 0;
2. (c) P = – = ...(i)
V 2V 2V W= P V =0
nB RT nB RT nB RT Now Q= U W
and 1.5 P = – = ...(ii)
V 2V 2V or 0= U + 0 U=0
From above equations So, T = 0.
P 13. (a) In free expansion,
n
= A W = P V = 0 × V = 0.
1.5 P nB
Also Q=0
mA So, Q= U +W U=0
= m
B 14. (d) P, V and T are the parameter to characterize the thermodynamic
m B = 1.5 mA state of matter.
15. (b) Change in internal energy and entropy depends on initial and
3. (a) Cp – Cv = work done final state.
3R 16. (c) PV = nRT, is equation of ideal gas, so it true of any process.
4. (c) U = nCv T = n T
2 Vf
17. (b) W = nRT n
3 Vi
=1 8.32 (100 – 0)
2
= 1248 J 20
= 1 × 8.31 × 300 n
10
P
= 1728 J
5. (a) During the process, 18. (a) P = KT3; and
W = P V ; V increasing, PV
PV = nRT T=
so work done increases. nR
3
V PV
6. (d) The required fraction, So P= k
nR
Cv 1 1 5
F= = = = or PV 3/ 2 = constant
Cp 7 /5 7
= 3/2.
7. (a) dQ = dU + dW
19. (d) In reversible isochoric process, v = 0; so
As dW = 0 and dQ < 0, dU < 0, so internal energy and
hence temperature decreases. W = 0. Also U = 0.
8. (d) Polyatomic gas has non-zero moment of inertia so there are Q=0+0=0
rotational degrees of freedom in addition to translation. 20. (b) The heat reject by the refrigerator to source (room)
9. (b) In isothermal process, Q 1 = Q2 + W,
U = 0, So room will be heated.
Q=0+W T2 300
Area of P – V is greater for 1, so W and hence Q1 is greatest. 21. (d) =1– =1– = 0.95.
T1 6000
10. (c) Change in entropy,
Q Q2 1 1–
S = . 22. (b) = =
T1
=
T W
For adiabatic process –1
T2
Q = 0, and 1 – 110
Q2
so S =0 or = 1
10 10
11. (c) For cyclic process,
Q 2 = 90 J
U = 0,
Q= U W
www.crackjee.xyz
510 MECHANICS & THERMODYNAMICS
23. (b) The area of 2 is larger than 1, so work done will be negative.
PAV A PBVB
24. (a) 30. (c) TA = TB
T
2T0 15 5 10 15
273 427 = T2
Q3 Q1
T 2 = 1400 K
T0
Q2 15 10
S WAB = 10 = 125 atm-
S0 2S0 2
= 125 × 105 × 10–3
2T0 T0 3
Q1 = S0 T0 S0 = 12500 J
2 2
Thus Q= U W
Q 2 = T0S0 and Q3 = 0
or 5500 × 4.2 = U + 12500
Q1 – Q2 2 1
= U = 10600 J.
Q1 = 1– 3 = 3
U = 10600 = m Cv T
25. (d) W = WAODA + WOBCO
5
=+W–W=0 = Cv (700)
1000
26. (a) Q= U W
or C v = 3.03 kJ/kg – °C
or 40 = U 30 , U = 10 J.
31. (a) U = n Cv T
27. (d) The slope of adiabatic curve will be greater than slope of
isothermal curve. = 2 × 103 × 20 × 273
= 10.9 MJ.
nRT
28. (b) P(V – b) = nRT, P= . 32. (b) WABD = WAB + WBD
V –b
= 0 + 6 × 104 × 4 × 10–3
2V 2V = 240 J.
dV
Thus W= PdV = nRT
V –b Now, Q= U W
V V
or 500 + 150 = U 240
2V
= nRT n V – b V U = 410 J.
33. (a) WAB = 0,
2V – b
= nRT n R
V –b WBC = Ti – T f
–1
2V – b
= 2 RT n R
V –b T –T ,
=
–1 2 3
29. (b) dQ dU PdV dT CV dT PdV WCA = P V = R T = – R(T3 – T1)
T
RT W = WAB + WBC + WCA
dT Cv dT dV
T V R
= T2 – T3 – R T3 – T1
1
–1
C VT –1
e / RT After substituting values, we get
W = 75 R.

Solutions Exercise7.1 Level -2


1. (b) The double the volume, the temperature will also be doubled. 5 –1
–1
So, T = 2T – T = T = 273 K T1 L2 L2 3

Q = nC T T2 = L1 L1

1 2
= 3 4 273 L2 3
2 =
= 1638 J. L1
–1 –1
2. (d) Using, TV
1 1 = T2V2 5 3R
3. (d) U= 2 RT 4
–1 –1 2 2
or T1 AL1 = T2 AL2
= 11 RT
www.crackjee.xyz
LAWS OF THERMODYNAMICS 511
= 3.75 min.
P
4. (b) The slope of adiabatic curve = – 12. (a) For tiny glass tube,
V
P1V 1 = P2V 2
The r for monoatomic gas is greater, so 1 for O2 and 2 for
He. PV
1 1 4.5 0.50
P2 = V =
5. (c) The given processes are shown in figure. Clearly P2 > P1 2 500
and work done is negative.
= 0.0045 atm.
P Thus P = 1atm + 0.0045 atm
= 1.0045 atm = 76.34 cm of Hg.
P2
13. (b) W= P V = m C p – Cv T
P1
= 4(0.219 – 0.157) × 4200 × (120 – 20)
104 × 103J
V 14. (c) Slope of adiabatic curve
V1 V2
= × slope of isothermal curve
= 1.4 × (–400) = – 560 MPa/m3
6. (a) W = WAB + WBC + WCA 15. (d) For maximum positive work, the area enclosed in P – V
= WAB + 0 + WCA diagram should be maximum. By observation it will be
WAB = P ( V ) = 10 × (2 – 1) = 10 J. between the curves c and e.
Now, Q= U W 16. (b) Along ab, Q = 0 + 5 = 5. U ab = 0
or 5 = 0 + (10 + WCA)
WCA = – 5 J. Along bc, 0 = Ubc+ 4, U bc = – 4J

Q1 T1 273 227 In cyclic process abca,


7. (d) Q2 = T2 = 273 127 U ab + Ubc + Uca = 0

500 5 or, 0–4+ Uca = 0


= =
400 4
U ca = + 4J
5 4
Q 2 = Q1 = 6 104 = 4.8 10 4 cal Thus U ac = – 4 J
4 5
W = Q1 – Q2 = 1.2 × 104 cal. dQ
17. (a) S ; As T is smaller for (i), so
W T2 273 27 2
T
8. (b) Q1 = 1 – T1 = 1– 273 627 3
change in entropy is greatest in case (i)
2 2 18. (b) Qa = U +0= U
W= Q = × 3000 = 2000 kcal.
3 1 3 and Qb = U P V
= 8.4 × 106 J
As Qb Qa , Change in entropy is greater in case(b).
PV
9. (b) n= 19. (c) Q = mL = 1 × L =L; W = P(V2 – V1)
RT
Now Q= U W
1.6 106 0.0083
= = 5.33 or L= U + P(V2 – V1)
8.31 300
U = L – P(V2 – V1)
Q = nCv T
20. (d) W= P V = 1 × 105 × (3.34 – 2 × 10–3) = 340 × 103J.
3 8.31
or 2.49 × 104 = 5.33 × × T 21. (c) – 20 = U + 50 U = – 70.
2
From 2 1, U = 70 kJ.
or T = 375 K
Now 10 = 70 + W W = – 60 kJ.
T f = Ti + 375 = 675 K.
10. (c) Using, P1V 1 = P2V 2 P0 P2
(100 × 103 + 1000g × 20) × 3 = (100 × 103) × V2 22. (b) = P 2 = 2P0
T0 2T0
V 2 = 8.88 mm3
11. (b) Pt = mC T Now tension F= P A = P0A.
mC T 4 4200 5 23. (b) Q = Q1 + Q2 + Q3 + Q4
t= =
P 0.5 746 = 6000 – 5500 – 3000 + 3500
www.crackjee.xyz
512 MECHANICS & THERMODYNAMICS
= + 1000 J = – 29.9 kJ.
W = W1 + W2 + W3 + W4 W = WAB + WBC + WCA + WDA
= 2500 – 1000 – 1200 + x 40 kJ.
= + 300 + x Va Vd Va Vb
In cyclic process, 31. (a) We know that Vb = Vc =
Vd Vc
U =0
32. (c) WAB = P V
Now, Q= U W
or 1000 = 0 + (300 + x) =1 R T = RT
T.
x = 700 J
Pi
WBC = nR (2T ) n
W Pf
=
Q1 Q4
= 1 2 RT n 2 = 2 RT n 2
1000 WCD = P V
= = 10.5%
6000 3500
= nR T = 1 R T – 2T
24. (d) Q= U+0
= i RT = 12 × 100 × (5 × 60)
2 = –RT
= 3 × 104 J. Pi 1
W DA = nRT n =1 R T n
7R Pf 2
25. (d) Q = nCP × 30 = n 30
2 = – RT n 2
5R W = WAB + WBC + WCA + WDA
and Q= n T
2 = RT n 2
7R 5R r1r2 1 1
n 30 = n T 33. (c) W= =
2 2 2 2
or T = 42 K. = /2 J
26. (a) A C B: T B = 4T 0 34. (d) The change in internal energy U is same in all process.

Now, Q= U + W = nCv T P V Q ACB = U + WACB ,


Q ADB = U,
3
= R 3T0 RT0 Q AEB =
2 U + WAEB
Here WACB is positive and WAEB is negative.
11RT0
= Hence QACB > QADB > QAEB.
2
27. (a) Process AB is isobasic and BC is isothermal, CD isochoric 35. (a) WAB = 0, WBC = P V = nR T = – nRT0
and DA isothermic compression.
Vf
28. (c) Process AB is isobasic, AC isothermic and CB isochoric. WCA = nRT n = nR 2T0 n 2
Vi
29. (d) Q a b = nCv T
nR
8.31 Q BC = nC p T = T0
or 7000 = n 5 1000 – 300 –1
2
n = 0.48 W 2 n2 – 1
Efficiency, = =
Q / –1
Now, Q ca = nC p T
36. (a) dQ = dU + dW
7 8.31
= 0.48 1000 – 300 or nCdT = nCvdT + PdV
2
9800 J. P dV
C = Cv
30. (c) WAB = 0, WBC = P ( V ) n dT

= nR T = 6 × 8.31 × (2200 – 800) Differentiating TV 2 = constant, w.r.t. T, we get


= 69.8 kJ. dV V
= –
WCD = 0, W DA = – P ( V ) dT 2T
= nR T = 6 × 8.31 × (600 – 1200)
www.crackjee.xyz
LAWS OF THERMODYNAMICS 513
Q12 = U2 + W
P RT
Also, PV = nRT = or 10 = U 21 – 5
n V
U 21 = 15
RT V
Now, C = Cv + × – For 2 1:
V 2T
Q21 = U12 + W
= –15 – 3
3R R
= – = R. = –18 J
2 2 Total energy change = Q12 + Q21 = 10 – 18 = –8 J
nRT dU
37. (b) PV = nRT or P = 44. (d) Cv 2R
V dT
nRT 3R 5R
(i) V 2 = K or VT = C1 1 1
V alsoCv = 2 2 = 2R
3 2
nRT V
(ii) = KV2 or = C2 45. (d)
V T P2 T or P T
Clearly final temperature in (ii) will be greater than (i). 2T0 2T0
RdT0 RT0
T0 W PdV
38. (b) T 2 2
Q CdT a ln 0 a ln T0 T0

T0
T0
3R RT0
Q = U + W = PV T + W = T0 = 2RT0
R 2 2
U CV T ( 1)T0
1 V0

1 46. (a) The work done of thermodynamics,


W Q U a ln RT0 P0
1
Q= U W
PV
1 1 P2V2 PV P2V2
39. (c) W 1 1 or 0 = nCv ( T ) – P0V0
2 1 1
R
nR nR T P2V2 PV or 0= n (T – T0 ) nRT0
U T 1 1 1
( 1) ( 1) 1
T= T0
W 47. (b) Q = mC T 1.5 0.12 4200 (400 – 25)
0.4 ( for diatomic gas = 1.4)
U = 2.83 × 105 J
1 k W = P ( V ) P (V T )
40. (b) P P PV 2 k = 105 × (5 × 10–2)3 × 3.5 × 10–5 × 375 = 0.164 J
V2 V2
Thus Q= U+W
PV .V k nRTV k TV k1 or 2.83 × 105 = U + 0.164 ; U = 282 kJ
48. (b) The power input P = V i = 12× 22
Since temperature increases therefore volume decreases. = 264 W.
U = 264 – 12 = 252 W.
Pi
Pf 100 300
41. (c) Pav = = 103 Pa = 200 × 103 Pa
2 2 5V0
49. (b) V0 + 4V0 = (V + 3V) V=
and V = 100 × 10–3 m3 4
P1 + P1 = P2 P1 + P1= P2
W = Pav V 200 103 100 10 3

= 20 kJ RT0 RT0
Here P1 = and P2 = V
4V0 0
Vb Pa
42. (d) Wab = RT n R 400 n
Va Pb RT0 1
PP = V 1 – 4 ... (i)
Wbc = Pb V Pb (Vc – Vb ), 0

and Wca = P V 0 RT 1
Also PP = 1– ...(ii)
V 3
W = Wab Wbc Wca
From above equations, we get T = 420 K.
After substituting the values and simplifying, we get
W = 2116 J 50. (b) VP = (V + V0)P1
43. (d) From 1 2;
VP
P1 =
V V0

and VP1 = (V + V0)P2


www.crackjee.xyz
514 MECHANICS & THERMODYNAMICS

Solutions Exercise 7.2


1. (a,b,c,d) The change in internal energy depend on initial and final P
11. (a,d) VP2 = (4U) P22 P2 =
state and so it is same in all. 2
2. (a,b) In each path, the change in temperature is We have, PV = nRT
T = T2 – T1, U1 = U2 = U3 = U4. nRT
or V = ,
3. (b,c) At 0°C, water contracts, so work is to be done by P
atmosphere.
nRT
As Q = U–W P2 = K
P
U = Q + W.
or PT = constant.
4. (b,d) WDAB is +ve and WBCD is –ve. As WDAB > WBCD, so net
It represents hyperbola between P and T.
work done is positive.
Q = U +WBCD , as WBCD is negative, so Q will decreases. P2 P22 P
12. (b,d) = P2 =
/2 2
n0
5. (a,b,c) PV = nRT = RT = n0KT
N P
As PV = mRT orP = RT ; = .
RT
PV
or n0 = .
KT P2
Thus = K
So n0 is same for all at equal P, V and T. P / RT
or PT = constant;
3
Also E = kT. It represents hyperbola.
2
It is same for each gas. P
Also PT = × T2 , T2 = 2T
6. (a, d) As T = 0, so U = 0 2
Pressure is increasing, so volume will decrease. The work 13. (b,d) UA = UB = UC = UD = U
done will be negative. Q A = U + WA, and QB = U + WB,
Also PV = constant. Q C = U + WC, and QD = U + WD,
7. (c,d) VAB = VBC.; So area under AB is greater than area Also, WA > WB > WC > WD.
under BC. So WAB > WBC. QA – QD = WA – WD,
For A to B, V T. and QA > QB > QC > QD.
14. (a,b,c) The area under isothermal is greater than area under
8. (a,c) U = 0; Q = –W = – RT0 n2
adiabatic curve.
W = WBC + WDA = –2RT0 n2 + RT0 n2 = – RT0 n2 So Wisothermal > Wadiabatic.
9. (a,c) If H is the rate of heat supplied, then P
Q1 = H t1, Q2 = H t2,
T
Q3 = H t3, Q4 = H t2
As t1 > t3, so Q1 > Q3 and Isothermal
uid

so Csolid > Cliquid


liq

Adiabatic
As t4 > t2, so Q4 > Q2 and
id

V
sol

so L vapour > Lfusion


t
t1 t2 t3 t4 PV
In isothermal pressure, P2 = V
2

10. (a,b,d) The area under isotherm is smaller than area under straight
line process. V
In adiabatic process, P2 = P
Others are explained earlier. V2
P
For V2 > V, > 1 so P2 < P2.
P
15. (a, b)
Isotherm
P/2 300 1 400 1
16. (a, d) 1=
1– = ; 2 =1– =
400 4 500 5
V 500 1 600 1
V 2V 1– =1–
3= = ;
600 6 4 =
800 4
www.crackjee.xyz
LAWS OF THERMODYNAMICS 515

Solutions Exercise-7.3
1. (d) Zeroth law of thermodynamics tells about thermal 8. (a) The electrical energy first convert into kinetic energy of fan
equilibrium. and then into heat energy due to collisions with air molecules.
2. (a) Explanation is in the statements. 9. (c) In adiabatic process, Q = 0
3. (a) Explanation is in the statements. 0 = U + W or U = –W.
4. (a) Temperature will change in adiabatic process.
5. (d) In isothermal process, T = 0 and so U = 0. 10. (b) In adiabatic process, Q = 0.
Thus Q = 0 + W = W. Q
S= =0
6. (a) The opening of bottle is the rapid or adiabatic process. In T
the process temperature falls.
T2
7. (a) The quike process will be adiabatic process. 11. (a) = 1 ; clearly when T2 is decreases will increase.
T1

Solutions Exercise-7.4
Passage (Q1 – 3) : Passage (Q.7 – 9) :
1. (c) WAB = P( V)AB = nR T 7. (c) acb : 80 = Uab + 30 = 50 J.
= 2R × (500 – 300) U ab = 50 J
= 400 R. U ba = – 50 J
Pi 1 In path adb, Q = U + W = 50 + 10 = 60 J.
2. (a) W DA = nRT n = 2 R 300 n
Pf 2 8. (d) In path b to a : Q = Uba + W = – 50 – 20 = – 70J
= –600 R n 2 9. (d) U ad = 40 – 0 = 40 J
= – 414 R Q = U + W = 40 + 10 = 50 J.
3. (b) WBC = nR (500) n 2 = 2 R 500 n 2 Passage (Q.10 – 12) :
Vf
= 693 R 10. (b) W = nRT n
WCD = P( V)CD = nR T Vi
= 2R × (300 – 500) 1
= 0.10 R 273 n
= – 400 R. 5
W = WAB + WDA + WBC + WCD = – 365 J.
= 276 R 11. (b) Q = U + W = 0 – 366 = – 366 J.
Passage for (Q. 4 -6) : 12. (d) As the process is isothermal, so U = 0,
4. (d) The correct choice is (d) Passage (Q.13 -15) :
1 Q1 T1 400
5. (b) For an adiabatic pressure TP = constant. 13. (b) Q2 = T2 = 300
2/5 2/ 5 3 3
T0 P0 gH = T0 P0 g ( H – y)
Q2 = Q = × 1200 = 900 cal.
4 1 4
2/5
P0 g (H – y ) T1 4
Which gives T = T0
( P0 gH ) 14. (d) Q 1 = T Q2 = × 1200 = 1600 cal.
2 3
So the correct choice is (b).
Q2 1
6. (b) Buoyant force F = weight of liquid displaced = Vg, V = 15. (a) = =
W T1
volume of the bubble –1
T2
nRT
From PV = nRT, we have V Therefore,
P 1200 1
or =
nRT g W 400
F –1
P 300
W = 400 cal.
2 /3
nR g P0 g ( H y) Passage (Q.16 -18) :
T0
[ P0 g (H y )] P0 gh 16. (b) n1Cv (T – T0) + n2 Cv (T – 2T0) = 0
3
mR g T T0
2
( P0 gH )2 / 5[ P0 g (H y )]3 / 5
PT
i f 3
Hence the correct choice is (b) Pf P0
Ti 2
www.crackjee.xyz
516 MECHANICS & THERMODYNAMICS
17. (c) Q n1Cv (T f – T0 ) = 1150.7 J Ans.

P0V0 3 3 3 In closed cycle, U = 0.


R T0 – T0 P0V0 Now by first law of thermodynamics
2 RT0 2 2 8
Q= U W
18. (c) Let V is change in volume in any compartment then = 0 + 1150.7
= 1150.7 J. Ans.
V0
Pf – V Passage (Q.25 - 27) :
P0V0 2 25. (b) Work done for path A – B – C
n1 and
2 RT0 RT f WAB = PDV
= 0,
V0 WBC = PB (VC – VB)
Pf V
2 P0V0 2 = 15 (6 – 2)
n2 V 0 = 60 J,
2 RT0 RT f
WABC = WAB + WBC
Passage (Q.19 -21) : = 60 J
19. (a) Process AB, UP = constant Work done for the path A – C
P RT 1
and U WAC = [15 5] 4
M t 2
P = const = 40 J
Process BC isochoric Clearly work done along path AC is minimum.
Process CA isothermal 26 (c) Along path A – C
20. (a) Q = QAB + QBC + QCA Q= U+W
or 200 = (UC – 10) + 40
10U 0
Q = – 5U0 + 3U0 + ln 2.5 U C = 170 J.
3 Ans.
21. (b) WAB = QAB – UAB = – 5U0 – (–3U0) = – 2U0 27 (d) Along path A – B
Passage (Q.22 - 24) :
Q= U+W
22. (b) 23. (a) 24. (d) = (20 – 10) + 0
Initial volume of the gas = 10 J.
nRT Ans.
VA = Passage (Q.28 - 30) :
P
2 8.31 300 28. (b) If T is the change in temperature of the metal object, then
= Q = mc T
2 1.013 105
= 0.025 m3. or 20000 = 1 × 400 × T
TB T = 50 oC.
Volume at B V B = T VA
A The final tempratureTf = Ti + T
= 20 + 50
400 = 70 oC.
= 0.025
300
m
= 0.003 m3 29 (c) Volume of the object,V=
Work done: WAB = PB (VB – VA)
= 2 × 1.013 × 105 (0.033 – 0.025) 1
= m3
= 1620.8 J 9000
PB If DV is the change in volume of the object, then
WBC = nRTB ln P V =V T
C
= 2 × 8.31 × 400 ln 2
= 4608.0 J 1 5
WCD = nR (TD – TC) = 9 10 50
900
= 2 × 8.31 (300 – 400)
= 5 × 10–7 m3.
= – 1620.8 J
Work done in expansion
PD
W DA = nRTD ln P W =P V
A = 105 × 5 × 10–7
= 2 × 8.31 × 300 ln 1/2 = 0.05 J Ans.
= – 3456.0 J 30 (d) Now, Q= U +W
Total work done W = WAB + WBC + WCD + WDA
U =Q–W
www.crackjee.xyz
LAWS OF THERMODYNAMICS 517
= 2000 – 0.05
P0 L0
= 19999.95 J Ans. P1 = ( L – H ) … (1)
0
Passage for (Q. 31 -33) :
31. (a) Since the cylinder has a hole at the top, it is open to Also P2 = atmospheric pressure + pressure due to a column of
atmosphere. Hence the pressure in the cylinder = atmospheric length (L0 – H) of water
pressure P0, which is choice (a). = P0 g ( L0 – H ) … (2)
32. (d) Let A = R2 be the cross-sectional area of the cylinder. Let x
be the distance of the piston from the top when equilibrium g ( L0 – H )2 P0 ( L0 – H ) – P0 L0 0
is attained. Since the process is slow, it is isothermal. Thus
Hence the correct choice is (c).
P1V1 = P2V2 Passage (Q.34 - 36):
where P1 = P0, V1 = A × (2L) mA =100 gm ; VA = VB = 10–2 m3
PA = 105 N/m2.
2R In cylinder A, the process is adiabatic, so

1 1 = P2V2
PV
10 5(10 –2)1.5 = P2 (25 × 10–4)1.5
P2 x P 2 = 8 × 105
34. (d) For cylinder B,
F 8000
Pf = A =
P1 = P2 2 200 10 – 4
= 4 × 105 N/m2
WB = fx = WA
or 8000x = 2000 x = 0.25m
VB = Ax = 200 × 10–4 × 0.25
= 50 × 10–4 m3
Piston Vf = 10–2 + 50 × 10–4 = 250 × 10–4m3
For cylinder A,
At equilibrium, the downward force equals the upward force 100
i.e. Mg = (P0 – P2)A 105 × 10–2 = RT0 ...(i)
M
Mg For cylinder B,
Which gives P2 = P0 – , Also V2 Ax .
A mB
4 105 250 10 –4 = RT0 ...(ii)
Substituting for P1, V1, P2 and V2 in Eq. (1) we have M
After solving above equations, we get
Mg
P0 × A × (2 L) = P0 Ax m B = 1000 gm.
A
1 1 – P2V2
PV
35. (a) WA =
P0 (2 L ) P0 R 2 –1
x= (2 L)
Mg R 2 P0 Mg
P0 – 105 10 –2 – 8 105 25 10 –4
A =
1.5 – 1
Hence the correct choice is (d)
= – 2000 J
33. Let A be the corss-sectional area of the cylinder. At equilibrium. Now Q A = U + WA
Pressure at the top level of water inside the cylinder =
or 0 = U – 2000 U = 2000 J.
pressure just below the level of water i.e.,
36. (c) So at equilibrium, the force on the connecting rod
P1 = P2
F = P2A2 = 8 × 105 × 1000 × 10–4
= 8000N
37. (A) (q)
As the ideal gas expands in vacuum, no work is done (W = 0). Also
the container is insulated therefore no heat is lost or gained
(Q = 0). According to first law of thermodynamics
P1 (L0 –H) U=Q+W
L0 U=0
There is no change in the temperature of the gas
P2 H
(B) (p, r)
Given PV2 = constant ....(i)

From Boyle’s law, we have PV


Also for an ideal gas = constant ....(ii)
T
Pt A( L0 – H ) = P0 L0 A From (i) & (ii) V × T = constant
As the gas expands its volume increases and temperature decreases
(p) is the correct option.
www.crackjee.xyz
518 MECHANICS & THERMODYNAMICS
To find whether heat is released or absorbed let us find a (C) q As the sides of the rod are insulated, so
relationship between Q and change in temperature T.
Q
We know that Q = nC T ...(i) S= = 0.
where C = molar specific heat T
Also for a polytropic process we have (D) s The total change in entropy
R S = 610 – 446 = 164 J/K
C = Cv + and PVn = constant
1– n 1
Here PV2 = Constant.Therefore n = 2 39. (A) r Cv = / °C same for all gases
273
R
C = Cv + = Cv – R 1
1- 2 (B) p PV = cos A P
V
3 (C) q Ideal gas obey gas law at each range of temperature.
For monoatomic gas Cv = R
2 (D) s
At high temperature, intermolecular forces become zero and
3 R so real gas behaves like ideal gas.
C= R –R=
2 2 40. A r; B p; C q; D s
Substituting this value in (1), we get 41. A p, r; B p; C q, r; D q
R 42. A p, s ; B s; C r, s ; D p,q,r,s
Q=n× × T. 43. A p; B r, s; C p, q; D p,q
2
In this case the temperature decreases i.e. T is negative. Therefore 44. (A) q Isothermal process occur at constant temperature.
Q is negative. This in turn means that heat is lost by the gas during (B) p For constant entropy, Q = 0.
the process. (r) is the correct option. (C) s Isochoric process is at constant volume.
(C) (p, s) (D) r Isobaric process is at constant pressure.
Proceeding in the same way we get in this case 45. Explanation is given in theory of the chapter.
V1/3× T = constant
46. (A) p
As the gas expands and volume increases, the temperature
decreases. Therefore (p) is the correct option. It is the isothermal expansion at T1.
(B) s
4
In this process, x =. Adiabatic expansion in which temperature falls from T1 to T2.
3 (C) r
R 3 3R 3 3R Isothermal compression at T2.
C = Cv + = R+ = R – 3R =
4 2 1 2 2 (D) q
1
3 Adiabatic compression at T2.
47. (A) (q,r,t)
3R
Q=n Δt As temperature is constant, so U = 0. Volume is decreasing,
2 W = –ve. Thus Q = 0 – W = – W.
As T is negative, Q is positive. This in turn means that heat is (B) s
gained by the gas during the process (s) is the correct option. V = 0, W = 0. Also Q = U + 0 = U. As temperature is
(D) (q, s) increasing, so U = + ve and so Q > W.
PV (C) (p,s,t)
Also T =
nR As temperature is constant, so U = 0. Volume is increasing in the
Here (PV) is positive T is positive temperature increases. process, so W = + ve.
So, (q) is the correct option Now Q = + W = W, Q > 0.
From the graph it is clear that during the process the pressure (D) (q,r)
of the gas increases which shows that the internal energy of the It is isobasic process in which volume is decreasing, so, W = – ve.
gas has increased. Also the volume increases which means work is Q = U – W.
done by the system which needs energy. From these two
48. (A) s
interpretation we can comfortably conclude that the gas gains heat
In the process, pressure increasing which V = 0, so temperature
during the process. (s) is the correct option.
will increase.
Q – mL (B) q
38. (A) r S= =
T T In the process, temperature and volume both decrease, and so
–0.5 4200 80 U = 0 and W = – ve.
= (C) r
373
– 446 J/K In the process temperature decreases and volume increases, so
U = –ve and W = + ve.
Q – mL
(B) p S= = (D) q
T T
In the process VB < VA, V = – ve and so W = – ve. P and V both
–0.5 4200 80 decrease, so T must be decreased. Therefore U = –ve.
= 610 J/K
273
www.crackjee.xyz
LAWS OF THERMODYNAMICS 519

Solutions Exercise-7.5
1 For the adiabatic compression, let P2 be the final pressure, then
1. (i) We know that V1 T1 = V2 1T2
P1(4V) = P2V
T P 2 = P1(4)1.5
or –1 ×
V T = (5.66V)
1
2 = 8 P1
The change in internal energy,
= 1.4 (diatomic gas)
Thus degrees of freedom of the gas is 5. U = nCV T
Work done in the process
R
=n [T2 T1]
PV
1 1 P2V2 1
W=
1
P2V2 PV1 1
= = 400 J.
PV P2V2 1
Also = T
T 2
1
4V
P For adiabatic process, T2 = T0 = 2 T0 Ans.
V
P2 =
11.32
5. For the gas = 5/3.
P PV P2V2
PV 5.66V 1 1
Now W= 11.32 =1.25 PV. Work done W=
1
1.4 1
After substituting the values and simplifying, we get
Ans.
W = 957 J Ans.
WA T1 T2 6. Given, V = kT 2/3
2. A = Q = T
1 1 2 1/ 3 2 1/ 3
dV = k T dT = kT dT
3 3
WB
and 2 = ' Work done dW = PdV
Q1
RT
= dV
T1' T2' V
=
T1'
RT 2 2
(i) Putting WA = WB and known values, we get = kT 1/ 3
dT = R ( dT )
kT 2 / 3 3 3
T = 600K
(ii) Putting = , 2 T2
A B Total work done W= R dT
3 T1
T = 650 K. Ans.
3. Work done in the process 2
= R [T2 T1]
1 2 3
W = (PA)x + kx
2 = 2 × 1.99 × 30 = 39.8 cal = 167 J.
Ans.
1
or 50 = 105 × 4 × 10–3 × 0.1 + k (0.1) 2 7. In the process 1 to 2 and 3 to 4,
2
P T,
k = 2000 N/m. Ans. and so volume of the gas remains constant.
Also U = nCV T W 1 2 = W34 = 0
W 2 3 = P2(V3 – V2)
3R and W 4 1 = P1(V1 – V4)
= 2 50 = 1245 J
2 Thus total work W = P2(V3 – V2) + P1(V1 – V4)
Thus Q= W = 1245 + 50 = 1295 J.
U We have P1V 1 = 3RT
Ans. P1V 4 = 3RT 4
4. If P1 be the pressure after expansion, then P2V 2 = 3RT 2
PV = P1 (3V + V) and P2V 3 = P3V 3
= 3RT 3
P Substituting these values into the expression, we get
P1 = .
4 W = 3R(T1 + T3 – T2 – T4)
= 2 × 10 4 J. Ans.
www.crackjee.xyz
520 MECHANICS & THERMODYNAMICS
8. The work done in the complete cycle is equivalent to two cycles
( P2 P0 ) 2
1 0 2 1 and 0 4 3 0.
W 2 = – W1 2
In the first cycle the work is positive while in the second cycle it P0 P1
is negative. The work done in the first cycle Total work done W = W1 + W2
P1 )(V2 V1 )
( P0
W1 = . 1 ( P2 P0 ) 2
2
= W1 2
For the second cycle, the triangle on the P-V diagram similar to the P0 P1
triangle corresponding to the first cycle. Therefore the work done
750 J. Ans.
in the second cycle

Solutions Exercise-5.6
1. Answer is the solution. 5. The two processes are shown in figure.
2. We know that PV = nRT
P Isothermal
PV 1.6 106 0.0083 16
n= = =
RT 8.31 300 3
5R
Given Cp = Adiabatic
2
V
3R (i) From the P-V diagram, it is clear that pressure in isothermal
Cv = expansion is greater than adiabatic expansion.
2
(ii) Area under P-V graph is greater for isothermal expansion,
If T be the change in temperature, then and so work done is greater in this case.
Q = nCv T (iii) As T is proportional to P, so temperature is greater in case of
16 3 isothermal expansion.
or 2.49 × 104 = R T 6. Given P= V ................(i)
3 2
The work done in the process
T = 375 K
V V
Final temperature T' = T + T W= PdV = VdV
V V
= 300 + 375 = 675 K
P T V2 2
At constant volume = = 1
P' T' 2
The change in internal energy
T' 675
P' = P = 1.6 106 R
T 300 U = nCv T =1 Tf Ti
= 3.6 × 106 N/m2 Ans. 1

n1Cv n2Cv Pf V f PV
i i
1 2
3. Cv mix = = 1
n1 n2

2
3R 5R V2 V2 2
2
1
1 3 = = V
= 2 2 1 1
1 3
The heat exchange in the process
9R 9 Q= U+W
= = 8.31 =18.7 J/mol-K Ans.
4 4
V2 2 V2 2
4. For adiabatic process = ( 1) [ 1]
( 1) 2
1 1 = P2V2
PV
V2 2 1
1.5 = ( 1)
PV
1 1 1600 2 1
P2 = = 150 = 1200 kPa
V2 400 By PV = nRT,
Work done
PV
PV P2V2 T= (n = 1)
1 1 R
W =
1
V V V2
3 6 3 6 Ti = =
150 10 1600 10 1200 10 400 10 R R
= =– 480 J.
1.5 1
2
Ans. and Tf = V2
R
www.crackjee.xyz
LAWS OF THERMODYNAMICS 521
By the definition of heat capacity
2 8.3
= [600 300] = 7450 J
Q R 1 5
C= = Ans. 1
Tf Ti 2 1 3
7. (i) Let P 1,V 1 and T 1 are the new pressure, volume and Total work done W = WAB + WBC
temperature of the right chamber. Then = 12450 J. Ans.
1/ 2
P0V0 = PV
1 1 9. (i) Given P= T
By ideal gas equation PV = RT
243
or P0V0 = P0 V1 or T 1/ 2 V = RT
32
8
or V = RT 1/2
V1 = V0
27 R –1/ 2
or dV = T dT
2
Again V0 1T0 = V1 1
T1
R
1 or ( T 1/ 2 )dV = dT
V0 9 2
T1 = T0 = T0 Ans.
V1 4 R
or P(dV) = dT
(ii) Work done by the gas in right chamber 2
P0V0 PV1 1
T2
W= Work done W= PdV
1 T1

T2 R R
243P0 8V0 = dT = [T2 T1 ]
P0V0 15
T1 2 2
32 27 P0V0
= =
5 8 8.31
1 = 50 = 207.7 J
3 2
(ii) If C be the molar specific heat, then by first law of
Ans. thermodynamics
8. For perfect gas PV = nRT
Q= U W
nRT
P= R
V or C T = CV T T
2
2 8.3 300
= = 2.5 × 105 N/m2 R R R 1
20 10 3
C= = . Ans.
1 2 2 1
P 10. (a) First law of thermodynamics
(i)
5 A B dQ = dU + dW
2.5 × 10 If C be the heat capacity of the gas, then
5 CdT = CV dT + PdV
0.44 × 10 C
dV
or C = CV + P
dT

RT dV
V = CV +
0.02 0.04 0.113 V dT
(ii) At B: PB = 2.5 × 105 N/m2 Given T = T0 + V
VB = 2V One differentiating w. r. t. T, we get
= 2 × 20 × 10–3 = 0.04 m3 dV
T B = 600 K 1=0+
dT
Now VB 1TB = VC 1TC
dV 1
1 =
dT
TB 1
VC = VB R (T0 V) 1
TC Now C = CV
V
= 0.04 × 2 2 = 0.113 m3, RT0
= CV R
(iii) Work done WAB = P V V
= (2.5× 105) × (40 – 20) × 10–3 RT0
= 5000 J = (CV R)
V
nR
WBC = [TB TC ] RT0
1 = CP . Ans.
V
www.crackjee.xyz
522 MECHANICS & THERMODYNAMICS
(b) Given, T = T0 V WBC = P (VC – VB)
Also PV = RT (n = 1) PA
= (VA – 2VA)
RT0 2
From above P= R = – PA VA / 2 = – nRTA / 2
V
3 8.3
V2 = TA = – 12.45 TA
Work done W= P.dV 2
V1
WCA = 0
V2 RT0
= R dV Total work done W = WAB + WBC + WCA
V1 V
= 4.81 TA
= RT0 ln(V2 / V1 ) R(V2 V1 ) As U=0
The heat transferred Q = W. Ans.
Q = U W = CV T W T 350
13. =1 2 = 1 100 = 12.50%
= CV [ V] W T1 400
R 300
= [ (V2 V1 )] W T2 '
1 and '= 1 = 1 100 = 14.3%
T1 ' 350
V2 Difference in efficiencies
= (V2 V1 ) C P RT0 ln Ans.
V1 '– = 1.8%. Ans.
11. If V1 and V2 are the final volumes under isothermal condition and
T2 300
P' is the common pressure, then 14. (i) Efficiency = 1 T =1 = 50%
P × 5V = P'V 1 ........... (i) 1 600
and 8P × V = P'V 2 ........... (ii) (ii) The heat taken in each revolution
Also V1 + V2 = 5V + V ............(iii) Q = 600 kcal
After solving above equations, we get
W
30 We know that h =
V1 = V Q
13
W= Q
24 = 0.5 × (600 × 103) × 4.2
V2 = V
13 = 1.26 × 106 J Ans.
(iii) Heat rejected Q 2 = Q1 – W
13P = 600 – 300 = 300 kcal
P '= Ans.
6
W
Under adiabatic condition:P (5V) = P" V1 (iv) Power output P=
t
and 8 PV = P" V2 500 1.26 106
Also V1 + V2 = 5V + V =
60
After solving above equations, we get
= 1.05 × 104 kW Ans.
10V 1 1
V1 = 15. (i) We can write V1 T1 = V2 T2
3
8V V1
1
V2 = T2 =
3 V2
P" = 1.84 P Ans.
12. (a) 1.4 1
T2 = 1 × 600 = 293 K Ans.
V T
2
B TA B A
2VA T2 293
(ii) Efficiency, =1 =1 = 51.2 % Ans.
T1 600
TA/2 16. Given, T 1 = 273 K,
VA A C
C T 2 = 15 + 273 = 280 K
Useful power = hP
O P O P = 0.6 × 500 = 300 J/s
PA/2 PA PA/2 PA
We know that coefficient of performance
For the isothermal process
PA VA = PB (2VA) Q2 T2
= =
PB = P / 2 . W T1 T2
A
VB 273
(b) Work done WAB = nRTA ln = 300 = 5460 J/s
VA 288 273
= 3 × 8.3 × TA ln 2 = 17.26 TA
www.crackjee.xyz
LAWS OF THERMODYNAMICS 523
Heat needed to melt the ice (ii) In the process B to C: As volume is constant, so
Q = mL = 15 × 333 ×103 J PB PC
Time taken in freezing water =
TB TC
Q 15 333 103
t= = = 914.8 s PC 1/ 3
Q2 5640 TC = TB P = 850 × = 425 K
Ans. B 2/ 3
17. Suppose initially the engine is working between the temperatures The change in internal energy
300 K and 600 K. Thus U = CV T
T 300
=1 2 =1 = 0.50 3R
T1 600 = (TC TB )
2
Now temperature of source is increased by 10 K, 3 8.31
= (425 850) = – 5297.63 J.
300 2
1 =
1 = 0.508.
610 Now by first law of thermodynamics, we have
When temperature of sink is decreased by 10K Q= U W
290 or Q = – 5297.63 + 0
2 = 1- = – 5297.63 J Ans.
600
= 0.517. 1 1
PA PB
(iii) For path A to B, = ......(i)
Clearly 2 > 1. Ans. T T
T2 400 PB PC
18. Efficiency of the engine =1 = 1 = 20% For path B to C, T = T .....(ii)
T1 500 B C

1
W PC PD 1
By the definition = Q For path C to D, = .....(iii)
1 TC TD
W 10000 PA PD
Q1 = = = 5 × 104 J, For path A to D, T = T .....(iv)
0.20 A D

and Q2 = Q1 – W = 4 × 104 J. Ans. From these equations, we have


TA TC = TD T B
19. Q = Q 1 + Q2 + Q3 + Q4
= 5960 – 5585 – 2980 + 3645 TATC
= 1040 J TD =
TB
The total work done W = W1 + W2 + W3 + W4
= 2200 – 825 – 1100 + W4 1000 425
= 275 + W4 = = 500 K. Ans.
850
As the process is cyclic, so 21. In the first case, U =0
U = 0. Q=0+W =W
Thus Q= U + W
V2
or 1040 = 0 + 275 + W4 = nRT ln
W 4 = 765 J V1
and W = 275 + 765 = 1040 = nR × 300 ln 2 .....(i)
work done In the second case, W = 0,
= heat absorbed and U = nCV T

W 3R
1040 = n T
= Q Q = =10.82% 2
1 4 5960 3645
From (i) and (ii),
Ans.
20. For the adiabatic process A to B: 3R
nR 300ln 2 = n T
1 2
PA PB 1
= T = 138.6 K
TA TB The final temperature of the second vessel
1 5/ 3 1 T = Ti T
PB 2
T B = TA = 1000 = 850 K. = 300 + 138.6 = 438.6 K
PA 3 The temperature of the combined gas
R[TA TB ] n1T1 n2T2
Work done WAB = Tf =
1 n1 n2

8.31(1000 850) 1 300 1 438.6


= =1869.83 J Ans. = =369.3 K. Ans.
5/3 1 1 1
www.crackjee.xyz
524 MECHANICS & THERMODYNAMICS
Now using PV = nRT, we get nR
Pf = 2.46 × 105 N/m2 Ans. WCD = (TC TD ) = – 5 R 1027
1 2
1
22. (i) Work done, W= [4 105 8 105 ](0.5 0.2) WDA = 0
2
W = WAB + WBC + WCD + WDA
= 1.8 × 105 J
= 1954 R
2
P(N/m ) work done W
Efficiencey = heat absorbed = Q
5 AB
8 × 10
1954 R
5 = = 61.4% Ans.
4 × 10 3182 R
24. (a) Sketch of the process is given in the answer.
[ PV
1 1 P2V2 ]
(b) Work done: WAB = … (i)
3 ( 1)
0 V(m )
0.2 0.5
We have 1 1 = P2V2
PV
(ii) The change in internal energy,
U = nCV T PV
1 1
P2 = …(ii)
V2
3R From equations (i) and (ii), we have
= n [T2 T1]
2
1
V1
3 1 1 1
PV
= [ nRT2 nRT1 ] V2
2
WAB =
3 ( 1)
= [ P2V2 PV 1 1]
2 5/3 1
= 4.8 × 105 J. Ans. V1
1 1 1
PV
(iii) Amount of heat supplied V2
Q= U W =
(5/ 3 1)
= 4.8 × 105 + 1.8 × 105
= 6.6 × 105 J. 3 V1
2/3

Q Q
= 1 1 1
PV Ans.
2 V2
(iv) C= = = 17 J. Ans.
T P2V2 PV
1 1
Now by first law of thermodynamics
R R
Q= U W
23. For adiabatic process, we have
V –1T = constant, U =Q – W
1 1 2/3
or V A TA = VB TB 3 V1
=Q 1 1 1
PV Ans.
1 2 V2
VA
T B = TA 25. The number of moles of the gas is given by
VB
= 300 × (16)1.4 – 1 = 909 K. PV
n=
RT
VC
At constant pressure, T C = TB = 909 × 2 = 1818 K.
VB PV
1 1
n1 =
RT1
a 1.4 1
VC 1 P2V2
Also T D = TC =1818
VD 8 and n 2 = RT
2
= 791.3 K. If T is the final temperature, then
Heat absorbed Q BC = nCP (TC – TB)
P(V1 V2 )
R n=
=1 (TC TB ) = 3182 R RT
( 1) For isolated system
Heat released Q DA = nCV (TD – TA) n1 + n2 = n
R 2455 R PV
1 1 P2V2 P(V1 V2 )
=1 (TD TA ) = . or RT1 RT2 = .....(i)
( 1) 2 RT
nR Also by conservation of energy for isolated system
Work done: WAB = [TA TB ] = 5 R 609 U1 + U2 = U
1 2
3 3 3
WBC = nR[TC – TB] = 909 R n1 kT1 n2 kT2 = ( n1 n2 ) kT
2 2 2
www.crackjee.xyz
LAWS OF THERMODYNAMICS 525
T mN 2 RT
PV
1 1 P2V2 PV
1 1 P2V2 PN 2 = ,
or RT T1 T2 = M N2 V
1 RT2 RT1 RT2
mO2 RT
PV
1 1 P2V2 and PO2 =
T = T1T2 M O2 V
1 1T2
PV P2V2T1
According to Dalton's law,
PV
1 1 P2V2 P 1 = PH 2
From equation (i), P= V1 V2 . Ans. 1.3 × 109 Pa
26. (a) Given PT = k …(i) P 2 = PH 2 + PO2
Also PV = nRT …(ii) 4.5 × 109 Pa
From equations (i) and (ii), we get And P 3 = PH 2 + PN 2
nR 2 2.0 × 109 Pa Ans.
V= T
k 3
m 2 10
2nRT 29. (i) n = = = 500
dT M 4
dV =
k By the equation, PV = nRT, we have
Work done WAB = PdV PV
T=
nR
T2
k 2nRT
dT (5 104 ) 10
= T k TA =
T1 500 8.31
T2 = 120.33 K
= 2nR T1
dT = 2nR [T –T ]
2 1 Similarly, T B = 240.66 K,
T C = 481.32 K,
= 2 × 2R [300 – 600] = – 1200 R.Ans.
T D = 240.66 K.
(b) Change in internal energy
(ii) No.
U AB = nCV T (iii) The change in internal energy is given by
3R U = nCV T .
= 2 ( 300) = – 900 R
2 Thus [ U ] ABC = nCV T
Now Q = U AB WAB
3
= – 900 R – 1200R = – 2100 R Ans. = 500 × R × [TC – TA]
27. Work done in the complete cycle, 2
W = Area of the trapeziam = 2.25 × 106 J
and [ U ] ADC = 2.25 × 106 J
(V4 V1 ) (V3 V2 )
= ( P2 P1 ) WABC = 10 × (10 × 104) = 106 J
2
And WADC = 5 × 104 (20 – 10)
V2T3 V1T3 = 0.5 × 106 J
Here V3 = = T , Now Q = DU + W
T2 2
Q ABC = 3.25 × 106 J
V1T4 V1T2 and Q ADC = 2.75 × 106 J. Ans.
and V4 = T = T , 30. As the piston does not conduct heat so the temperature of left
1 1
part does not change. If D T is the change in temperature of right
PT
1 2
P2 = T , part, then
1 U = nCV D T
After substituting these values, we get
3R
T2 T1 T2 T3 = n T
2 2
W = PV
1 1
T1 T1 T2 By first law of thermodynamics, we have
(Q – Q') = U W
For n-moles, 1 1 = nRT1
PV
1 2
T T3 Here work done, W= 2 kx
W = nR(T2 T1 ) 2 2 Ans. 2
T1 T2
2
1 k
mRT = 2 k ( / 2) 2 =
28. Pressure is given by P= , 2 4
MV 2
3R k
mH 2 RT Q – Q' = n T ...........(i)
Partial pressures: PH 2 = , 2 4
MH2 V For the equilibrium of the piston

P2A = P1A + 2 k
2
www.crackjee.xyz
526 MECHANICS & THERMODYNAMICS

P2 where M = 2 S mer
P1
W = W1 + W2
5 7 2
= P0 S mer gS
2 4
7.7 J. Ans.
k VA VB
or P 2 = P1 + 33. For the process A to B: T = T
A A B
2nR(T T) 2nRT k
or = + ......(ii) VB
3A A A T B = TA = 300 × 2
VA
From equations (i) and (ii), we get
= 600 K = 327oC
5 2
Q' = Q – 3nRT k Ans. U AB = nCV T
2
31. If T is the increase in temperature of the gas, then increase in 3R
= 2 300 = 900 R
internal energy of the gas, 2
U1 = nCV T nRTA
PA = V
3R A
=1× T
2 Work done, WAB = PA (VB – VA)
3R T nRTA VB
=
2 = V (VB V A ) = nRTA
VA
1
A
The energy stored in the spring in compressing from x1 to x2, = 2 R × 300 [2 – 1] = 600 R
1 2 2 Thus heat absorbed Q = U + W = 600 R + 900 R = 1500 R
U 2 = k ( x2 x1 ) For the process B to C:
2
We know that for pressure P of the gas VC
PA = k x WBC = nRTB ln
VB
PA
x=
k VD
= nRTB ln
P ( Ax) PV RT VB
or x2 = = = = 2R × 600 ln 2 = 831.6 R.
k k k
As the temperature is constant from B to C, so
R R T
U2 = (T2 T1 ) = U BC = 0.
2 2
The total energy spend Heat absorbed in the process
Q = U BC WAB
U = U1 + U 2 = 2R T
C = 2R. Ans. = 0 + 831.6 R = 831.6 R. Ans.
32. The work done by the gas is the sum of work done W1 against the For the process C to D :
force of atmospheric pressure and the work done W2 against the WCD = P (VD – VC) = 0
gravity. Thus total work done, and U CD = nCV T
W = W1 + W2
3R
The mercury-gas interface is shifted upon the complete displace- = 2 ( 300) = – 900 R.
ment of mercury 2
5 Heat absorbed Q = U WCD
s= 2 + /2 = , = – 900 R + 0 = – 900 R. Ans.
2
and hence W 1 = Fs For the process D to A :
5 5 P0 S VA
= P0S × = W DA = nRTD ln V
2 2 D
The work done W2 against the gravity is equal to the change in the 1
potential energy of mercury as a result of its displacement. The = 2 R 300ln = – 831.6 R
4
whole of mercury rises as a result of displacement by l relative to
the horizontal part of the tube. This quantity regarded as the final U DA = 0
height of the centre of gravity of the whole mercury. The initial Heat absorbed Q = U DA WDA
position of centre of gravity of murcury is = /8. = – 831.6 R. Ans.
Thus W 2 = U2 – U1 Total work done W = WAB + WBC + WCD + WDA
7 = 600 R. Ans.
Mg ( – /8) = Mg .
8
www.crackjee.xyz
www.crackjee.xyz
528 MECHANICS, HEAT, THERMODYNAMICS & WAVES
8.1 MODES OF HEAT TRANSFER
There are three modes of heat transfer. These are :
(i) Conduction, (ii) convection and (iii) radiation.
Heat transfer by the process of conduction mainly occurs in solids. Liquid and gases
heated by the process of convection. The heat transfer by radiation requires no
intervening medium. The heat from sun to earth comes by radiation. Radiation is the
fastest mode of heat transfer. Radiation is the universal mode of heat transfer which
always occurs.

8.2 THERMAL CONDUCTION


Conduction is a process in which heat is transmitted from one part of a body to other
part at a lower temperature by molecular collisions, without transfer of the material
medium.

Molecular theory of thermal conduction


When one end of the rod is heated, the molecules at the hot end vibrate with larger
amplitude and hence with greater kinetic energy. As these molecules collide with the
neighbouring molecules, the energy being shared between them. So the kinetic energy
of the neighbouring molecules increases. These molecules transfer the energy to the
next molecules and so on. In this way, heat is transferred from hot to the colder end of the
rod.

8.3 STEADY STATE AND TEMPERATURE GRADIENT


Consider a metal rod, whose sides are covered with an insulating material so that
convection and radiation are prevented. When rod starts heating from its one end, the
temperature of subsequent cross-sections of the rod starts increasing. The rod is said to
be in the variable state of heat conduction. After some time, the temperature of every
cross-section of the rod becomes constant and there is no further absorption of heat in
any part of the rod. This is the steady state of the rod.
At steady state :
(i) The temperature of two different cross-sections of the rod are different, but
Fig. 8.1
temperature of each cross-section remain constant.
(ii) The temperature of the cross-section decreases when we move from hot end.
(iii) The rate of flow of heat through every cross-section remain constant.
If T1 and T2 are the temperatures of two cross-sections separated by distance x,
then temperature gradient is defined as :
T1 T2 T
Temperature gradient = =
x x

Note:
(i) If the temperature T1 and T2 are in kelvin or if they are 1°C and 2°C, then
T1 T2 1 2
=
x x
T dT d
(ii) For small x, temperature gradient = = =
x dx dx

Fig. 8.2
www.crackjee.xyz
HEAT TRANSFER 529
8.4 RATE OF FLOW OF HEAT : HEAT CURRENT
Fig. 8.2 represents a rod of material of cross-sectional area A and length L.
Let the left end of the rod be kept at a temperature T1 and the right end at a lower
temperature T2. The direction of flow of heat current is from left to the right through the
rod.
Experiments shows that, the rate of flow of heat through the rod in the steady state
is proportional to the area of cross-section of rod A, and the temperature difference
(T1 – T2), and inversely proportional to the length L. If Q is the heat flows in time t, then
heat current.

Q (T T ) T
H= = KA 1 2 = KA … (1)
t L L

The SI unit of H is J/s or W. Here K is called coefficient of thermal conductivity. It is a


material property which does not depend on size and shape of the specimen. The SI unit
of K is W/m-K.

Note:
Equation (1) may be used to compute rate of flow of heat through a homogeneous
body of uniform cross-section perpendicular to the direction of flow of heat. In
case of non-uniform cross-section, the temperature does not necessarily change
uniformly along the direction of flow of heat. The equation (1) can be used as :

H = dT
KA
dx
when dT is the temperature difference across the distance dx. The negative sign
indicates that temperature of the rod decreases with x.

8.5 THERMAL RESISTANCE


We know that heat current
(T1 T2 )
H = KA
L
(T1 T2 )
or H = L … (1)
KA
In electricity, electric current is given by
(V1 V2 ) Fig. 8.3
i = … (2)
R
where V1 V2 is the potential difference across the resistor R.

L
If we compare equation (1) with equation (2), we find that is a type of resistance, is
KA
called thermal resistance RH. Fig. 8.4

L
So RH =
KA
Temperature difference is analogous to the potential difference in electricity.

K K
SI unit of RH = = .
J /s W
www.crackjee.xyz
530 MECHANICS, HEAT, THERMODYNAMICS & WAVES
Temperature of a section of rod at a distance x
Consider a rod of length L at steady state. Let T1 is the temperature of left end and T2 is
the temperature of right end of the rod, with T1 > T2. If T is the temperature of the section
under consideration, then heat current through AB will equal to heat current through BC,
so we have
T1 T T T2
KA = KA
x ( L x)
After solving, we get
T1 T2
T = T1 x
Fig. 8.5 L
Above equation represents a straight line between T and x with negative slope, so the
temperature of the rod falls linearly from T1 and T2 in a distance L.
Ex. 1 Two metal cubes A and B of same size are arranged as Total heat entering into the box in 6h
shown in Fig. 8.6 . The extreme ends of the combination are KA(T1 T2 )
maintained at the identical temperatures. The arrangement is Q = .t
L
thermally insulated. The coefficient of thermal conductivity of A
and B are 300 W/m – °C and 200 W/m – ºC respectively. After steady 0.01 0.54 45 6 3600
state is reached, what will be the temperature T of the interface? =
0.05
= 104976 J
If m be the amount of ice melted, then
Q = mL
Q 104976
or m = =
L 335 103
Fig. 8.6
= 0.313 kg
Sol. Mass of the ice left after
In steady state;
6h = 4 – 0.313 kg = 3.687 kg Ans.
Rate of flow of heat through A = Rate of flow of heat through B
100 T (T 0) Ex. 3 A brass boiler has a base area of 0.15 m2 and thickness
or K1 A = K2 A
x x 1.0 cm. It boils water at the rate of 6.0 kg/min, when placed on a gas.
(100 T ) (T 0) Estimate the temperature of the part of the flame in contact with
or 300 A = 200 A the boiler. Thermal conductivity of brass = 109 J/sm-°C and heat of
x x
vapourisation of water = 2256 J/g.
or 300 – 3T = 2T
T = 60°C Ans. Sol.
Let T1 be the temperature of the part of the flame in contact with boiler.
Ex. 2 A ‘thermocole’ cubical icebox of side 30 cm has a thickness The amount of heat flows into water in 1 minute
of 5.0 cm. If 4.0 kg of ice are put in the box, estimate the amount of
ice remaining after 6h. The outside temperature is 45°C and T1 T2
Q = KA t
coefficient of thermal conductivity of thermocole = 0.01 J/ sm - °C. L
Given heat of fusion of water = 335 × 103 J/kg. 109 0.15 (T1 100) 60
= J
Sol. 0.01
Area of six faces of the box Mass of the water boiled in 1 minute = 6 kg = 6000 g
Heat required to boil the water Q = mL = 6000 × 2256 J
= 6 2 = 6 (0.30) 2
= 0.54 m2 109 0.15 (T1 100) 60
= 6000 × 2256
and L = 5.0 cm = 0.05 m 0.01
time, t = 6h = 6 × 3600 s or T1 – 100 = 138
T1 T2 = 45 – 0 = 45°C or T 1 = 238° C Ans.

8.6 DETERMINATION OF THERMAL CONDUCTIVITY


1. Ingen- Hausz experiment
Ingen-Hausz devised an experiment to compare the thermal conductivities of the
metals. If 1, 2 ,..... are the lengths of wax melted on the metal rods, then the ratio
of thermal conductivities is K1 : K 2 : K3 ... = 12 : 22 : 32 : ....
www.crackjee.xyz
HEAT TRANSFER 531
2. Searle’s method
Consider a rod XY whose thermal conductivity is to be determined. The left end X
of the rod is placed in a steam jacket and steam is continuously passed through it.
Heat is transferred along the rod and when steady state is reached, the thermometers
T1, T2, T3 and T4 record constant temperatures. Let in time t, m be the mass of water
collected.

Fig. 8.7

Fig. 8.8
Quantity of heat flows from the section at P and Q in one second,
KA(T1 T2 )
Q = 1 … (1)
L
where L is the distance between sections of the rod at temperatures T1 and T2.
Amount of heat gained by water in one second
= mass of water collected in one second
× specific heat × temperature rise

m
= c (T4 T3 ) … (2)
t
From equations (1) and (2)

(T1 T2 ) mc(T4 T3 )
KA =
L t

mcL(T4 T3 )
or K = At (T1 T2 )
Wiedemann - Franz law
The ratio of the thermal and electrical conductivities is the same for all metals at the same
temperature. Moreover, the ratio is directly proportional to the absolute temperature of
the metal. If K and are the thermal and electrical conductivities of a metal at a temperature
T, then
K
T

K
or = constant
T
At low temperatures the ratio K/ decreases and its value becomes zero at absolute zero.
www.crackjee.xyz
532 MECHANICS, HEAT, THERMODYNAMICS & WAVES
8.7 COMBINATION OF METALLIC RODS
1. Series combination : Suppose n number of rods each of cross-sectional area A and
lengths 1 , 2 ,...... n and conductivities K1, K 2 ,......., K n are placed in series.

Fig. 8.9
(i) Equivalent thermal resistance : If R1 , R2 ,.......... are the thermal resistances
of the rods, then equivalent resistance
RH = R1 R2 ....... Rn
(ii) Heat current : The heat current remains same for all rods.
(T1 T2 )
H = Q = H1 = H2 = ...... =
t RH
(iii) Equivalent thermal conductivity :
We know that RH = R1 R2 ...... Rn

1 2 ... n 1 2 n
or = ....
KA K1 A K2 A Kn A

1 2 ...... n
or K = = /K
1 2 ..... n
K1 K2 Kn

2 K1K 2
(iv) For two rods : K =
K1 K 2
2. Parallel combination : Suppose n number of rods or slabs each of length and
area of cross-sections A1 , A2 ,....., An and thermal conductivities K1, K 2 ,......., K n
are placed in contact.

Fig. 8.10
(i) Equivalent thermal resistance : If R1, R2, ......, Rn are the resistances of the
rods, then equivalent resistance
1 1 1 1
RH
= ......
R1 R2 Rn
(T1 T2 )
(ii) Heat current : H1 = R1
,

(T1 T2 ) (T1 T2 )
H2 = , ........, Hn=
R2 Rn
(T1 T2 )
and H = H1 H 2 ....... H n =
RH
www.crackjee.xyz
HEAT TRANSFER 533
(iii) Equivalent thermal conductivity :
We know that
1 1 1 1
= .........
RH R1 R2 Rn
1 1 1 1
or = .....

K ( A1 A2 ..... An ) K1 A1 K 2 A2 K n An
K1 A1 K2 A2 ...... K n An
or K = A1 A2 ....... An
KA
=
A
K1 A1 K 2 A2
(iv) For two slabs : K =
A1 A2

Ex. 4 An electric heater is used in a room of total wall area


1 2 3
137 m2 to maintain a temperature of +20°C inside it, when the
outside temperature is –10°C. The walls have three different layers K = 1 2 3
K1 K2 K3
materials. The innermost layer of wood of thickness 2.5 cm, the
middle layer is of cement of thickness 1.0 cm and the outermost
0.025 0.01 0.25
layer is of brick of thickness 25.0 cm. Find the power of the electric
= 0.025 0.01 0.25
heater. Assume that there is no heat loss through the floor and the
ceiling. The thermal conductivities of wood, cement and brick are 0.125 1.5 1.0
0.125, 1.5 and 1.0 W/m –°C respectively.
0.285
Sol. = = 0.624 W/m-°C
0.457
Equivalent thermal conductivity of the wall The rate of flow of heat is given by
T1 T2
H = KA
L

[20 ( 10)]
= 0.624 137
0.285

0.624 137 30
= = 9000 W Ans.
0.285

Fig. 8.11
8.8 REDIAL FLOW OF HEAT
Consider two thin spherical shells of radii r1 and r2. A medium of thermal conductivity K
is contained between these shells. A heater is placed at the centre of the shells. Heat is
conducted through the medium radially from inner to the outer shell. Let the temperatures
of the inner and the outer shells be T1 and T2 at steady state.
Choose an element of radial thickness dr at a radial distance r from the centre of shells.
Let dT is the temperature difference across it. The rate of flow of heat through the
element
dT
H = KA
dx
Here A = 4 r 2 are dx = dr, H is constant
dT
H = K (4 r 2 )
dr
dr 4 K Fig. 8.12
or = dT … (1)
Integrating equation (1), we have
r2 H

r2 T2
dr 4 K
dT
r2 = H
r1 T1
www.crackjee.xyz
534 MECHANICS, HEAT, THERMODYNAMICS & WAVES
r
1 2 4 K T2
r r1 = TT
H 1

1 1 4 K
or = (T1 T2 )
r1 r2 H

H (r2 r1 )
or K = … (2)
4 r1r2 (T1 T2 )

8.9 CYLINDRICAL FLOW OF HEAT


Consider a cylindrical tube of length , r1 and r2 are the inner and outer radius of the
tube. The thermal conductivity of material of the tube is K. A heater is placed inside the
tube. After steady state, let temperatures of inner and outer surfaces are T1 and T2
respectively. Heat is conducted radially across the tube.
Choose an elements of radial thickness dr at a distance of r from the axis of the tube. Let
dT is the temperature difference across it. The rate of flow of heat through the element
dT
H = KA
dx
Fig. 8.13 Here A 2 r and dx = dr, H is constant
dT
H = K (2 r )
dr
dr 2 K
or = dT … (1)
r H
Integrating equation (1), we have

r2
dr T2
2 K
r = dT
r1 H
T1

r 2 K T
or ln r r2 = T T2
1 H 1

2 K
or ln r2 – ln r1 = (T1 T2 )
H

r2
H ln
or K = r1 … (2)
2 (T1 T2 )

8.10 FORMATION OF ICE ON PONDS


When temperature of the atmosphere falls below 0°C, the water in pond starts freezing
from the uppermost layer.
Consider a layer of water of thickness y which has already freezed. Let dy thick layer of
water is under process of freezing.
Let the temperature of air over the top face of ice be –T°C and that of water below ice 0°C.
Suppose (dy) thick ice will form in time dt. If A is the area of the pond, then mass of ice
formed, m = (Ady)
Heat lost by water in process of formation of ice
= mL = ( Ady ) L … (1)
Fig. 8.14
www.crackjee.xyz
HEAT TRANSFER 535
where L is the latent heat of fusion of ice. This heat is conducted across y thick layer of
ice into the atmosphere. If K is the thermal conductivity of ice, then heat conducted in
time dt
[0 ( T )]
= KA dt
y
T (dt )
= KA … (2)
y
Equating equations (1) and (2),we have
(dt )
KAT = ( Ady ) L
y

L
or dt = ydy … (3)
KT
If t is the time to form y2 – y1 thick ice, then
t y2
L
dt ydy
= KT
0 y1

L 2 y2
or t = y
2 KT y1

L
t = ( y2 2 y12 )
2 KT
If t1 is the time to form top 1m of ice and t2 is time to form next 1 m of ice, then
L 2 L
t1 = (1 02 ) =
2 KT 2 KT

L 3 L
and t2 = (2 2 12 ) =
2 KT 2 KT
t2 = 3t1

Ex. 5 Fig. 8.15 shows a large tank of water at a constant This heat increases the temperature of the water by d , then
Q = mc (d ) …(ii)
temperature 0 and a small vessel containing a mass m of water at
Equating equations (i) and (ii), we have
an initial temperature 1 ( 0 ) . A metal rod of length L, area of
cross-section A and thermal conductivity K connects the two vessels. KA( 0 )
dt = mcd
Find the time taken for the temperature of the water in the smaller L
vessel to become 2 ( 1 2 0) . Specific heat capacity of water mcL d
or dt = KA ( 0 )
is c and all other heat capacities are negligible.
Sol. If t is the required time, then
Let be the temperature of the water in the smaller vessel at any time t. t 2
In next time interval dt, the amount of heat flow mcL d
dt = KA ( 0 )
( ) 0 1
0
Q = KA dt … (i)
L
2
mcL ln( 0 )
t = KA ( 1)
1

mcL 0 1
t = ln Ans.
KA 0 2
Fig. 8.15
www.crackjee.xyz
536 MECHANICS, HEAT, THERMODYNAMICS & WAVES
Ex. 6 A rod CD of thermal resistance 5.0 K/W is joined at the
middle of an identical rod AB as shown in Fig. 8.16. The ends A, B
and D are maintained at 100°C, 0°C and 25°C respectively. Find
the heat current in CD.
Fig. 8.18
CD
Sol. Given, AC =CB =
2
Let H1 and H2 are the heat in straight and bent parts of the frame, then
H1 6R = H 2 7 R
or 6H1 = 7H 2 … (i)
Given H1 + H1 = 130 … (ii)
Solving equations (i) and (ii), we get
H 1 = 70 J/s
and H 2 = 60 J/s Ans.

Ex. 8 Find the resistance of the rod as shown in the Fig. 8.19.
Fig. 8.16 Thermal conductivity of material of the rod is K.

As resistance of the rod is proportional to the length, so


RCD 5
RAC = RCB = = = 2.5 K/W
2 2
Let H1 is the heat current in CD and H2 in CB, then
T C TD TC 25 Fig. 8.19
H1 = = … (i)
RCD 5 Sol.
TC TB TC 0 Choose an element of thickness dx at a distance of x from the left end of
and H2 = RCB = 2.5 … (ii) the rod. The radius of the element
If H is the heat current through AC, then r2 r1
r x = r1 + y = r1 x
L
TA TC 100 TC
H = The resistance of the element
RAC = 2.5
length dx
We know that H = H1 + H2 dR = KAx = K rx 2
100 TC TC 25 TC The resistance of whole rod
=
2.5 5 2.5 L
dx 1
L
dx
or TC = 45°C R = =
0K rx2 K 2
From (i) 0 r2 r1
r1 x
45 – 25 L
H1 = = 4.0 W Ans.
5 L 2
1 r2 r1
= r1 x dx
Ex. 7 Consider the situation shown in Fig. 8.17. The frame is K
0
L
made of the same material and has a uniform cross-sectional area
L
everywhere. Calculate the amount of heat flowing per second 1
r2 r1
through a cross-section of the bent part if the total heat taken out r1 x
1 L
per second from the end at 100°C is 130 J.
= K r2 r1
( 1)
L
0
L
L 1
Fig. 8.17 K ( r2 r1 ) r2 r1
= r1 x
Sol. L 0
Suppose resistance of 10 cm length of frame is R, then
L 1 1
R AB = REF = 2R, RBE = 6R = K ( r2 r1 ) r2 r1
Resistance of bent part (length = 60 + 5 + 5 = 70 cm) R = 7R
L
The equivalent network of resistors is shown in figure = Ans.
Kr1r2
www.crackjee.xyz
HEAT TRANSFER 537
Ex. 9 Two bodies of masses m1 and m2 with heat capacities Sol.
C1 and C2 are interconnected by a rod of length L, cross-sectional Light piston will maintain the pressure of the gas equal to the atmospheric
area A, thermal conductivity K and negligible heat capacity. The pressure (Pa).
whole system is thermally insulated. At time t = 0, the temperature Let at any instant, the temperature of the gas is . In next small time dt,
of the first body is T1 and the temperature of the second body is its temperature increases by d .
T2 (T 1 > T 2). Find the temperature difference between the two Heat gained by gas = Heat conducted into the cylinder
bodies at time t.

Sol.
Let T1 and T2 be the instantaneous temperatures of the bodies at time t.
If dT1 is the decrease in temperature of first body and dT2 is the increase
in temperature of the second body, then
dT1 T1 T2
m1C1 = KA …(i)
dt L

Fig. 8.20

dT2 T1 T2
and m2C2 = KA … (ii) Fig. 8.21
dt L
Equations (i) and (ii) can be written as
Ts
dT1 KA or nC p ( d ) = KA dt
= (T1 T2 ) …(iii) x
dt m1C1L
For monoatomic gas,
dT2 KA 5R
and = (T1 T2 ) … (iv) Cp =
dt m2C2 L 2
On adding equations (iii) and (iv), we have
5R T
n ( d ) = KA s dt
KA 1 1 2 x
d (T1 T2 )
(T1 T2 ) =
dt L m1C1 m2C 2 d 2 KA
or (dt )
(Ts ) = 5nRx … (i)
d (T1 T2 ) KA 1 1
or = ( dt ) … (v) Let after time t, the temperature of the gas is T, then
T1 T2 L m1C1 m2C2
T t
d 2 KA
Integrating equation (v), = 5nRx ( dt )
T0
(Ts )
(T1 T2 ) final 0
t
d (T1 T2 ) KA 1 1
= dt 2 KA
T1 T2 L m1C1 m2C2 ln(Ts
T
)T t
(T1 T2 )initial 0 or =
0 5nRx
(T1 – T2) denotes temperature difference
Ts T 2 KAt
(T T )final KA 1 1 or ln =
ln 1 2 = t Ts T0 5nRx
(T1 T2 )initial L m1C1 m2C2
2 KAt
KA 1 1 or (Ts T ) = (Ts T0 )e 5 nRx
t
or (T1 T2 )final = L m1C1 m2C2 Ans.
(T1 T2 )initial e
If we write Ts T T , then
Ex. 10 n moles of a monoatomic gas at an initial temperature
2 KAt
T 0 is enclosed in a cylindrical vessel filled with a light piston.
T = (T T )e 5 nRx …(ii)
The surrounding air has a temperature T s ( > T 0 ) and the s 0
atmospheric pressure is P a. Heat may be conducted between the By equation of state, we have
surrounding and the gas through the bottom of the cylinder. The P V = nR T
bottom has a surface area A, thickness x and thermal conductivity
K. Assuming all changes to be slow, find the distance moved by the nR
or V = T … (iii)
piston in time t. P
www.crackjee.xyz
538 MECHANICS, HEAT, THERMODYNAMICS & WAVES
From equations (ii) and (iii), we get
2 KAt
nR 5nRx
(Ts T0 )e
V = P

Let y is the displacement of the piston in time t, then


Ay = V

2 KAt
V nR 5nRx
(T T0 )e
y = = PA s Ans.
A

Ex. 11 A monoatomic ideal gas is contained in a rigid container Ex. 12 A closed cubical box is made of perfectly insulating
of volume V with walls of total inner surface area A, thickness x material and the only way for heat to enter or leave the box is
and thermal conductivity K. The gas is at an initial temperature T0 through two solid cylindrical metal plugs, each of cross-sectional
and pressure P0. Find the pressure of the gas as a function of time area 12 cm2 and length 8 cm fixed in the opposite walls of the box.
if the temperature of the surrounding air is Ts. All temperatures The outer surface of one plug is kept at a temperature of 100°C
are in absolute scale. while the outer surface of the other plug is maintained at a
Sol. temperature of 4°C. The thermal conductivity of the material of
3R the plug is 2.0 W/m-°C. A source of energy generating 13W is
The volume of the gas is constant, so Cv = . If the temperature of the enclosed inside the box. Find the equilibrium temperature of the
2
gas at time t is and next time dt, let temperature of the gas increases by inner surface of the box assuming that it is the same at all points
d then on the inner surface.
Heat gained by gas = Heat conducted into the container
Sol.
KA(Ts ) Let the temperature of the interior of the box is . If H1 and H2 are the
or nCV (d ) = dt
x rate of flow of heats from left and right plugs respectively, then at
equilibrium
3 (T )
or n R ( d ) = KA s dt H1 + P = H 2 … (i)
2 x
Here P is the power of the source inside the box.
d 2 KA
or dt
(Ts ) = 3 xR
Integrating above equation, we have
T t
d 2 KA
dt
T0
(Ts ) = 3 xR 0

T 2 KA
or ln(Ts )T = t Fig. 8.22
0 3 xR
We know that,
T T 2 KA KA( ) KA(
or ln s = t 1 2)
Ts T0 3 xR H1 = and H2 =
x x
2 KA
t Substituting these values in equation (i), we have
or Ts T = (T T )e 3 xR
s 0
( 1 ) KA( 2)
2 KA KA P =
t x x
or T = T (Ts T0 )e 3 xR
s

As the volume remains constant, so Here 1 100 C , 2 4 C and P = 13W,,

P P0 K = 2.0 W/m-C, A = 12 × 10–4 m2 , x = 0.08 m


= T
T 0
(12 10 4 ) (100 – ) 2 (12 10 4 ) ( 4)
2 13
P0 0.08 0.08
or P = T T
0 3 10 2 (100 – ) = 3 10 2 ( 4) 13
2 KA
P0 t
= T Ts (Ts T0 )e 3 xR
Ans. After solving, we get
0 = 268.67°C Ans.
www.crackjee.xyz
HEAT TRANSFER 539
8.11 APPLICATION OF CONDUCTIVITY IN DAILY LIFE
1. A new quilt is warmer than an old quilt : New quilts and bed clothing filled with
cotton become bad conductor of heat due to air traps in pores. This prevent the
conduction of heat from the surroundings and therefore we feel warmer.
2. Ice is packed in saw dust : Saw dust and air trapped inside it becomes poor
conductors of heat. This prevents the conduction of heat from surroundings to the
ice and therefore ice will not melt.
3. In winter, a metallic handle appears colder than the wooden door : In winter, the
temperature of human body remains higher than the surrounding objects. When
we touch the metallic handle, heat flows from our body to the handle and feels cold.
But in case of wooden door, on being bad conductor, heat does not flow, so it does
not feel as cold as the metallic handle.
Fig. 8.23
8.12 CONVECTION
It is the process by which heat flows from the region of higher temperature to the region
of lower temperature by the actual movement of the particles of the medium. There is no
simple equation for convective heat transfer as there is for conduction. Convective heat
transfer depends on many factors, such as the shape and orientation of the surface, the
mechanical and thermal properties of the fluid. For practical calculations, we can define
heat current due to convection as ;
H = hA T
where h is called convection coefficient. A is the surface area and T is the temperature
difference between the surface and the main body of the fluid.
Experiment1 : Take a glass tube, filled with water and put some charcoal powder in the
tube, and starts heating it from one of its sides (see Fig. 8.23). The heated water molecules
rise up (due to low density) and cold water molecules take their places, coming from the
side tube and therefore a convection current is set-up in the whole tube. The direction of
movement of water molecules is indicated by the movement of the charcoal particles.
Experiment 2 : Take a flask containing water and put some crystals of potassium
permagnet. When flask starts heating, coloured streaks of water rise up due to lower Fig. 8.24
density. The denser cold water takes its place by moving downwards. Thus convection
current is setup in the water.
Natural convection : If the material of the medium moves due to difference in density
caused by difference in temperature, the process of heat transfer is called natural or free
convection. Natural convection currents always move upward due to difference in density
and gravity. Different types of winds in the atmosphere are originated due to natural
convection.
Forced convection : If the heated material is forced to move by a machine like a blower or
a pump, the process of heat transfer is called forced convection. Heat convector, hair
drier, air-conditioning are the examples of forced convection.
8.13 PHENOMENON BASED ON CONVECTION
(i) Monsoon :In summer, the surface of the earth of the Indian subcontinent becomes
hotter than the Indian ocean. This sets up convection current with hot air from the
land rising and moving towards the Indian Ocean, while the moisture laden air from
the ocean moves towards the land. When obstructed by mountains, the moist air
rushes upwards to great height and gets cooled. In this process moisture condenses
and causes rains in all over India.
(ii) Trade winds : The surface of the earth gets heated more at the equator than at the
poles. Warm air at the equator moves up and cold air from the poles moves towards
the equator. In the northern hemisphere, it is coming from the north and due to the
rotation of the earth from west to east, the wind appears to come from north-east.
In the southern hemisphere, the wind appears to be from south-west. These winds
are called trade winds because they were used by traders for sailing their vessels in
ancient days.
www.crackjee.xyz
540 MECHANICS, HEAT, THERMODYNAMICS & WAVES
(iii) Land and sea breezes : Sun shines almost uniformly on the land mass near coastal
regions, giving equal amount of heat energy. However the temperature of land
rises more rapidly as compared to sea, because specific heat capacity of land is
much smaller than that of water. Thus the air above the land becomes hot and light
and hence rises up. This results decrease in pressure over land mass. So the colder
air starts blowing from sea towards land and thereby setting up sea breeze.

Fig. 8.25
During night the land as well as sea water radiate out heat energy. However the
temperature of land decreases more paridly as compared to sea water due to higher
specific heat of water. Thus at night the temperature of sea water becomes more than
land. The air above sea water become warm and light and rises up. The cold air from land
takes its place. This set-up land breeze.

Fig. 8.26
www.crackjee.xyz
HEAT TRANSFER 541
8.14 RADIATION
Radiation is the process by which heat is transmitted from one place to another without
heating and transferring the intervening medium.
Properties of thermal radiation
(i) These are electromagnetic waves having wavelength range from 1µm to 100µm.
There are also called infrared waves.
(ii) Thermal radiations travel in straight line with the speed of light.
(iii) They obey the laws of reflection and refraction like light does.
(iv) They show the phenomenon of interference, diffraction and polarisation.

Note: Word ‘radiation’ uses for process and energy both.

Reflectance, absorptance and transmittance


When thermal radiations falls on a body, they are partly reflected, absorbed and rest get
transmitted. Let Q amount of thermal energy is incident on a body. Suppose the part R is
reflected, A is absorbed and T is transmitted, then
R+A+T = Q … (1)
Dividing both sides of equation (1), by Q we have
R A T
= 1 … (2)
Q Q Q
R
where = r, is called reflectance
Q
A
Q
= a, is called absorptance and

T
= t, is called transmittance.
Q
Thus equation (2) takes the form r + a + t = 1
For any specific wavelength , we can write,
r a t = 1.
Special cases :
(i) If a body does not transmit the radiations, t = 0, then r + a = 1.
It shows that if r is more, a is less and vice versa. That is good reflectors will be bad
absorbers and viceversa.
(ii) If a body neither reflects nor transmits any radiation, r = 0 and t = 0, then a = 1, such
a body is called a black body.
8.15 SPECTRAL ABSORPTIVE POWER
The absorptive power of any body for a given wavelength is defined as the ratio of
amount of heat energy absorbed by certain surface area of the body in a given time to the
total heat energy incident on that area and in same time within a unit wavelength range
around the wavelength . It can be denoted by a .
If dQ is the quantity of heat radiations incident on the surface in one second and Q1 is
the quantity of heat absorbed by the surface in a wavelength range to + d , then

Q1
a = or Q1 = a (dQ )
(dQ)
8.16 SPECTRAL EMISSIVE POWER
The emissive power of a body at a given temperature and for a given wavelength is
defined as the amount of radiant energy emitted by unit surface area of the body per unit
time within a unit wavelength range around the wavelength . If e is the emissive power
of the body, then the radiant energy emitted by it in one second = e (d ) .
The SI unit of emissive power is W/m2-Å.
www.crackjee.xyz
542 MECHANICS, HEAT, THERMODYNAMICS & WAVES
8.17 EMISSIVITY
It is defined as the ratio of the heat energy radiated per unit surface area per second by
the given body to the amount of the heat energy radiated per unit area per second by a
black body of the same temperature. If e and E are the emissive powers of any body and
black body respectively, then emissivity
e
=
E
It is a dimensionless quantity. Its value ranges from 0 to 1. For a black body, it is 1.

8.18 BLACK BODY


A perfectly black body is one which absorbs all the heat radiation incident on it. When
such a body is placed inside an isothermal enclosure, it will emit all the radiations of the
enclosure after it is in equilibrium with the enclosure.
Fery’s black body
It is not possible to construct a perfectly black body, but a body showing close
approximation to a perfectly black body can be constructed. Fery constructed such a
black body. It has a hollow copper sphere and coated with lamp black on its inner
surface. A fine hole is made and a pointed projection is made just in front of the hole.
When the radiations enter the hole, they suffer multiple reflections and are completely
absorbed. This body behaves as a black body.
When this body is heated, the heat radiations come out of the hole. It should be
remembered that only the hole, not the walls of the body, acts as the black body emitter.

8.19 KIRCHHOFF’S LAW


Fig. 8.27
It states that at any temperature, the ratio of the emissive power to the absorptive power
of any body is a constant, and equal to the emissive power of the perfectly black body
at the same temperature.
Consider any body (not black body) which is suspended in a hollow enclosure maintained
at a constant temperature.
The amount of energy absorbed per unit area per unit time = a (dQ )
The amount of energy emitted per unit area per second = e (d )
As the body is in thermal equilibrium, so e (d ) = a (dQ ) … (1)
Suppose a black body at the same temperature is suspended in the enclosure. For this
body, a 1 and e E , so equation (1) becomes
E (d ) = (dQ) … (2)
Dividing equation (1) by (2), we get
e
E
= a … (3)

e
or a
= E (constant)

The above equation can be stated as ; good emitter is a good absorber.


e
As = ,
E
from equation (3), we have
a =
www.crackjee.xyz
HEAT TRANSFER 543
Thermos flask
A thermos flask is constructed by a double walled glass bottle. The space in between the
two walls is evacuated and sealed. By doing this, heat can not go out by conduction and
convection. The inner surface of the outer wall and outer surface of inner wall are highly
polished, to prevent heat loss by radiation. When a hot liquid is kept in the bottle, it
remains hot for a long time. Similarly ice kept inside the flask will not melt for a long time.
8.20 PREVOST THEORY OF HEAT EXCHANGE (1792)
According to this theory, all bodies radiate thermal radiation at all temperatures. Besides,
body also absorbs radiations from its surroundings. This is known as theory of heat
exchange. According to this theory:
(i) All bodies at temperature above 0 K emit radiation to the surroundings and gain Fig. 8.28
from the surroundings at all the time.
(ii) The amount of heat radiated per second depends on the nature of the surface, its
area and its temperature, and it does affect by the presence of surrounding bodies.
(iii) The rise or fall in temperature of a body is the net result of exchange of heat
radiations between body and the surroundings.

8.21 STEFAN -BOLTZMANN LAW


According to this law, the rate of emission of heat energy by unit area of a perfectly black
body is proportional to the fourth power of its absolute temperature of its surface. Thus

E T4
or E = T 4 W/m 2
Here is a universal constant called Stefan’s constant. Its values is 5.67 × 108 W/m2-K4.
If H is the rate of energy radiated by a blackbody of surface area A, then
H = EA = AT 4
If T0 is the temperature of the surrounding and is the emissivity of the body, then the
net rate of loss of energy per unit area will be ;
Fig. 8.29
E
net
= (T 4 T0 4 ) W/m 2

8.22 NEWTON’S LAW OF COOLING


It states that the rate of cooling of the body is directly proportional to the temperature
difference between body and the surrounding, provided it to be small.
If T and T0 are the temperatures of the body and the surrounding respectively and A is
the surface area of the body, then by Stefan-Boltzmann law, the rate of loss of heat due
to radiation
H1 = A(T 4 T0 4 )
If T is the temperature difference between body and surroundings, then we can write

T = T T0 or T = T0 T

H1 = A (T0 T )4 T04

4
4 T
= A T0 1 T04
T0
4
T T
If T is small, then 1 1 4
T0 T0
www.crackjee.xyz
544 MECHANICS, HEAT, THERMODYNAMICS & WAVES
Thus, we have
4 T
H1 = A T04 1 T04
T0

= A T04 4T03 T T04

or H1 = 4 AT03 ( T )

or H1 = k1 A(T T0 ) … (1)

where k1 is a constant. The body may loose heat due to convection also. The rate of loss
of heat by convection can be written as
H2 = k2A(T – T0) … (2)
The net rate of loss of heat energy

H = H1 + H2 = (k1 k 2 )A(T T0 ) … (3)

If c is the specific heat capacity of the body and m its mass, then

dT
H = mc
dt

dT H (k1 k2 ) A(T T0 )
or = = … (4)
dt mc mc

dT
or = k (T T0 ) … (5)
dt
The constant k depends on the surface involved and the surrounding conditions.

Note:
1. From equation (3), it is clear that the rate of loss of heat H A . Thus if two
bodies of equal surface areas, one solid and other hollow are kept at same
temperature difference, then their ratio of loss of heat will be equal.
2. For spherical body, A 4 r2 . H r2
dT A
3. From equation (4), (T T0 ) . Thus two bodies of same material and
dt mc
equal surface areas, one solid other hollow (ms mH ) , then rate of cooling of
solid body will be less.

A 4 r2 1
4. For spherical body, = 4 3 .
mc r
r c
3
Rate of cooling of body is inversely proportional to its radius, provided other
things remain constant.
5. If temperature of the body falls from T1 and T2 in time t, then we can write
T1 T2
T= ,
2
T1 T2 T1 T2
and = k T0
t 2
www.crackjee.xyz
HEAT TRANSFER 545

dT
6. We have, = k (T T0 )
dt
Tf t
dT
or = kdt
T T0
Ti 0

T
ln(T T0 ) T f = –kt
i

T f – T0
or ln = –kt
Ti – T0

or (T f T0 ) = (Ti T0 )e kt

or = T0 kt
Tf (Ti T0 )e Fig. 8.30

Ex. 13 A spherical body with radius 12 cm radiates 450 W power In first case ; T1 = 60°C, T2 = 40°C, T0 = 10°C and t = 7 minute
at 500 K. If the radius were halved and the temperature doubled, 60 40
60 40
what would be the power radiated? = k 10
7 2
Sol. 1
By Stefan’s law, power radiated or k =
14
E = AT 4 = (4 r 2 )T 4 In second case; T1 = 40°C and T2 = ?, T = 7 minute
When radius is halved and temperature is doubled, power radiates 40 T2 1 40 T2
= 10
2 7 14 2
r 4
E = E' = 4 (2T ) = 4E
2 T2
or 80 2T2 = 20 10
= 4 × 450 = 1800 W 2
or T 2 = 28°C Ans.
Ex. 14 A thin brass rectangular sheet of sides 15.0 and 12.0 cm
is heated in a furnace to 600°C, and taken out. How much electric Ex. 16 A hot body placed in air is cooled down according to
power is needed to maintain the sheet at this temperature, given Newton’s law of cooling, the rate of decrease of temperature being
that its emissivity is 0.250 ? Neglect heat loss due to convection k times the temperature difference from the surrounding. Starting
(Stefan - Boltzmann constant, = 5.67 ×10–8 W/m 2 - k 4 ). from t = 0, find the time in which the body will lose half the
maximum heat it can lose.
Sol.
Area of the both sides of the plate Sol.
We have,
A = 2 (15.0) (12.0) 10 4 m 2 dT
= k (T T0 )
= 3.60 × 10–2 m2 dt
The energy radiated by the plate where T 0 is the temperature of the surrounding. If T 1 is the initial
temperature and T is the temperature at any time t, then
= AT 4
T t
= 0.250 × 5.67 × 10–8 × 3.60 × 10–2 × dT
= k dt
(600 + 273)4 T1
(T T0 )
0
–12 4
= 5.10 × 10 × 873 = 296.4 W Ans.
T
Ex. 15 A body cools in 7 minute from 60°C to 40°C. What will or ln(T T0 ) T = –kt
1
be its temperature after the next 7 minute? The temperature of the
surroundings is 10°C. Assume that Newton’s law of cooling holds T T0
good throughout the process. or ln = –kt
T1 T0
Sol. kt
or T = T0 (T1 T0 )e … (i)
Newton’s law of cooling can be written as :
The body continues to lose heat till its temperature becomes equal to
T1 T2 T1 T2
= k T0 that of the surrounding. The loss of heat
t 2
Q = mc(T1 T0 )
www.crackjee.xyz
546 MECHANICS, HEAT, THERMODYNAMICS & WAVES
If the body loss half of the maximum lose that it can, then decrease in (1.0 – 0)
temperature or M 10 (0.1) = 0.50 0.50 3
2 10
T1 T0 or M = 12.5 kg Ans.
Q
= mc
2 2
Ex. 18 A copper sphere is suspended in an evacuated chamber
If body loses this heat in time t, then its temperature at time t will be maintained at 300 K. The sphere is maintained at a constant
T1 T0 T1 T0 temperature of 500 K by heating it electrically. A total of 210 W of
T1 = electric power is needed to do it. When the surface of the copper
2 2
sphere is completely blackened, 700 W is needed to maintain the
Putting these values in equation (i), we have same temperature of the sphere. Calculate the emissivity of copper.
T1 T0 kt ' Sol.
= T0 (T1 T0 )e
2 Let is the emissivity of the sphere then by Stefan’s law
T1 T0
or = (T1 T0 )e kt '
2
1
or kt ' =
e 2
ln 2
or t' = Ans.
k
Ex. 17 Fig. 8.31 shows water in a container having 2.0 mm Fig. 8.32
thick walls made of a material of thermal conductivity 0.50 W/m-
°C. The container is kept in a melting ice bath at 0°C. The total E = A(T 4 T0 4 )
surface area in contact with water is 0.05 m2. A wheel is clamped
inside the water and is coupled to a block of mass M as shown in the or 210 =A(5004 3004 ) … (i)
figure. As the block goes down, the wheel rotates. It is found that When sphere is blackened, it behaves like a perfectly blackbody, so we
after some time a steady state is reached in which the block goes have
down with a constant speed of 10 cm/s and the temperature of the 700 = A(500 4 3004 ) … (ii)
water remains constant at 1.0°C. Find the mass M of the block.
Assume that the heat flows out of the water only through the walls Dividing equation (i) and (ii), we get
in contact. Take g = 10 m/s2. = 0.3 Ans.
Ex. 19 A solid copper sphere (density and specific heat c) of
radius r at an initial temperature 200 K is suspended inside a
chamber whose walls are almost at 0 K. Calculate the time required
for the temperature of sphere to drop to 100 K.
Sol.
dT
Let T is the temperature at any time and is the rate of fall of
dt
temperature. Then the rate of loss of heat
dT
= mc … (i)
dt
Fig. 8.31
The rate of heat lost by sphere due to radiation only
Sol.
At steady state, Rate of doing work by gravity on block E = AT 4 … (ii)
M = Rate of energy produced Equating equations (i) and (ii), we have
d d 1 dT
or Mgh = Q Mv 2 mc = A(T 4 04 ) = AT 4
dt dt 2 dt

dh dQ M dv dT A
or Mg = 2v … (i) or 4 = dt
dt dt 2 dt T mc
Integrating both sides of above equation
dh dv
As block goes down with constant velocity, so = v and =0 100 t
dt dt A
T 4dT = dt
dQ 200
mc 0
Mgv = …(ii)
dt 100
3
T A
dQ T = t
Here = KA 3 mc
dt L 200
100
T mc 1
Mgv = KA t =
L 3 A T3 200
www.crackjee.xyz
HEAT TRANSFER 547

4 3 (T1 T2 )
r c or KA = A (T2 4 Ts 4 ) … (i)
3 1 1 L
= Given T2 = Ts + T
2
3 4 r 1003 2003 4
4 T
T2 4 = (Ts T )4 = Ts 1 Ts
r c 1
= 1
9 10 6 8 As T Ts , so by Binomial theorem, we get
4 T
7 r c 6 T2 4 = Ts 1 4 T
= 10 s Ans.
72 s

or T2 4 Ts 4 = 4Ts 3 T
Ex. 20 One end of a rod of length L and cross-sectional area A
Also T 2 = Ts + T
is kept in a furnace of temperature T1. The other end of the rod is Substituting these values in equation (i), we have
kept at a temperature T2. The thermal conductivity of the material [T1 (Ts T )]
KA
L
= A (4Ts3 T )
of the rod is K and emissivity is . It is given that T 2 = Ts T,
where T Ts , T s being the temperative of surrounding. If (T1 Ts ) K
or K T= 4 Ts 3 T
T (T1 Ts ), find the proportionality constant. Consider that L L
heat is lost only by radiation at the end where the temperature of (T1 Ts ) K
the rod is T2. or K = 4 Ts 3 T
L L
K (T1 Ts )
or T = … (ii)
(4 LTs 3 K)
It is given that T = C (T1 Ts )
Fig. 8.33 Constant of proportionality
Sol. At steady state rate of heat gained by left end of the rod K
= Rate of heat radiates by right end of the rod C = Ans.
4 LTs 3 K

8.23 WIEN’S DISPLACEMENT LAW


The intensity of energy radiated by a black body is not uniformly distributed over all the
wavelengthsbut it ismaximum for a particular wavelength m. The value of m decreases
with the increase of temperature.
According to Wien’s law the product of the wavelength corresponding to maximum
intensity and absolute temperature is a constant i.e.,

m T = b (constant)
where b is Wien’s constant. Its value is 2.9 × 10 –3 m-K.
8.24 SOLAR CONSTANT
It is the amount of radiant energy that a unit area of a perfectly black body placed at a Fig. 8.34
mean distance of the earth from the sun would receive per second in the absence of the
atmosphere with its surface held perpendicular to the sun rays.
Temperature of the sun
Let Rs be the radius and T be the temperature of sun, then the solar energy radiated per
second E = 2 4
AT 4 = 4 Rs T
If r is the mean distance of the earth from the sun then surface area over which solar
energy will spread = 4 r2
Let S is the solar constant, then by the definition
E 4 Rs 2 T 4
S = =
4 r2 4 r2
2
Rs
or S = T4
r
1/ 4
r2S
or T =
Rs 2
By this formula the surface temperature of sun is found to be 5742 K. Fig. 8.35
www.crackjee.xyz
548 MECHANICS, HEAT, THERMODYNAMICS & WAVES
Ex. 21 The spectral energy distribution of the sun has a Ex. 23 Two bodies A and B have thermal emissivity of 0.01 and
maximum at 4753 Å. If the temperature of the sun is 6050 K, what 0.81, respectively. The outer surface areas of the two bodies are the
is the temperature of a star for which this maximum is at 9506 Å? same. The two bodies emit total radiant power at the same rate.
The wavelength B corresponding to maximum spectral radiancy
Sol. Given m = 4753 Å, T = 6050 K in the radiation from B is shifted from the wavelength
corresponding to maximum spectral radiancy in the radiation from
'
m = 9506 Å
A by 1.00 µm. The temperature of A is 5802 K. Find temperature of
B and wavelength corresponding to maximum spectral radiancy.
If T is the temperature of star, then Sol.
= ' If TA and TB are the temperatures of the bodies A and B respectively, then
mT mT '
A TA4 = B TB 4
mT 4753 6050
or T = ' = = 3025 K Ans. 1/ 4 1/ 4
m 9506 A 0.01
TB = TA = 5802
B 0.81
Ex. 22 An indirectly heated filament is radiating maximum
= 1934 K
energy of wavelength 2.16 × 10–5 cm. Find the net amount of heat
By Wien’s displacement law, we have
energy lost per second per unit area, the temperature of
ATA = BTB
surrounding air is 13°C. Given b = 0.288 cm - K, =
5 or 5802 = 1934
5.77 10 erg/s - cm2 - K 4 . A B

Sol. or =
B
… (i)
A 3
If T is the surface temperature of the filament, then
It is also given
mT = b = 10 –6 … (ii)
B A
b 0.288 Solving equations (i) and (ii) , we get
or T = =
= 13333.3 K
2.16 10 5
m B = 1.5 10 6 m = 1.5 m Ans.
The temperature of surrounding air T0 = 13 + 273 = 286K
The net amount of energy radiated per unit area per second
Note:
E = (T 4 T04 ) Solar spectrum : When light from the sun is seen through a
spectrometer, there observed several dark lines over continuous
5
= 5.77 10 (13333.3)4 (286)4 spectrum. These dark lines are called Franhoffer lines. By comparing
the wavelengths of these dark lines with those emitted by elements on
= 1.824 1012 erg/s-cm2d Ans. the earth, we have identified various elements like H, He, Na, N2 etc. in
the atmosphere of the sun.

Review of formulae & Important Points


1. Rate of heat flow in conduction 6. Formation of ice on pond
(T1 T2 ) T L
H = KA KA ty22 y12=
L L 2 KT
L 7. Radiation is the universal and fastest mode of heat transfer.
2. Thermal resistance RH = e
KA
3. Equivalent thermal conductivity : When two rods of thermal 8. Kirchoff's law : a = constant
conductivities K 1 and K 2 are placed in
9. Stefan's-Boltzmann law : Net loss of heat
2K1K 2
(i) Series : K = K1 K 2 Enet = (T 4 T04 ) W/m2
10. Newtons's law of cooling
K1 A1 K 2 A2 dT
(ii) Parallel : K = = k (T T0 )
A1 A2 dt
here (T – T 0) is small.
H (r2 r1)
4. In radial flow of heat K = 11. Wien's displacement law
4 r1r2 (T1 T2 ) = constant
mT

r2 2
Rs
H n
r1 12. Solar constant, S = T4
5. Cylindrical flow of heat K = r
2 (T2 T1 )
www.crackjee.xyz
HEAT TRANSFER 549

MCQ Type 1 Exercise 8.1


Level -1
Only one option correct (b) High specific heat and high conductivity
1. The temperature of the two outer surfaces of a composite slab (c) Low specific heat and low conductivity
consisting of two materials having coefficient of th ermal (d) Low specific heat and high conductivity
conductivities K and 2K, thickness x and 4x respectively are T 2 6. A slab consists of two parallel layers of copper and brass of the
and T1 (T2 > T1). The rate of heat transfer through the slab, in a same thickness and having thermal conductivities in the ratio 1 : 4.
If the free face of brass is at 100°C and that of copper at 0°C, the
A(T2 – T1 ) K temperature of interface is
steady state is f , the value of f is
x (a) 80°C (b) 20°C
(c) 60°C (d) 40°C
7. Two metal cubes A and B of same size are arranged as shown in the
T2 K 2K T1 figure. The extreme ends of the combination are maintained at the
indicated temperatures. The arrangement is thermally insulated.
The coefficients of thermal conductivity of A and B are 300 W/
x 4x m°C and 200 W/m°C, respectively. After steady state is reached,
the temperature of the interface will be
1
(a) 1 (b) (a) 45°C
2
(b) 90°C
2 1
(c) (d) (c) 30°C
3 3
2. A black body at a high temperature T K radiates energy at the rate (d) 60°C
T 8. There are two identical vessels filled with equal amounts of ice.
of E W/m 2. When the temperature falls to K , the radiated The vessels are of different metals. If the ice melts in the two
2
2
energy in W/m will be vessels in 20 and 35 minutes respectively, the ratio of the
coefficients of thermal conductivity of the two metals is
E E
(a) (b) (a) 4 : 7 (b) 7 : 4
4 2 (c) 16 : 49 (d) 49 : 16
E 9. In which of the following process, convection does not take place
(c) 2 E (d)
16 primarily?
3. The following solid objects made of the same material are maintained (a) Sea and land breeze
at a temperature of 300 K in an environment whose temperature is (b) Boiling of water
400 K : a cube of edge r, a sphere of radius, r and (c) Warming of glass of bulb due to filament
a hemisphere of radius r. The object in which the heat exchange is (d) Heating air around a finance
the greatest ?
10. There is a rough black spot on a polished metallic plate. It is
(a) Cube (b) Sphere heated upto 1400 K approximately and then at once taken in a
(c) Hemisphere (d) Equal in all dark room. Which of the following statements is true?
4. Two identical rods of metal are welded end to end as shown in (a) In comparison with the plate, the spot will shine more
figure (i), 20 calories of heat flows through it in 4 minutes. If the (b) In comparison with the plate, the spot will appear more
rods are welded as shown in figure (ii), the same amount of heat black
will flow through the rods in (c) The spot and the plate will be equally bright
(d) The plate and the black spot can not be seen in the dark
room
11. An ideal black body at room temperature is thrown into a furnace.
It is observed that
(a) 1 minute (b) 2 minute (a) Initially it is the darkest body and at later times the brightest
(c) 4 minute (d) 16 minute (b) It is the darkest body at all times
5. For cooking the food, which of the following type of utensil is (c) It cannot be distinguished at all times
most suitable? (d) Initially it is the darkest body and at later times it cannot be
(a) High specific heat and low conductivity distinguished

Answer Key 1 (d) 3 (b) 5 (d) 7 (d) 9 (c) 11 (a)


Sol. from page 561 2 (d) 4 (a) 6 (a) 8 (b) 10 (a)
www.crackjee.xyz
550 MECHANICS, HEAT, THERMODYNAMICS & WAVES
12. Colour of shining bright star is an indication of its (a) K1 = K2 and K2 = K3 C
(a) Distance from the earth (b) Size K1 K2
(c) Temperature (d) Mass (b) K1K4 = K2K3
13. A metal ball of surface area 200 cm2 and temperature 527°C is (c) K1K2 = K3K4 A K5 B
surrounded by a vessel at 27°C. If the emissivity of the metal is
K1 K2
0.4, then the rate of loss of heat from the ball is (d) K3 K4
K4 K3
( = 5.67 × 10–8 J/m2 – s – K4)
D
(a) 108 joule approx. (b) 168 joule approx.
(c) 182 joule approx. (d) 192 joule approx. 20. Radius of a conductor increases uniformly from left end to right
end as shown in fig.
14. Assuming the sun to be a spherical body of radius R at a
temperature of T K, evaluate the total radiant power, incident on
Earth, at a distance r from the Sun
T1 T2
(a) r02 R 2 T 4 / r 2 (b) r02 R2 T 4 / 4 r 2
x
(c) R2 T 4 / r2 (d) 4 r02 R 2 T 4 / r 2
Material of the conductor is isotropic and its curved surface is
where r0 is the radius of the earth and is stefan’s constant.
thermally insulated from surrounding. Its ends are maintained at
15. Suppose the sun expands so that its radius becomes 100 times its temperatures T1 and T2 (T1 > T2) : If, in steady state, heat flow rate
present radius and its surface temperature becomes half of its
is equal to H, then which of the following graphs is correct
present value. The total energy emitted by it then will increase by
a factor of
H H
(a) 10 4 (b) 625
(c) 256 (d) 16
16. Two rods (one semi-circular and other straight) of same material (a) (b)
and of same cross-sectional area are joined as shown in the figure.
The points A and B are maintained at different temperature. The O x O x
ratio of the heat transferred through a cross-section of a semi-
circular rod to the heat transferred through a cross section of the
H H
straight rod in a given time is
(a) 2: i circular ro
Sem d (c) (d)
(b) 1:2
O x O x
(c) :2
21. A long metallic bar is carrying heat from one of its ends to the other
(d) 3:2 end under steady-state. The variation of temperature along the
A Straight rod B
length x of the bar from its hot end is best described by which of
17. The only possibility of heat flow in a thermos flask is through its
the following figures?
cork which is 75 cm2 in area and 5 cm thick. Its thermal conductivity
is 0.0075 cal/cmsec°C. The outside temperature is 40°C and latent
heat of ice is 80 cal g–1. Time taken by 500 g of ice at 0°C in the
flask to melt into water at 0°C is (a) (b)
(a) 2.47 hr x x
(b) 4.27 hr
(c) 7.42 hr
(d) 4.72 hr (c) (d)
18. One end of a copper rod of length 1.0 m and area of cross-section x x
10–3 m2 is immersed in boiling water and the other end in ice. If
the coefficient of thermal conductivity of copper is 92 cal/m–s–°C 22. Three metal rods made of copper, aluminium and brass, each 1 m
and the latent heat of ice is 8 × 104 cal/kg, then the amount of ice long and 4 cm in diameter, are placed end to end with aluminium
which will melt in one minute is between the other two. The free ends of copper and brass are
(a) 9.2 × 10–3 kg (b) 8 × 10–3 kg maintained at 100°C and 0°C respectively. Assume that thermal
(c) 6.9 × 10 kg–3 (d) 5.4 × 10–3 kg conductivity of copper is twice that of aluminium and four times
19. Five rods of same dimensions are arranged as shown in the figure. that of brass. The equilibrium temperature of copper-aluminium
They have thermal conductivities K1, K2, K3, K4 and K5. When and aluminium-brass junction are
points A and B are maintained at different temperatures, no heat (a) 68° C and 75° C (b) 75° C and 68° C
flows through the central rod if (c) 57° C and 86° C (d) 86° C and 57° C

Answer Key 12 (c) 14 (d) 16 (a) 18 (c) 20 (b) 22 (d)


Sol. from page 561 13 (c) 15 (b) 17 (a) 19 (b) 21 (b)
www.crackjee.xyz
HEAT TRANSFER 551
23. A 100 W lamp is immersed in an insulated container holding 500 g 24. A boiler is made of a copper plate 2.4 mm thick with an inside
of alcohol at 20°C. The time it takes to warm the alcohol to 50°C coating of a 0.2 mm thick layer of tin. The surface area exposed to
is (specific heat of alcohol 0.572 cal/g°C) gases at 700°C is 400 cm2. The maximum amount of steam that
(a) 150 s (b) 360 s could be generated per hour at atmospheric pressure is (Kcu = 0.9
(c) 100 s (d) 200 s and Ktin = 0.15 cal/s-cm-°C and Lsteam = 540 cal/g)
(a) 500 kg (b) 1000 kg
(c) 4000 kg (d) 5000 kg

Answer Key
Sol. from page 561 23 (b) 24 (c)

Level -2
Only one option correct
c 1
1. A wall has two layers A and B, each made of a different material . (a) (b)
r3 r3 c
Both the layers have the same thickness. The thermal conductivity
of the material of A is twice that of B. Under thermal equilibrium, 1
the temperature difference across the wall is 36°C. The temperature (c) 3r3 c (d)
r c
difference across the layer A is
(a) 6°C (b) 12°C 5. A black body is at a temperature of 2880 K. The energy of
(c) 18°C (d) 24°C radiation emitted by this body with wavelengths between 499 nm
and 500 nm is U1, between 999 nm and 1000 nm is U 2 and
2. A cylinder of radius R made of a material of thermal conductivity
between 1499 nm and 1500 nm is U 3. The Wien constant
K1 is surrounded by a cylindrical shell of inner radius R and outer
b = 2.88 × 106 nm K. Then
radius 2R made of material of thermal conductivity K2. The two
(a) U1 = 0 (b) U3 = 0
ends of the combined systems are maintained at two different
(c) U1 > U2 (d) U2 > U1
temperatures. There is no loss of heat across the cylindrical surface
6. A spherical black body with a radius of 12 cm radiates 450 W
and the system is in steady state. The effective thermal
power at 500 K. If the radius were halved and the temperature
conductivity of the system is
doubled, the power radiated in watt would be
K1 3 K 2 (a) 225 (b) 450
(a) K1 + K2 (b) (c) 900 (d) 1800
4
7. Two metallic spheres S1 and S2 are made of the same material and
K1K 2 3K1 K2 have got identical surface finish. The mass of S1 is thrice that of
(c) (d)
K1 K 2 4 S2. Both the spheres are heated to the same high temperature and
placed in the same room having lower temperature, but are
3. The graph shown in figure represents the variation of temperature thermally insulated from each other. The ratio of initial rate of
(T) of two bodies x and y having same surface area with time (t) cooling of S1 to that of S2 is
due to emission of radiation. Find the correct relation between the
emissive power and absorptive power of two bodies 1 1
(a) (b)
3 3
T
1/3
3 1
y (c) (d)
1 3
8. Three rods of identical cross–sectional area and made from the
x same metal form the sides of an isosceles triangle ABC, right
t angled at B. The points A and B are maintained at temperatures T

(a) ex > ey, ax < ay (b) ex < ey, ax > ay and 2 T respectively, in the steady state, the temperature of
(c) ex > ey and ax > ay (d) ex < ey and ax < ay points C is Tc. Assuming that only heat conduction takes place
4. A sphere of density , specific heat capacity c and radius r is hung Tc / T is
by a thermally insulating thread in an enclosure which is kept at a 1 3
lower temperature than the sphere. The temperature of the sphere (a) (b)
2 2 –1 2 1
starts to drop at a rate which depends upon the temperature
difference between the sphere and the enclosure and the nature of 1 1
the surface of sphere and is proportional to (c) (d)
3( 2 – 1) 2 1

Answer Key 1 (b) 3 (c) 5 (d) 7 (d)


Sol. from page 562 2 (b) 4 (d) 6 (d) 8 (b)
www.crackjee.xyz
552 MECHANICS, HEAT, THERMODYNAMICS & WAVES
9. Two identical conducting rods are first connected independently 16. An insulated tub is divided into two sections by a watertight
to two vessels containing metal at 100°C and other containing ice partition of dimension 20cm × 40cm made of copper 3 mm
at 0°C. In the second case the rods are joined end to end and thick.On one side is water which is boiling and on the other side is
connected to the same vessel. Let q1 and q 2 g/s be the rate of a mixture of 2 kg of crushed ice being constantly stirred in 5 kg of
q1 water. All the ice will melt in (KCu = 0.92 cal s–1 cm–1°C–1) :
melting of ice in the two cases respectively, the ratio q2 is
(a) 1.32 s (b) 1.6 s
1 2 (c) 0.33 s (d) 0.65 s
(a) (b)
2 1 17. Figure shows a composite slab of three different materials,
4 1 a, b and c with equal thickness and with thermal conductivities
(c) (d) 4 ka > kb > kc. The transfer of energy through them as heat is non-
1
zero and steady. The temperature difference is the greatest across
10. Three discs A, B and C having radii 2 m, 4 m and 6 m respectively
are coated with carbon black on their outer surfaces . The wavelength
corresponding to maximum intensity are 300 nm, 400 nm and 500 a b c
nm respectively. The power radiated by them are QA, QB and QC
respectively (a) a (b) b
(a) QA is maximum (b) QB is maximum (c) c (d) a and c
(c) QC is maximum (d) QA = QB = QC 18. The intensity of radiation emitted by the sun has its maximum
value at a wavelength of 510 nm and that emitted by the north star
11. A spherical body of area A and emissivity 0.6 is kept inside a
has the maximum value at 350 nm. If these stars behave like black
black body. What is the rate at which energy is radiated per second
bodies, then the ratio of the surface temperature of the sun and
at temperature T
north star is
(a) 0.6 AT4 (b) 0.4 AT4
(a) 1.46 (b) 0.69
(c) 0.8 AT 4 (d) 1.0 AT4
(c) 1.21 (d) 0.83
12. A room is maintained at 20°C by a heater of resistance 20
19. Three rods made of the same material and having the same cross
connected to 200 V mains. The temperature is uniform throughout
section have been joined as shown in the figure. Each rod is of the
the room and the heat is transmitted through a glass window of same length. The left and right ends are kept at 0°C and 90°C
area 1 m2 and thickness 0.2 cm (K for glass is 0.2 cal/m°C. s). The respectively. The temperature of the junction of the three rods
temperature outside is will be 90°C
(a) 20°C (b) 10°C B
(a) 45°C °C
(c) 12.2°C (d) 15.2°C (b) 60°C
13. A ‘cold box’ in the shape of a cube of edge 50 cm is made of A
(c) 30°C C
‘thermocool’ material 4.0 cm thick. If the outside temperature is 90°C
(d) 20°C
30°C, the quantity of ice that will melt each hour inside the ‘cold
20. Five identical rods are joint as shown in the figure. Point A and C
box’ is (the thermal conductivity of the thermocool material is
0.050 W/m2.K) are maintained at temperature 120°C and 20°C respectively. The
temperature of junction B is
(a) 1.52 kg (b) 0.605 kg
(a) 70°C
(c) 2.520 kg (d) 0.512 kg
(b) 80°C 120°C 20°C
14. The body of an unclothed person has a surface area of 1.50 m2 C
(c) 30°C A
and an emissivity of 0.80. His skin temperature is 37°C. He B
stands in an air–conditioned room where the temperature is (d) 20°C
maintained at 17°C. The amount of heat he loses per minute, is 21. Three rods of same dimensions are arranged as shown in figure.
( = 5.67 × 10–8 W/m2 . K4) They have thermal conductivities K1, K2 and K3. The points P
and Q are maintained at different temperatures for the heat to
(a) 8.8 kJ (b) 4.4 kJ
flow at the same rate along PRQ and PQ then which of the following
(c) 2.2 kJ (d) 3.3 kJ
options is correct?
15. Water contained in a closed thin walled cylindrical copper tank, of
radius 30 cm and height 1 m, is maintained at 60°C by means of 1
(a) K3 K1 K2 R
an electric heater immersed in water, the outside temperature 2
being 20°C. The tank’s outer curved surface is covered with 1 cm (b) K3 = K1 + K2
thick felt (Kfelt = 9 × 10–5 cal/s.cm.°C). Neglect all other losses. K1 K2
The wattage of the heater is K1K 2
(c) K3
(a) 100 (b) 1000 K1 K 2
(c) 220 (d) 284 (d) K3 = 2(K1 + K2) P K3 Q

Answer Key 9 (c) 11 (a) 13 (b) 15 (d) 17 (c) 19 (b) 21 (c)


Sol. from page 562 10 (b) 12 (d) 14 (a) 16 (d) 18 (b) 20 (a)
www.crackjee.xyz
HEAT TRANSFER 553
22. A metal rod AB of length 10x has its one end A in ice at 0°C and the nE nE
other end B in water at 100°C. If a point P on the rod is maintained
at 400°C, then it is found that equal amounts of water and ice
(a) (b)
evaporate and melt per unit time. The latent heat of evaporation
of water is 540 cal/g and latent heat of melting of ice is 80 cal/g. If nT nT
the point P is at a distance of x from the ice end A, find the value nE
of . [Neglect any heat loss to the surrounding]. nE
(a) 9 (b) 2
(c) 6 (d) 1 (c) (d)
nT
23. The figure shows a system of two concentric spheres of radii r1 nT
and r2 and kept at temperatures T 1 and T 2, respectively. The
25. A system S receives heat continuously from an electrical heater of
radial rate of flow of heat in a substance between the two concentric power 10W. The temperature of S becomes constant at 50°C
spheres is proportional to when the surrounding temperature is 20°C. After the heater is
r1 r2 switched off, S cools from 35°C to 34.8°C in 1 minute. The heat
(a) r2 – r1 capacity of S is
(a) 100 J/°C (b) 300 J/°C
(b) r2 – r1 r1
(c) 750 J/°C (d) 1400 J/°C
T1
(c) r2 – r1 r1 r2 26. The sun rays are focussed by a concave mirror of diameter 12 cm
fixed with its axis towards the sun onto a copper calorimeter,
r2 r2 T2 where they are absorbed. If the thermal capacity of the calorimeter
(d) ln
r1 and its contents is 59 cal/°C and the temperature rises by 8°C in 2
24. Which of the following graphs correctly represents the relation minutes, the heat received in 1 minute by a square meter of the
between ( n E) and ( n T) where E is the amount of radiation earth surface when the rays are incident normaly is
emitted per unit time from unit area of a body and T is the absolute (a) 20860 cal (b) 25540 cal
temperature
(c) 10430 cal (d) 51180 cal

Answer Key
22 (a) 23 (a) 24 (d) 25 (d) 26 (a)
Sol. from page 562

MCQ Type 2 Exercise 8.2


Mulitiple options correct through a hole in the roof of the dark room. Assuming that there
1. Two bodies A and B have thermal emissivities of 0.01 and 0.81 is no change in the ambient temperature of the room, which of the
respectively. The outer surface areas of the two bodies are the following statement is/are correct?
same. The two bodies emit total radiant power at the same rate. (a) The quantity of radiation absorbed by the black body in unit
The wavelength B corresponding to maximum spectral radiancy time will increase.
in the radiation from B is shifted from the wavelength (b) Since emissivity = absorptivity, hence the quantity of
corresponding to maximum spectral radiancy in the radiation from radiation emitted by black body in unit time will increase
A by 1.00 m. If the temperature of A is 5802 K.
(c) Black body radiates more energy in unit time in the visible
(a) The temperature of B is 1934 K spectrum
(b) B 1.5 μm (d) The reflected energy in unit time by the black body remains
(c) The temperature of B is 11604 K same
(d) The temperature of B is 2901 K 3. For transmission of heat from one place to the other, medium is
2. In a dark room with ambient temperature T0 a black body is kept required in
at a temperature T. Keeping the temperature of the black body (a) conduction (b) convection
constant (at T) sunrays are allowed to fall on the black body (c) radiation (d) all

Answer Key
1 (a, b) 2 (a, b, c, d) 3 (a, b)
Sol. from page 564
www.crackjee.xyz
554 MECHANICS, HEAT, THERMODYNAMICS & WAVES
4. Three rods of material X and three rods of material Y are connected 6. A black body of temperature T is inside chamber of temperature
as shown in the figure. All the rods are of identical length and T0 initially. Sun rays are allowed to fall from a hole in the top of
cross-sectional area. The end A is maintained at 60°C and the chamber. If the temperature of black body (T) and chamber (T0)
junction E at 10°C. The thermal conductivity of X is 0.92 cal/sec- remains constant, then
cm-°C and that of Y is 0.46 cal/sec-cm-°C. Choose the correct
option(s).
C T0
X X
T
A B E
Y X
Y Y
D (a) Black body will absorb more radiation
(a) The temperature of the junction B is 40°C (b) Black body will absorb less radiation
(b) The temperature of the junction C is 20°C (c) Black body emits more energy
(c) The temperature of the junction D is 20°C (d) Black body emits energy equal to energy absorbed by it
(d) The temperature of the junction B is 30°C d
7. In Newton's law of cooling , = – k ( – 0) , the constant k is
5. Two identical objects A and B are at temperatures T A and T B dt
respectively. Both objects are placed in a room with perfectly proportional to
absorbing walls maintained at a temperature T (TA > T > TB). The (a) A , surface area of the body
objects A and B attain the temperature T eventually. Select the (b) S , specific heat of the body
correct statements from the following (c) 1/m , m being mass of the body
(a) A only emits radiations, while B only absorbs it until both (d) e , emmisivity of the body
attain the temperature T 8. A body takes 5 minutes for cooling from 50°C to 40°C. Its
(b) A loses more heat by radiation than it absorbs, while B absorbs temperature comes down to 33.33°C in next 5 minutes. Now,
more radiation than it emits, until they attain the temperature choose the correct statement(s) from the following ?
T (a) The temperature of surrounding is 20°C
(c) Both A and B only absorb radiation, but do not emit it, until (b) The temperature of surrounding is 25°C
they attain the temperature T (c) The temperature of the body in further 5 minutes becomes
(d) Each object continues to emit and absorb radiation even 18.88°C
after attaining the temperature T (d) The temperature of the body in further 5 minutes becomes
28.88°C

Answer Key
4 (b, c, d) 5 (b, d) 6 (a, c, d) 7 (a, c) 8 (a, d)
Sol. from page 564

Statement Questions Exercise 8.3


Read the two statements carefully to mark the correct option out of the options given below:
(a) If both the statements are true and the statement - 2 is the correct explanation of statement - 1.
(b) If both the statements are true but statement - 2 is not the correct explanation of the statement - 1.
(c) If statement - 1 true but statement - 2 is false.
(d) If statement - 1 is false but statement - 2 is true.

1. Statement 1 3. Statement 1
A body at 25 ° C radiates in a room, where the room temperature Temperature near the sea coast are moderate.
is 30° C. Statement 2
Statement 2 Water has high thermal conductivity.
Each body radiates energy at all temperature. 4. Statement 1
2. Statement 1 A body that is a good radiator is also a good absorber of radition at
If the temperature of a star is doubled then the rate of loss of heat a given wavelength.
from it becomes 16 times. Statement 2
Statement 2 According to Kirchoff's law the absorptivity of a body is equal to
Specific heat varies with temperatures its emissivity at a given wavelength.
www.crackjee.xyz
HEAT TRANSFER 555
5. Statement 1 Statement 2
Bodies radiate heat at all temperature. Peak emission wavelength of a blackbody is proportional to the
Statement 2 fourth power of absolute temperature.
Rate of radiation of heat is proportional to the fourth power of 9. Statement 1
absolute temperature. The radiation from the sun's surface varies as the fourth power of
6. Statement 1 its absolute temperature.
Woollen cloths keep the body warm in winter. Statement 2
Statement 2 The sun is not a black body.
Air is bad conductor of heat. 10. Statement 1
7. Statement 1 A hollow metallic closed container maintained at a uniform
It is hotter over the top of a fire than at the same distance on the temperature can act as a source of black body radiation.
sides. Statement 2
Statement 2 All metals acts as a black body.
Air surrounding the fire conducts more heat upwards. 11. Statement 1
8. Statement 1 Blue star is at high temperature than red star.
For higher temperature, the peak emission wavelength of a black Statement 2
body shifts to lower wavelengths. 1
Wein's displacement law states that T .
m

Answer Key 1 (a) 2 (b) 3 (b) 4 (a) 5 (b) 6 (a)


Sol. from page 564 7 (c) 8 (c) 9 (c) 10 (c) 11 (a)

Passage & Matrix Exercise 8.4

2. The temperature of the oil is


Passage for (Q. 1 - 3) : (a) 439.15 K (b) 419.83 K
Hot oil is circulated through an insulated container with a wooden (c) 523 K (d) 318.2 K
lid at the top whose conductivity K = 0.149 J/(m-ºC-sec), thickness 3. If area of the top of the lid be 0.5 m2 then the rate of heat flow
t = 5 mm, emissivity = 0.6. Temperature of the top of the lid is maintained through radiation is
at T = 127ºC, if the ambient temperature Ta = 27ºC. (a) 297.5 W (b) 595 W
17 (c) 892.5 W (d) 669.4 W
(Given = 10 –8 )
3
Passage for (Q. 4 - 6) :
A hot body placed in air is cooled down according to Newton’s law of
cooling, the rate of decrease of temperature being k times the temperature
difference from the surrounding. Assume that cooling starts at t = 0 :

4. If T0 and T1 be the temperature of the surrounding and the initial


Ta = 27ºC temperature of the body respectively, then the temperature of the
Hot Oil body at any time t is given by

(a) kt (b) kt
T0 (T1 – T0 )e T0e
(c) kt (d) kt
1. The rate of heat loss per unit area due to radiation from the lid is T1(T0 – T1)e T1 (T0 – T1)e
(a) 415 W / m2 (b) 555 W / m2
(c) 595 W / m 2 (d) 315 W / m2

Answer Key
1 (c) 2 (b) 3 (a) 4 (a)
Sol. from page 565
www.crackjee.xyz
556 MECHANICS, HEAT, THERMODYNAMICS & WAVES
5. The time in which the body will lose half the maximum heat it can
Passage for (Q. 10 - 12) :
lose is
ln 2 A metal block of heat capacity 80 J/°C placed in a room at 20°C is
(a) ln 2 (b) heated electrically. The heater is switched off when the temperature
k
reaches 30°C. The temperature of the block rises at the rate of 2°C/s
(c) k ln 2 (d) k ln 2
just after the heater is switched on and falls at the rate of 0.2–°C/s just
6. If k = ln 2, then the time (in proper unit) in which the body will after the heater is switched off. Assume Newton’s law of cooling to
lose half the maximum heat it can lose is hold.
(a) 2 (b) 1 10. The power of the heater is
(c) 4 (d) 3
(a) 100 W (b) 160 W

Passage for (Q. 7 - 9) : (c) 190 W (d) 220 W


n moles of a monoatomic gas at an initial temperature T0 is enclosed in 11. The power radiated by the block just after the heater is switched
a cylindrical vessel filled with a light piston. The surrounding air has a off is
temperature Ts ( > T0) and the atmospheric pressure is Pa. Heat may (a) 4W (b) 8W
be conducted between the surrounding and the gas through the bottom (c) 12 W (d) 16 W
of the cylinder. The bottom has a surface area A, thickness x and
12. The power radiated by the block when the temperature of the
thermal conductivity K. Assume all changes to be slow. block is 25°C is
(a) 4W (b) 8W
7. If T be the temperature of gas after time t, then the difference
(c) 12 W (d) 16 W
(Ts – T = T ) is given by
2 KAt 2 KAx
(a) (b) Passage for (Q. 13 - 14) :
(Ts – T0 )e – 5nRx (Ts – T0 ) e – 5 nRt
The rate at which the radiant energy reaches the surface of earth
2 KtR 3 Ktx from the sun is 1.4 kW/m 2. The distance of earth from the sun is
(c) – 5nAx (d) – 5nRA 1.5 × 1011 m and the radius of sun is 7 × 108 m. Calculate
(Ts – T0 )e (Ts – T0 )e
8. The change in volume of the gas in cylinder is
13. Rate of radiant energy emitted from sun’s surface per unit area
2 KAx 2 KAt per second,
nR nR
(a) (Ts – T0 ) e – 5nRt (b) (Ts – T0 ) e – 5nRx (a) 3.32 × 107 W/m2 (b) 6.43 × 107 W/m2
Pa Pa
(c) 5.83 × 107 W/m2 (d) none of these
2 KA 5 KAt 14. Assuming sun as a black body, what is surface temperature of the
nR – 5nRtx nR – 2 nRx sun ?
(c) (Ts – T0 )e (d) (Ts – T0 )e
Pa Pa (a) 4000 K (b) 5200 K
9. The distance moved by the piston in time t is given by (c) 5803 K (d) 6430 K
2 KAx 2 KAt
nR nR
(a) (Ts – T0 ) e – 5 nRT
(b) (Ts – T0 ) e – 5 nRx
Pa A Pa A
2 KA 5 KAt
nR nR
(c) (Ts – T0 ) e – 5nRtx
(d) (Ts – T0 ) e – 2 nRx
Pa A Pa A

Answer Key 5 (b) 7 (a) 9 (b) 11 (d) 13 (b)


Sol. from page 565 6 (b) 8 (b) 10 (b) 12 (b) 14 (c)

15. A copper rod (initially at room temperature 20°C) of non-uniform cross section is placed between a steam chamber at 100°C and
ice-water chamber at 0°C.

100°C
0°C
Steam
Ice water
Chamber
Chamber
B
A C
www.crackjee.xyz
HEAT TRANSFER 557
Column I Column II
dQ
A. Initially, rate of heat flow will be (p) maximum at section A
dt

dQ
B. At steady state, rate of heat flow will be (q) maximum at section B
dt

dT
C. At steady state, temperature gradient will be (r) maximum at section C
dx
D. At steady state, rate of change of temperature (s) minimum at section B
dT
at a certain point will be (t) same for all section
dt
16. A ball has surface temperature T initially at time t = 0, that is less than surrounding constant temperature T0. On the vertical axis of the graph
shown has either thermal energy radiated/absorbed per unit time or total energy radiated/absorbed till time t by the ball. Correctly match the
curves marked in the graph

3
4

t
Column I Column II
A. Thermal energy emitted per unit time (p) 1
B. Thermal energy absorbed per unit time (q) 2
C. Total energy emitted till time t (r) 3
D. Total energy absorbed till time t (s) 4
17. Match columns I and II (regarding Newton's law of cooling)
Column I Column II
Y

A. Curve between log ( 0) and time,


plotted on X- and Y - axes respectively (p)

X
O
Y

B. Curve between temperature of body ( ) and time, (q)


plotted on X - and Y - axes respectively
X
O

C. Curve between the rate of cooling (R) and body (r)


temperature ( ), plotted on X-and Y-axes respectively
X
O

D. Curve between the rate of cooling (R) and temperature (s) O X


difference between body ( ) and surrounding ( 0), k
plotted on X - and Y-axes respectively

15 (A) p, r; (B) t; (C) q, r; (D) t 16 (A) p; (B) q; (C) s; (D) r


Answer Key
Sol. from page 565 17 (A) q; (B) p; (C) s; (D) r
www.crackjee.xyz
558 MECHANICS, HEAT, THERMODYNAMICS & WAVES

Subjective Integer Type Exercise 8.5


Solution from page 566

1. A pitcher with 1 mm thick porous walls contains 10 kg of water. 5. Three rods of material x and three of material y are connected as
Water comes to its outer surface and evaporates at the rate of shown in figure. All the rods are identical in length and cross–
0.1 g/s. The surface area of the pitcher (one side) = 200 cm2. The sectional area. If the end A is maintained at 60°C and the junction
room temperature = 42°C, latent heat of vaporization = 2.27 × 106 E at 10°C, calculate the temperature of the junction B. The thermal
J/kg and the thermal conductivity of the porous walls = 0.80 J/m–s– conductivity of x is 800 W/m–°C and that of y is 400 W/m–°C.
°C. Calculate the temperature of water in the pitcher when it attains
a constant value. Ans : 28°C.

2. A metal rod of cross–sectional area 1.0 cm2 is being heated at one


end. At one time, the temperature gradient is 5.0 °C/cm at cross–
section A and is 2.5°C/cm at cross–section B. Calculate the rate at
which the temperature is increasing in the part AB of the rod. The
heat capacity of the part AB = 0.40 J/°C, thermal conductivity of Ans : 40°C.
the material of the rod = 200 W/m–°C. Neglect any loss of heat to 6. An iron ball having a surface area of 200 cm2 and at a temperature
the atmosphere. Ans : 12.5°C/s. of 527°C is placed in an enclosure at 27°C. If the surface emissivity
3. Steam at 120°C is continuously passed through a 50 cm of iron be 0.4, at what rate is heat being lost by radiation by the
long rubber tube of inner and outer radii 1.0 cm and 1.2 cm. T h e ball ?
room temperature is 30°C. Calculate the rate of heat flow Ans. 44 cal/s.
through the walls of the tube. Thermal conductivity of rubber
= 0.15 J/m–s–°C. Ans :233 J/s. 7. An electric heater is used in a room of total wall area 137 m2 to
maintain a temperature of + 20°C inside it when the outside
4. A cylindrical rod of length 50 cm and cross–sectional area 1 cm2 is temperature is –10°C. The walls have three layers of different
fitted between a large ice chamber at 0°C and an evacuated chamber materials. The innermost layer is of wood of thickness 2.5 cm, the
maintained at 27°C as shown in figure. Only small portions of the middle layer is of cement of thickness 1.0 cm and the outermost
rod are inside the chambers and the rest is thermally insulated layer is of brick of thickness 25.0 cm. Find the power of the
from the surrounding. The cross–section going into the evacuated electric heater. Assume that there is no heat loss through the floor
chamber is blackened so that it completely absorbs any radiation and ceiling. The thermal conductivities of wood, cement and brick
falling on it. The temperature of the blackened end is 17°C when are 0.125, 1.5 and 1.0 W/m–°C respectively.
steady state is reached. Stefan’s constant = 6 × 10–8 W/m2–K4.
Find the thermal conductivity of the material of the rod. Ans. 9 kW.

Ans : 1.8 W/m–°C.


www.crackjee.xyz
HEAT TRANSFER 559

Subjective Exercise 8.6


Solution from page 567
1. An aluminium container of mass 100 g contains 200 g of ice at 8. Four identical rods AB, CD, CF and DE are joined as shown in
– 20°C. Heat is added to the system at a rate of 100 cal/s. What figure. The length, cross–section area and thermal conductivity of
is the temperature of the system after 4 minutes ? Draw a rough each rod are l, A and K respectively. The ends A, E and F are
sketch showing the variation in the temperature of the system as maintained at temperatures T1, T2 and T3 respectively. Assuming
a function of time. Specific heat capacity of ice = 0.5 cal/g–°C, no loss of heat to the atmosphere, find the temperature at B.
specific heat capacity of aluminium = 0.2 cal/g–°C, specific heat
capacity of water = 1 cal/g–°C and latent heat of fusion of
ice = 80 cal/g.
Ans : 25.5°°C
2. Two vessels A and B of different materials but having identical
shape, size and wall–thickness are filled with ice and kept at the
same place. Ice melts at the rate of 100 g/min and 150 g/min in A
and B respectively. Assuming that heat enters the vessels through
the walls only, calculate the ratio of thermal conductivities of their
materials. Ans. 2 : 3.
3. An electric bulb with tungsten filament having an area of 0.25 cm2
is raised to a temperature of 3000 K, when a current passes through
it. Calculate the electrical energy being consumed in watt, if the
3T1 2(T2 T3 )
emissivity of the filament is 0.35. Stefan’s constant, = 5.67 × Ans : .
10 –5/erg/cm 2–K 4. If due to fall in main voltage the filament 7
temperature falls to 2500 K, what will be wattage of the bulb ? 9. A hot body placed in a surrounding of temperature 0 obeys
Ans. 40.19 W, 19.38 W. Newton’s law of cooling d /dt = – k( – 0). Its temperature at
4. A layer of ice 2 cm thick is formed on a pond. The temperature of t = 0 is 1. The specific heat capacity of the body is s and its mass
air is – 20°C. Calculate how long it will take for the thickness of is m. Find
ice to increase by 1 mm. [Density of ice = 1 g/cm3, latent heat of (a) the maximum heat that the body can lose and
ice = 80 cal/g, conductivity of ice = 0.008 cal /cm–s–°C]
(b) the time starting from t = 0 in which it will lose 90% of this
Ans : 102.5 s
maximum heat.
5. One end of a copper rod of a uniform cross–section and of length
1.5 m is kept in contact with ice and the other end Ans : (a) ms ( 1 – 0) (b) ln 10 / k.
with water at 100°C. At what point along its length should a 10. Consider a cubical vessel of edge a having a small hole in one of its
temperature of 200°C be maintained so that in steady state, the walls. The total thermal resistance of the walls is r. At time t = 0,
mass of ice melted be equal to that of the steam produced in the it contains air at atmospheric pressure pa and temperature T0.
same interval of time ? Assume that the whole system is insulated The temperature of the surrounding air is Ta(> T 0 ). Find the
from the surroundings. Latent heat of fusion of ice = 80 cal/g, amount of the gas (in moles) in the vessel at time t. Take Cv of air
latent heat of vaporization of water = 540 cal/g. to be
Ans : 1.396 m. 5 R / 2.
6. Water is 50°C, is filled in a closed cylindrical vessel of height 10 2Ta
– t
cm and cross–sectional area 10 cm2. The walls of the vessels are pa a 3 Ta 7 rpa a3
Ans : n = 1 –1 e
adiabatic but the flat parts are made of 1 mm thick aluminium RTa T0
(K = 200 J/m–s–°C). Assume that the outside temperature is 11. A cylindrical block of length 0.4 m and area of cross–section
20°C. The density of water is 1000 kg/m3 and the specific heat 0.04 m2 is placed coaxially on a thin metal disc of mass 0.4 kg and
capacity of water = 4200 J/kg–°C. Estimate the time taken for the of the same cross–section. The upper face of the block is maintained
temperature to fall by 1.0 °C. Make any simplifying assumptions at a constant temperature of 400 K and the initial temperature of
you need but specify them. the disc is 300 K. If the thermal conductivity of the material of the
Ans : 0.035 s. cylinder is 10W/m–K and the specific heat of the material of the
7. Three rods of lengths 20 cm each and area of cross–section 1cm2 disc is 600 J/kg–K, how long will it take for the temperature of the
are joined to form a triangle ABC. The conductivities of the rods disc to increase to 350 K ? Assume for purpose of calculation, the
are KAB = 50 J/m–s–°C, KBC = 200 J/m–s–°C and KAC = 400 J/m– thermal conductivity of the disc to be very high and the system to
s–°C. The junctions A, B and C are maintained at 40°C, 80°C and be thermally insulated except for the upper face of the cylinder.
80°C respectively. Find the rate of heat flowing through the rods Ans. 2 min. 46 s.
AB, AC and BC.
Ans : 1W, 8W, zero.
www.crackjee.xyz
560 MECHANICS, HEAT, THERMODYNAMICS & WAVES
12. A body initially at 80°C cools to 64°C in 5 minutes and to 52°C in 14. A rod of length l with thermally insulated lateral surface is made of
10 minutes. What will be the temperature after 15 minutes and a material whose thermal conductivity varies as K = /T, where
what is the temperature of surroundings ? is a constant. The ends of the rod are kept at temperatures T1 and
Ans. 43°C, 16°C. T2. Find the temperature function T(x), where x is the distance
from the end whose temperature is T 1 and also find the heat
13. A solid body X of heat capacity C is kept in an atmosphere whose
flowing per second per unit cross-sectional area.
temperature is TA = 300 K. At time t = 0, the temperature of X is
T0 = 400 K. It cools according to Newton’s law of cooling. At time x/
t, the temperature is found to be 350 K. At this time (t1), the body T2 – T2
Ans. Tx = T1 , H= log e
X is connected to a large box Y at atmospheric temperature TA, T1 T1
through a conducting rod of length L, cross–sectional area A and
thermal conductivity K. The heat capacity of Y is so large that any 15. Find the temperature distribution in a substance placed between
variation in its temperature may be neglected. The cross-sectional two parallel plates kept at temperature T 1 and T 2. The plate
area A of the connecting rod is small compared to the surface area separation is equal to l, the coefficient of thermal conductivity of
of X. Find the temperature of X at time t = 3 t1. the substance varies as K T .

KA ln 2 2/3
Ans. T = 300 50exp –2t1 . 3/ 2
CL t1 x T2
Ans. T = T1 1 –1
T1
www.crackjee.xyz
HEAT TRANSFER 561

Solutions Exercise8.1 Level -1


1. (d) R = R1 + R2 and so emits them in the dark room while the polished shining
part reflects radiation and absorbs nothing and so does not
x 4x x 4x radiate.
or =
KA KA 2 KA 11. (a) Initially black body absorbs all the incident energy and so it
5K is the darkest one. Black body radiates maximum energy if
K = conditions are same.
3 12. (c) Bright body emits colour of short wavelength.
The rate of heat flow, 13. (c) E = A (T4 – T04)
= 0.4 × 200 × 10–4× 5.67 × 10–8 × [8004 – 3004]
T2 T1
H = K A = 182J.
5x 14. (d) The total power radiated by sun
5K T T = T 4 4 R2
= A 2 1
3 5x 2
The power incident on earth of surface area 4 r0 .
K T T
= A 2 1 T4 4 R2
4 r0 2
.
3 x = 2
4 r
1
Thus f = .
3 4 r0 .2 R2 T 4
=
r2
4
2. (d) E = T
15. (b) E = 4 R2 T4
4
T E
E = 4
2 16 2 T
and E = 4 100 R
2
2
3. (b) The surface area of cube, A1 = r 2 ; sphere = 4 r = 625 E.
hemisphere = 2 r 2 r2 3 r2 . Ct
16. (a) Q = Ht = CKt =
As E surface area, and so heat transfer is greatest for R
sphere.
4. (a) If R is the resistance of each rod, then Q1 R2 2r 2
Q2 = R1 = r
R
R1 = R + R = 2R and R2 =
2 T
17. (a) mL = KA
x
V2 4 t
Like Q = t ; 2R = R / 2 t = 1 minute
R 40
or 500 × 80 = 0.0075 × 75 t
5. (d) For quick and large amount of heat needed low specific heat 5
and high conductivity. t = 8888 s
6. (a) If T be the temperature of interface, then = 2.47 hr.
T
T 0 100 T 0°C 100°C T
KA
x = 4 KA K 4K 18. (c) mL = KA t
x x
x x
T = 80º C.
7. (d) If T be the temperature of interface, then 3 100
or m × 8 × 104 = 92 10 60
1
100 T T 0
300 A = 200A m = 6.9 × 10–3kg.
x x 19. (b) For no heat flows across CD,
T = 60°
8. (b) Q = H1t1 = H2t2 R1 R3
R2 = R4
or K1t1 = K2t2
K1 t2 35 7 1/ K1 1/ K3 K2 K4
= = = or =
K2 t1 20 4 1/ K 2 1/ K 4 K1 K3
9. (c) Warming of the bulb is due to radiation process. K2K3 = K1K4
10. (a) Black spot on heating absorbs radiations from surroundings
www.crackjee.xyz
562 MECHANICS & THERMODYNAMICS

20. (b) The rate of heat flow does not depend on area of cross- After substituting the values and simplify,
section of specimen. we get T1 = 86°C and T2 = 57°C
23. (b) Q = 500 × 0.572 × (50 – 20)
1 2 = 8580 cal.
21. (b) KA = KA
x l x As Pt = Q
Q
1 2 t =
P
x –x = 360 s

2 1 700 T T 100 700°C T 100°C


= 1 x 24. (c) K tin A = K cu A
l 0.2 2.4
It represents straight line with negative slope. T = 500°C
T1 T2 700 500
22. (d) 100°C 0°C Q = 0.15×(400)× ×3600
1
0.2 10
Cu Al Brass = 2.16 × 109 cal 0.2 2.4
100 T1 T2 0 mm mm
K cu A = K brass A Q 2.16 109
l l m= = = 4 × 106 g
L 540
T1 T2 = 4000 kg.
= K Al A
l

Solutions Exercise 8.1Level -2


T1 T T2
T1 T T T2 6. (d) E = T4A
1. (b) 2K A = KA 2K K
x x E1 = T 4 4 r2
or 2(T1 – T) = (T – T2) x x
or 3(T1 – T) = (T – T2) + T1 – T 2
4 r
= (T1 – T2) and E2 = 2T 4
2
T1 T2 36
T1 – T = 12 = 4E1 = 4 × 450 = 1800 W.
3 3
4 3 4 3
1 1 1 2K 7. (d) r1 = 3 r2
2. (b) = 3 3
R R1 R2 K
or KA = K1A1 + K2A2 r1
r2 = 3
3
2 2 2 2
or K 4 R = K1 × R + K2 × 4 R R
dT / dt 1 r2 1
K1 3K 2 dT / dt = 3
K = 2
r1 3
4
8. (b) For steady state
3. (c) The temperature of x is decreasing faster than y, so its emissive
power is greater than y. H1 = H2 Tc C
As ex > ey, so ax > ay. Tc T
2T Tc KA H2 H1
KA =
dT A l 2l
4. (d) = T T0
dr mc T
Tc 3 2T
= A B
4 r2 T 2 1
T T0
= 4 3 9. (c) In first case R1 = R/2 and in second case R2 = 2R.
r c
3
q1 R2 2R
Thus, = 4
dT 1 q2 R1 R/2
dr rc 4
5. (d) The mean wavelength b
10. (b) QA = T 4A A
500 1500 A
= 1000nm
2
4
So the intensity of radiation is greatest for 1000 nm. b K
= A
Thus U2 > U1 and U3. 300 81
www.crackjee.xyz
HEAT TRANSFER 563

4 90°C
b K
QB = 4A 19. (b) H = H1 + H2 0°C T H1
400 64
H H2
T 0 90 T
4 KA = KA 2 90°C
b 9K
QC = 9A
500 625 T = 60°C.
QB > QC > QA 20. (a) The equivalent combination is as follows :
HAB = HBC A R T R C
11. (a) E = eA T 4 0.6 A T 4
120 T T 20 120°C B 20°C
12. (d) If T be the outside temperature, then or =
R R
V2 T T T = 70°C.
= KA
R x 21. (c) H PRQ = H PQ

or K PRQ = K PQ
2002 0.2 4.2 1
20 T
or = 2
20 0.2 10 K1K 2
T = 15.2°C. or K1 K 2 = K3

T 400°C
13. (b) mL = kA t
x A B
22. (a)
30 0 0°C x 10 x 100°C
or m × (80 × 4200) = 0.050 6 0.52 3600
0.04
mwater evaporate = mice =m
m = 0.605 kg. melt

4 400 100
14. (a) E = eA T T0 4 KA 1
m × 540 = 10 x .............(i)
8
= 0.80 1.50 5.67 10 3104 290 4 60
400 0
and m × 80 = KA 1 .............(ii)
= 8.8 kJ x
From above equations, we get
T T
15. (d) P = KA = K 2 Rh = 9
x x
23. (a) The radial flow of heat is given by
5 60 20 T1 T2
= 9 10 420 2 0.30 1
H = K 4 r1r2
0.01 r2 r1
= 284 W.
24. (d) E = A T4
T
16. (d) mL = KA t
x nE = n A 4 nT
It represents straight line.
100°C 0°C 25. (d) If P is the power radiates at 35°C, then

3 mm 10 = A 3234 2934 .............(i)


or 2 × 80 × 4200
and P = A 3084 2934 ............(ii)
100 0
= 0.92 420 0.20 0.40 t
3 10 3 From above equations, we get
P = 4.65 W
t = 0.65 s
Now P t = mc T
Ta Tb Tc or 4.65 × 60 = mc × 0.2
17. (c) Q = Ka A = Kb A = Kc A
x x x mc = 1400 J/°C.

As Kc is least, so Tc greatest d2
26. (a) T4 2 60 = 59 × 8 .............(i)
18. (b) = 4
sunTsun starTstar

Tsun star 350 35 and T 4 1 60


Q = ............(ii)
Tstar = 510 51 After solving above equations, we get
sun
Q = 20860 cal.
= 0.69.
www.crackjee.xyz
564 MECHANICS & THERMODYNAMICS

Solutions Exercise 8.2


C
1. (a,b) A TA4 = B TB4 TB =
30°C. 60°C 10°C
A R/2 R/2
R B E
1 1 R
TA Also T C = TD , R
B
4 0.81 4 D
TB = = =3
A 0.01 30 TC TC 10 60°C R
and = 2R/3 10°C
R/2 R/2 A E
TB
TA 5802
TB = T C = 20°C
3 3
(5) (b,d)Every object emit and absorb the radiations simultaneously,
= 1934 K if energy emitted is more than energy absorbed, temperature
From Wein’s displacement law, falls and vice versa.
(6) (a, c, d) Since sun rays fall on the black body, it will absorb more
ATA = BTB radiation and since, its temperature is constant it will emit
more radiation. The temperature will remain same only when
A TB 1934 1 energy emitted is equal to energy absorbed.
or = TA = 5802 = 3 ...........(i)
B (7) (a,c) On comparing the given equation with the equation

Also B A = 1.00 ..........(ii) dT A


= T T0 , we find that
dt mc
After solving above equations, we get
A
B = 1.5 μm . K =
mc
(2). (a,b,c,d) With the incident radiation, the temperature of black
body try to increase, so body emits more energy per unit 1
K A and K
time. To keep the temperature constant it must absorbs m
incident radiations with increased rate. The reflection depends
50 40 50 40
on the nature of surface not on the temperature. (8) (a,d) = k T0 ..........(i)
5 2
(3) (a,b) Conduction and convection need medium for transmission
of heat. 40 33.33 40 33.33
and = k T0 ..........(ii)
5 2
Ry R
(4) (b,c,d) Kx = 2K y , so Rx = on solving above equations, we get
2 2
The equivalent circuit is shown in figure. T0 = 20°C

60 TB TB 10 33.33 T 33.33 T
= Also = k 20 ..........(iii)
R 2R / 3 5 2
T = 28.88°C.

Solutions Exercise-8.3
(1). (a) According to Stefan’s law, the energy radiated per second (6) (a) Air enclosed inside clothes prevent the heat due to poor
through unit area conductivity.
(7) (c) Hot air rises up due to smaller density.
E = T4
1
4 (8) (c)
(2) (b) E = T4 , E = 2T = 16 E. T
(3) (b) Both are experimentally proved. (9) (c) Sun is a black body, and so E T4
(4) (a) According to Kirchoff’s law
(10) (c)
e (11) (a) The wavelength of blue colour is shorter than red colour, and
= constant.
a so temperature of blue star will be higher.

(5) (b) E T 4 , as T is always greater than 0 K, so


E always be greater than zero.
www.crackjee.xyz
HEAT TRANSFER 565

Solutions Exercise-8.4
Passage (Q. 1 - 3) : Putting these values in equation (i), we have
1. (c) The rate of heat loss per unit area per second due to radiation
T1 T0 kt '
is given by Stefan's-Boltzmann law T0 (T1 T0 )e
2
E = e s T 4 T04
T1 T0 kt '
or (T1 T0 )e
2
17
= 0.6 × 10 8 [(400) 4 (300) 4 ] = 595 watt/m2
3 kt ' 1
or e
2. (b) Let Toil be the temperature of the oil. Then rate of heat flow 2
through conduction = Rate of heat flow through radiation
ln 2
or t'
KA(Toil T ) k
= 595 × A where A is the area of the top of lid
If k = ln 2, t ' 1 unit
3 Passage (Q. 7 - 9) :
595 595 5 10
Toil = T 400 = 419.83 K
K 0.149 7. (a) 8. (b) 9. (b)

3. (a) Rate of heat flow through radiation Light piston will maintain the pressure of the gas equal to the
atmospheric pressure (Pa).
= (595 W/m2) × (0.5 m2 ) = 297.5 W.
Let at any instant, the temperature of the gas be q. In next small
Passage (Q. 4 - 6) :
time dt, its temperature increases by dq.
4. (a) 5. (b) 6. (b)
Heat gained by gas = Heat conducted into the cylinder
dT
We have, k (T T0 )
dt
where T0 is the temperature of the surrounding. If T1 is the initial
temperature and T is the temperature at any time t, then

T t
dT
k dt
(T T0 )
T1 0

T
or ln(T T0 ) T kt
1

T T0
or ln kt
T1 T0
Ts
kt or nC p ( d ) KA dt
or T T0 (T1 T0 )e … (i) x

The body continues to lose heat till its temperature becomes equal
5R
to that of the surrounding. The loss of heat For monoatomic gas, Cp
2
Q mc (T1 T0 )
5R Ts
If the body loses half of the maximum loss that it can, then decrease n (d ) KA dt
in temperature 2 x

Q T1 T0 d 2KA
mc or (dt ) … (i)
2 2 (Ts ) 5nRx

If body loses this heat in time t, then its temperature at time t ' Let after time t, the temperature of the gas be T, then
will be
T t
d 2 KA
T1 T0 T1 T0 ( dt )
T1 (Ts ) 5nRx
2 2 T0 0
www.crackjee.xyz
566 MECHANICS & THERMODYNAMICS
(c) Power rediated is proportional to the temperature differ-
T 2 KA
or ln(Ts )T t ence between block and the room.
0 5nRx
and so in second case power radiated = 8 W.
Ts T 2 KAt (d) The heat energy absorbed by the block
or ln
Ts T0 5nRx Q = (mc) D T
= 80 × 10 = 800 J
2 KAt
5nRx Passage for (Q. 13 - 14) :
or (Ts T ) (Ts T0 )e
13. (b) 14. (c)
If we write Ts T T , then
2
Rs
We know that, solar constant S = T4
2 KAt r
T (Ts T0 )e 5nRx …(ii)
2
By equation of state, we have 7 108
or 1.4 × 103 = 5.67 × 10–8 × T4 1.5 1011
P V nR T

nR T 5800 K. Ans.
or V T Pa P … (iii)
Pa The energy radiated from the sun per unit area per second

From equations (ii) and (iii), we get E = T4

= 5.67 × 10–8 [5800]4


2 KAt
nR 5nRx = 6.43 × 107 W/m2 Ans.
V (Ts T0 )e
Pa 15. (A) p, r; (B) t; (C) q, r; (D) t
(A) Initially more heat will enter through section A but the metal
Let y is the displacement of the piston in time t, then will absorb some heat and less heat will leave from C.
Ay V dQ
(B) At steady state heat accumulation is same for all
dt
2 KAt
V nR sections.
5nRx
y (Ts T0 )e
A Pa A
dQ dT dT 1 dQ
(C) At steady state kA or
dt dx dx KA dt
Passage for (Q. 10 - 12) :
10. (b) 11. (d) 12. (b) dT
is inversely proportional to area of cross-section. Hence
dx
DT
(a) Power of the heaterP = ( mc ) is maximum at B and minimum at C.
t
(D) At steady state heat accumulation = 0
= 80 × 2
= 160 W dT
So 0 for any section.
(b) The power radiated by the block, dx
16. (A) p; (B) q; (C) s; (D) r
DT
P = ( mc ) 17. (A) q; (B) p; (C) s; (D) r
t
= 80 × 0.2 = 16 W

Solutions Exercise-8.5
1. The heat lost in evaporation per second T
= (0.1 × 10–3)× 2.27 × 106 227 = 0.80 × (200 × 10–4)× 3
= 227 J/s 10
If T is the temperature of the pitcher, then for steady state T 14oC
T or T0 – Ti = 14
H = kA Ti = T0 – 14
x
= 42 – 14
= 28oC Ans.
www.crackjee.xyz
HEAT TRANSFER 567

T 60°C r 1.5 r 10°C


2. We can write CDq = KA
x
t
A B E
If TB is the temperature of B, then
T
or C = KA
t x 60 TB TB 10
=
r 1.5r
KA T
= TB = 40oC Ans.
t C x
6. The rate at which heat radiated
4
200 1 10 (5 2.5) = A[T 4 T04 ]
=
0.40 100
= 0.4 × 5.67 × 10–8 × 200 ×10–4[8004 – 3004]
= 12.50 oC/s Ans.
= 182.12 J
3. The rate of flow of heat is given by
44 cal/s.
2 (T1 T2 ) 7. The resistance offered by the wall
H = K
r2 R = R1 + R2 + R3
ln
r1
L L1 L2 L3
or = K1 A K2 A K3 A
KA
= 0.15 (2 0.50)[120 30]
1.2
ln
1

233 J/s. Ans.

Cement
Wood

Brick
4. At steady state 20°C
Heat conduct through the rod = heat absorbed at the end

T
or KA = A(T 4 T04 )
x 2.5 cm 1 cm 2.5 cm

17 0
or K (10 4 ) = 6 ×10–8(10–4) (3004 – 2904) L1 L2 L2 L1 L2 L3
0.50 or =
K K1 K2 K3
K = 1.8 W/m-oC. Ans.
5. Suppose the resistance of each of rod of material is r, then resis- 2.5 1 25 2.5 1 25
or =
tance of each of material y will be 2r. The temperature of junction K 0.125 1.5 1
C and D will be equal and so there in no heat flow in between. The K = 0.624 W/m-oC
equivalent system of rods is shown in figure.
T
C The power of the heater H = KA
x
r 2r
30
= 0.624 × 137 2
60°C 10°C 28.5 10
E
A r B = 9 kW Ans.
r
2r

D
Solutions Exercise-8.6
1. The amount of heat added in 4 min Heat taken by aluminimum container
Q = 100 ×( 4 × 60 ) Q 2 = 100 × 0.2 × (T + 20).
= 2.4 × 104 cal. We can write Q = Q1 + Q2
Suppose temperature of the system becomes T. The heat taken or 2.4 × 104 =[200 × 0.5 × 20 + 200 × 80] + [ 100 × 0.2 × (T +
by ice 20)]
Q1 =200 × 0.5 × (20) + 200 × 80 + 200 × 1 × (T – 0) T = 25.5oC Ans.
www.crackjee.xyz
568 MECHANICS & THERMODYNAMICS
2. In one minute heat enters into the vessels T
Q 1 = m1L 7. H AB = KAB A
x
T C 80°C
= k1A
x
and Q2 = m2 L
T
= k2 A
x 40°C 80°C
A B
k1 m1
k2 = m2 80 40
= 50 × (1× 10–4) ×
0.20
100
= = 1W
150
Similarly, H AC = 8 W,
2 HBC = 0. Ans.
= Ans.
3
3. The electrical energy consumed per second 8. Suppose resistance of each of AB, CD, CF is r. The resistance of
E1 = 4 3r
A T part BDE or BCF is . The equivalent system is shown in
2
= 0.35 × (0.25) × 5.67 × 10–5 × (3000)4
figure.
= 40.19 W. Ans.
By consenation of heat energy, we can write
In the second case
H = H1 + H2
E 2 =0.35 × 0.25 × 5.67 0 × 10 –5 × ( 2500)4
= 19.38 W Ans. T3 T2
4. If t be the time taken to form the ice, then
L 2 3r/2 3r/2
t = ( y2 y12 )
2kT B
After substituting the given values, we get
t = 102.5 s Ans. r
5. If m is the mass of ice or steam produced then
(200 0) T1
m × 80 = KA …(i)
x
T1 TB TB T3 TB T2
(200 100) = +
and m × 540 = KA …(ii) r 3r / 2 3r / 2
1.5 x
3T1 2(T2 T3 )
0°C 200°C 100°C TB = Ans.
x 1.5 – x 7
From equations (i) and (ii), we get
x 1.4 m Ans. 9. (a) The maximum heat lost by the body
6. The mass of the water in the vessel Q = ms D T
m = V = ms ( 1 0 )
= 1000 × (10 × 10 × 10–6) (b) If DT be the fall in temperature in lossing 90% of the maxi-
= 0.1 kg. mum heat, then
The amount of heat lost in falling the temperature
0.90 ms ( 1 0 ) = ms D T
Q = mC T
= 0.1 × 4200 ×1 DT = 0.90 ( 1 0 ).
= 420 J. Thus final temperature = 1– 0.90 ( 1 0)
If t is the time to fall this temperature, then
(0.1 1 + 0.9 0 )
=
T
Q = kA t Now by Newton's law of cooling
x
d
50 20 = –k( 0 )
t dt
or 420 = 200×(2×10×10–4) 3
10
t = 0.035 s Ans.
www.crackjee.xyz
HEAT TRANSFER 569

(0.1 0.9
After simplifying above equations, we get T0 = 16°C
1 0) t
d If T ' be the temperature after 15 minutes, then
or = k dt
( 0)
0 80 T ' 80 T '
1
= k 16 …(iii)
15 2
(0.1 0.9 0)
or ln( 0)
1
1
= –kt From equations (i) and (iii), we get
T ' = 43 oC. Ans.
ln10
t = Ans.
k
13. If T is the temperature of the body at a time t, then by Newton's
10. As the gas can leak out from the hole, so the pressure in the vessel law of cooling
becomes equal to atomspheric pressure Pa. Let n be the number of
moles in the vessel at time t. Let an amount Q of heat is given to dT
= – k (T – TA)
the gas in time dt, its temperature increases by dT, then dt

Q = nCPdT dT
or C = – kC (T – TA)
If the temperature of the gas is T at time t, then dt
Substituting kC = – b,
Q Ta T
= dT
dt r C
We have = – b (T – TA)
dt
nC p dt Ta T
or = ... (i) 350
dt r dT t1
b
By Gas law, PV = nRT, or = dt
(T TA ) C 0
400
We have Pa(a 3) = nRT
T = Paa3/nR .... (ii) 350 b
or ln (T TA ) 400 = t1
or ndT + Tdn = 0 C
or ndT = – Tdn .... (iii)
C ln 2
b =
Pa a 3 t1
The initial amount of gas n0 = .
RT0 Now when the body X is connected to body Y, the body X loses
heat by radiation and by conduction. Thus
From above equations, we have
KA b ln 2
n dn t dt CdT = (T TA ) dt (T TA ) dt
= L t1
n0 (nRTa Pa a3 ) 0 CP rPa a3
After integrating and substituting the known values, we get
2Tat
Pa a 3 Ta kPa a3 KA ln 2
n = 1 1 e T = 300 50exp 2t1
RTa T0 CL t1
Ans.
Ans.
14. If A is the cross-sectional area of the rod, then
11. Suppose T is the temperature of the lower face of the cylinder,
and in time dt the temperature of disc increases by dT, then dT
H = KA
dT 400 T dx
mc = KA
dt
dT
= A
mcl 350 dT T dx
or t =
kA 300 (400 T ) x T
A dT
After simplifying and substituting the given values, we get or dx = …(i)
0 H T1 T
t = 2 min 46 s Ans.
12. If T0 be the temperature of the surroundings, then A T
or x = ln
H T1
dT
= k (T – T0) For x = ,
dt
T = T2
80 64 80 64
or = k T0 …(i)
5 2 A T2
= .ln
H T1
80 52 80 52
and = k T0 …(ii)
10 2
www.crackjee.xyz
570 MECHANICS & THERMODYNAMICS
From equation (i) and (ii), we have 2 CA 3/ 2
or x = T1 T23/ 2 …(i)
x/ 3 H
T
T = T1 1 Ans. For x = , T = T2
T1
Also heat flow per second, per unit area 2 CA 3/ 2
= T1 T23/ 2 …(ii)
3 H
T
H = ln 1 Ans. From equations (i) and (ii), we have
T2
x T13/ 2 T 3/ 2
dT =
15. We have H = KA
dx T13/ 2 T23/ 2

dT 2 /3
3/ 2
= C TA x T2
dx or T = T1 1 1 Ans.
T1
x CA T
or dx = T 1/ 2 dT
0 H T1

Vous aimerez peut-être aussi